You are on page 1of 644

ECG Masters’ Collection

Favorite ECGs from Master Teachers Around the World


ECG Masters’ Collection
Favorite ECGs from Master Teachers Around the World

ECG Masters’
Editors:
Mohammad Shenasa, MD • Mark E. Josephson, MD • N. A. Mark Estes III, MD
Ezra A. Amsterdam, MD • Melvin Scheinman, MD

Collection
230 exceptional electrocardiogram case studies curated from the libraries of 60
internationally recognized master teachers of ECG interpretation are brought together
in this one-of-a-kind resource for student and teacher alike.
Organized by disease type, ECG case studies are presented in a clinical context fol-
lowed by questions and discussion. Medical students, residents, fellows, physicians
— anyone who is involved in caring for patients with various cardiovascular diseases
and other systemic pathologies — will find this unique collection with a global
perspective useful and practical in developing the skills necessary to reading ECGs.
Favorite ECGs from
Key Topics
Master Teachers
n  onduction Disturbances: Sinus Node Disease/Sick Sinus Syndrome, AV Conduction
C
Disturbances, AV Blocks, Bundle Branch Blocks, and Fascicular Blocks
Around the World
n  iscellaneous Phenomena: Concealed Conduction, Superabnormalities, Aberrancy
M

n
Conduction, Premature Atrial and Ventricular Contractions (PACs and PVCs)
Preexcitation Syndromes
Editors:
n Early Repolarization Mohammad Shenasa
Long and Short QT Syndromes
Mark E. Josephson
n

n Brugada Syndrome
n

n
Narrow QRS Complex Arrhythmias
Wide Complex Arrhythmias
N. A. Mark Estes III
n Ischemia and Infarction
Shenasa
Ezra A. Amsterdam
Electrolyte Disturbances, Pharmacological and Recreational Agents
Melvin Scheinman
n

n Paced Rhythms and Device Troubleshooting


Josephson
n Heart Failure, LVH, and Cardiomyopathies Estes
n Congenital Heart Diseases Amsterdam
Special Considerations: Age, Race, Gender, and Athletes Scheinman
n
Forewords by
n Syncope and ECG Troubleshooting
Francis E. Marchlinski
and Samuel Lévy
This book includes a free digital
edition for use by the first buyer.
3405 W. 44th Street For additional information regarding
Minneapolis, Minnesota 55410 personal or institutional use, please
www.cardiotextpublishing.com visit www.cardiotextpublishing.com.
+1 (612) 925-2053
ECG Masters’ Collection
Favorite ECGs from Master Teachers
Around the World

Mohammad Shenasa, MD
Mark E. Josephson, MD
N. A. Mark Estes III, MD
Ezra A. Amsterdam, MD
Melvin Scheinman, MD

Minneapolis, Minnesota
© 2017 Mohammad Shenasa, Mark E. Josephson, N. A. Mark Estes III, Ezra A. Amsterdam, Melvin Scheinman

Cardiotext Publishing, LLC


3405 W. 44th Street
Minneapolis, Minnesota 55410
USA

www.cardiotextpublishing.com

Any updates to this book may be found at: www.cardiotextpublishing.com/ecg-masters-collection

Comments, inquiries, and requests for bulk sales can be directed to the publisher at: info@cardiotextpublishing.com.

All rights reserved. No part of this book may be reproduced in any form or by any means without the prior
permission of the publisher.

All trademarks, service marks, and trade names used herein are the property of their respective owners and are
used only to identify the products or services of those owners.

This book is intended for educational purposes and to further general scientific and medical knowledge, research,
and understanding of the conditions and associated treatments discussed herein. This book is not intended to
serve as and should not be relied upon as recommending or promoting any specific diagnosis or method of
treatment for a particular condition or a particular patient. It is the reader’s responsibility to determine the proper
steps for diagnosis and the proper course of treatment for any condition or patient, including suitable and
appropriate tests, medications or medical devices to be used for or in conjunction with any diagnosis or
treatment.

Due to ongoing research, discoveries, modifications to medicines, equipment and devices, and changes in
government regulations, the information contained in this book may not reflect the latest standards,
developments, guidelines, regulations, products or devices in the field. Readers are responsible for keeping up to
date with the latest developments and are urged to review the latest instructions and warnings for any medicine,
equipment or medical device. Readers should consult with a specialist or contact the vendor of any medicine or
medical device where appropriate.

Except for the publisher’s website associated with this work, the publisher is not affiliated with and does not
sponsor or endorse any websites, organizations or other sources of information referred to herein.

The publisher and the authors specifically disclaim any damage, liability, or loss incurred, directly or indirectly,
from the use or application of any of the contents of this book.

Unless otherwise stated, all figures and tables in this book are used courtesy of the authors.

Library of Congress Control Number:  2017937042

ISBN: 978-1-942909-08-8

eISBN: 978-1-942909-18-7

Printed in The United States of America


Dedication

We dedicate this book in memory of Mark E. Josephson, MD (January 27, 1943 – January 11, 2017),
who inspired and mentored many of us. He gave the electrogram a new look - “ECG as a mapping
tool.” He is also the senior editor on this book.
Contents

Contributors............................................................................................................................vii
Forewords................................................................................................................................xv
Dr. Francis Marchlinski
Dr. Samuel Lévy
Preface....................................................................................................................................xix
Abbreviations.........................................................................................................................xxi
Video Legends......................................................................................................................xxiii
Section 1 Introduction to the Interpretation of the Electrocardiogram���������������������������1
Section 2 Conduction Disturbances: Sinus Node Disease/Sick Sinus Syndrome,
AV Conduction Disturbances, AV Blocks, Bundle Branch Blocks, and
Fascicular Blocks....................................................................................................3
Section 3 Miscellaneous Phenomena: Concealed Conduction, Superabnormalities,
Aberrancy Conduction, Premature Atrial and Ventricular
Contractions (PACs and PVCs).............................................................................41
Section 4 Preexcitation Syndromes....................................................................................77
Section 5 Early Repolarization (ECG Pattern and the Syndrome)................................... 135
Section 6 Long and Short QT Syndromes......................................................................... 153
A. Long QT Syndrome...................................................................................... 153
B. Short QT Syndrome..................................................................................... 173
C. Torsades de Pointes..................................................................................... 175
D. Other Proarrhythmias.................................................................................. 193
Section 7 Brugada Syndrome............................................................................................ 197
Section 8 Narrow QRS Complex Arrhythmias..................................................................227
A. Inappropriate Sinus Tachycardia.................................................................227
B. Sinus Node Reentrant Tachycardia.............................................................229
C. Atrial Tachycardia/Atrial Flutter..................................................................231
D. Atrioventricular Nodal Reentrant Tachycardia...........................................255
E. Atrioventricular Reentrant Tachycardia......................................................283
F. Atrial Fibrillation..........................................................................................287
G. Junctional Rhythms.....................................................................................295

 v
Section 9 Wide Complex Arrhythmias..............................................................................297
A. Ventricular Tachycardia/Fibrillation............................................................297
B. Supraventricular Tachycardia (SVT) with Bundle Branch Block (BBB).......355
C. Preexcited Tachycardia................................................................................ 371
D. Idioventricular Rhythm................................................................................373
Section 10 Ischemia and Infarction.....................................................................................375
Section 11 Electrolyte Disturbances, Pharmacological and Recreational Agents........... 413
Section 12 Paced Rhythms and Device Troubleshooting..................................................425
Section 13 Heart Failure, LVH, and Cardiomyopathies......................................................459
A. Arrhythmogenic Right Ventricular
Dysplasia/Cardiomyopathy (ARVD/C)..........................................................459
B. Hypertrophic Cardiomyopathy (HCM)........................................................479
C. Dilated Cardiomyopathy (DCM)..................................................................495
D. Chagas Cardiomyopathy.............................................................................501
E. Takotsubo (Stress) Cardiomyopathy............................................................ 511
F. Non-Compaction Cardiomyopathy.............................................................523
G. Pericarditis....................................................................................................525
H. Other Cardiomyopathies.............................................................................529
Section 14 Congenital Heart Diseases................................................................................533
Section 15 Special Considerations: Age, Race, Gender, and Athletes..............................583
Section 16 Syncope and ECG Troubleshooting..................................................................593
Appendix...............................................................................................................................621

vi  u  Contents
Contributors

Editors Contributors
Mohammad Shenasa, MD, FACC, FHRS, Ahmed Abdel Aziz, MD, PhD
FAHA, FESC Professor, Critical Care Medicine Department,
Attending Physician, Department of Cairo University Hospitals, Cairo, Egypt
Cardiovascular Services, O’Conner Hospital;
Heart & Rhythm Medical Group, San Jose, Simon Abou Jaoude, MD
California Cardiology Department, Hotel-Dieu de France,
Saint Joseph University, Beirut, Lebanon
Mark E. Josephson, MD, FACC, FHRS, FAHA
Director, Harvard-Thorndike Electrophysiology Baris Akdemir, MD
Institute and Arrhythmia Service; Cardiac Electrophysiology Fellow, Cardiology,
Chief Emeritus, Division of Cardiovascular University of Minnesota, Minneapolis,
Medicine, Beth Israel Deaconess Medical Minnesota
Center; Herman C. Dana Professor of Medicine,
Jason Andrade, BSc, MD, FRCPC, FHRS
Harvard Medical School, Boston,
Clinical Assistant Professor, Department of
Massachusetts
Medicine, University of British Columbia,
N. A. Mark Estes III, MD, FACC, FHRS, Vancouver, British Columbia, Canada; Adjunct
FAHA, FESC Professor, Université de Montréal, Clinical
Professor of Medicine, Tufts University School Electrophysiology Service, Montreal Heart
of Medicine; Director, New England Cardiac Institute, Montreal, Quebec, Canada
Arrhythmia Center, Tufts Medical Center,
Samuel J. Asirvatham, MD, FACC, FHRS
Boston, Massachusetts
Consultant, Department of Cardiovascular
Ezra A. Amsterdam, MD Medicine, Division of Pediatric Cardiology,
Distinguished Professor, Associate Chief Department of Physiology and Biomedical
(academic affairs), Division of Cardiovascular Engineering; Professor of Medicine and
Medicine, University of California, Davis, Pediatrics, Mayo Clinic College of Medicine;
Medical Center, Sacramento, California Program Director, EP Fellowship Program,
Mayo Clinic, Rochester, Minnesota
Melvin Scheinman, MD, FACC, FHRS
Professor of Medicine, Walter H. Shorenstein Nitish Badhwar, MD, FACC, FHRS
Endowed Chair in Cardiology; Chief of Director, Cardiac Electrophysiology Training
Cardiology Genetics Arrhythmia Program, Program, Professor of Medicine, University of
University of California, San Francisco, California, San Francisco, San Francisco,
San Francisco, California California

  vii
Adrian Baranchuk, MD, FACC, FRCPC, FCCS Catalin A. Buzea, MD, PhD
Professor of Medicine (Tenure), Head, Heart Cardiology Consultant, Cardiology
Rhythm Service, Queen’s University, Kingston, Department, Colentina University Hospital;
Ontario, Canada Associate Professor, “Carol Davila” University of
Medicine, Bucharest, Romania
Raimundo Barbosa-Barros, MD
Chief of Coronary Center, Messejana Hospital David J. Callans, MD, FHRS
Dr. Carlos Alberto Studart Gomes, Fortaleza, Professor of Medicine, Perelman School of
Ceará, Brazil Medicine; Associate Director of
Electrophysiology, University of Pennsylvania
Antoni Bayés de Luna, MD, PhD, FESC, FACC Health System, Philadelphia, Pennsylvania
Senior Investigator, Catalan Institute of
Cardiovascular Sciences, St. Pau Hospital, Francesc Carreras Costa, MD
Barcelona, Spain Cardiology Department, Hospital Santa Creu
i Sant Pau, Universitat Autónoma de Barcelona,
Bernard Belhassen, MD, FHRS Barcelona, Spain
Department of Cardiology, Tel-Aviv Sourasky
Medical Center; Sackler Faculty of Medicine, Lily Chen, MD
Tel-Aviv University, Tel-Aviv, Israel Resident of Internal Medicine, University
of California, Davis, Medical Center,
David G. Benditt, MD, FACC, FHRS, FRCPC, Sacramento, California
FESC
Cardiac Arrhythmia Center, Cardiovascular Alan Cheng, MD, FACC, FHRS, FAHA
Division, University of Minnesota Medical Associate Professor of Medicine, Associate
School, Minneapolis, Minnesota Professor of Pediatrics, Johns Hopkins
University School of Medicine, Baltimore,
Dan Blendea, MD, PhD, FHRS Maryland
Cardiac Arrhythmia Service, Department of
Medicine, Massachusetts General Hospital, Paolo China, MD
Boston, Massachusetts Unit of Electrophysiology and Cardiac Pacing,
Ospedale Dell’Angelo, Venice, Italy
Pedro Brugada, MD, PhD
Chairman, Cardiovascular Division, UZ Xavier Copie, MD
Brussel, Brussels, Belgium Cardiologist, Arrhythmia Department, Centre
Cardiologique du Nord, Saint-Denis, France
Jonathan Bui, MD
Research Assistant, Division of Cardiovascular Jane E. Crosson, MD
Medicine, University of California, Davis, Associate Professor, Pediatrics, Johns Hopkins
Medical Center, Sacramento, California Hospital, Baltimore, Maryland

Haran Burri, MD Mohammad Dalili, MD


Assistant Professor, Cardiology Department, Rajaie Cardiovascular Medical and Research
University Hospital of Geneva, Geneva, Center, Iran University of Medical Sciences,
Switzerland Tehran, Iran

viii  u  Contributors
Andrei G. Dan, MD, PhD, FESC, FAHA, Andrew E. Epstein, MD, FACC, FHRS, FAHA
FEHRA Professor of Medicine, Electrophysiology
Head of Cardiology Department and Internal Section, Cardiovascular Division, University of
Medicine Clinic, Colentina University Hospital; Pennsylvania, Philadelphia, Pennsylvania
Professor, “Carol Davila” University of
Medicine, Bucharest, Romania Sabine Ernst, MD, PhD, FESC
Consultant Cardiologist, Reader in Cardiology,
Judith Daniels, RN, CRDS, CEPS Lead EP Researcher, Royal Brompton Hospital,
Registered Nurse, EP Lab, Milpark Hospital, National Heart and Lung Institute, Imperial
Johannesburg, Gauteng, South Africa College, London, United Kingdom

Luiz Carlos de Abreu, PhD Luis Alberto Escobar Robledo, MS


Chief, Design of Studies and Scientific Writing Fellow, Catalan Institute of Cardiovascular
Laboratory, ABC School of Medicine, Santo Sciences, St. Pau Hospital, Barcelona, Spain
André, São Paulo, Brazil; Program in Molecular
and Integrative Physiological Sciences (MIPS), Tamer S. Fahmy, MD, PhD
Department of Environmental Health, Harvard Associate Professor, Critical Care Medicine
T. H. Chan School of Public Health, Boston, Department, Cairo University Hospitals, Cairo,
Massachusetts Egypt

Arnaud Denis, MD Guy Fontaine, MD, PhD, HDR


Hôpital Cardiologique du Haut Lévèque, CHU Centre Hospitalier Pitié-Salpêtrière and
de Bordeaux, Université de Bordeaux and Université Pierre et Marie Curie, Department of
LYRIC Institute, Bordeaux, France Rhythmology, Paris, France

Nicolas Derval, MD Robert Frank, MD


Hôpital Cardiologique du Haut Lévèque, CHU Institut de Cardiologie, Hopital Pitié
de Bordeaux, Université de Bordeaux and Salpétrière, Paris, France
LYRIC Institute, Bordeaux, France
Javier García-Niebla, RN
Marc Dubuc, MD, FRCPC, FACC, FHRS Servicios Sanitarios del Area de Salud de El
Associate Professor of Medicine, Université de Hierro, Valle del Golfo Health Center, Canary
Montréal; Clinical Electrophysiology Service, Island, Spain
Montreal Heart Institute, Montreal, Quebec,
Diego Goldwasser, MD
Canada
Institut Cátala de Ciences Cardiovasculars, Sant
Kenneth A. Ellenbogen, MD, FHRS Pau Hospital; Cardiology Department, Hospital
Kontos Professor of Medicine, Chair, Division Quiron Barcelona, Barcelona, Spain
of Cardiology, Virginia Commonwealth
Judith A. Groeneweg, MD, PhD
University, Pauley Heart Center, Richmond,
University Medical Center Utrecht, The
Virginia
Netherlands

Contributors  u  ix
Majid Haghjoo, MD, FACC, FESC Heikki V. Huikuri, MD, PhD
Director, Department of Cardiac Professor, Medical Research Center Oulu,
Electrophysiology, Rajaie Cardiovascular Research Unit of Internal Medicine, University
Medical and Research Center, Tehran, Iran of Oulu and Oulu University Hospital, Oulu,
Finland
Michel Haïssaguerre, MD, PhD
Hôpital Cardiologique du Haut Lévèque, CHU James E. Ip, MD
de Bordeaux, Université de Bordeaux and Assistant Professor of Medicine, Division of
LYRIC Institute, Bordeaux, France Cardiology, Cardiac Electrophysiology
Laboratory, Cornell University Medical Center,
Frederick T. Han, MD, FACC, FHRS New York Presbyterian Hospital, New York,
Assistant Professor of Medicine, Division of New York
Cardiovascular Medicine, University of Utah
Health Sciences Center, Salt Lake City, Utah Pierre Jaïs, MD, PhD
Hôpital Cardiologique du Haut Lévèque, CHU
Richard N. Hauer, MD, PhD, FESC de Bordeaux, Université de Bordeaux and
Professor of Cardiology, Netherlands Heart LYRIC Institute, Bordeaux, France
Institute, University Medical Center Utrecht,
The Netherlands Mohammad-Ali Jazayeri, MD
Division of Cardiovascular Diseases, University
Shahriar Heidary, MD, FACC of Kansas Hospital & Medical Center, Kansas
Adjunct Clinical Instructor, Department of City, Kansas
Internal Medicine, Division of Cardiovascular
Medicine, Stanford University Medical School, Mohammad-Reza Jazayeri, MD
Stanford, California Heart, Lung & Vascular, Bellin Health, Green
Bay, Wisconsin
Hein Heidbuchel, MD, PhD, FESC, FEHRA
Professor and Chair, Department of Cardiology, Charles Jazra, MD, FACC, FESC
Antwerp University Hospital, Antwerp, Cardiology Department, Saint Joseph Hospital,
Belgium; Guest Professor, Cardiology, Hasselt Bauchrieh, Beirut, Lebanon
University, Hasselt, Belgium
Juhani Junttila, MD, PhD
Mélèze Hocini, MD Associate Professor, Medical Research Center
Hôpital Cardiologique du Haut Lévèque, CHU Oulu, Research Unit of Internal Medicine,
de Bordeaux, Université de Bordeaux and University of Oulu and Oulu University
LYRIC Institute, Bordeaux, France Hospital, Oulu, Finland

Henry H. Hsia, MD, FACC, FHRS Gautham Kalahasty, MD


Health Science Professor of Medicine, Program Director, Cardiology Fellowship
University of California, San Francisco; Program, Virginia Commonwealth University,
Chief, Arrhythmia Service, VA Medical Center, Pauley Heart Center, Richmond, Virginia
San Francisco, California

x  u  Contributors
Jonathan Kalman, MBBS, PhD, FRACP, FHRS Balaji Krishnan, MD
Professor of Medicine and Director of Cardiac Cardiac Arrhythmia Center, Cardiovascular
Arrhythmia Service, Department of Cardiology, Division, University of Minnesota Medical
Royal Melbourne Hospital and Department of School, Minneapolis, Minnesota
Medicine, University of Melbourne, Melbourne,
Australia Gilles Lascault, MD
Cardiologist, Arrhythmia Department, Centre
Demosthenes G. Katritsis, MD, PhD, FRCP, Cardiologique du Nord, Saint-Denis, France
FESC, FACC
Beth Israel Deaconess Medical Center, Harvard Robert Lemery, MD, FHRS, FESC, FRCPC
Medical School, Boston, Massachusetts Cardiac Electrophysiology, University of Ottawa
Heart Institute, Ottawa, Canada
George D. Katritsis, MBChB, BSc
Oxford University Clinical Academic Graduate Antoine Lepillier, MD
School, Radcliffe Hospital, Oxford, UK Cardiologist, Arrhythmia Department, Centre
Cardiologique du Nord, Saint-Denis, France
Tuomas Kenttä, PhD
Post doc researcher, Medical Research Center Bruce B. Lerman, MD
Oulu, Research Unit of Internal Medicine, H. Altschul Master Professor of Medicine;
University of Oulu and Oulu University Chief, Division of Cardiology; Director, Cardiac
Hospital, Oulu, Finland Electrophysiology Laboratory, Cornell
University Medical Center, New York
Bradley P. Knight, MD, FACC, FHRS Presbyterian Hospital, New York, New York
Medical Director, Center for Heart Rhythm
Disorders, Bluhm Cardiovascular Institute, Mohamed Magdy, MSc, L’AFSA, PhD, MD
Northwestern Memorial Hospital, Cooley Electrophysiology Fellow, Nancy CHU France;
Professor of Medicine, Lecturer, Cairo University Hospital,
Northwestern University, Feinberg School of Egypt; Head of EP Lab, Al Qassimi Hospital,
Medicine, Chicago, Illinois United Arab Emeriates

Pieter Koopman, MD Moussa Mansour, MD


Electrophysiologist, Heart Center Hasselt, Jessa Cardiac Arrhythmia Service, Department of
Hospital, Hasselt, Belgium Medicine, Massachusetts General Hospital,
Boston, Massachusetts
Peter R. Kowey, MD, FACC
Professor, Jefferson Medical College, Lankenau Frank I. Marcus, MD
Institute for Medical Research, Professor of Medicine, Section of Cardiology,
Philadelphia, Pennsylvania  University of Arizona Medical Center, Tucson,
Arizona
Andrew D. Krahn, MD, FRCPC
Heart Rhythm Services, Department of Albert Massó van Roessel, MD
Medicine, Division of Cardiology, St. Paul’s Department of Medicine, Universitat
Hospital, University of British Columbia, Internacional de Catalunya, Barcelona, Spain
Vancouver, British Columbia, Canada

Contributors  u  xi
John M. Miller, MD, FHRS, FACC Inger Olson, MD
Professor of Medicine, Indiana University Clinical Associate Professor, Department of
School of Medicine; Director, Clinical Cardiac Pediatrics (Cardiology), Stanford University
Electrophysiology, Indiana University Health, School of Medicine, Stanford, California
Indianapolis, Indiana
Ali Oto, MD, FESC, FACC, FHRS, FISHNE
Fred Morady, MD Professor of Cardiology, Chairman, Department
McKay Professor of Cardiovascular Disease, of Cardiology, MHG, Memorial Ankara
Professor of Medicine, University of Michigan Hospital, Ankara, Turkey
Health System, Ann Arbor, Michigan
Santosh K. Padala, MD
Daniel J. Murphy, Jr., MD Cardiac Electrophysiology Fellow, Division of
Professor, Department of Pediatrics Cardiology, Virginia Commonwealth
(Cardiology) Stanford University School of University, Pauley Heart Center, Richmond,
Medicine, Stanford, California Virginia

Robert J. Myerburg, MD Carlos Alberto Pastore, MD, PhD, FESC


Professor of Medicine and Physiology, Division Director, Clinical Unit of Electrocardiography,
of Cardiology, American Heart Association Heart Institute (InCor), Hospital das Clínicas
Chair in Cardiovascular Research, University of da Faculdade de Medicina da Universidade de
Miami Miller School of Medicine, Miami, São Paulo, São Paulo, Brazil
Florida
Olivier Paziaud, MD
Yuji Nakazato, MD, PhD, FESC Cardiologist, Arrhythmia Department, Centre
Professor, Department of Cardiology, Heart Cardiologique du Nord, Saint-Denis, France
Center, Juntendo University Urayasu Hospital,
Urayasu City, Chiba, Japan Horácio Gomes Pereira Filho, MD
Assistant Physician, Clinical Unit of
Amit Noheria, MBBS, SM Electrocardiography, Heart Institute (InCor),
Assistant Professor of Medicine, Cardiac Hospital das Clínicas da Faculdade de Medicina
Electrophysiology, Washington University da Universidade de São Paulo, São Paulo, Brazil
School of Medicine, St. Louis, Missouri
Andrés Ricardo Pérez-Riera, MD, PhD
I. W. P. Obel, MBChB, FCP(SA), FACC Post Graduate Advisor at Design of Studies and
Specialist Cardiologist/Electrophysiologist, Scientific Writing Laboratory, ABC School of
Milpark Hospital, Johannesburg, Gauteng, Medicine, Santo André, São Paulo, Brazil 
South Africa
Femi Philip, MD
Sercan Okutucu, MD Division of Cardiovascular Medicine, Kaiser
Department of Cardiology, Cardiology Permanente, Medical Center, Sacramento,
Specialist, Memorial Ankara Hospital, Ankara, California
Turkey
Olivier Piot, MD
Cardiologist, Arrhythmia Department, Centre
Cardiologique du Nord, Saint-Denis, France

xii  u  Contributors
Philip Podrid, MD Scott Sakaguchi, MD, FHRS, FACC, FACP
Professor, Boston University School of Professor, Department of Internal Medicine,
Medicine; Lecturer in Medicine, Harvard University of Minnesota, Minneapolis,
Medical School, Boston, Massachusetts; Minnesota
Attending Physician, West Roxbury VA
Hospital, West Roxbury, Massachusetts Nelson Samesima, MD, PhD
Supervising Physician, Clinical Unit of
Guillem Pons-Lladó, MD Electrocardiography, Heart Institute (InCor),
Cardiology Department, Hospital Santa Creu Hospital das Clínicas da Faculdade de Medicina
i Sant Pau, Universitat Autònoma de Barcelona, da Universidade de São Paulo, São Paulo, Brazil
Barcelona, Spain
Luca Santini, MD, PhD
Sergio Richter, MD Cardiology Division, G.B. Grassi Hospital,
Associate Professor of Medicine and Ostia-Lido, Rome, Italy
Cardiology, Department of Electrophysiology,
Heart Center, University of Leipzig, Leipzig, Massimo Santini, MD, FESC, FACC
Germany Past-President, World Society of Arrhythmias,
Rome, Italy
Michael P. Riley, MD, PhD
Assistant Professor of Medicine, Peter J. Schwartz, MD, FACC, FHRS, FAHA,
Electrophysiology Section, Cardiovascular FESC
Division, University of Pennsylvania, Director, Center for Cardiac Arrhythmias of
Philadelphia, Pennsylvania Genetic Origin, IRCCS Istituto Auxologico
Italiano, Milan, Italy
Magdi M. Saba, MD, FHRS
Consultant Cardiac Electrophysiologist, Hossein Shenasa, MD, MS, FACC
St. George’s Hospital and University of London, Staff Cardiologist, Electrophysiologist, Heart &
London, England Rhythm Medical Group, San Jose Area
Hospitals, San Jose, California
Frédéric Sacher, MD, PhD
Hôpital Cardiologique du Haut Lévèque, CHU Mariah Smith
de Bordeaux, Université de Bordeaux and Heart & Rhythm Medical Group, San Jose,
LYRIC Institute, Bordeaux, France California

Mohammad Ali Sadr-Ameli, MD Christian Steinberg, MD, FRCPC


Rajaie Cardiovascular Medical and Research Heart Rhythm Services, Department of
Center, Iran University of Medical Sciences, Medicine, Division of Cardiology, St. Paul’s
Tehran, Iran Hospital, University of British Columbia,
Vancouver, British Columbia, Canada
Johan Saenen, MD, PhD
Staff Member, Department of Cardiology, Benjamin Stripe, MD
Antwerp University Hospital, Antwerp, Fellow of Division of Cardiovascular Medicine,
Belgium University of California, Davis, Medical Center,
Sacramento, California

Contributors  u  xiii
Sakis Themistoclakis, MD Edward P. Walsh, MD, FHRS
Director, Unit of Electrophysiology and Cardiac Chief, Cardiac Electrophysiology, Boston
Pacing, Ospedale Dell’Angelo, Venice, Italy Children’s Hospital, Boston, Massachusetts;
Professor of Pediatrics, Harvard Medical
Vassil Traykov, MD, FEHRA School, Boston Massachusetts
Head of Department of Electophysiology and
Pacing, Clinic of Cardiology, Tokuda Hospital, David E. Ward, MD, FACC
Sofia, Bulgaria Consultant Cardiologist and
Electrophysiologist, Retired, London, England
Zian H. Tseng, MD, MAS
Murray Davis Endowed Professor, Associate Christopher E. Woods, MD, PhD, FHRS
Professor of Medicine in Residence, Cardiac Medical Director, Cardiac Electrophysiology,
Electrophysiology Section, Cardiology Division, Palo Alto Medical Foundation, Burlingame,
University of California, San Francisco, California
San Francisco, California
Begüm Yetiş Sayın, MD
George F. Van Hare, MD Department of Cardiology, Cardiology
Division Chief, Pediatric Cardiology, Louis Specialist, Memorial Ankara Hospital, Ankara,
Larrick Ward Professor of Pediatrics, Turkey
Washington University School of Medicine,
St. Louis, Missouri Li Zhang, MD
Associate Professor, Jefferson Medical College,
Nishant Verma, MD, MPH Lankenau Institute for Medical Research,
Assistant Professor of Medicine-Cardiology, Philadelphia, Pennsylvania
Cardiac Electrophysiology, Bluhm
Cardiovascular Institute, Northwestern
Memorial Hospital, Feinberg School of
Medicine, Northwestern University, Chicago,
Illinois

xiv  u  Contributors
Foreword

It is exciting to see the publication of this case-based collection of ECGs from “Masters” in
electrocardiography and electrophysiology.  It is also a pleasure for me to contribute the foreword to
this important book.  This collection should be read by both early trainees and experienced
electrocardiographers. There are pearls littered throughout and the explanations and interpretations
grounded in physiology and fundamental vector analysis. The clinical relevance is made obvious by
the case format.  The outstanding group of Editors have done a superb job with the organization of
the text in dividing it into focused sections to maximize ease of review and educational value. The
ECG recordings are characteristically the best from the experts’ collections. The tracings show the
incredible  value of this simple yet elegant tool for diagnosing and localizing arrhythmias and
recognizing signature ECG patterns associated with unique genetically determined and acquired
arrhythmogenic syndromes.
It is important for me to also pay a special tip of the hat to one of the Co-Editors, Dr. Mark
Josephson. Not only is he the father of modern cardiac electrophysiology, but Mark has also been
inspirational in his love of the 12-lead ECG and his desire to maximize its full potential.  For more
than 40 years, he has mentored a long collection of trainees on the correct interpretation of the 12-lead
ECG. Such phrases as “burn it in your brain” for a unique ECG pattern that was critical to recognize
within a second of display  are always remembered with a smile throughout one’s career. I was a
lucky  trainee who has many critical ECG images “burned in my brain.” His participation in
this important text adds to the glow of the other stellar editors and ECG aficionados and provides the
‘Grand Cru’ stamp to this effort. 
This book should serve as an important reference, and I guarantee it will be pulled from the shelf
for decades to come.  It is a gem and should be enjoyed by even those with only a modest interest in
the 12-lead ECG and the care of patients with cardiac arrhythmias. The true fans of the ECG will be
awed by the experience.
Francis E. Marchlinski, MD, FACC, FHRS, FAHA
University of Pennsylvania School of Medicine
Philadelphia, Pennsylvania

  xv
Foreword

The father of electrocardiography is Willem Einthoven (1860–1927), who first recorded the first
human ECG in 1902 at the University of Leiden, the Netherlands, where he used to teach. He received
the Nobel Prize in 1924 for his major invention. Since then, the ECG has fascinated a number of
cardiologists by the number of information that can be derived. Some of these famous
electrocardiographers such as Alfred Pick and Richard Langendorf, have described a number of
phenomena such as “concealed conduction” or “tachycardia/bradycardia-dependent bundle branch
block,” which were found later on to be correct using invasive electrophysiology. My generation is
fortunate to have met some of them and to learn from them. It is refreshing that Mohammad Shenasa,
Mark E. Josephson, N. A. Mark Estes III, Ezra A. Amsterdam, and Melvin Scheinman, the Editors of
this ECG Masters’ collection, have emphasized that the ECG remains an invaluable tool for clinicians
despite the advances made in the field of arrhythmias. This contribution, with the participation of
experts from around the world, will be extremely useful to clinicians, fellows in cardiology, and all
those who are involved in the management of cardiac arrhythmia patients.
This book is not a simple collection of ECGs. It is, in fact, the report of clinical situations in which
the ECG guides the diagnosis, signals the choice of the appropriate tests, and leads to the appropriate
management. The cases presented are not rare or unusual. They represent clinical settings that
cardiologists and clinicians will encounter in their daily practice—and this, in my view, adds to the
educational value of this book. I found it very interesting and enjoyable to read the ECG tracings put
into their clinical context.
The field of cardiac arrhythmias has been enriched by the major advances made in the last four
decades in better understanding tachycardia mechanisms due to the advent of intracardiac recordings,
invasive clinical electrophysiology, ablation, and new mapping techniques. The authors of the cases
refer to these techniques to support their interpretation and document the concepts used in their
interpretation, adding, when necessary, references and suggested reading.
I have no doubt that the authors have succeeded in providing the reader with an interesting,
enjoyable, and useful collection of clinical situations in which the correct ECG interpretation has
played a major role.
Samuel Lévy, MD, FACC, FESC, FAHA
Aix -Marseille Université
Marseille School of Medicine, France

  xvii
Preface

Each year, several new books or new editions are published on electrocardiography. Since the invention
of the electrocardiogram (ECG) by Willem Einthoven almost 110 years ago, the ECG has become the
most commonly used test worldwide, and its use continues to increase. The medical community has
subsequently gained a wealth of knowledge from the ECG for the diagnosis of many cardiac and non-
cardiac conditions, ranging from acute ischemia and infarction on the one hand to arrhythmias on the
other. Furthermore, the ECG is the first step in evaluating patients arriving at the emergency department,
as the results are immediately available. Likewise, the ECG has been used as a screening test for athletes
and is also used to identify patients at a high-risk of arrhythmias and sudden cardiac death.
Despite the emergence of other imaging modalities, the ECG remains a benchmark diagnostic test
and is an integral part of a risk stratification algorithm in almost all guidelines of all disciplines of
medicine.
Since the success of our 2015 book, The ECG Handbook of Contemporary Challenges, many of our
colleagues and friends encouraged us to provide a case-based collection of ECGs. Thus, we have
invited the most renowned physicians from around the world who read and interpret ECGs (i.e.,
electrocardiographers) to provide their most insightful examples. We also asked them to include their
interpretation of the ECG findings with appropriate, up-to-date references. All tracings are well
annotated and described. In addition, we suggested providing questions for the readers relating to the
ECGs and a discussion that makes this book useful for trainees at all levels.
Although the main theme of this book is electrocardiography, other imaging techniques are
discussed to validate the authors’ interpretations. We are extremely grateful that all of our colleagues
have unanimously accepted our invitation and provided their best cases.
The cases in this book are arranged according to topics in electrocardiography and arrhythmias.
Areas of focus include ECGs of inherited arrhythmia syndromes, athletic ECGs, and ECGs in
congenital heart disease. The book also discusses new ECG criteria/markers and syndromes related
to recently discovered channelopathies such as Brugada syndrome, early repolarization syndrome, and
the like. We are confident that this collection of ECGs from masters of electrocardiography from
around the world will prove useful and of great educational value to clinicians in many areas of
medicine. We believe this unique collection is similar to receiving a master art collection from the
Louvre or the Metropolitan Museum of Art that should be on everyone’s shelf as an ECG museum.
Finally, we wish to thank the Cardiotext staff, namely Mike Crouchet and Carol Syverson, for their
professionalism and for providing the text and figures in a high-quality format.
The Editors

  xix
Abbreviations

ACS acute coronary syndrome


AF atrial fibrillation
AP accessory pathway
APC atrial premature contraction
APD action potential duration
ARVC/D arrhythmogenic right ventricular cardiomyopathy/dysplasia
AT atrial tachycardia
AVNRT atrioventricular nodal reentrant tachycardia
AVRT atrioventricular reentrant tachycardia

BB bundle branch
BBB bundle branch block
BS Brugada syndrome

CAD coronary artery disease


CHF congestive heart failure
CL cycle length

DDD dual-chamber pacing

EADs early afterdepolarizations


ECG electrocardiogram
EF ejection fraction

HCM hypertrophic cardiomyopathy


HPS His-Purkinje system
HR heart rate

ICD implantable cardioverter-defibrillator

LA left atrial
LAD left axis deviation
LAFB left anterior fascicular block
LBB left bundle branch
LBBB left bundle branch block
LQTS long QT syndrome

  xxi
LV left ventricle
LVEF left ventricular ejection fraction
LVH left ventricular hypertrophy
LVOT left ventricular outflow tract

MI myocardial infarction
MRI magnetic resonance imaging

OD once daily
ORT orthodromic reciprocating tachycardia

PABs premature atrial beats


PAC premature atrial contraction
PJRT permanent form of junctional reciprocating tachycardia
PV pulmonary vein
PVC premature ventricular contraction
PVI pulmonary vein isolation

RA right atrial
RBB right bundle branch
RBBB right bundle branch block
RV right ventricle
RVA right ventricular apex
RVH right ventricular hypertrophy
RVOT right ventricular outflow tract

SCD sudden cardiac death


SQTS short QT syndrome
SR sinus rhythm
SVT supraventricular tachycardia

TdP torsades de pointes

VF ventricular fibrillation
VT ventricular tachycardia

WCT wide complex tachycardia


WPW Wolff–Parkinson–White syndrome

xxii  u  Abbreviations
Video Legends

Video 13B.5.1 Short axis shows concentric LVH


Video 13B.5.2  ong axis 3-chamber view shows concentric LVH and pronounced apical
L
hypertrophy consistent with apical hypertrophic cardiomyopathy

Video 13B.6.1 Short axis of mild left ventricular hypertrophy

Video 13B.6.2 Long axis 3-chamber view of hypertrophic cardiomyopathy

Video 13E.3.1  pical, three-chamber view echocardiogram showing akinesis in the mid to distal
A
anteroseptum and apex

Video 13E.3.2 Enlarged view of Video 13E.3.1

Video 13E.3.3 Coronary angiogram

Video 13E.3.4 Right coronary

Video 13E.3.5 Left coronary

Video 15.6.1 Echocardiogram showing LVEF of 74% with mild concentric LVH

Video 15.6.2 MRI, 4-chamber view

  xxiii
SECTION 1
Introduction to the Interpretation of the Electrocardiogram

CASE
Mohammad Shenasa, MD 1.1

The first and most important step in ECG interpretation is the differentiation between “normal”
and “abnormal.”
The second step consists of differentiation between the various abnormal ECG patterns and
their correlation with known pathologic conditions. In particular, the recent discoveries with small
subtle significant markers for adverse events such as early repolarization, Brugada-type ECGs, and
other channelopathies.
Information about the ECG in disease is much more complex than knowledge of normal
variation. Yet, it is in the differentiation between normal and abnormal that difficulties in ECG
interpretation frequently arise.
Below are two examples of normal ECGs.
Heart rate: 64 bpm
PR interval: 154 ms
QRS duration: 98 ms
QT/QTc: 406/415 ms
Normal ST-T wave patterns

Figure 1.1.1

ECG Masters’ Collection: Favorite ECGs from Master Teachers Around the World © 2017 Mohammad Shenasa, Mark E. Josephson,
N. A. Mark Estes III, Ezra A. Amsterdam, Melvin Scheinman. Cardiotext Publishing, ISBN: 978-1-942909-08-8.  1
Heart rate: 80 bpm
PR interval: 148 ms
QRS duration: 92 ms
QT/QTc: 364/420 ms

Figure 1.1.2

It is important to have a systematic approach when analyzing and interpreting ECGs.


1. Baseline findings in sinus rhythm.
2. Observations during tachycardias.
3. Analysis of the changes of the cardiographic morphologies (transient changes).
4. Mode of spontaneous initiation and termination.
5. Maneuvers during tachycardias.
In a stepwise approach to ECG or rhythm analysis, one should determine the rate of the
tachycardia (fast or slow), the QRS duration (wide or narrow) and morphology, and the relationship
of the P wave to the QRS, whether it is before, during, or after and if there is a one-to-one
relationship between the P wave and the QRS.
Other important points regarding interpretation of the ECG:
1. Determine the origin and initiation of cardiac arrhythmias.
2. Look for myocardial ischemia and infarction.
3. Evidence of electrolyte imbalance and reversible causes.
4. Systemic and myocardial disorders.
5. Measure; do not eyeball the intervals.
6. Focus on zone of transition.

References
1. Wellens HJ, Gorgels AP. The electrocardiogram 102 years after Einthoven. Circulation. 2004;109(5):562–564.
2. Yong CM, Froelicher V, Wagner G. The electrocardiogram at a crossroads. Circulation. 2013;128(1):79–82.
3. Stern S. Electrocardiogram: Still the cardiologist’s best friend. Circulation. 2006;113(19):e753–e756.

2  u  Case 1.1
SECTION 2
Conduction Disturbances: Sinus Node Disease/Sick Sinus Syndrome,
AV Conduction Disturbances, AV Blocks, Bundle Branch Blocks,
and Fascicular Blocks

CASE
2.1
Amit Noheria, MBBS, SM
Samuel J. Asirvatham, MD

Patient History
A 36-year-old male with congenitally corrected transposition of great arteries (CC-TGA) presents
with fatigue.

Questions
1. What can you say about antegrade and retrograde AV conduction from ECGs in Figure 2.1.1
and 2.1.2?
2. What relevance does this have for decisions on pacemaker implantation?

Figure 2.1.1

ECG Masters’ Collection: Favorite ECGs from Master Teachers Around the World © 2017 Mohammad Shenasa, Mark E. Josephson,
N. A. Mark Estes III, Ezra A. Amsterdam, Melvin Scheinman. Cardiotext Publishing, ISBN: 978-1-942909-08-8.  3
Figure 2.1.2

Discussion, Interpretation, and Answer


Levo-looping of the embryonic bulboventricle from the developing heart tube results in
morphologic ventricular inversion—morphologic right ventricle positioned on the left side and the
morphologic left ventricle and the left bundle branches on the right. L-looping frequently occurs
with double switch—atrioventricular and ventriculoarterial—in context of normally positioned atria
(situs solitus) and transposition of great arteries, and is often referred to as CC-TGA. CC-TGA can
be associated with membranous ventricular septal defect (VSD), pulmonary stenosis, and Ebstein’s
anomaly of the morphologic tricuspid valve.
With situs solitus, the sinus node anatomy and atrial activation may be normal with a normal
P-wave axis and morphology, as is seen in this case. L-looping can, however, occur in patients with situs
inversus or situs ambiguus (heterotaxy syndromes with asplenia or polysplenia) that might impact the
location of the sinus node and P-wave axis. The ventricular septal activation in CC-TGA occurs from
the morphologic left bundle branch from right to left. As seen in the junctional escape complexes in
Figure 2.1.3, conducted QRS complexes in CC-TGA are characterized by Q waves in leads V1–V2 and
inferior leads (red arrows), and absence of “septal Q waves” in the left lateral leads (blue arrows).

4  u  Section 2: Conduction Disturbances: Sinus Node Disease


Figure 2.1.3

One-third of patients with CC-TGA develop complete AV block. The AV conduction system in
CC-TGA (and double-inlet left ventricle) is characterized by an anteriorly displaced AV node outside
the triangle of Koch at the base of the right atrial appendage.1,2 This gives rise to an anteriorly
displaced AV bundle (bundle of His) that goes around the lateral/superior aspect of the valve of the
transposed pulmonary artery that arises in juxtaposition to the mitral valve of the morphologic left
ventricle (subpulmonary). This AV bundle continues and splits into the left and right bundle
branches along the superior aspect of a membranous VSD that is often present. The normally
located AV node in the triangle of Koch typically does not connect to the ventricle on account of
malalignment of the atrial and ventricular septae. However, in certain patients this can function as
the second AV nodal connection to a separate distinct bundle of His that courses along the inferior
margin of the VSD.1,3
Figure 2.1.1 shows sinus rhythm (Figure 2.1.3, vertical black arrows) with complete AV nodal
conduction block. There is escape junctional rhythm with narrow complexes (black horizontal
arrows, ~1900 ms or 32 bpm). The junctional rhythm is intermittently interrupted by wide QRS
complexes (asterisks). The wide QRS complexes have a slurred onset and suggesting an “extra-
fascicular” myocardial origin. The positive precordial concordance (upright QRS in leads V1 and
V4 –V6) and the left frontal place axis localize the wide complexes to the posterior paraseptal region
along the left-side AV annulus. As both wide QRS complexes in this tracing have a short, fixed
preceding PR interval, it is quite likely these ventricular activations occur over a left posterior
paraseptal accessory AV pathway. Only two P waves conduct over this accessory pathway (asterisks),
and the others are blocked. The accessory pathway has weak antegrade conduction properties and is
low risk for sudden death from rapid conduction of atrial fibrillation. Another observation in this
ECG is the ventriculophasic sinus arrhythmia shown by the changing PP interval (orange double-
headed arrows, L—long, S—short).
The level of AV block is thought to most commonly occur at the level of the AV bundle. The
narrow escape complexes, albeit with a left axis (rS in lead II) would suggest either AV block at the

Case 2.1  u  5
level of the anterior AV node with junctional escape complexes with a left axis on account of variant
ventricular anatomy, remodeling, or intraventricular conduction abnormalities, or alternatively
block at the anterior AV bundle but escape complexes from a preserved posterior AV conduction
system (expected to have left superior axis due to the relatively posterior location).
All the junctional escape complexes in Figure 2.1.1 time coincidentally with or immediately
after sinus P waves. This precludes any assessment of retrograde VA conduction, as the atrial tissue
is refractory and unexcitable after sinus activation. The first QRS complex in Figure 2.1.2, however,
allows assessment of retrograde conduction. This QRS complex is followed by a retrograde P wave
with a short RP interval (Figure 2.1.4, arrow). This P wave is negative in inferior leads (II > III),
isoelectric in lead I, positive in lead V1, and would localize to the left posteroseptal region,
consistent with the location of the atrioventricular accessory pathway. The subsequent QRS
complex is followed by P-wave fusion between sinus rhythm and retrograde atrial activation
(asterisk). Following pacemaker implantation, the brisk retrograde conduction can lead to
pacemaker syndrome or pacemaker-mediated tachycardia.

Figure 2.1.4

This patient had CC-TGA without evidence of VSD or pulmonary stenosis. The systemic
ventricle (morphologic right ventricle) was hypertrophied and had reduction in systolic function
(ejection fraction 30%). He received an atrial-biventricular cardiac resynchronization therapy
pacemaker (he declined defibrillator implantation). On electrophysiology study, he had a left-sided
posteroseptal accessory pathway with good retrograde conduction properties (effective refractory
period 270 ms) that was successfully ablated.

References
1. Anderson RH, Becker AE, Arnold R, et al. The conducting tissues in congenitally corrected transposition.
Circulation. 1974;50(5):911–923.
2. Anderson RH, Arnold R, Thapar MK, et al. Cardiac specialized tissue in hearts with an apparently single ventricular
chamber (double inlet left ventricle). Am. J. Cardiol. 1974;33(1):95–106.
3. Hosseinpour AR, McCarthy KP, Griselli M, et al. Congenitally corrected transposition: Size of the pulmonary trunk
and septal malalignment. Ann. Thorac. Surg. 2004;77(6):2163–2166.

6  u  Section 2: Conduction Disturbances: Sinus Node Disease


Antoni Bayés de Luna, MD CASE
2.2
Luis Alberto Escobar Robledo, MS
Albert Massó van Roessel, MD

Patient History
A 65-year-old patient with chronic obstructive pulmonary disease (COPD) that present in a long
strip recording of lead II evident brusque changes of P wave (Figure 2.2.1).

Figure 2.2.1  Continuous lead II. Observe the abrupt and repeated change in P-wave morphology and polarity not related to the
respiration in a patient with chronic obstructive pulmonary disease (COPD). Two types of P-wave morphology are observed: peaked P wave
and flat P wave.

Question
How might these changes be explained?
These ECG changes of P wave may be due to:
1. Changes with respiration
2. Fusion beats between sinus and ectopic rhythm
3. Artifacts
4. Sinus rhythm with atrial aberrancy

Discussion, Interpretation, and Answer


The correct answer is 4.
These changes are due to atrial aberrancy. This concept was coined more than 40 years ago by
Chung in 19721 as “the bizarre configuration of the P wave of a sinus beat immediately after an
atrial, AV, or ventricular premature complexes, and is equivalent in the atria to the concept of
ventricular aberrancy”.

Case 2.2  u  7
The concept of atrial aberrancy also encompasses brusque changes of P-wave morphology that
may appear in either the same or separate ECG strip, and may or may not be related to the changes
in heart rate.2,3 These changes in morphology (aberrancy) are due to transient changes in the way of
transmission of the sinus impulses in the atria. This can sometimes be due to transient interatrial
block; however, in this case where there is no ECG pattern of interatrial block,4–7 it may be due to
changes in the transmission of stimulus probably through right atrium. These changes may appear
in the same recording, (Figure 2.2.1), in different days (Figure 2.2.2), or immediately after an atrial
or ventricular premature beat (Figures 2.2.3–2.2.5). In the setting of Chung,1 aberrant atrial
conduction is an infrequent ECG finding that usually occurs in the elderly with organic heart
disease, especially ischemic heart disease. It can also be in chronic cor pulmonale.2 The clinical
significance is uncertain, but usually occurs in patients with heart disease and atrial involvement.
The current case has the following characteristics:
The P-wave changes are not an artifact, are not related with respiration, and are not fusion beats.
1. The recording starts with 3 flat P wave followed by 13-peaked P wave, 17-flat P wave, and finally
3 peaked waves. This is not a respiratory cycle.
2. The changes from one morphology to another are sudden or with only one complex (the third of
second strip) that may correspond to a minor degree of aberrancy. Therefore, there are no fusion
beats.
The type of aberrancy of Figure 2.2.2 may explain that patients with important subacute cor
pulmonale and pathological P wave of right atrium enlargement (Figure 2.2.2A) may present
transient or permanent disappearance of the ECG criteria of the right atrial enlargement
(Figure 2.2.2A, 2.2.2B, and 2.2.2C).

Figure 2.2.2  A. A 45-year-old patient with subacute cor pulmonale. Note the right axis of P wave (ÂP) around + 80º (right) few days
later (B), the ÂP was left, returning to the right in a third ECG (C) recorded at 15 days. This example shows that criteria of right atrial
enlargement may be concealed due to atrial aberrancy.

8  u  Section 2: Conduction Disturbances: Sinus Node Disease


Finally, in Figures 2.2.3 to 2.2.5, we can see different changes of the P wave that appear after an
premature atrial complex (PAC) or an premature ventricular complex (PVC). These bizarre P waves
are not an artifact or an escape beat. They most likely correspond to atrial aberrancy due to them
being recorded at different times with the same morphology. Figure 2.2.3 shows that after a PAC
there is a change in the refractory period of the atria, and the next impulse (x) is partially blocked in
the same part of the right atrium and presents a very different pattern (more peaked) than the
other ones.

Figure 2.2.3  A P wave that appeared after a PAC that has different morphology from previous and successive ones. It is not an artifact
or escape complex because it was recorded many times with the same PR, and is most likely explained by atrial aberrancy.

Figure 2.2.4 also shows a transient change of P wave after a PAC. The P wave changes from a
pattern of advanced interatrial block (aIAB) to another of partial interatrial block (pIAB) (from ± to
bimodal (x)) that occur is due to the refractory period of the upper part of atria being shortened due
to PAC. The next P wave may be conducted with lower degree of interatrial block (IAB) (transient or
second-degree A-IAB).

Figure 2.2.4  In the presence of an A-IAB pattern (first P ± in VF) after a PAC there is a pause followed by transient P-IAB pattern that is
followed again by a pattern of A-IAB.

In other occasions (Figure 2.2.5) the presence of A-IAB (P ± in II) after one PVC appears as a
pause followed by a peaked P wave that has a normal PR interval and is not an artifact. This
presumably corresponds to an atrial aberrancy (right atrium).

Figure 2.2.5  In a case of A-IAB after two sinus beats with ± morphology of P wave, a PVC appears which is followed by a P wave with
normal P-R conduction and different morphology, only positive (peaked P). This is due to atrial aberrancy.

Case 2.2  u  9
Conclusion
• Carefully watching the P wave is a useful way to perform the correct diagnosis. Cases with atrial
aberrancy that appears transiently explain that the diagnosis of RAE may be temporally masked6
(Figure 2.2.2).
• Cases of atrial aberrancy usually appear in the elderly with organic heart disease and atrial
involvement.

References
1. Chung E. Aberrant atrial conduction. Unrecognized electrocardiographic entity. Br. Heart J. 1972;34:341–346.
2. Júlia J, Bayés de Luna A, Candell J, et al. Aberrancia auricular: A proposito de 21 casos. Rev. Esp. Cardiol.
1978;31(2):207.
3. Bayés de Luna A. Bloqueo a nivel auricular. Rev. Esp. Cardiol. 1979;32(1):5–10.
4. Bayés de Luna A, Fort de Ribot R, Trilla E, et al. Electrocardiographic and vectorcardiographic study of interatrial
conduction disturbances with left atrial retrograde activation. J. Electrocardiol. 1985;18(1):1–13.
5. Bayés de Luna A, Platonov P, García-Cosio F, et al. Interatrial blocks. A separate entity from left atrial enlargement:
A consensus report. J. Electrocardiol. 2012;45:445–451.
6. Bayés de Luna A. Clinical electrocardiography. Sussex, U.K.: Wiley-Blackwell. 2012;103.
7. Bayés de Luna A, Massó-van Roessel A, Escobar Robledo LA. The diagnosis and clinical implications of interatrial
block. Eur. Cardiol. Rev. 2015;10(1):54–59.

10  u  Section 2: Conduction Disturbances: Sinus Node Disease


CASE
Bernard Belhassen, MD 2.3
Patient History
The following electrocardiogram (ECG) tracings were recorded during electrophysiology studies
performed without sedation in a 71-year-old male with obstructive hypertrophic cardiomyopathy
and recurrent syncope.

Figure 2.3.1

Discussion
Baseline ECG shows sinus rhythm with normal PR and QRS duration. The His potential has normal
configuration; AH is prolonged at 150 ms and HV normal (50 ms). Incremental atrial pacing showed
supra-Hissian block with Wenckebach sequence at 100/minute.
Catheter-induced atrial fibrillation (AF) then occurred. Heart rate initially ranged from 50 to
85/minute then atrioventricular (AV) block lasting 3 seconds occurred (Figure 2.3.1). This event was

Case 2.3  u  11
reproducibly documented and not related to any significant change in the patient’s respiratory
status. After AF spontaneously converted after #5 minute, short bursts of rapid atrial pacing
reproducibly induced transient AV block on very late sinus beats (Figure 2.3.2).
Despite the fact the His potential could not be recorded during AV block, the assumption was a
phase 4 dependent block located at the proximal His area. A similar case was previously reported.1

Figure 2.3.2

Reference
1. Belhassen B, Danon L, Shoshani D, et al. Paroxysmal atrioventricular block triggered by orthostatic hypotension.
Am. Heart J. 1986;112:1107–1109.

12  u  Section 2: Conduction Disturbances: Sinus Node Disease


CASE
2.4
Andrei G. Dan, MD, PhD
Catalin A. Buzea, MD, PhD

Patient History
An 86-year-old female with history of hypertension and hypercholesterolemia is admitted for
orthostatic syncope preceded by dizziness. The patient has left hemiparesis after an ischemic
stroke. She is under treatment with perindopril, indapamide, rosuvastatin, and clopidogrel.

Questions
1. What is the underlying rhythm?
2. Could help this tracing in explaining the etiology of syncope?

Figure 2.4.1  Sinus rhythm with second-degree atrioventricular block with Luciani periods (standard leads).

Figure 2.4.2  Sinus rhythm with second-degree AV atrioventricular with Luciani periods (precordial leads). The second QRS complex has
different morphology suggesting aberrancy.

Case 2.4  u  13
Discussion, Interpretation, and Answers
The two tracings are simultaneous depicting standard and precordial leads. The atrial rhythm is
sinus (positive P wave in II, III, and aVF). The PP is regular (red marks) and the rhythm is sinus
tachycardia at 107 bpm. There is a group beating. The first QRS of the group (A) has right bundle
branch block morphology with the leftward axis and clockwise rotation in the horizontal plan
suggesting anterosuperior haemiblock. The second QRS complex (B) is slightly modified suggesting
increased right intraventricular delay. The reason for this could be phase 3 aberrancy because it
follows a long-short cycle (similar to Ashman phenomenon in atrial fibrillation). The PR interval
(green marks) of the first QRS is 160 ms, the second is 200 ms and the third P is not followed by a
QRS complex. This is highly suggestive for a Mobitz type I, second-degree atrioventricular block.
Therefore, a bifascicular block is accompanied by a second-degree atrioventricular block; the last
one could be localized at the atrioventricular node level (indicating a multilevel block) or less likely
at the level of posteroinferior fascicle of the left bundle branch (indicating an incomplete
trifascicular block). For this patient, the ischemic etiology is more probable than a degenerative one
because of his risk factors and history. There is a risk for progression to more advanced block and
there is a high index of suspicion that the patient suffered an arrhythmic syncope. A pacemaker
implantation is advisable.

14  u  Section 2: Conduction Disturbances: Sinus Node Disease


CASE
2.5
Andrei G. Dan, MD, PhD
Catalin A. Buzea, MD, PhD

Patient History
An 81-year-old patient is admitted in the neurology department for altered peripheral sensitivity. She
is under treatment with valproate for epilepsy and described several episodes of lightheadedness.
Two consecutive electrocardiogram (ECG) tracings were obtained during her hospitalization.

Questions
1. What could explain the progression to the second tracing?
2. What is the nature of the premature beats (red star)?

Figure 2.5.1  Normal sinus rhythm with right bundle branch block and left anterior fascicular block.

Case 2.5  u  15
Figure 2.5.2  Complete atrioventricular block in same patient with supernormality phenomenon.

Discussion, Interpretation, and Answers


There is normal sinus rhythm in the first tracing. QRS morphology indicates right bundle branch
block morphology with secondary type repolarization abnormalities. The QRS axis is leftward
(−60º) indicating left anterosuperior hemiblock. The PR interval is 210 ms (longer than normal for
the rate). The above mentioned findings indicate bifascicular block and first-degree atrioventricular
block. The conduction abnormality could be at two levels or indicating incomplete trifascicular
block.
In the second tracing, the atrial rhythm is sinus (93 bpm) and the ventricular rhythm is slow
(36 bpm). There is no relation between the atrial and ventricular rhythm (variable PR interval,
atrioventricular dissociation). This is indicative for complete atrioventricular block. As the QRS
morphology is similar with that observed in the first tracing, the subsidiary pacemaker is situated at
the left inferoposterior left branch level. Some atrial impulses (red star) are slightly delayed and are
conducted to the ventricle giving the false impression of extrasystoles. These beats could represent
“supernormal” conduction as this conduction is unexpected. Several explanations exist for
supernormal conduction including gap phenomenon, supernormal excitability and concealed
conduction.1 The patient has indication for pacemaker implantation.

Reference
1. Bayés de Luna A. Clinical Electrocardiography: A Textbook. 4th ed. Sussex, U.K.: Wiley-Blackwell. 2012.

16  u  Section 2: Conduction Disturbances: Sinus Node Disease


CASE
2.6
Andrew E. Epstein, MD
Michael P. Riley, MD, PhD

Patient History
An 86-year-old asymptomatic male referred for pacemaker implantation for progressive
atrioventricular (AV) block.

Figure 2.6.1

Discussion
There is marked first-degree AV block. Sinus rhythm is confirmed by a normal P-wave axis and
premature ventricular complexes that do not reset the sinus rate but do have concealed retrograde
conduction leading to AV block of the subsequent sinus beat. A pacemaker was not implanted.

Case 2.6  u  17
CASE
2.7 N. A. Mark Estes III, MD

Patient History
ECG of an 82-year-old asymptomatic male with pulmonary sarcoidosis for 50 years.

Questions
1. What is the ECG abnormality?
2. What are the common ECG manifestations of cardiac sarcoidosis?

Figure 2.7.1

Discussion
This ECG demonstrates left bundle branch block with a QRS duration of 176 ms and first-degree
atrioventricular block (PR 248 ms). Infra-Hisian conduction system disease is the most common
ECG manifestation of cardiac sarcoidosis, which has a tendency to involve the basal portion of the
intraventricular septum and manifest with conduction system involvement.

18  u  Section 2: Conduction Disturbances: Sinus Node Disease


CASE
N. A. Mark Estes III, MD 2.8
Patient History
ECG of a 50-year-old male with no prior cardiac history resuscitated from cardiac arrest due to
sustained ventricular tachycardia.
Echocardiogram demonstrated a left ventricular ejection fraction of 25% with anteroseptal and
inferior left ventricular hypokinesis. Coronary angiography demonstrated no obstructive disease.

Questions
1. What is the ECG abnormality?
2. What cardiovascular condition is likely to cause these ECG abnormality and cardiac arrest?

Figure 2.8.1

Discussion
The ECG demonstrates right bundle branch block (RBBB) (QRS 144 ms) and first-degree
atrioventricular (AV) block (PR 264 ms). Multiple premature ventricular contractions with a left
bundle branch block morphology are present. The ECG shows a RBBB with a left axis/left anterior
hemiblock with a first-degree AV block. With the extensive conduction system disease, impaired left
ventricular function, multiple regional wall motion abnormalities, and sustained ventricular
tachycardia, cardiac sarcoidosis would be at the top of the differential diagnosis. This was confirmed
by cardiac magnetic resonance imaging and positron emission tomography scan. Pulmonary
sarcoidosis was diagnosed by pulmonary function tests and a chest computed tomography.

Case 2.8  u  19
CASE
2.9
Mohammad-Ali Jazayeri, MD
Mohammad-Reza Jazayeri, MD

Patient History
Two cases form the basis for this scenario. The first case is from a 69-year-old female who presented
with bradycardia and two month history of exertional dyspnea. She denied any syncope or
presyncope. Figure 2.9.1 top is her 12-lead ECG at the time of presentation showing 2:1 AV block,
persistent PP alternans, PR interval of 200 ms, and complete right bundle branch block (RBBB). An
echocardiogram showed her left ventricle with mild asymmetric hypertrophy, normal systolic
function, and mild to moderate diastolic dysfunction. The second case is an 81-year-old gentleman
diagnosed with severe aortic stenosis who underwent a transcatheter aortic valve insertion (TAVI)
with a 29-mm Medtronic Corevalve system (MCS) without any complication except for a transient
Mobitz type I, second-degree AV block at the conclusion of the procedure. Figure 2.9.1 bottom is a
three-day postoperative telemetry tracing.

Figure 2.9.1  Second-degree AV block with 2:1 and 3:2 conduction. The top panel depicts sinus rhythm (SR) with 2:1 AV conduction and
right bundle branch block (RBBB). Of note, there is PP alternans with the intervals being shorter (760 ms) when a QRS complex is present
in between. The bottom panel is a rhythm strip (ECG leads II and V1) showing grouped beating of Wenckebach periodicity during SR
(arrows) at a cycle length of 800 ms. Lead II shows persistent left axis deviation. Note that during each group, the first P is conducted with
a PR interval of 360 ms and RBBB (#1, 3, 5). The second P wave is conducted with a longer PR interval of 560 ms and left BBB. The third P
is completely blocked.

20  u  Section 2: Conduction Disturbances: Sinus Node Disease


Question
What is the most appropriate action to be taken for these two patients?
1. Permanent pacemaker implantation (PPI) for both patients
2. Electrophysiologic study (EPS) in case 1 and PPI for case 2
3. EPS for both patients
4. None of the above

Discussion
Case 1 underwent EPS to determine the site of her AV block before committing her to PPI. Her
baseline rhythm was sinus with a cycle length (CL) of 905 ms and 1:1 AV conduction, AH interval of
97 ms, and HV interval of 70 ms. During rapid atrial pacing, 1:1 AV conduction was present at a CL
of 820 ms. with block commencing in the AV node at CL of 810 ms. The patient was then started on
procainamide infusion at 100 mg/min. After infusion of 350 mg of procainamide, her HV interval
progressively prolonged to 105 ms and she started to have 2:1 AV block below the HB recording site,
which progressed to third-degree AV block (Figure 2.9.2). Case 2 underwent PPI because there was a
progressive worsening of the His-Purkinje system (HPS) conduction over 7 days postoperatively and
he clearly demonstrated the ECG signs of “bilateral bundle branch block (BBBB).” Figure 2.9.3 shows
preoperative progression of the HPS conduction abnormality over a period of one month, which
resulted in complete RBBB with a left axis deviation.

Figure 2.9.2  Spontaneous and drug-induced advanced AV block. Panel A shows sinus rhythm with a cycle length of 760 ms and 1:1
AV conduction on the left (first five beats), which spontaneously converts to 2:1 AV block. Of note, the PR remains unchanged after the
conversion, which favors the His-Purkinje system (HPS) as being the site of block. Panel B is obtained during an electrophysiological study,
when the patient after being given intravenous procainamide developed third-degree AV block. This observation further confirms the HPS
involvement as the site of block in this patient. It should be mentioned that the paper speed is different in panels A and B.

Case 2.9  u  21
Figure 2.9.3  Preoperative and postoperative ECGs. The ECGs are arranged in a chronological order. Panels A to C are preoperative
ECGs showing gradual progression of the His-Purkinje system conduction disturbances from left axis deviation (LAD) in panel A to right
ventricular conduction delay in addition to LAD shown in panel B to right bundle branch block and left anterior fascicular block in panel
C, which occurred over a one-month period. Panel D is a 7-day postoperative ECG showing left bundle branch block with marked PR
prolongation of 402 ms.

Discussion
The critical issue presented in these cases is whether or not these two patients had advanced
conduction abnormalities in the HPS and, therefore, were susceptible to third-degree AV block.
A 2:1 AV block is a form of second-degree AV block and may be localized to the AV node or the
HPS. A normal PR interval of the conducted beats favors the HPS as being the site of block.
However, a prolonged PR interval would not be helpful in localizing the site of block. Coexistence of
2:1 AV block with BBB does not necessarily translate into the site of block being in the HPS as in
15%–20% of these patients the site of 2:1 block is the AV node.1 Exercise stress testing may
differentiate the site of block by worsening the degree of AV block when the HPS is the site or by
improving the AV conduction when the AV node is the culprit. Pharmacological stress testing of
the HPS with intravenous administration of a class IA antiarrhythmic agent during EPS is another
technique to identify a group of patients who maybe at risk of complete AV block and in need of
PPI.2,3 Of note, the PP alternans seen in Figure 2.9.1A is a phenomenon known as “ventriculophasic
sinus arrhythmia” occurring in patients with heart block, in which the PP intervals with a QRS
complex sandwiched in between is shorter than those with no QRS complex in between.4,5
BBBB is an old terminology, in which the occurrence of third-degree AV block maybe eminent.6
A large number (up to 80%) of complete AV block have been attributed to BBBB.7 Different degrees
of conduction delay in bundle branches may electrocardiographically manifest as “bifascicular
block” or “alternating bundle branch block.” The latter situation is an intriguing phenomenon, in

22  u  Section 2: Conduction Disturbances: Sinus Node Disease


which properly timed 3:1 conduction in one BB with a shorter propagation time, and 3:2 conduction
in the contralateral BB with a longer propagation time would manifest as a grouped beating pattern
which mimics Wenckebach periodicity but in the HPS (Figure 2.9.1B). In the setting of an acute
insult to the conduction system such as acute myocardial infarction or iatrogenic injury during
surgical procedures and valve replacement/implantation, the conduction abnormalities may be
completely or partially reversible. On the contrary, however, during chronic and degenerative
disease processes, the conduction disturbances are usually permanent and progressive.
Over the past decade, TAVI has emerged as a valuable and effective strategy to replace the aortic
valve in elderly patients with severe aortic valve stenosis who are high-risk surgical candidates. This
technique, however, has been associated with high incidence of postoperative AV conduction
disturbances. Obviously, understanding the anatomy of the conduction system and its close
proximity to the aortic valve apparatus is essential for prevention of AV block or prediction of a
need for PPI. The bundle of His (BH) and its left bundle branch are located in the central fibrous
body, adjacent to the non-coronary cusp of the aortic valve. The AV node/BH junction and the
uppermost portion of the RBBB are also in the vicinity of this region. Currently, there are two
prototypical transcatheter aortic valves available in the market: the self-expanding MCS and
balloon-expandable Edwards SAPIEN valve (ESV). The reported incidence of new left BBB, third-
degree AV block, and PPI has been 29%–65%, 15%–44%, and 9%–49%, respectively for MCS and
6%–18%, 0%–27%, and 0%–27%, respectively for ESV.8 The higher incidence of conduction
disturbances has been attributed to a deeper insertion and perhaps further radial forces of
expansion exerted by the MCS stent cage to the neighboring conduction system, particularly the
LBB.9 Consequently, in the presence of preexisting RBBB, newly developed LBBB may ultimately
lead to third-degree AV block.

References
1. Barold SS, Hayes DL. Second-degree atrioventricular block: A reappraisal. Mayo Clin Proc. 2001;76(1):44–57.
2. Twidale N, Heddle WF, Tonkin AM. Procainamide administration during electrophysiology study-utility as a
provocative test for intermittent atrioventricular block. Pacing Clin. Electrophysiol. 1988;11:1388–397.
3. Englund A, Bergfeldt L, Rosenqvist M. Pharmacological stress testing of the His-Purkinje system in patients with
bifascicular block. Pacing Clin. Electrophysiol. 1998;21:1979–1987.
4. Rosenbaum MB, Lepeschkin E. The effect of ventricular systole on auricular rhythm in auriculoventricular block.
Circulation. 1955;11:240–261.
5. Schamroth L. Ventriculophasic atrial extrasystoles associated with complete artrioventricular block. Am. J. Cardiol.
1968;21:593–596.
6. Rosenbaum MB, Lepeschkin E. Bilateral bundle branch block. Am. Heart J. 1955;50:38–61.
7. Lopez JF. Electrocardiographic findings in patients with complete atrioventricular block. Br Heart J. 1968;30:20–28.
8. Nuis RJ, Van Mieghem NM, Schultz CJ, et al. Timing and potential mechanisms of new conduction abnormalities
during the implantation of the Medtronic CoreValve System in patients with aortic stenosis. Eur. Heart J.
2011;32:2067–2074.
9. Fraccaro C, Buja G, Tarantini G, et al. Incidence, predictors, and outcome of conduction disorders after
transcatheter self expandable aortic valve implantation. Am. J. Cardiol. 2011;107:747–754.

Case 2.9  u  23
CASE
2.10
Nishant Verma, MD, MPH
Bradley P. Knight, MD

Patient History
A two-lead rhythm strip was recorded in an elderly male admitted to the hospital for acute
abdominal pain (Figure 2.10.1). You are consulted for bradycardia. What is the mechanism of
the transient bradycardia and resolution of the intraventricular conduction delay?

Figure 2.10.1  Lead rhythm strip showing transient bradycardia and QRS narrowing.

Discussion
The rhythm strip shows sinus rhythm at a rate of approximately 90 bpm and a bundle branch block
(BBB), followed by seven beats of bradycardia at a rate that is about half the initial rate and is
associated with transient resolution of the BBB. At first glance, it appears that the patient has sinus
node disease. However, there are a series of premature atrial contractions (PACs) denoted by the
arrows. The first one conducts with an IVCD; thereafter, they occur in a bigeminal pattern with AV
block. Note that the fifth QRS complex is preceded by a PAC that deforms the T-wave. The sixth
QRS complex is followed by a PAC that blocks in the AV node and sets up a bigeminal pattern of
blocked PACs resulting in ventricular bradycardia. The bradycardia is associated with transient
resolution of the BBB. Clues that the bradycardia is caused by atrial bigeminy with blocked PACs
rather than sudden sinus node dysfunction are the subtle presence of the blocked PACs as well as
the PAC that conducts before the fifth QRS complex.

24  u  Section 2: Conduction Disturbances: Sinus Node Disease


CASE
2.11
Gilles Lascault, MD
Olivier Piot, MD

Patient History
A 48-year-old female patient was hospitalized in a state of emergency for recurrent presyncopal
episodes during the last 24 hours. The history of the patient revealed nothing except she had
complained for several weeks of scapular and spinal unexplained pain relieved by anti-
inflammatory drugs. She had no fever, no tick bites, no cutaneous eruption (such as erythema
migrans rash). She did not walk in the woods and forests. At admission clinical examination,
echocardiogram and standard blood tests were normal (including c-reactive protein). Cardiac
magnetic resonance imaging and positron emission tomography scan showed no cardiac
abnormalities.
The ECG (Figure 2.11.1) at admission showed high degree block. The basal PR interval is
increased and the blocked P wave is preceded by a progressive increase in PR interval (type I block).
There is a complete right bundle branch block (QRS duration around 140 ms) with a normal QRS
axis. An electrophysiology study was done and confirmed an atrioventricular (AV) nodal block and
a normal HV interval (40 ms). During hospitalization heart rhythm was continuously monitored.
After 4 days, atrio-ventricular and intra-ventricular AV conduction disturbances had progressively
vanished along with the symptoms (Figure 2.11.2). No pacemaker implantation was indicated. Blood
tests were eventually positive for recent infection by Borrelia burgdorferi (IgM positive), confirming
the diagnosis of Lyme disease. An appropriate antibiotic treatment was then started.

Figure 2.11.1  AVB Lyme 1.

Case 2.11  u  25
Figure 2.11.2  AVB Lyme 2.

Discussion
Acute AV block can occur during adulthood and has various causes. Among the different causes,
Lyme disease must be systematically suspected and blood tests performed. This diagnosis is
important since the conduction disturbances are reversible after days or weeks. It may be difficult,
as in the present patient, to suspect Lyme disease.

Question
Before atrioventricular conduction disturbances, when is it appropriate to suspect Lyme disease?

Answer
In the presence of atrioventricular block (first-, second-, or third-degree), it is important to carefully
analyze the history of the patient. It is easy to suspect Lyme disease when the patient is at risk of
exposure to infected ticks, or has had a tick bite followed by erythema migrans rash and secondarily
by cardiac, musculo-skeletal or neurological manifestations. This sequence may not be present.
Lyme disease has to be suspected when there is no evidence of (many) other causes of AV block.
Lyme disease must be considered before the sudden appearance of AV conduction disturbances of
unknown origin in a young and previously healthy patient. The final diagnosis relies on a positive
Lyme antibody test.

Reference
1. Krause PJ, Bockenstedt LK. Lyme disease and the Heart. Circulation. 2013;127:e451–e454.

26  u  Section 2: Conduction Disturbances: Sinus Node Disease


CASE
Fred Morady, MD 2.12
Patient History
This ECG (Figure 2.12.1) was recorded during preoperative evaluation of a 53-year-old female
scheduled to undergo shoulder surgery. She had a septal myocardial infarction 2 years earlier.

Figure 2.12.1

Question
What is the explanation for variability in the PR intervals and QRS complexes?

Explanation
There are two different PR intervals (200 and 360 ms) that alternate. The QRS complexes also
alternate, with a right bundle branch block configuration always associated with the longer of the
two PR intervals. The most plausible explanation for the varying PR intervals is that there are dual
atrioventricular nodal pathways, with atrioventricular conduction occurring on an alternating basis
through the fast and slow pathways. Because of the longer AH interval when there is conduction
through the slow pathway, the HH interval preceding the QRS complex that occurs at the end of the
longer PR interval is also longer. This results in phase 4 block in the right bundle. Phase 4 block is a
pathologic, not a physiologic phenomenon. It probably occurred in this patient because of the prior
septal infarction, with some degree of involvement of the right bundle.

Case 2.12  u  27
CASE
2.13 Fred Morady, MD

Patient History
This electrocardiogram (Figure 2.13.1) was recorded in a 76-year-old male who was hospitalized for
treatment of prostate cancer.

Question
What is the mechanism of the atrioventricular (AV) block?

Figure 2.13.1

Explanation
There is sinus rhythm at 75 bpm, initially with 1:1 AV conduction and a left bundle branch block.
A premature atrial complex (PAC) that is not conducted to the ventricles occurs and is followed by
2:1 AV block. The 2:1 AV block is caused by functional infranodal block. The PAC was blocked in
the AV node, which created a “long–short” sequence in the bundle of His (see ladder diagram,
Figure 2.13.2). The long diastolic interval resulted in prolongation of refractoriness in the His
bundle, which lead to infranodal AV block with the next sinus beat. The ongoing 2:1 AV block is
caused by perpetuation of the “long–short” sequence in the His bundle, as depicted in Figure 2.13.2.
Note that the QRS complexes become narrow during the 2:1 AV block. This indicates that the left

28  u  Section 2: Conduction Disturbances: Sinus Node Disease


bundle branch block was caused by phase 3 block in the left bundle that resolved with the
bradycardia.
This type of block can occur when the refractory period of the His bundle is mildly longer than
that of the AV node and is not necessarily an indication for pacemaker implantation in an individual
who is asymptomatic.

Figure 2.13.2

Case 2.13  u  29
CASE
2.14 Fred Morady, MD

Question
What is the most likely mechanism of the atrioventricular (AV) block on this 6-lead
electrocardiogram (Figure 2.14.1)?

Figure 2.14.1

Explanation
The first two complexes the ECG are sinus rhythm with a normal PR interval and with a narrow
QRS complex. There is an atrial premature depolarization that is followed by complete AV block
during the remaining portion of the recording. This is a typical example of paroxysmal AV block.
Paroxysmal AV block is third-degree AV block that occurs suddenly and unexpectedly following
a period of normal AV conduction. The AV block is often initiated by a premature atrial or
ventricular depolarization and usually resumes only after an escape beat.
In this ECG, there are two sinus beats followed by a conducted premature atrial depolarization.
The premature atrial depolarization results in lengthening of the subsequent PP interval by
approximately 200 ms. In patients susceptible to paroxysmal AV block, lengthening of the PP
interval allows hypopolarization due to spontaneous diastolic depolarization in the His-Purkinje

30  u  Section 2: Conduction Disturbances: Sinus Node Disease


system, causing phase 4 block. Although not shown on this ECG, the patient eventually had an
escape beat which resulted in resumption of 1:1 AV conduction. The escape beat depolarizes the
His-Purkinje system and “resets” normal AV conduction.
It is noteworthy that the sinus cycle length during the AV block is not longer than when there
was AV conduction. This rules out vagally-mediated AV block.

Reference
1. Rosenbaum MB, Elizari MV, Levi RJ, et al. Paroxysmal atrioventricular block and spontaneous diastolic
depolarization. Chest. 1973;63:678–688.

Case 2.14  u  31
CASE
2.15 Yuji Nakazato, MD, PhD

Patient History
This 34-year-old female arrived to the hospital with palpitations. She had a history of atrial septal
defect closure at 12 years old and myocarditis at 30 years old. A Holter ECG was recorded for the
evaluation of arrhythmias.

Question
How can you explain the findings (arrow and star) on Figure 2.15.2?

Figure 2.15.1

32  u  Section 2: Conduction Disturbances: Sinus Node Disease


Figure 2.15.2

Case 2.15  u  33
Figure 2.15.3

34  u  Section 2: Conduction Disturbances: Sinus Node Disease


Figure 2.15.4

Discussion
In Figure 2.15.1, Mobitz type II atrioventricular (AV) block is observed on the Holter ECG. The
rhythm strip of leads V1 and V2 are demonstrated in Figure 2.15.2. As the arrow indicated in the
upper panel, premature atrial contraction (PAC) without ventricular conduction followed by
junctional escape is clearly observed. In the lower panel, the arrow indicates His extrasystole just
preceding QRS (same as sinus rhythm which) conducted to the ventricle faster than atrium. For
reference, standard 12 leads ECG are presented in Figure 2.15.3. According to this behavior,
extrasystole of His bundle was strongly suspected. Confirmation of the diagnosis was made by a
recording of His bundle electrogram (Figure 2.15.4). The electrogram of His extrasystole
(He: arrow) and following retrograde conduction to the atrium are clearly demonstrated.
Nonconducted PAC with a long coupling interval is often mistaken as true AV block. Therefore,
this phenomenon is called pseudo AV block. If AV block-like findings were shown on a surface ECG,
extrasystole of the His bundle should not be forgotten.

Reference
1. Damato AN, Lau SH, Bobb G. Cardiac arrhythmia simulated by concealed bundle of His extrasystole in the dog. Cir
Res. 1971;28:316.

Case 2.15  u  35
CASE
2.16 Philip Podrid, MD

Patient History
A 58-year-old male presents with a history of substernal chest pressure radiating to his arms
and jaw. It is associated with shortness of breath. Upon presentation to the emergency
department an electrocardiogram is obtained.

Questions
1. What is the underlying abnormality of the QRS complexes?
2. What accounts for the long RR intervals?

Figure 2.16.1

Diagnosis
Acute inferior wall ST segment elevation myocardial infarction (STEMI), second-degree AV block
(Mobitz type II), sinus node exit block.

36  u  Section 2: Conduction Disturbances: Sinus Node Disease


Discussion
The rhythm is irregular as a result of short RR intervals which are the same and long RR intervals
(↔,┌┐) which are the same. The rhythm is regularly irregular. The short RR intervals have a rate of
90 bpm. The QRS complex duration is normal (0.08 seconds) and there is a normal morphology and
axis between 0° and +90° (positive QRS complex in leads I and aVF). There is a tall R wave in lead V2
(←) which is termed early transition or counterclockwise rotation. This is due to a shift of the
electrical axis in the horizontal plan. This is established by imagining the heart as if viewed from
under the diaphragm. With counterclockwise rotation the left ventricular forces are shifted
anteriorly and become prominent in the anterior precordial leads, especially V2. The QT/QTc
intervals are normal (360/440 ms). There is J point and ST segment elevation seen in leads II, III,
and aVF (↓) consistent with an acute inferior wall STEMI. There are Q waves in these leads,
indicating that there is already infarcted tissue. There is ST segment depression in leads I and aVL
(↑) which represent reciprocal changes associated with the STEMI. There is also ST segment
depression in leads V2–V3 (^) which suggests posterior wall involvement with the inferior wall
infarction, although these might also be reciprocal changes. There is a P wave before each QRS
complex (+) with a stable PR interval (0.28 seconds). The P waves are positive in leads I, II, aVF, and
V4 –V6. Following QRS complexes 6–8 there are P waves (*) that are not conducted, accounting for
the long RR intervals (┌┐). The PP intervals are constant (└┘) with a rate of 90 bpm. There is an
underlying normal sinus rhythm. The presence of an occasional non conducted P wave defines a
second-degree AV block. The second-degree AV block presents with a pattern of 2:1 AV conduction
or block. This may be either a Mobitz type I or II. Since all the PR intervals are the same when there
are several sequential P waves that are conducted, this represents Mobitz type II. There first two
long RR intervals (↔) are not the result of a non conducted P wave as there is no P wave seen during
these long intervals. Therefore this is not second-degree AV block, Mobitz type II but rather is a
sinus node pause. As the PP interval around the pause is equal to two sinus intervals (└┘) this is a
sinus node exit block.

Case 2.16  u  37
CASE
2.17 Vassil Traykov, MD

Patient History
A 75-year-old hypertensive male with no significant structural heart disease reported recurrent
syncope without prodromal symptoms.
During one of the episodes the patient suffered a mild facial trauma due to syncope. His medical
treatment included angiotensin receptor blocker, diuretic, and a beta-blocker. The patient was
referred to a general cardiologist who recorded the ECG shown on Figure 2.17.1. This recording
shows right bundle branch block (RBBB) and left anterior fascicular block (LAFB) alternating with
RBBB and left posterior fascicular block. PR interval is borderline at 200 ms, slightly longer with
RBBB LAFB configuration. Beta-blocker therapy was discontinued and the patient was sent for
pacemaker implantation. When the patient was admitted to the clinic for the procedure, the
conduction disturbance had progressed with a block in the left anterior fascicle resulting in 2:1
atrioventricular block shown on Figure 2.17.2. Red arrows denote the P waves and blue lines show
blocked P waves.

Figure 2.17.1

38  u  Section 2: Conduction Disturbances: Sinus Node Disease


Figure 2.17.2

Case 2.17  u  39
SECTION 3
Miscellaneous Phenomena: Concealed Conduction,
Superabnormalities, Aberrancy Conduction, Premature Atrial and
Ventricular Contractions (PACs and PVCs)

CASE
David J. Callans, MD 3.1
Patient History
A 67-year-old male with no structural heart disease, complained of palpitations caused by
premature ventricular complexes.

Figure 3.1.1

ECG Masters’ Collection: Favorite ECGs from Master Teachers Around the World © 2017 Mohammad Shenasa, Mark E. Josephson,
N. A. Mark Estes III, Ezra A. Amsterdam, Melvin Scheinman. Cardiotext Publishing, ISBN: 978-1-942909-08-8.   41
Discussion
In the electrophysiology (EP) lab, activation mapping from the anterior intraventricular vein (AIV)
was slightly earlier than the endocardium just anterior to the aorta (−20 vs. −13 ms from QRS
onset). Pacemapping from the AIV had a qS morphology in lead I, consistent with epicardial
stimulation; pacemapping from the LV endocardium had a dominant R wave in lead I. Neither site
matched the premature ventricular contraction (PVC) morphology (rS) in lead I, as the PVC seemed
to be “in between” the pacemaps. In fact, endocardial ablation resulted in late disappearance of the
PVCs, followed by a reduction in frequency but not elimination. Afterwards, ablation from the AIV
resulted in elimination of the PVCs suggesting a site of origin “in between” the two sites.

42  u  Section 3: Miscellaneous Phenomena: Concealed Conduction


CASE
Mohammad Dalili, MD 3.2

Figure 3.2.1

Nonconducted atrial premature contraction (APC; red arrows). Note the pause that is less than
compensatory.

Comments
The APC has to get into the sinus node; it resets the node and then the sinus node fires and
activates the atrium. The time it takes for the next P wave to appear allows for a junctional escape
(yellow arrows)—note the shorter PR interval after the pause.

Case 3.2  u  43
CASE
3.3
Arnaud Denis, MD
Pierre Jaïs, MD, PhD

Patient History
A 63-year-old female was referred to our institution for dyspnea (NYHA II) without any other
symptom. Transthoracic echocardiography revealed a moderately reduced left ventricular ejection
fraction (40%).
Figure 3.3.1 – Figure 3.3.4 are presented with Lewis diagrams (abbreviations—A: atrium, AVN:
atrioventricular node, HPS: His-Purkinje system, and V: ventricle).

Figure 3.3.1

44  u  Section 3: Miscellaneous Phenomena: Concealed Conduction


Heart rate is 75 bpm with narrow QRS complexes. There is no P wave just before the QRS.
A sinus P wave (positive in leads I, II, III, aVF and negative in leads V1 and aVR, solid arrow) is
present just after each QRS and is fused with the ST segment and the beginning of the T wave.
Two hypotheses can be offered:
1. The P wave is conducted to the ventricle with a PR interval of 600 ms through a slow pathway. In
this hypothesis, the fast pathway is blocked antegrade because of a concealed retrograde
conduction coming from the previous beat.
2. Isorhythmic dissociation may be present; that is, a junctional rhythm that is a little bit faster
than the sinus rate. In that case, there is no relationship between the sinus P wave and QRS.

Figure 3.3.2

Sinus rhythm is stable at 75 bpm. The ventricular rhythm is irregular, and there are more QRS
complexes than P waves.
1. The first P wave is conducted to the ventricle with a normal PR interval (160 ms), the second with
a PR interval of 340 ms, and the third one with PR interval of 180 ms. The variation of the PR
interval can be explain by dual AV nodal physiology properties or concealed junctional ectopies.

Case 3.3  u  45
2. Same sequences described in Figure 3.3.1. The coexistence of P wave conducted to the ventricle
with normal, prolonged or very prolonged PR interval support the hypothesis of dual AV nodal
physiology. Two narrow QRS complexes (with the same morphology as the normal sinus QRS)
are apparently not preceded by P waves (solid arrow). These QRS complexes can be explained by
dual AV nodal physiology properties (double ventricular response due to a single atrial
depolarization through both AV nodal pathways) or junctional ectopy conducted to the ventricle
but not to the atrium. All of these ECG patterns cannot be explained by junctional ectopies,
especially when the P wave is conducted to the ventricle with a very prolonged PR interval.
All ECG patterns are explained by the presence of dual AV nodal pathways capable of
conduction through the fast pathway, the slow pathway or both at the same time. There is always a
concealed retrograde conduction that explains the variation of the PR interval and the self-
perpetuation of the mechanism.

Figure 3.3.3

46  u  Section 3: Miscellaneous Phenomena: Concealed Conduction


1. The first P wave is conducted to the ventricle through the slow pathway with a very prolonged PR
interval. Then, the second P wave blocks in the AV node. The third P wave is conducted only
through the fast pathway, resulting in a normal PR interval.
2. Double ventricular response to a single P wave occurs through both fast and slow pathways. The
second QRS complex that is conducted through the slow pathway is conducted to the ventricle
with a complete right bundle branch block (RBBB) and left anterior fascicular aberrancy. This is
followed by an atrial ectopy (dotted arrow) that conducts to the ventricle through the fast
pathway.
3. Double ventricular response occurs as described in step 2. The following sinus P wave (solid
arrow) is fused with the previous T wave and is conducted through the fast pathway with an
RBBB and through the slow pathway without ventricular aberrancy.
4. The following P wave is conducted to the ventricle through the slow pathway as described in
Figure 3.3.1. Indeed, the fast pathway is blocked anterogradely by the concealed retrograde
conduction through the fast pathway, which is coming from the previous beat.

Figure 3.3.4

Case 3.3  u  47
This is the ECG immediately after a slow pathway ablation, which shows sinus P-wave conduction
through only the fast pathway (with a normal PR).
All ECG patterns are explained by the presence of dual AV nodal pathways. P-waves can be
conducted to the ventricle either through the fast pathway, the slow pathway (which is very slow), or
both pathways at the same time. The patient’s symptoms can be explained by a pseudo pacemaker
syndrome that was due to the loss of AV synchrony. A slow pathway ablation was performed with an
endpoint of complete slow pathway elimination. Patient was asymptomatic after ablation and her
left ventricular ejection fraction was normalized 2 months after the ablation, supported the
hypothesis of rhythmic cardiomyopathy.

48  u  Section 3: Miscellaneous Phenomena: Concealed Conduction


CASE
3.4
Arnaud Denis, MD
Frédéric Sacher, MD, PhD

Patient History
A 55-year-old male without medical history presented with recurrent episodes of palpitation.
Twelve-lead ECGs during palpitations are presented below.

Question
What is the mechanism (1) of the palpitation, (2) of the blocked P wave, and (3) of the wide QRS?
Does the patient need an antiarrhythmic drug and/or a pacemaker implantation?
ECGs are presented with Lewis diagrams (abbreviations: A: atrium, AVN: atrio-ventricular
node, HPS: His-Purkinje system, and V: ventricle).

Figure 3.4.1

Case 3.4  u  49
1. Premature wide QRS complexes that are not preceded by atrial activity. The morphology of the
wide QRS is compatible with left bundle branch block (LBBB) aberrancy. This QRS complex is
either a premature ventricular contraction (PVC) or ectopy from the His with only anterograde
conduction and LBBB aberrancy. This is then followed by a sinus beat that conducts to the
ventricle with PR prolongation (PR = 320 ms). The PR prolongation is explained by the concealed
conduction of the ectopy (His or PVC) in the AV node but not in the atrium.
2. Premature wide QRS complex that is again not preceded by atrial activity. The morphology is
compatible with right bundle branch block (RBBB) aberrancy. It is unlikely that these two wide
QRS complexes are two different PVC morphologies mimicking typical LBBB and RBBB.
The main hypothesis is His ectopy with only anterograde conduction with RBBB (or LBBB in 1)
aberrancy. This is then followed by a sinus beat that conducts to the ventricle with PR
prolongation (PR = 280 ms). At this stage, it is impossible to know if the ectopy is junctional or
para-Hisian. Electrophysiological study confirmed the diagnosis of parahisian ectopy.
3. PR prolongation followed by a nonconducted P wave that would ordinarily indicate a second-degree
AV block, Mobitz type I. But in this patient with suspected His ectopy, the P wave more than likely
fails to conduct because of a concealed His ectopy (no resultant QRS or retrograde P wave). PR
prolongation is explained by the same mechanism: concealed His ectopy with concealed conduction
in the AV node resulting in sinus P-wave conduction to the ventricle with PR prolongation.
4. His ectopy with only anterograde conduction with RBBB and left anterior fascicular aberrancy.
This is followed by a sinus beat that conducts to the ventricle with PR prolongation (P-R=280 ms)
due to concealed retrograde conduction of the His ectopy in the AV node.

Figure 3.4.2

50  u  Section 3: Miscellaneous Phenomena: Concealed Conduction


The basic rhythm is sinus with a PR of 160 ms.
1. His ectopy that is conducted retrogradely to the atrium (negative P wave in inferior leads) but
not conducted to the ventricle because the His-Purkinje system is refractory. There is a
compensatory pause after His ectopy which indicates that the sinus node has been depolarized
by the retrograde conduction of the His ectopy. This beat is followed by a second His ectopy with
only anterograde conduction which occurs just before the sinus P wave. The sinus P wave is
blocked in the AV node because of a concealed conduction in the AV node coming from a second
His ectopy.
2. His ectopy with both anterograde (with RBBB aberrancy) and retrograde conduction (negative
P-wave in inferior leads). This is followed by a second His ectopy as described in Figure 3.4.1.

Figure 3.4.3

Case 3.4  u  51
1. Sudden and unexpected PR prolongation is explained by concealed His ectopy. His ectopy is not
conducted in the ventricles or the atria but it is conducted in the AV node only. When the sinus
P-wave occurred, the AV node was still in a relative refractory period and therefore the
conduction time in the AV node is prolonged. The following P-wave is conducted with a normal
PR interval.
2. His ectopy with only anterograde conduction and an incomplete RBBB aberrancy followed by a
sinus beat conducted to the ventricle with PR prolongation (PR = 320 ms).

Figure 3.4.4

52  u  Section 3: Miscellaneous Phenomena: Concealed Conduction


1. His ectopy with both anterograde (with RBBB and left posterior fascicular aberrancy) and
retrograde conduction (negative P wave in inferior leads). Note that the retrograde conduction is
slower in the first His ectopy when compared to the second one. The first His ectopy a) occurs
slightly earlier then the previous sinus beat, b) the AV node is still in a relative refractory period,
and c) the conduction time in the AV node is prolonged.
An EP study confirmed the diagnosis of ectopy located below the His. His ectopy can be
concealed, conducted to the atrium and/or to the ventricle. Wide QRS complexes are explained by
different aberrancy locations.
This patient was managed with antiarrhythmic drugs and was asymptomatic while under
treatment.

Case 3.4  u  53
CASE
3.5
James E. Ip, MD
Bruce B. Lerman, MD

Patient History
A 37-year-old female with mitral valve prolapse experienced palpitations. An echocardiogram
showed mild left ventricular dysfunction with an ejection fraction of 40%–45%. A Holter monitor
recorded frequent premature ventricular contractions (PVCs) 23 100 beats (26% of total). Her
electrocardiogram is shown in Figure 3.5.1.

Figure 3.5.1  12-lead electrocardiogram showing PVCs.

Question
Where is the site of origin of the PVC?

Discussion
The most common form of idiopathic monomorphic ventricular arrhythmia arises from the
ventricular outflow tracts. These types of arrhythmias can manifest as frequent unifocal PVCs,
repetitive monomorphic ventricular tachycardia (VT), or sustained exercise-induced VT. Most of
these focal arrhythmias (80%) arise from the right ventricular outflow tract (RVOT) rather than the
left ventricular outflow tract (LVOT). Therefore, the majority of these arrhythmias will have a left
bundle branch block configuration with an inferiorly directed axis.
The RVOT is a conical-shaped structure that merges with the pulmonary trunk. The
description of RVOT locations differs based on fluoroscopic orientation versus the attitudinal
position in the chest. The attitudinally posterior aspect is often referred to as the “septal” wall and
an attitudinally anterior aspect is often referred to as the “free-wall”. The attitudinally leftward

54  u  Section 3: Miscellaneous Phenomena: Concealed Conduction


aspect of the RVOT is sometimes described as “anterior” (based on fluoroscopic orientation)
whereas the attitudinally rightward aspect is considered “posterior.” Most RVOT arrhythmias
originate from “septal” aspect, whereas approximately 10% originate from the “free-wall.”
Since the unipolar ECG lead V1 is positioned on the right side of the sternum, activation from
the RVOT is primarily directed away from the lead, resulting in an rS or QS wave. Therefore, a free-
wall RVOT focus, which is positioned along the most attitudinally anterior aspect of the heart is
associated with a QS wave in lead V1. As the arrhythmia focus originates more posteriorly in the
chest, along the posterior or “septal” RVOT, there are greater initial anterior forces observed in
leads V1–V3 and, resulting in a small r waves in V1. Therefore, a “septal” RVOT focus will have an
earlier precordial R wave transition (≥ V3) compared to a free wall focus (≥ V4). It also follows that a
“free wall” focus is often associated with larger S waves because of unopposed posterior forces
(S wave amplitude ≥ 3.0 in lead V2). Another distinguishing feature of a “septal” versus “free wall”
site is that the former has a relatively narrow QRS complex (≤ 140 ms) because it is in closer
proximity to the conduction system. On the other hand, a “free wall” location has a relatively wider
QRS complex (>140 ms) and can be associated with notched QRS complexes, particularly in the
inferior leads.
The frontal plane axis is used to help localize whether the arrhythmia focus is “posterior”
(attitudinally rightward) or “anterior” (attitudinally leftward). The former will have more positive
polarity in lead I and aVL (qR, RS), because the forces are directed toward those leads, whereas a
leftward focus will generate forces away from these leads, generating QS, Qr, or rS waves.
This patient’s ECG shown in Figure 3.5.1 has a late precordial transition in lead V4 (blue arrow).
The negligible r waves in leads V1 and V2 are suggestive of a free wall RVOT location. This location
is further supported by the presence of notched QRS complexes in the inferior leads (red arrows).
Her PVC site of origin localized to the anterior, leftward free wall of the RVOT. Following successful
radiofrequency ablation, her symptoms and her cardiomyopathy resolved.

References
1. Dixit S, Gersteinfeld EP, Callans DJ, et al. Electrocardiographic patterns of superior right ventricular outflow tract
tachycardias: Distinguishing sepal and free-wall sites of origins. J. Cardiovasc. Electrophysiol. 2003;14:1–7.
2. Tada H, Ito S, Naito S, et al. Prevalence and electrocardiographic characteristics of idiopathic ventricular arrhythmia
originating in the free wall of the right ventricular outflow tract. Circ. J. 2004;68:909–914.
3. Kamakura S, Shimizu W, Matsuo K, et al. Localization of optimal ablation site of idiopathic ventricular tachycardia
from right and left ventricular outflow tract by body surface ECG. Circulation. 1998;98:1525–1533.

Case 3.5  u  55
CASE
3.6
James E. Ip, MD
Bruce B. Lerman, MD

Patient History
A 71-year-old male complained of lightheadedness, shortness of breath, fatigue, and decreased
exercise tolerance. An echocardiogram showed an ejection fraction of 40%–45%. A cardiac
catheterization showed nonobstructive coronary disease. A Holter monitor showed frequent PVCs
with a daily burden of 20%. His ECG is shown in Figure 3.6.1.

Question
Where is the site of origin of this PVC?

Figure 3.6.1 12-lead ECG showing frequent PVCs.

Discussion
The PVCs shown in Figure 3.6.1 have LBBB morphology with an inferior axis, suggesting an origin
in the ventricular outflow tract. The precordial transition in lead V3 along with a calculated V2
transition ratio of 0.68 (as discussed in case 9A.8) point to an origin in the left ventricular outflow
tract (LVOT) rather than the right ventricular outflow tract.
Crescents of ventricular myocardium that extend into the right and left coronary sinuses of
Valsalva are frequent sources of LVOT arrhythmias. Since the noncoronary sinus and the posterior
aspect of the left coronary sinus of Valsalva lack myocardial tissue, these regions are rarely
associated with LVOT arrhythmias. The right coronary cusp (RCC) is attitudinally more anterior in
the chest than the left coronary cusp (LCC) and the noncoronary cusp (NCC).
Because the LCC and RCC are relatively more posterior in the chest compared to the RVOT,
arrhythmias originating from these cusps are associated with an earlier precordial transition, with
the emergence of R waves in lead V2 or V3. LCC arrhythmias are characterized by an R/S wave
amplitude in lead V1 or V2 (≥ 30%) or R wave duration index ≥ 50% (ratio of longest R wave in V1 or
V2 to total QRS duration). The precordial r/R wave and transition occur earlier with LCC

56  u  Section 3: Miscellaneous Phenomena: Concealed Conduction


arrhythmias (in lead V1 or V2) compared to those from the RCC because of its relatively more
posterior location. In addition, because the LCC is slightly more superior and closer to the septum,
arrhythmias originating from this cusp have relatively larger R-wave amplitudes in the inferior leads
and a narrower QRS complexes. Since the LCC is located more leftward in the chest compared to
the RCC, forces are directed more toward the right, with a QS or rS morphology inscribed in lead I.
In contrast, RCC arrhythmias may have a relatively larger R wave in lead I. An example of a LCC
PVC is shown in Figure 3.6.2. ECG characteristics of arrhythmias originating from the right and left
coronary cusp commissure include notching of the downward deflection in lead V1, a precordial
transition in lead V2 or V3 and an R or Rs wave in lead I.

Figure 3.6.2  12-lead electrocardiogram showing frequent PVCs from a left coronary cusp. Note the transition in lead V2.

Intracardiac echocardiography has been very helpful in accurately identifying the site of origin
of these arrhythmias and in facilitating the safety of ablation by identifying the origin of the left
main coronary ostium. This patient had successful radiofrequency ablation performed at the site of
origin that localized to the RCC (Figure 3.6.3).

Figure 3.6.3  Electroanatomical map showing site of successful ablation in the right coronary cusp. The location of the cusps and left
main coronary artery were identified with intracardiac echocardiography. LCC = left coronary cusp, LM = left main coronary artery,
NCC = noncoronary cusp, and RCC = right coronary cusp.

Case 3.6  u  57
References
1. Ouyang F, Fotuhi P, Ho SY, et al. Repetitive monomorphic ventricular tachycardia originating from the aortic sinus
cusp. J. Am. Coll. Cardiol. 2002;39:499–508.
2. Lin D, Ilkhanoff L, Gerstenfeld E, et al. Twelve-lead electrocardiographic characteristics of the aortic cusp region
guided by intracardiac echocardiography and electroanatomic mapping. Heart Rhythm. 2008;5:663–669.
3. Bala R, Garcia FC, Hutchinson MD, et al. Electrocardiographic and electrophysiologic features of ventricular
arrhythmias originating from the right/left coronary cusp commissure. Heart Rhythm. 2010;7:312–322.
4. Yamada T, Yoshida N, Murakami Y, et al. Electrocardiographic characteristics of ventricular arrhythmias originating
from the junction of the left and right coronary sinuses of Valsalva in the aorta: The activation pattern as a rationale
for the electrocardiographic characteristics. Heart Rhythm. 2008;5:184–192.

58  u  Section 3: Miscellaneous Phenomena: Concealed Conduction


CASE
3.7
James E, Ip, MD
Bruce B. Lerman, MD

Patient History
A 68-year-old female with hypertension developed intermittent light-headedness and chest
heaviness. An ECG showed frequent premature ventricular contractions (PVCs) [Figure 3.7.1].
A 24-hour Holter monitor recorded monomorphic PVCs (29% burden) with occasional runs of
nonsustained ventricular tachycardia. A cardiac MRI revealed a mildly dilated left ventricle with
mildly reduced left ventricular function (LVEF = 51%), no myocardial scar/infarction, normal right
ventricular size and systolic function and no myocardial fibro-fatty infiltration.

Figure 3.7.1  12-lead electrocardiogram showing PVCs.

Question
Where is the site of origin of the PVC?

Discussion
A subset of RVOT arrhythmias can also originate from the tricuspid annulus, most frequently from
the para-Hisian region, the most rightward aspect of the RVOT. Since the para-Hisian area lies in
close proximity to the right coronary cusp and non-coronary cusp, ventricular arrhythmias
originating from these structures may share similar ECG findings. However, the rightward and
inferior position of the para-Hisian area relative to the remainder of the RVOT leads to a slightly
difference ECG appearance. In particular, the frontal axis is not as inferiorly directed in contrast to
most other forms of RVOT tachycardia. There may be a positive (or isoelectric) QRS deflection in
lead aVL because of its relatively rightward and inferior para-Hisian location. Because the wave of
depolarization is from right to left, the R wave in lead II is typically greater than that lead III. Lastly,
para-Hisian ventricular arrhythmias, being closer to the conduction system, typically have more

Case 3.7  u  59
narrow QRS complexes (as opposed to arrhythmias originating from the free wall of the tricuspid
annulus).
This patient’s PVC shows an RS wave in lead aVL (Figure 3.7.1, blue arrow) and the site of origin
localized to the para-Hisian area where successful ablation was performed (Figure 3.7.2).

Figure 3.7.2  Fluoroscopy of successful catheter ablation location in the para-Hisian area (PVC origin) in the right anterior oblique (left
panel) and left anterior oblique (right panel) views. Abl = ablation catheter. His = His catheter.

References
1. Yamauchi Y, Aonuma K, Takahashi A, et al. Electrocardiographic characteristics of repetitive monomorphic right
ventricular tachycardia originating near the His-bundle. J. Cardiovasc. Electrophysiol. 2005;16:1041–1048.
2. Komatsu Y, Otomo K, Taniguchi H, et al. Catheter ablation of ventricular arrhythmias arising from the right
ventricular septum close to the His bundle: features of the local electrogram at the optimal ablation site.
J. Cardiovasc. Electrophysiol. 2011;22:878–885.
3. Tada H, Tadokoro K, Ito S, et al. Idiopathic ventricular arrhythmias originating from the tricuspid annulus:
Prevalence, electrocardiographic characteristics, and results of radiofrequency catheter ablation. Heart Rhythm.
2007;4:7–16.

60  u  Section 3: Miscellaneous Phenomena: Concealed Conduction


CASE
3.8
James E. Ip, MD
Bruce B. Lerman, MD

Patient History
A 74-year-old male with nonischemic cardiomyopathy and an ejection fraction of 30% was found to
have a 25% PVC burden. His ECG is shown in Figure 3.8.1.

Figure 3.8.1  12-lead electrocardiogram showing PVCs.

Question
Where is the site of origin of his PVCs?

Discussion
Idiopathic outflow tract ventricular arrhythmias can also originate from the tricuspid and mitral
valve annuli. Although these types of arrhythmias are less prevalent (≈5%–10%) than the RVOT and
coronary cusps, they behave similarly in that they are sensitive to adenosine, verapamil and vagal
maneuvers and are inducible with catecholamines and programmed stimulation. Mitral annular
(MA) ventricular arrhythmias can originate from the aorto-mitral continuity (AMC) as well as
from the anterolateral, lateral, and posterior annulus.
This patient’s PVCs have an inferior axis with an RBBB morphology. The characteristic finding
of a qR pattern in lead V1 is suggestive of a site of origin at the AMC. The AMC is a fibrous
connection between the aorta and anterior mitral valve leaflet, which extends from the left fibrous
trigone (posterior aspect of left coronary cusp) to the right fibrous trigone (junction of the right and
noncoronary cusps).
All mitral annular locations with the exception of the posterior region have an inferior axis. In
addition, ventricular arrhythmias from the AMC, anterior MA, and anterolateral MA result in a

Case 3.8  u  61
RBBB morphology in lead V1. A focus arising from the AMC area has a characteristic qR
morphology in lead V1. In contrast, anterior and anterolateral MA locations have monophasic R
waves.
As described in the previous cases, the frontal axis changes as the location of the arrhythmia
focus becomes more leftward in the chest. In lead I, the AMC location has relatively more positive
forces (Rs morphology), the anterior MA location has equiphasic forces (rs morphology), and the
anterolateral MA has more prominent negative forces (rS or QS morphology).
This patient had PVCs with the characteristic qR pattern in lead V1 (Figure 3.8.1, red arrow)
that were localized to the AMC region using a retrograde aortic approach (Figure 3.8.2). The
electroanatomical map demonstrated the site of successful ablation at the location of earliest
activation in this region just below the aortic valve (Figure 3.8.3).

Figure 3.8.2  Fluoroscopy of successful catheter ablation location in the aorto-mitral continuity in the right anterior oblique (left panel)
and left anterior oblique (right panel) views. Abl = ablation catheter, CS = coronary sinus catheter placed in the distal coronary sinus,
His = His catheter, and RV = right ventricular catheter.

Figure 3.8.3  Electroanatomical map and intracardiac electrograms from the site of successful ablation in the aorto-mitral continuity. Note
the qR pattern in lead V1 (yellow arrow). LCC = left coronary cusp, MA = mitral annulus, and RCC = right coronary cusp.

62  u  Section 3: Miscellaneous Phenomena: Concealed Conduction


References
1. Ip JE, Liu CF, Thomas G, et al. Unifying mechanism of sustained idiopathic atrial and ventricular annular
tachycardia. Circ. Arrhythm Electrophysiol. 2014;7:436–444.
2. Chen J, Hoff PI, Rossvoll O, et al. Ventricular arrhythmias origination from the aortomitral continuity: An
uncommon variant of left ventricular outflow tract tachycardia. Europace. 2012;14:388–395.
3. Dixit S, Gerstenfeld EP, Lin D, et al. Identification of distinct electrocardiographic patterns from the basal left
ventricle: Distinguishing medial and lateral sites of origin in patients with idiopathic ventricular tachycardia. Heart
Rhythm. 2005;2:485–491.

Case 3.8  u  63
CASE
3.9 Fred Morady, MD

Question
What is the most likely cause of the irregular rhythm in this electrocardiogram (Figure 3.9.1)?

Figure 3.9.1

Explanation
There is a consistent pattern of grouped beating, with all QRS complexes having the same
morphology. There are sinus P waves that have a regular rate of approximately 60 bpm. The most
likely explanation is that there are dual atrioventricular nodal pathways and a “double-fire”
phenomenon, with each atrial depolarization leading to two ventricular depolarizations. The first
QRS complex after each P wave is a result of conduction through the fast pathway and the second
QRS complex is a result of conduction through the slow pathway (Figure 3.9.2). Junctional bigeminy
is also a possible explanation, but this is considerably less common than the double-fire
phenomenon.
It is interesting to note that the QRS complexes alternate in amplitude in several of the leads.
This most likely is a manifestation of a slight degree of aberrant conduction created by the
“long-short” sequences that result from the double-fire phenomenon and that cause mild
lengthening of refractoriness in the His-Purkinje system.

64  u  Section 3: Miscellaneous Phenomena: Concealed Conduction


Figure 3.9.2

Case 3.9  u  65
CASE Begüm Yetiş Sayın, MD

3.10
Sercan Okutucu, MD
Ali Oto, MD

Patient History
A 58-year-old patient presents with palpitations, shortness of breath, and chest discomfort.
Echocardiography was normal and coronary angiography revealed normal coronary arteries.

Figure 3.10.1

66  u  Section 3: Miscellaneous Phenomena: Concealed Conduction


Figure 3.10.2

Discussion
Electrophysiology study revealed premature ventricular contractions originating from the aortic
cusps. Ablation was performed successfully from the area between the noncoronary cusp and left
coronary cusp.

Case 3.10  u  67
CASE
3.11 Philip Podrid, MD

Patient History
A 45-year-old female with a history of rheumatoid arthritis being treated medically and
hypertension being treated with a calcium channel blocker and hydrochlorthiazide presents to her
primary care physician for a routine physical examination. Her pulse is noted to be rapid and a
12-lead ECG is obtained (Figure 3.11.1). As a result, she is sent to the emergency department when
an ECG is repeated (Figure 3.11.2).

Questions
1. What is the rhythm seen in Figure 3.11.1?
2. What is the etiology for the aberrated complexes in Figure 3.11.1?
3. What is the etiology for the aberrated complexes in Figure 3.11.2?

Figure 3.11.1  Normal sinus rhythm, LVH, interpolated premature junctional complexes in a bigeminal pattern (junctional bigeminy), and
Ashman’s phenomenon.

68  u  Section 3: Miscellaneous Phenomena: Concealed Conduction


Figure 3.11.2  Normal sinus rhythm, LVH, interpolated premature junctional complexes in a trigeminal pattern (junctional trigeminy), and
Ashman’s phenomenon.

Diagnosis
Figure 3.11.1 shows an irregular rhythm, although there is pattern to the irregularity as there are
two different RR intervals; one is slightly longer (↔) than the other (┌┐). All the slightly longer
intervals are the same and the shorter intervals are all the same. There are also three much longer
RR intervals (=) which are all the same. Therefore, the rhythm is regularly irregular with an average
rate of about 150 bpm. All of the QRS complexes (except for complexes 2, 11, and 14 [*]) have the
same morphology. They are normal in duration (0.08 seconds) and a normal morphology and axis
between 0° and +90° (positive QRS complex in leads I and aVF). There is a deep S wave in lead V2
(30 mm) ( [ ) which meets one of the criteria for LVH (i.e., any S wave or R wave in the precordial
leads ≥25 mm). The QT/QTc intervals are normal (280/440 ms). A P wave (^) can be seen before the
QRS complex that ends the long RR interval (=) The PR intervals associated with these P waves are
the same (0.20 seconds) and the P waves are positive in leads I, II, aVF, and V4 –V6. Hence there is an
underlying sinus rhythm. During each of the slightly longer RR intervals (↔) there is positive
deflection (+) that has a morphology that is similar to the obvious P waves. These are P waves and
they have a stable PP interval (└┘) at a rate of 85 bpm. Hence there is an underlying normal sinus
rhythm. Every other QRS complex, which is associated with the slightly shorter RR interval (┌┐),
does not have an associated P wave. These are premature junctional complexes and they are in a
bigeminal pattern. As these premature complexes are not altering the PP interval and are not
associated with a pause, they are said to be interpolated. Complexes 2, 11, and 14 (*), which are
premature junctional complexes, are wide (0.12 seconds) and have a morphology of a typical right
bundle branch block with an RSR’ morphology in lead V1 (←) and a broad terminal S wave in lead I

Case 3.11  u  69
(→). The aberrancy is not rate related as the other premature junctional complexes are associated
with the same short RR interval (┌┐) but are not aberrated. However, it can be seen that each of the
complexes with a right bundle branch block are preceded by a long (=) short (┌┐) RR interval. This
is consistent with an Ashman’s phenomenon. In contrast to rate-related aberration, which is due to
some abnormality of His-Purkinje conduction (resulting from underlying myocardial disease,
antiarrhythmic drug, or hyperkalemia), the Ashman’s phenomenon is a normal physiologic change
in His-Purkinje refractoriness. When heart rate is faster His-Purkinje refractoriness shortens and
when the heart rate is slower, His-Purkinje refractoriness lengthens. When there is an abrupt
change in heart rate, going from slow (long RR interval) to fast (short RR interval) His-Purkinje
refractoriness may not adapt immediately and hence one or several QRS complexes will be
conducted with aberration.
Figure 3.11.2—The rhythm is irregular and there is group beating, i.e., three QRS complexes and
a pause (↔). Therefore the rhythm is regularly irregular. The first and third QRS complexes are
narrow (0.08 seconds) and have the same axis, morphology and QT/QTc intervals as the narrow
complexes in Figure 3.11.1. These QRS complexes are preceded by a P wave (+) with a stable PR
interval (0.20 seconds). The P waves are positive in leads I, II, aVF, and V4 –V6; they are sinus P
waves. The PP interval (└┘) is constant with a rate of 95 bpm. Therefore, there is an underlying
normal sinus rhythm. The second (middle) QRS complex has a right bundle branch block
morphology with an RSR’ morphology in lead V1 (←) and a broad terminal S wave in leads I and
V5–V6 (→). These QRS complex are not preceded by a P wave and are identical in morphology to the
aberrated QRS complexes in Figure 3.11.1. Therefore these are premature junctional complexes
which are not associated with a pause and are not altering the PP interval; hence they are
interpolated. As every third QRS complex is a premature junctional complex, this is junctional
trigeminy. These junctional complexes are preceded by a long (↔) short (┌┐) RR interval. The
aberration is not rate related as the RR interval before the first and second QRS complex is the same
as the RR interval between the second and third QRS complex (┌┐). If the aberration was rate
related, then the third QRS complex would also have right bundle branch block morphology.
However, only the second complex has a right bundle branch block as it follows a long (↔) short
(┌┐) cycle. Hence the right bundle branch block aberration is the result of the Ashman’s
phenomenon.

70  u  Section 3: Miscellaneous Phenomena: Concealed Conduction


CASE
Philip Podrid, MD 3.12
Patient History
A 54-year-old female with a long history of occasional palpitations presents to her primary care
physician for a routine physical examination. She states that the etiology of the palpitations has
never been established because the episodes usually self-terminate. Her physical examination is
unremarkable, however her pulse is noted to be irregular. A 12-lead ECG is obtained.

Questions
1. What accounts for the irregular rhythm?
2. What is the likely etiology for the palpitations?

Figure 3.12.1

Case 3.12  u  71
Diagnosis
Normal sinus rhythm, right bundle branch block, left anterior fascicular block, and dual AV nodal
pathways.

Discussion
The rhythm is irregular, but there is a repeating pattern of long (↔) and short (=) RR intervals. The
rhythm is regularly irregular. The QRS complexes are wide (0.12 seconds) and there is a right
bundle branch block morphology with an RSR’ in V1 (→) and a broad terminal S wave in leads I and
V5–V6 (←). The axis is extremely leftward between –30° and –90° (positive QRS complex in lead I
and negative complex in leads II and aVF). There are two etiologies for an extreme left axis, i.e., a
left anterior fascicular block (with an rS morphology in leads II and aVF) or an old inferior wall
myocardial infarction with deep Q waves in leads II and aVF. Although there is a QS morphology in
lead II, there are small R wave in leads III and aVF. Therefore, this is most likely a left anterior
fascicular block rather than an inferior wall myocardial infarction. The QT/QTc intervals are
prolonged (420/500 ms) but are only slightly prolonged (400/475 ms) when the widened QRS
complex duration is considered. There is a P wave before each QRS complex (+) and the P wave is
positive in leads I, II, aVF, and V4 –V6. The PP intervals are regular (└┘) at a rate of 85 bpm.
Therefore there is an underlying normal sinus rhythm. However, there are two different PR
intervals present, i.e., 0.16 seconds alternating with 0.32 seconds. As there is a stable sinus rhythm,
the two different RR intervals result from the two different PR intervals. One explanation is that the
sinus complex has a PR interval of 0.32 seconds and this is alternating with a premature junctional
complex that occurs after the sinus P wave. The more likely explanation is that the changes in the
PR interval is a manifestation of dual AV nodal pathways, i.e., one that conducts faster and one that
conducts slowly. The presence of dual AV nodal pathways is the underlying anatomic substrate for
atrioventricular nodal reentrant tachycardia. This is therefore the likely etiology for the palpitations.

72  u  Section 3: Miscellaneous Phenomena: Concealed Conduction


CASE
Philip Podrid, MD 3.13
Patient History
A 67-year-old female presents to the emergency department with a complaint of palpitations that
have been present for several hours. She notes that she has had a history of palpitation for several
years and has been treated with sotalol. However, this episode has lasted longer than usual. Upon
presentation her heart rate is noted to be rapid and irregular. A 12-lead ECG is obtained shortly
after she received intravenous beta blocker.

Figure 3.13.1

Questions
1. What is the rhythm seen in this ECG?
2. What accounts for the wide QRS complexes?

Diagnosis
Atrial fibrillation reverting to atrial flutter with 1:1 and 2:1 AV conduction, right bundle branch
block due to Ashman’s phenomenon, and low voltage in limb leads.

Case 3.13  u  73
Discussion
The initial part of the ECG (limb leads) shows a rhythm that is irregularly irregular. There are only
three supraventricular rhythms that are irregularly irregular.
1. Sinus arrhythmia, which is a respirophasic arrhythmia, has one P-wave morphology and a stable
PR interval.
2. Wandering atrial pacemaker or multifocal atrial rhythm (heart rate < 100 bpm) or multifocal
atrial tachycardia (heart rate > 100 bpm) in which there are ≥ 3 different P-wave morphologies
(and often PR intervals) with no dominant P-wave morphology.
3. Atrial fibrillation in which there are no organized P waves, but only fibrillatory waves which are
irregular in morphology, amplitude, and interval.
As no P waves are seen, the initial rhythm is atrial fibrillation with an average rate of about
160 bpm. In the second part of the ECG (precordial leads) the rhythm become regular, although
there are occasional intervals that are longer (↔, but which have the same duration). The rhythm is
therefore regularly irregular. The rate is 240 bpm. Noted in lead V1 are distinct atrial waveforms (*)
that have a stable PP interval (┌┐) and a rate of 240 bpm, which is identical to the ventricular rate.
Similar atrial waveforms at the same rate can be seen during the long pause in leads V4 –V6 (^).
These waveforms appear to be continuously undulating and the rhythm is thus atrial flutter. The
atrial flutter waves can also be seen in the lead II rhythm strip (o). As the atrial and ventricular rates
are identical, this is atrial flutter with 1:1 conduction and the long intervals represent 2:1 AV block
or conduction. Most of the QRS complexes (except for the five wide complexes in leads V1–V3) have
a normal duration (0.08 seconds) and a normal axis between 0° and +90° (positive QRS complex in
leads I and aVF). There is low voltage in the limb leads, defined as a QRS complex amplitude < 5 mm
in each limb lead. The QT/QTc intervals are slightly prolonged (280/470 ms), likely the result of
sotalol therapy.
The wide QRS complexes seen in leads V1–V3 have a right bundle branch block morphology,
with a tall broad R wave in lead V1 (←). However, this is not a rate related aberration as there are
other RR intervals that follow which are equally short (=) and are not aberrated. It can be seen that
the aberrated complexes are preceded by a long-short cycle (↔,└ ┘) and are the result of the
Ashman’s phenomenon. In contrast to rate related aberration, which is due to an abnormality of
His-Purkinje conduction (resulting from underlying myocardial disease, antiarrhythmic drug, or
hyperkalemia), the Ashman’s phenomenon is a normal physiologic change in His-Purkinje
refractoriness related to rate. When heart rate is faster His-Purkinje refractoriness shortens and
when the heart rate is slower, His-Purkinje refractoriness lengthens. When there is an abrupt
change in heart rate, going from slow (long RR interval) to fast (short RR interval) His-Purkinje
refractoriness may not adapt immediately and hence one or several QRS complexes will be
conducted with aberration. The explanation for the aberration continuing for five complexes is that
when there is a right bundle branch block and conduction to the ventricles via the left bundle, the
impulse may retrogradely enter the right bundle causing it to have a longer refractory period for
several complexes. Most often the Ashman’s phenomenon is associated with a right bundle branch
block aberration, probably because right bundle refractoriness is slightly longer than that of the left
bundle.

74  u  Section 3: Miscellaneous Phenomena: Concealed Conduction


CASE
3.14
Magdi M. Saba, MD
David E. Ward, MD

Patient History
A 45-year-old male with a recent pulmonary vein isolation for paroxysmal atrial fibrillation presents
with occasional palpitations.

Figure 3.14.1

Question
What is the diagnosis?

Discussion
The patient was asked to stand up and take a few deep breaths.
The subsequent three-lead rhythm traces reveal both conducted (red arrow) and blocked (black
arrows) atrial premature complexes. The conducted premature beats also exhibit bundle branch
block (long-short cycle).
Concealed atrial bigeminy is seen in the first ECG, with the P wave only visible in lead V1. The
change in autonomic tone allowed the diagnosis to be confirmed. This resolved spontaneously and
no specific treatment was indicated.

Case 3.14  u  75
Figure 3.14.2

Figure 3.14.3

76  u  Section 3: Miscellaneous Phenomena: Concealed Conduction


SECTION 4
Preexcitation Syndromes

CASE
4.1
Amit Noheria, MBBS, SM
Samuel J. Asirvatham, MD

Patient History
Resting ECG in a 27-year-old female with paroxysmal episodes of tachypalpitations (Figure 4.1.1).

Questions
1. What is the rhythm?
2. Where is the accessory pathway located?

Figure 4.1.1

ECG Masters’ Collection: Favorite ECGs from Master Teachers Around the World © 2017 Mohammad Shenasa, Mark E. Josephson,
N. A. Mark Estes III, Ezra A. Amsterdam, Melvin Scheinman. Cardiotext Publishing, ISBN: 978-1-942909-08-8.   77
Discussion, Interpretation, and Answers
This ECG demonstrates a rhythm with normal narrow QRS morphology except the last QRS
complex, which reveals preexcitation (Figure 4.1.2, asterisk). The preexcited complex has R>S in
lead V1 that localizes the accessory pathway on the free wall of the left atrioventricular annulus. The
strongly positive delta wave in inferior leads (lead II, asterisk) makes a posterior location unlikely,
suggesting a left lateral accessory pathway.1 The reason for the accelerated junctional rhythm in this
patient is unclear, but healthy young patients can have a robust junctional rhythm and also have
marked variability in the sinus rate. An ECG during sinus rhythm at a slightly different time
(Figure 4.1.3) shows consistent ventricular preexcitation over a left anterolateral accessory pathway.1

Figure 4.1.2

Figure 4.1.3

78  u  Section 4: Preexcitation Syndromes


Figure 4.1.2 begins with sinus P waves (black arrows). However, there are dissociated faster
narrow QRS complexes reflecting an accelerated junctional rhythm (blue arrows). Activation
originates from the AV node and directly proceeds to the His-Purkinje system without
encountering the AV accessory pathway; therefore, the QRS complexes are normal without evidence
of preexcitation. During this period of slowing of the spontaneous sinus node depolarizations and
junctional escape rhythm, the P waves are hidden within the QRS complexes, and the atria are
activated either from an isorhythmic sinus rhythm or retrogradely from the junctional rhythm.
Toward the end of the tracing, as the sinus rate accelerates, the sinus P waves are again seen
ahead of the QRS complexes (black arrows). It is not until the last beat on the tracing that the
P wave is well ahead of the expected junctional complex (red arrow) for antegrade conduction over
the left sided accessory pathway to manifest in the form of preexcitation.
Preexcitation due to an AV accessory pathway is a variable fusion between ventricular excitation
over the accessory pathway and the normal conduction system. Absence of preexcitation with
junctional complexes confirms that the accessory pathway inserts to the atrium and ventricle across
the atrioventricular annulus without involvement of the AV node or His-fascicular system. In
contrast, a junctional rhythm in the presence of a fasciculoventricular pathway will continue to
demonstrate delta waves. In fact, the degree of preexcitation in the fasciculoventricular tract is
fixed.

Reference
1. Arruda MS, McClelland JH, Wang X, et al. Development and validation of an ECG algorithm for identifying
accessory pathway ablation site in Wolff-Parkinson-White syndrome. J. Cardiovasc. Electrophysiol. 1998;9(1):2–12.

Case 4.1  u  79
CASE
4.2 Bernard Belhassen, MD

Patient History
Twelve-lead electrocardiogram (ECG) recordings during electrophysiology studies in an 18-year-old
female referred for evaluation of rapid palpitations.

Figure 4.2.1  Upper panel—Spontaneous ECG recording: a stable sinus rhythm (94/min) with normal PR (0.12 seconds) and normal
QRS complexes is followed by an escape rhythm consisting of seven wide QRS complexes with a left bundle branch block (LBBB)-left axis
pattern at a rate of 77/min. Note a fusion beat (*) on the next QRS complex before stable sinus rhythm (86/min) resumes. Lower panel—
Fifteen seconds after intravenous administration of 10 mg adenosine triphosphate: after a short sinus rate slowing to 56/min, sinus rate
increases and a succession of eight wide QRS conducted beats (94/min) is observed. The morphology of these wide QRS complexes is
identical to the escape beats observed in the upper panel. Note the longer PR interval (150 ms) preceding the wide QRS complexes. Also
note a fusion beat (*) on the next complexes before sinus rhythm with normal PR interval and QRS complexes resumes. Reproduced with
permission from John Wiley and Sons; Belhassen B, Ilan M, Glick A. Wide QRS rhythm in a young woman with recurrent palpitations:
What is the diagnosis? J Cardiovasc Electrophysiol. 2003;14:1376–1378.

80  u  Section 4: Preexcitation Syndromes


Discussion
This patient showed the presence of an atrio-fascicular pathway with long conduction time
(so-called Mahaim fiber) and attributed the LBBB-QRS complexes to antegradely preexcited beats
(lower panel). An accessory pathway potential was found at the posterolateral area of the tricuspid
annulus between A and V in sinus rhythm and preceding the QRS complexes during the escape
beat. Therefore, the wide QRS complex escapes in the upper panel were actually an escape rhythm
originating from the Mahaim fiber. The pathway was successfully ablated with radiofrequency
ablation and the escape rhythm did not recur.

Case 4.2  u  81
CASE
4.3 David J. Callans, MD

Patient History
A 28-year-old male with recurrent palpitations and ventricular preexcitation.

Figure 4.3.1

Discussion
Electrophysiology testing demonstrated a baseline HV interval of 20 ms. Bypass tract conduction
was anterograde only, and the pattern of preexcitation did not change with incremental atrial
pacing, the delivery of atrial premature beats, or with spontaneous junctional beats. These
observations, as well as the morphology of preexcitation, are consistent with a fasciculoventricular
pathway, which was incidental. His palpitations were secondary to AV nodal tachycardia, which was
treated with slow pathway modification.

82  u  Section 4: Preexcitation Syndromes


CASE
4.4
Tamer S. Fahmy, MD, PhD
Ahmed Abdel Aziz, MD, PhD

Patient History
A 30-year-old female patient complaining of recurrent attacks of rapid palpitations. Her basic ECG
showed no evidence of manifest preexcitation, while her ECG during tachycardia is shown in
Figure 4.4.1. She was admitted to the cardiac electrophysiology laboratory for diagnosis and ablation
of her tachycardia.

Figure 4.4.1  Surface ECG during tachycardia. Twelve-lead ECG of the patient during tachycardia at a rate of 210 bpm showing wide
complex tachycardia having RBBB morphology. Though it has a monophasic R in V1, yet the terminal delay typical of RBBB is not evident.

From the beginning of the procedure, the tachycardia was incessant, and proved to be AVNRT,
for which the slow pathway was successfully ablated. ECG after ablation of the slow pathway
intermittently showed the following (Figure 4.4.2):

Case 4.4  u  83
Figure 4.4.2  Surface ECG after slow pathway ablation. After slow pathway ablation, there was intermittent development of RBBB
morphology. Atrial pacing reproduces the same morphology as that which occurs spontaneously. As shown with atrial pacing, the surface
ECG shows RBBB morphology that returns to normal with cessation of pacing.

Questions
1. What does the ECG intermittently show?
2. What does this tracing suggest?

Answers
1. Intermittent conduction over a bystander pathway antegradely conducting with atriofascicular
accessory pathway decremental properties, resulting in RBBB-like morphology.
2. This tracing differs from the regular Mahaim fibers in the site of insertion, which is probably in
the proximal left bundle.

Discussion and Interpretation


The intracardiac recording excludes actual right bundle branch block (RBBB), which has a relatively
long HV interval, with antegrade activation of the His. The presence of prolonged AH with short
HV, and distal to proximal activation of the His with a wide complex morphology with
ventriculoatrial dissociation, indicates an antegradely conducting accessory pathway with
decremental conduction properties. Interestingly, the common insertion site of Mahaim fibers

84  u  Section 4: Preexcitation Syndromes


produces a left bundle branch block (LBBB) morphology with delayed transition to after V4. Thus,
the apical activation precedes the basal, and distal to proximal activation of the His. In this case, it
seems that the bystander pathway passing the His bundle is inserted in the conduction system on
the left side rather than the right, probably at the proximal left bundle. This is because the basal
right ventricle is still activated early, and the cause for terminal delay is due to activation of the
lateral left ventricle (LV). However, there was no LV catheter inserted to confirm the later finding.
Latent left-sided decremental atriofascicular accessory pathway has been previously described, but
contrary to our case, it was involved in the tachycardia.1

Figure 4.4.3  Intracardiac recordings after ablation. Surface ECG and intracardiac recording after ablation, where the ablation catheter
(MAP) is placed in the high right atrium (HRA). After the second beat, there is an atrial premature contraction (earliest in the MAP/HRA
catheter) that was non-conducted. This is followed by a normally conducted beat, with incomplete RBBB. The following beat is aberrantly
conducted with RBBB pattern on the surface ECG. At the HBE, the AH/AV is prolonged, and HV shortens with the distal His preceding
the proximal His, which disappears or is delayed just before the V. Note that despite the RBBB morphology, the LV (in distal CS) lead still
appears more delayed than the RV (see text for explanation).

Case 4.4  u  85
Figure 4.4.4  Intracardiac recording during change from narrow to wide complex. Surface ECG and intracardiac recording after ablation,
where the ablation catheter (MAP) is placed in the high right atrium (HRA). The first two beats are narrow, while the following two
beats are wide with RBBB morphology. The patient has sinus tachycardia and similar P wave on surface ECG (upright in I and aVF) and
the earliest atrial activation in the MAPp (H/LRA). There is autonomic modulation of the sinus rate with gradual increase in HR over the
four beats (520/500/495/490 ms). The first beat is normally conducted with normal AH/HV and proximal to distal activation of the His.
In the second beat with slight acceleration of the sinus rate, there is more AH prolongation, and is still proximal to distal activation of
the His; however, the HBEd shows double H activation, indicating intra-Hisian conduction delay. This is followed by conduction over the
atriofasicular pathway in the third and fourth beat, with AH prolongation, HV shortening, and distal-to-proximal activation of the His.

Reference
1. Goldberger JJ, Pederson DN, Damle RS, et al. Antidromic tachycardia utilizing decremental, latent accessory
atrioventricular fibers: Differentiation from adenosine-sensitive ventricular tachycardia. J. Am. Coll. Cardiol.
1994;24(3):732–738.

86  u  Section 4: Preexcitation Syndromes


CASE
Robert Frank, MD 4.5
Patient History
A 55-year-old female had brief episodes of tachycardia for many years. Her baseline ECG was
normal. However, a prolonged episode allowed her to arrive in the emergency department where a
12-lead ECG was recorded (Figure 4.5.1 and Figure 4.5.2).

Question 1
What is the tachycardia mechanism?

Figure 4.5.1

Figure 4.5.2

Case 4.5  u  87
Answer 1
This is a regular 170 bpm tachycardia with wide QRS and typical left bundle branch block (LBBB)
morphology. P waves can be seen in V1 as notches at the T-wave onset, 130 ms after the QRS onset
(Figure 4.5.2). This can be a ventricular tachycardia with 1/1 retrograde conduction, or a
supraventricular tachycardia with a functional LBBB, as sinus rhythm ECG was always normal. The
markedly superior axis and late transition is also compatible with an atriofascicular tachycardia.
The tachycardia was abruptly interrupted by IV adenosine (Figure 4.5.3).

Question 2
What is the tachycardia mechanism?

Figure 4.5.3

88  u  Section 4: Preexcitation Syndromes


Answer 2
VT interruption by adenosine is unusual, but adenosine may interrupt a triggered focus, usually
found in the ventricle from the outflow tract, which is not suggested by the frontal plane QRS
orientation. As the adenosine blocks the AV node, a reentrant tachycardia using the AV node should
be suspected.
Fortunately, tachycardia resumed later and was recorded by the monitoring system
(Figure 4.5.4).

Question 3
What is the mechanism?

Figure 4.5.4

Answer 3
The tachycardia is triggered by an atrial extrasystole. It is followed by an AV prolongation and a
LBBB QRS. Retrograde P wave can be seen in the V1 precordial lead, after a long ventriculoatrial
conduction time, suggesting an accessory pathway.
A later EP study confirmed a posteroseptal accessory pathway with predominant retrograde
conduction. Reinspection of Figure 4.5.4 shows evidence of antegrade preexcitation just after the
adenosine interrupted tachycardia.
Comment: The identification of a tachycardia is easier when a 12-lead ECG can record its onset
and its interruption.

References
1. Coumel P, Attuel P. Reciprocating tachycardia in overt and latent preexcitation. Influence of functional bundle
branch block on the rate of the tachycardia. Eur. J. Cardiol. 1974;1(4):423–436.
2. Sung RJ, Castellanos A, Gelband H, et al. Mechanism of reciprocating tachycardia initiated during sinus rhythm in
concealed Wolff-Parkinson-White syndrome: Report of a case. Circulation. 1976;54(2):338–344.

Case 4.5  u  89
CASE
4.6
Frederick T. Han, MD
Melvin Scheinman, MD

Patient History
A 36-year-old female presented for treatment of daily palpitations and syncope. Her baseline ECG is
shown in Figure 4.6.1. Where is the pathway located, and what is the likely successful site of
ablation?

Figure 4.6.1  Baseline electrocardiogram.

Discussion
The baseline ECG reveals the presence of preexcitation with a negative delta wave in leads V1,
V3–V6, as well as leads I, III, and aVF. There is a positive delta wave in leads V2 and I. The negative
component of the delta wave in lead II (Figure 4.6.2, green arrow) and the positive component in V2

90  u  Section 4: Preexcitation Syndromes


(Figure 4.6.2, green arrow) is consistent with an inferior paraseptal epicardial accessory pathway,
likely from the coronary sinus.1 A unique feature of this ECG is the negative component of the delta
wave in leads V5–V6 (Figure 4.6.2, red arrow), although in some beats there is a small r wave in lead
V6. This suggests that the pathway has an apical ventricular insertion as opposed to a typical basal
ventricular insertion of the accessory pathway.

Figure 4.6.2  Baseline electrocardiogram analysis.

A subsequent electrophysiology study mapped the earliest Kent potential (Kp), localizing the
pathway location to the basal inferior paraseptal location 5 mm inside the coronary sinus; Figure
4.6.3 shows the Kp during 2:1 retrograde ventriculoatrial block in the accessory pathway and Figure
4.6.4 shows the retrograde Kp after induction of orthodromic atrioventricular reentrant tachycardia
with ventricular programmed stimulation at an interval of 600/350 ms. Mapping within the middle
cardiac vein from an apical to a basal location failed to identify a Kp. Ablation at the Kp within the
coronary sinus eliminated both anterograde and retrograde pathway conduction (Figure 4.6.5). The
unique negative delta wave in leads V5–V6 is consistent with the accessory pathway insertion into an
apical ventricular location. The proximal portion of the accessory pathway was successfully mapped
to the proximal coronary sinus. The work of Sun et al. has shown that posteroseptal accessory
pathways contain muscle sleeves that course within the cardiac veins.2 In this patient, the
ventricular insertion of the accessory pathway produces a QRS morphology consistent with a
ventricular tachycardia exiting from the apical crux.3

Case 4.6  u  91
This pathway most likely coursed from the atrium through the coronary sinus en route to the
apical ventricular crux. If this pathway could not be localized with endocardial mapping, epicardial
mapping and ablation has been reported to eliminate epicardial pathway conduction successfully.4–6

Figure 4.6.3  Intracardiac electrograms with Kent potential during ventricular pacing.

Ventricular pacing from the right ventricular outflow tract was associated with 2:1 retrograde
block (cycle length 580 ms). The green arrow identifies the Kent potential mapped to a location 5
mm within the coronary sinus. CS 17,18 was located at the coronary sinus ostium. The coronary
sinus catheter is a 20 pole catheter with 2-mm spacing. The RVa catheter was in the right
ventricular outflow tract.

92  u  Section 4: Preexcitation Syndromes


Figure 4.6.4  Intracardiac electrograms with Kent potential during supraventricular tachycardia.

Orthodromic atrioventricular reentrant tachycardia induced with ventricular programmed


extrastimulus testing at interval of 600/350 ms. The green arrow identifies the Kent potential
mapped to a location 5 mm within the coronary sinus. CS 17,18 was located at the coronary sinus
ostium. The coronary sinus catheter is a 20-pole catheter with 2-mm spacing. The RVa catheter was
in the right ventricular apex.

Case 4.6  u  93
Figure 4.6.5  Right anterior oblique and left anterior oblique views of accessory pathway location.

Kent potential at the site of successful ablation was mapped to a location 5 mm within the
coronary sinus (in the left anterior oblique [LAO] view) and the anterior aspect of the coronary
sinus (in the right anterior oblique [RAO] view).

References
1. Arruda MS, McClelland JH, Wang X, et al. Development and validation of an ECG algorithm for identifying
accessory pathway ablation site in Wolff-Parkinson-White syndrome. J. Cardiovasc. Electrophysiol. 1998;9:2–12.
2. Sun Y, Arruda M, Otomo K, et al. Coronary sinus-ventricular accessory connections producing posteroseptal
and left posterior accessory pathways: Incidence and electrophysiological identification. Circulation.
2002;106:1362–1367.
3. Kawamura M, Gerstenfeld EP, Vedantham V, et al. Idiopathic ventricular arrhythmia originating from the cardiac
crux or inferior septum: Epicardial idiopathic ventricular arrhythmia. Circ. Arrhythm. Electrophysiol. 2014;7:
1152–1158.
4. de Paola AA, Leite LR, Mesas CE. Nonsurgical transthoracic epicardial ablation for the treatment of a resistant
posteroseptal accessory pathway. Pacing Clin. Electrophysiol. 2004;27:259–261.
5. Sapp J, Soejima K, Couper GS, et al. Electrophysiology and anatomic characterization of an epicardial accessory
pathway. J. Cardiovasc. Electrophysiol. 2001;12:1411–1414.
6. Scanavacca MI, Sternick EB, Pisani C, et al. Accessory atrioventricular pathways refractory to catheter ablation: Role
of percutaneous epicardial approach. Circ. Arrhythm. Electrophysiol. 2015;8:128–136.

94  u  Section 4: Preexcitation Syndromes


CASE
4.7
Arnaud Denis, MD
Michel Haïssaguerre, MD, PhD

Patient History
An 18-year-old male, without medical history, was admitted to the intensive care unit for aborted
sudden cardiac death after a night of binge drinking and cannabis use. The initial cardiac rhythm
was ventricular fibrillation, and five shocks were later delivered by an automated external
defibrillator. Blood alcohol level was 1.8 g/L. Coronary angiography was normal.

Question
What is the main hypothesis to explain the aborted sudden cardiac death?

Figure 4.7.1  The first ECG at admission in the intensive care unit. Body temperature was 32.5°C.

Case 4.7  u  95
Discussion, Interpretation, and Answer
The basic rhythm is sinus at 75 bpm with a preexcitation (short PR interval and delta wave). There is
a posteroseptal accessory atrioventricular pathway (delta wave is isoelectric in V1, positive in V2 to
V6, D1 and aVL, negative in inferior leads).
There is an elevation of the J point in V3 to V6 and in the inferior leads that is, at this stage, most
likely related to hypothermia (Osborn J wave) rather than early repolarization syndrome.
The main hypothesis to explain the ventricular fibrillation is the rapid conduction of atrial
fibrillation (holiday heart syndrome) over the accessory pathway, which would have a very short
refractory period resulting in ventricular fibrillation.

Figure 4.7.2  ECG 3 days after aborted sudden cardiac death. Body temperature was normal.

The rhythm is sinus at 100 bpm. The posteroseptal accessory pathway (AP) is present on the
first nine beats. Then the preexcitation suddenly disappears (solid arrow; prolongation of the PR
interval, apparition of a Q wave of septal activation in leads V5 and V6, smaller R wave in leads V2
and V3). Note the difference in repolarization between the preexcited QRS and the nonpreexcited
QRS.
Therefore, the AP is intermittent, which suggests the patient is at a low risk of rapid conduction
over the pathway and life-threatening ventricular arrhythmias in response to AF.1

96  u  Section 4: Preexcitation Syndromes


Figure 4.7.3

The delta wave in inferior leads is now positive (solid arrow), whereas it was negative in the
previous ECG. Two hypotheses can be mentioned:
• The existence of a second AP.
• Variable degree of preexcitation because of fusion between conduction over the posteroseptal
pathway and through the AV node.
In the inferior leads, the initial vector of the delta wave is completely changed (negative in
Figure 4.7.1 and Figure 4.7.2, positive in Figure 4.7.3). If it was a fusion between the posteroseptal AP
and conduction through the AV node, the initial vector of the delta wave should not be changed.
This ECG is in favor of the existence of a second intermittent AP.

Case 4.7  u  97
Figure 4.7.4  ECG during adenosine injection.

1. The first three QRS are not preexcited. The PR interval is getting longer due to the adenosine
injection.
Solid arrow: retrograde P wave (negative in inferior leads). As the PR interval is prolonged by
adenosine, retrograde conduction through the AP is possible because the atrium is no longer
refractory.
2. 2:1 AV block due to the adenosine injection and four wide QRS complexes with a stable
relationship with the preceding P wave. The morphology of these four QRS complexes are not
compatible with the posteroseptal AP because it is not negative in the inferior leads.
3. Wide QRS with a different morphology (positive in inferior leads and isoelectric in D1)
proceeded by a sinus P wave.
Two different hypotheses can explain these five wide QRS complexes:
• Two other intermittent APs (a left posterior and a left anterolateral AP). In this hypothesis,
P waves are conducted intermittently only through pathways because the AV node is blocked
by adenosine (2:1 AV block).
• Two different morphologies of PVCs triggered by the adenosine that are by chance just after
the P wave. The second hypothesis is of course less probable.
4. P wave is conducted 1:1 to the ventricle through the left posteroseptal AP without fusion in
conjunction with conduction through the AV node.

98  u  Section 4: Preexcitation Syndromes


Figure 4.7.5

In order to confirm the existence of three different intermittent APs, adenosine injection was
performed again.
1. Two P waves are conducted with fusion between their conduction through the AV node and the
left posterior AP (as described in Figure 4.7.3) and one P wave block in both the pathways and in
the AV node.
2. 1:1 conduction through the posteroseptal AP without fusion because the AV node is blocked by
adenosine. This is followed by a P wave which is blocked in the AV node and the three APs.
3. The second adenosine injection reproduced the same two different wide QRS morphologies that
support the hypothesis of the existence of multiple APs.
• P waves conducted only through the left posterior AP (dotted arrow).
• P waves conducted only through the left anterolateral AP (solid arrow).
This patient with aborted sudden cardiac death has three intermittent APs. Intermittent AP is
associated with low risk of rapid conduction over the pathway and life-threatening ventricular
arrhythmias in response to AF. However, multiple pathways are associated with increased risk of
sudden cardiac death.1
The most probable hypothesis to explain sudden cardiac death in this young male is rapid
conduction of atrial fibrillation (favored by alcohol intake) over three intermittent APs, resulting in
ventricular heterogeneity and ventricular fibrillation.
This case emphasizes the importance of closely investigating the morphology of delta waves on
12-lead ECG recordings (long printouts) to highlight the existence of multiple APs.

Case 4.7  u  99
Reference
1. Page RL, Joglar JA, Caldwell MA, et al. 2015 ACC/AHA/HRS guideline for the management of adult patients with
supraventricular tachycardia: A report of the American College of Cardiology/American Heart Association Task
Force on Clinical Practice Guidelines and the Heart Rhythm Society. Circulation. 2016;133:e471–e505.

100  u  Section 4: Preexcitation Syndromes


CASE
Henry H. Hsia, MD 4.8
Patient History
A 72-year-old male with episodes of irregular rapid palpitations associated with dizziness and near
syncope. He has a long history of tachycardia with progressively worsening symptoms over the last
10 years. An exercise stress test showed no evidence of myocardial infarction or stress-induced
ischemia. An echocardiogram revealed moderate left atrial and mild right atrial dilations, with
normal ventricular function and wall motion. Mild-to-moderate mitral regurgitation was noted.

Question
What is the diagnosis?

Figure 4.8.1

Case 4.8  u  101


Figure 4.8.2

102  u  Section 4: Preexcitation Syndromes


Figure 4.8.3

During symptomatic palpitations and patient-triggered events, Holter monitoring showed rapid,
regular, narrow complex supraventricular tachycardia (SVT) at rates of 225–235 bpm (Figure
4.8.1B), as well as sustained wide QRS complex tachycardias at 184–218 bpm (Figure 4.8.2A and
4.8.2B). He was diagnosed to have 1-to-1 atrial flutter and ventricular tachycardia (VT) and was
referred for atrial flutter ablation and VT management.

Interpretations and Discussion


Figure 4.8.1A showed atrial fibrillation without evidence of preexcitation. Occasional wide complex
beats suggestive of ventricular ectopy/nonsustained ventricular tachycardia (NSVT) were observed.
Figure 4.8.2A showed episodes of atrial flutter with variable AV block. Intermittent, regular
wide complex tachycardia was recorded at 150 bpm, at twice the underlying atrial flutter rate.
Figure 4.8.2B demonstrated a sustained wide complex tachycardia at ~188 bpm (320 ms cycle
length) with a different QRS morphology, suggestive of a left bundle branch block (LBBB) pattern.
This tachycardia spontaneously converted to a narrow complex SVT at a faster rate ~215 bpm (280
ms cycle length).
Although a “dual-tachycardia” with simultaneous SVT and VT was considered, the lack of
fusion beat makes the diagnosis less likely. The presence of aberrant conduction affecting the

Case 4.8  u  103


tachycardia rate suggests the involvement of His-Purkinje system as part of the tachycardia
mechanism, excluding AV nodal reentrant tachycardia or atrial tachyarrhythmias (with no
infranodal involvement). Slowing of the SVT with the development of LBBB (or acceleration with
LBBB resolution) implies the left bundle is part of the SVT circuit (Figure 4.8.2C) and thus confirms
the diagnosis of an orthodromic atrioventricular reentrant tachycardia (AVRT) involving a left-
sided accessory pathway. Upon further inspection, a subtle delta wave was noted in sinus rhythm
(Figure 4.8.2B).
Preexcitation was rarely observed during atrial fibrillation (Figures 4.8.1A and 4.8.3A), and was
only present at 2-to-1 conduction during atrial flutter at 150 bpm (Figure 4.8.2A). This suggests a
poor antegrade conduction property of the accessory pathway and identifies a low-risk patient for
sudden death in preexcitation syndrome.
Despite his age, the patient has both atrial fibrillation/flutter and symptomatic rapid
orthodromic AVRT involving a left-sided accessory pathway. Evidence of subtle preexcitation was
documented on another ECG (Figure 4.8.3B). He underwent a successful catheter ablation for both
right atrial flutter and the left lateral accessory pathway.

Reference
1. Coumel P, Attuel P. Reciprocating tachycardia in overt and latent pre-excitation: Influence of functional bundle
branch block on the rate of the tachycardia. Eur. J. Cardiol. 1974;1:423–443.

104  u  Section 4: Preexcitation Syndromes


CASE
Robert Lemery, MD 4.9
Patient History
A 43-year-old patient with a history of palpitations had an ECG (Figure 4.9.1) during complaints of
tachycardia. The tracing shows runs of wide complex tachycardia, suggesting VT. However, the
tracing shows that the narrow QRS complexes are not preceded by a P wave, suggesting possibly
SVT or AF. Furthermore, the wide complex tachycardia is irregular, with a distinctive pattern
(negative QRS complex in the inferior leads, positive in leads I and AVL, with a transition from
negative to positive in V1–V2). The following tracing shows narrow complex tachycardia
(Figure 4.9.2). The VA interval is prolonged, and the P wave is clearly seen in the inferior leads, and
is also suggested in V1.

Figure 4.9.1  Electrocardiogram showing atrial fibrillation with intermittent conduction over an accessory pathway in a patient with
Wolff-Parkinson-White syndrome.

Case 4.9  u  105


Figure 4.9.2  The ECG shows narrow complex tachycardia (see text).

Question
The electrocardiogram suggests the patient most likely has:
1. Ventricular tachycardia
2. Atrial fibrillation with intermittent conduction over a multiple accessory pathways
3. Atrial fibrillation with intermittent conduction over a left sided accessory pathway
4. Atrial fibrillation with intermittent conduction over a postero-septal accessory pathway

Answer
4

Discussion
The subsequent tracing (Figure 4.9.3) shows resolution of SVT after a bolus of intravenous
adenosine. Following antegrade block over the AV node, there is resumption of sinus rhythm, with a
preexcited beat in lead V2 that is positive, with the same vector of the QRS complex as the initial
tracing during wide complex tachycardia. The following ECG (Figure 4.9.4) shows preexcitation,
suggestive of a posteroseptal accessory pathway. Although the 12-lead ECG has limitations in being
able to predict accurate localization of the bypass tract, algorithms have been useful at determining
general locations of accessory pathways.1-3 The pattern of the retrograde P wave during SVT has
also been reported to predict localization of the accessory pathway.4,5

106  u  Section 4: Preexcitation Syndromes


Figure 4.9.3  Termination of tachycardia after a bolus of intravenous adenosine, followed by a preexcited beat.

Figure 4.9.4  In sinus rhythm, the electrocardiogram shows preexcitation consistent with a posteroseptal accessory pathway.

Case 4.9  u  107


The patient underwent radiofrequency (RF) ablation using remote magnetic navigation, with
loss of preexcitation with the first application of RF. A total of five applications were delivered for
persistent block over the accessory pathway. The postablation ECG (Figure 4.9.5) shows sinus
rhythm with loss of preexcitation. The tracing also shows negative T-wave changes, caused by
T-wave memory, with the localization of the negative T waves matching the leads showing a
negative QRS complex.6

Figure 4.9.5  Following radiofrequency ablation of the accessory pathway, the electrocardiogram shows loss of preexcitation, with
negative T waves in the inferior leads (see text).

References
1. Lemery R, Hammil S, Wood D, et al. Value of the resting 12-lead electrocardiogram and vectocardiogram for
locating the accessory pathway in patients with the Wolff-Parkinson-White syndrome. Br. Heart J. 1987;58:324–332.
2. Arruda MS, McClelland JH, Wang X, et al. Development and validation of an ECG algorithm for identifying
accessory pathway ablation site in Wolff-Parkinson-White syndrome. J. Cardiovasc. Electrophysiol. 1998;9:2–12.
3. Josephson ME. Josephson’s Clinical Cardiac Electrophysiology. 5th ed. Philadelphia, PA: Wolters Kluwer; 2016.
4. Puech P, Grolleau R, Cinca J. Reciprocating tachycardia using a latent left-sided accessory pathway: Diagnostic
approach to conventional ECG. In: Kulbertus, Ed. Reentrant Arrhythmia: Mechanisms and Treatment. Lancaster,
U.K.: MTP, 1977:117–131.
5. Tai CT, Chen SA, Chiang CE, et al. A new electrocardiographic algorithm using retrograde P waves for
differentiating atrioventricular node reentrant tachycardia from atrioventricular reciprocating tachycardia
mediated by concealed accessory pathway. J. Am. Coll. Cardiol. 1997;29:394–402.
6. Ghosh S, Rhee EK, Avari JN, Woodard PK, Rudy Y. Cardiac memory in patients with Wolff-Parkinson-White
syndrome. Noninvasive imaging of activation and repolarization before and after catheter ablation. Circulation.
2008;118:907–915.

108  u  Section 4: Preexcitation Syndromes


CASE
Mohamed Magdy, MSc, L'AFSA, PhD, MD 4.10
Patient History
An 18-year-old male is admitted to the emergency department with recurrent syncope attacks.

Figure 4.10.1  Left lateral accessory pathway and normalization during ablation.

Case 4.10  u  109


Figure 4.10.2  Postablation ECG with atrial pacing showing conduction over the accessory with atrial pacing.

Figure 4.10.3  Normalization of ECG during ablation.

110  u  Section 4: Preexcitation Syndromes


Figure 4.10.4  Atrioventricular normalization during ablation.

Figure 4.10.5  No more delta waves with atrial pacing.

Case 4.10  u  111


Figure 4.10.6  Retrograde ventricular pacing showing 2:1 conduction and no more accessory pathway.

112  u  Section 4: Preexcitation Syndromes


CASE
4.11
Dan Blendea, MD, PhD
Moussa Mansour, MD

Patient History
A 23-year-old male baseball player with preexcitation pattern on the electrocardiogram presented
for an electrophysiology study and possible ablation. The patient had a history of occasional
palpitations but no documented supraventricular tachycardia. The baseline electrocardiogram is
shown in Figure 4.11.1. The intracardiac electrograms at baseline and during atrial pacing are
shown in Figures 4.11.2, 4.11.3, and 4.11.4. No tachyarrhythmias were inducible during the EP study.

Figure 4.11.1  Baseline 12-lead ECG showing ventricular preexcitation with a PR interval of 110 ms, delta waves that were negative in
V1–V3 (pattern similar to Wolff-Parkinson-White type B), and a QRS duration of 104 ms.

Case 4.11  u  113


Figure 4.11.2  Intracardiac electrograms confirming ventricular preexcitation with a short HV interval = 26 ms.

Figure 4.11.3  Atrial pacing (first beat) prolongs the PR owing to AV nodal delay (increase in AH) but does not change the HV and the
degree of preexcitation.

114  u  Section 4: Preexcitation Syndromes


Figure 4.11.4  Junctional rhythm with preexcitation.

Questions
1. What is the diagnosis?
2. No ablation was performed. Why?

Answers
1. The diagnosis is supported by an increase in AH with no change in preexcitation pattern with
rapid atrial pacing. The absence of supraventricular tachycardia in this case does not necessarily
support the diagnosis but is consistent with fasciculoventricular accessory pathway (FVP).
2. Typically, these pathways do not participate in reentrant tachycardias or sudden death.

Discussion and Interpretation


The findings in this case are suggestive of a FVP. Electrophysiologic findings suggest FVPs insert
into the septal region. The 12-lead surface ECG during sinus rhythm is similar to the ECG of
patients with anteroseptal and midseptal bypass tracts.1 Although FVP is considered rare, the
results of a recent study indicate that many Wolff–Parkinson–White (WPW) syndrome patients
with a QRS width ≤120 ms may actually have FVP. Patients categorized as type B are more likely to
have FVP, whereas type A patients are most likely to have WPW syndrome.2 In our case, the QRS
was 104 ms and the electrocardiographic pattern was similar to WPW type B with a negative delta
wave in V1 and V2.

Case 4.11  u  115


References
1. Sternick EB, Rodriguez LM, Gerken LM, et al. Electrocardiogram in patients with fasciculoventricular pathways:
A comparative study with anteroseptal and midseptal accessory pathways. Heart Rhythm. 2005;2(1):1–6.
2. Suzuki T, Nakamura Y, Yoshida S, et al. Differentiating fasciculoventricular pathway from Wolff-Parkinson-White
syndrome by electrocardiography. Heart Rhythm. 2014;11(4):686–690.

116  u  Section 4: Preexcitation Syndromes


CASE
Robert J. Myerburg, MD 4.12
Patient History
This 46-year-old male experienced the sudden onset of a very rapid tachycardia, accompanied by
severe lightheadedness, while at a cocktail party. He had never had a previous episode of
tachycardia nor cardiac palpitations of any type. He had no history of heart disease and was not
diabetic, but did have a history of moderately elevated blood pressure, for which he had been treated
intermittently. His blood pressures were in the range of 145–155/80–90 mmHg when off of therapy.
His alcohol intake had not been excessive on the evening of this event.
Because of the lightheadedness and weakness, his wife drove him to a nearby emergency
department, where the following rhythm strip was recorded:

Figure 4.12.1  Baseline rhythm obtained in emergency department.

His blood pressure was 90/60 mmHg upon arrival in the ED, and he remained lightheaded and
was diaphoretic. He underwent immediate DC cardioversion, and the post-cardioversion ECG
follows.

Figure 4.12.2  ECG after cardioversion.

Case 4.12  u  117


Based upon the presence of delta waves and the PR interval, the diagnosis of Wolff–Parkinson–
White (WPW) syndrome was evident, and he was referred to a cardiac electrophysiologist for a
catheter ablation procedure. An ECG recorded after the procedure follows.

Figure 4.12.3  ECG after radiofrequency ablation procedure. (Used with permission from Junttila MJ, Castellanos A, Huikuri HV, et al. Risk
markers of sudden cardiac death in standard 12-lead electrocardiograms. Ann Med. 2012;44:717–732.)

Question
Which is the following is correct?
A. The ECG recorded during the tachycardia suggests that antegrade conduction is occurring in the
normal pathway.
B. The rate of the tachycardia requires that the accessory pathway have a very short refractory
period.
C. The tachycardia shown is most likely due to AV reentry, with antegrade conduction in the
accessory pathway and retrograde conduction in the normal (His-Purkinje > AV node) pathway.
D. In the majority of patients with WPW syndrome who have atrial fibrillation, conduction during
atrial fibrillation leads in the accessory pathway.
E. The pattern of this arrhythmia requires that both antegrade and retrograde conduction can
occur in the accessory pathway.

Discussion, Interpretation, and Answers


The arrhythmia shown is atrial fibrillation in a patient with WPW syndrome and an accessory
pathway having a very short refractory period. This is a potentially life-threatening arrhythmia,
since the rate of ventricular activation can be sufficient to induce ventricular fibrillation.
In the majority of instances of atrial fibrillation in patients with WPW, conduction proceeds
down the normal pathway because the refractory period of the accessory pathway tends to be
longer. In these cases, the QRS normalizes during atrial fibrillation, although rate-related aberrancy
may occur. It is only in those cases in which the refractory period of the accessory pathway is very
short that a rapid, wide QRS tachycardia of the type shown can occur. Therefore, the correct
answer is B.
For the common form of AV reentrant tachycardia in WPW, retrograde conduction is necessary,
but it is not necessary that retrograde conduction be intact for the pattern shown in this tracing.
Retrograde activation of the atrium in WPW can initiate atrial fibrillation, but that is not a

118  u  Section 4: Preexcitation Syndromes


requirement for this specific arrhythmia, and the individual prone to atrial fibrillation independent
of the WPW pattern, who has a short refractory period accessory pathway, is susceptible. Patients
with only antegrade accessory pathway conduction will have delta waves, but no history or
inducibility of paroxysmal AV reentrant tachycardias. Their arrhythmia risk is solely atrial
fibrillation.

Case 4.12  u  119


CASE
4.13 Philip Podrid, MD

Patient History
A 29-year-old female without any known cardiac disease and no cardiac symptoms is seen by her
primary care physician for a routine physical examination prior to training for a marathon. An ECG
is obtained (Figure 4.13.1), and as a result she is sent to a local emergency department where the
ECG is repeated (Figure 4.13.2).

Diagnosis
Wolff-Parkinson-White, probable left lateral accessory pathway, and concertina effect.

Questions
1. What is the etiology of the rhythm in Figure 4.13.1?
2. What accounts for the changing PR interval and changing QRS complex duration in
Figure 4.13.2?

Figure 4.13.1

120  u  Section 4: Preexcitation Syndromes


Figure 4.13.2

Discussion
In Figure 4.13.1, there is a regular rhythm at a rate of 54 bpm. There are no P waves seen before
or after any of the QRS complexes. The QRS complexes are wide (0.14 seconds), especially at the
base, as a result of a slurring of the upstroke. They have an abnormal morphology which does
not resemble either a typical right or left bundle branch block or an intraventricular ventricular
conduction delay. Therefore, it is not likely a result of typical aberration. In addition, there are
tall R waves in leads V1–V6 (←) which is not seen when there is conduction through the normal
His-Purkinje system; rather this indicates direct myocardial activation. This pattern (slurred
upstroke and positive concordance) is seen with a ventricular complex or Wolff–Parkinson–
White (WPW). A WPW pattern cannot be seen in the absence of initial atrial activity as the
accessory pathway begins in the atria. Therefore, the leading diagnosis would be an accelerated
idioventricular rhythm (AIVR). The axis is normal between 0° and +90° (positive QRS complex
in leads I and aVF). The QT/QTc intervals are slightly prolonged (480/450 ms) but are normal
when the prolonged QRS complex duration is considered (440/420 ms).

In Figure 4.13.2, there is a regular rhythm with a rate of 72 bpm at the beginning of the tracing, but
then the rate is regular at 60 bpm. The QRS complex morphology is the same throughout the ECG
and is identical to the QRS complexes seen in Figure 4.13.1, i.e., the complex is wide at the base with
a slurring of the upstroke (^). However, there is a change in the QRS width, which is initially 0.12
seconds (when the rate is faster) as a result of less slurring of the upstroke and a narrower base of

Case 4.13  u  121


the QRS complex. When the rate is slower, the QRS width increases to 0.14 seconds, identical to the
width seen in Figure 4.13.1. The widening is the result of a broader base of the complex due to more
slurring of the upstroke. In addition to the change in the QRS width it can be seen that the first six
QRS complexes are preceded by a P wave (+) while there is no P wave before the last five QRS
complexes. When the P wave is present, the PR interval is constant but very short (0.08 seconds)
without any PR segment. The short PR interval and widened QRS complex due to slurring of the
upstroke, termed a delta wave, is characteristic of a WPW pattern. As all of the QRS complexes are
identical, they all manifest a WPW pattern despite the absence of an initial P wave associated with
the last five complexes and all of the complexes in Figure 4.13.1. The change in the PR interval (i.e.,
shortening in this case) along with a change in the QRS complex width (i.e., widening in this case)
in WPW is termed the concertina effect. This occurs as a result of changes in conduction velocity
through the AV node. Importantly with WPW, the QRS complex is a fusion complex, resulting from
early ventricular activation via the accessory pathway (preexcitation). This direct myocardial
activation is slower than conduction through the His-Purkinje system and produces the slurred
upstroke or delta wave. There is also impulse conduction through the normal AV node His-Purkinje
system which occurs slightly later as a result of the slow conduction through the AV node. The
impulse generated via the normal pathway fuses with the impulse generated via the accessory
pathway. As conduction velocity through the accessory pathway is fixed (i.e., all or none) the AV
node controls the amount of preexcitation. When AV nodal conduction is slow, more of the
myocardium will be activated by the impulse coming through the accessory pathway. This results
in a wider delta wave and shorter PR interval. When AV nodal conduction is fast, less myocardium
will be activated by the impulse via the accessory pathway and hence there is less of a delta wave
and a longer PR interval. Changes in AV nodal conduction may occur as a result of changes in
vagal-sympathetic tone and such changes are frequent during the day and from day to day. In this
case, it is possible that there was sinus arrhythmia, accounting for the changes in heart rate from
72 to 60 bpm. The slower heart rate, due to enhanced vagal tone, resulted in more preexcitation and
as a result the P wave is not seen, as it is likely within the QRS complex (or delta wave) due to the
rapid AV conduction.

122  u  Section 4: Preexcitation Syndromes


CASE
4.14
Magdi M. Saba, MD
David E. Ward, MD

Figure 4.14.1

Patient History
A 65-year-old male with no previous history of structural heart disease presented to the emergency
department with a wide complex tachycardia (ECG) and near-syncope. The patient had a normal
echocardiogram.

Question
What is the differential diagnosis?

Answer
Ventricular tachycardia or antidromic atrioventricular tachycardia. The patient had a normal
baseline ECG, with no preexcitation.
A clinical diagnosis of ventricular tachycardia, arising near the mitral annulus, was made.

Case 4.14  u  123


Figure 4.14.2

Question
What mechanism does this intracardiac tracing rule out?

Answer
A regular wide complex tachycardia was induced with atrioventricular (AV) dissociation. Atrial
fibrillation is seen (coronary sinus recordings, top two traces) during a regular wide complex
tachycardia. This rules out any reentrant mechanism involving the atrium, namely AV reentrant
tachycardia (AVRT).

124  u  Section 4: Preexcitation Syndromes


Figure 4.14.3

During the electrophysiology study, atrial pacing resulted in occasional wide complex pre-
excited beats similar, but not identical, to the wide complex rhythm. The ability to reproducibly
capture the ventricle with a QRS morphology similar to that of the wide complex rhythm (arrows)
strongly suggests a communication between the atrium and ventricle via the AV node but not the
normal conduction system.
The mechanism is a left-sided nodoventricular pathway-mediated reentrant tachycardia, rather
than a mitral annular ventricular tachycardia.

Question
Can this be considered a form of ventricular tachycardia?

Case 4.14  u  125


Figure 4.14.4  Right anterior oblique view of the catheter position at ablation site.

Figure 4.14.5  Left anterior oblique view of the catheter at ablation site.

126  u  Section 4: Preexcitation Syndromes


CASE
4.15
Zian H. Tseng, MD, MAS
Melvin Scheinman, MD

Patient History
A 24-year-old medical student reports to the emergency department due to palpitations. There is no
history of cardiac disease.

Figure 4.15.1

Most Likely Diagnoses


• Torsades de pointes
• Atrial fibrillation with right and left bundle branch block (RBBB/LBBB)
• Polymorphic ventricular tachycardia (VT)
• Atrial fibrillation with multiple pathways
• Atrial fibrillation and VT

Case 4.15  u  127


Figure 4.15.2  After cardioversion.

Optimal Initial Therapy in Emergency Department


• IV Digoxin
• IV Verapamil
• IV Procainamide
• IV Amiodarone

Optimal Chronic Therapy


• Catheter ablation of accessory pathways
• Flecainide and beta-blockers
• Amiodarone
• Dofetilide

128  u  Section 4: Preexcitation Syndromes


Figure 4.15.3  After ablation of left lateral and decremental right lateral AP.

Discussion
Figure 4.15.1 shows an irregular wide complex tachycardia of two basic forms. Note from lead V1
there is an LBBB superior axis form as well as an RBBB with inferior axis. These forms do not show
a typical RBBB or LBBB pattern and with an initial slurred upstroke are not likely to be atrial
fibrillation with aberrancy. There are also short periods in the middle of the tracing with a more
narrow pattern suggestive of RBBB aberration. These tracings in an otherwise healthy young person
would be most compatible with Wolff-Parkinson-White Syndrome and rapid conduction over two
pathways.
Initial therapy should include the use of IV procainamide (or ibutilide) followed by direct
current conversion if there is no prompt response to therapy. This patient underwent an
electrophysiology study and was found to have two pathways; one at the anterolateral mitral
annulus, and the other was a slow conducting right lateral accessory pathway. Note that after acute
conversion to sinus rhythm the ECG pattern resembles the presence of a septal accessory pathway
(Figure 4.15.2). The most likely explanation for this pattern is a fusion between both left- and right-
sided accessory pathways.

Teaching Points
1. Atrial fibrillation is one of the best tools to suggest the presence of dual accessory pathways.
2. Atrial fibrillation in the patient with robust accessory pathway function results in a life
threatening situation demanding rapid attention, and the drugs of choice are IV procainamide or
ibutilide followed by prompt cardioversion if drugs are ineffective.
3. Catheter ablation of the pathways is mandatory in view of the malignant potential.

Case 4.15  u  129


CASE
4.16
Mohammad Shenasa, MD
Mariah Smith

Patient History
A 23-year-old male soccer player referred due to an abnormal ECG. He was asymptomatic with no
history of palpitations, syncope, or near-syncope and had a negative family history.
Heart rate: 59 bpm
PR interval: 98 ms
QRS: 164 ms
QT/QTc: 462/460 ms

Figure 4.16.1

Question
What is the ECG diagnosis?
1. Left bundle branch block
2. Left ventricular hypertrophy
3. Ventricular preexcitation with left anterior accessory pathway

Answer
Ventricular preexcitation with left anterior accessory pathway. Note the very short PR interval of
98 ms. ST-T wave abnormalities are consistent with preexcitation.

130  u  Section 4: Preexcitation Syndromes


Discussion
Radiofrequency ablation of accessory pathway is a reasonable option; however, the patient is
completely asymptomatic. Because of his level of exercise during sports participation, he may have
supraventricular tachycardias that can potentially cause syncope and in rare cases, ventricular
fibrillation. Since he has been playing for the last three years in a national team and had no events,
he has already proven to have a low risk profile. Two studies were recently published on the role of
radiofrequency catheter ablation in patients with Wolff-Parkinson-White (WPW) syndrome.1-3 The
current data from the references outlined below does not support a universal prophylactic use of
catheter ablation of asymptomatic patients with WPW syndrome.
There is insufficient data to be dogmatic about an athlete with asymptomatic WPW. Many
experts would do an electrophysiology study to define the refractory period of the bypass tract, see
if orthodromic reentrant tachycardia (ORT) is inducible, as well as atrial fibrillation with the fastest
preexcited RR interval. If there is short effective refractory period (ERP) (<250) or ORT, ablation
would be recommended. If there was an abrupt loss of delta wave on the surface ECG with the
ETT (with loss of the delta wave), this could be used as a surrogate of relatively long ERP (personal
communication with Mark Estes, MD). For further information, readers are encouraged to read
reference 6.

References
1. Pappone C, Vicedomini G, Manguso F, et al. Wolff-Parkinson-White syndrome in the era of catheter ablation:
Insights from a registry study of 2169 patients. Circulation. 2014;130(10):811–819.
2. Obeyesekere MN, Klein GJ. The asymptomatic Wolff-Parkinson-White patient: time to be more proactive?
Circulation. 2014;130(10):805–807.
3. Chevalier P, Cadi F, Scridon A, et al. Prophylactic radiofrequency ablation in asymptomatic patients with Wolff-
Parkinson-White is not yet a good strategy: a decision analysis. Circ. Arrhythm. Electrophysiol. 2013;6(1):185–190.
4. Obeyesekere MN, Leong-Sit P, Massel D, et al. Risk of arrhythmia and sudden death in patients with asymptomatic
preexcitation: A meta-analysis. Circulation. 2012;125(19):2308–2315.
5. Laks MM. On the need for a universal prospective ECG database. Circulation. 2012;125(19):2288–2290.
6. Pediatric and Congenital Electrophysiology Society (PACES); Heart Rhythm Society (HRS); American College of
Cardiology Foundation (ACCF), et al. PACES/HRS expert consensus statement on the management of the
asymptomatic young patient with a Wolff-Parkinson-White (WPW, ventricular preexcitation) electrocardiographic
pattern: Developed in partnership between the Pediatric and Congenital Electrophysiology Society (PACES) and the
Heart Rhythm Society (HRS). Endorsed by the governing bodies of PACES, HRS, the American College of
Cardiology Foundation (ACCF), the American Heart Association (AHA), the American Academy of Pediatrics
(AAP), and the Canadian Heart Rhythm Society (CHRS). Heart Rhythm. 2012;9(6):1006–1024.

Case 4.16  u  131


CASE
4.17
Mohammad Shenasa, MD
Hossein Shenasa, MD, MS

Patient History
An 81-year-old male presented to the emergency department with palpitations and near-syncope.
Heart rate: 60 bpm
PR interval: 138 ms
QRS duration: 156 ms
QT/QTc: 444/444 ms

Figure 4.17.1

Figure 4.17.1 shows ventricular preexcitation with a left posterior accessory pathway. The patient
underwent radiofrequency ablation of accessory pathway. His post ablation ECG is shown in
Figure 4.17.2.

132  u  Section 4: Preexcitation Syndromes


Heart rate: 65 bpm
PR interval: 192 ms
QRS: 146 ms
QT/QTc: 448/466 ms

Figure 4.17.2

Figure 4.17.2 shows sinus rhythm, complete left bundle branch block (LBBB) pattern; the PR
interval is longer compared to Figure 4.17.1.

Discussion
This patient has an established LBBB pattern and a manifest left-sided accessory pathway, which
masked the LBBB. Once the accessory pathway has been ablated, the LBBB is then visible.1 The
effect of manifest preexcitation in a patient with ipsilateral LBBB depends on the degree of fusion
and the arrival of impulses from the normal atrioventricular conduction and the accessory pathway.
A similar phenomenon has been reported previously in patients with ipsilateral right bundle branch
block.2

References
1. Mendoza I, Castellanos A, Sung R. Wolff-Parkinson-White syndrome type B with tachycardia-dependent (phase 3)
block in the accessory pathway and in left bundle-branch coexisting with rate-unrelated right bundle-branch block.
Br. Heart J. 1980;43(4):43481–43486.
2. Castillo C, Castellanos A, Befeler B, et al. Arrival of excitation at right ventricular apical endocardium in
Wolff-Parkinson-White syndrome type A, with and without right bundle-branch block. Br. Heart J.
1973;35(6):35594–35600.

Case 4.17  u  133


SECTION 5
Early Repolarization (ECG Pattern and the Syndrome)

CASE
N. A. Mark Estes III, MD 5.1
Patient History
ECG of a 26-year-old male with recurrent syncope who was resuscitated after experiencing cardiac
arrest while driving.

Questions
1. What is the ECG abnormality?
2. What is the likely cause of cardiac arrest?

Figure 5.1.1

Discussion
The ECG shows a pattern of early repolarization (ER) in the inferolateral leads, which is associated
with a higher risk of cardiac arrest. The current classification of early repolarization patterns on the
ECG is as follows:
• Type 1 is associated with ER in the lateral precordial leads. This form is common among healthy
male athletes and is thought to be largely benign.

ECG Masters’ Collection: Favorite ECGs from Master Teachers Around the World © 2017 Mohammad Shenasa, Mark E. Josephson,
N. A. Mark Estes III, Ezra A. Amsterdam, Melvin Scheinman. Cardiotext Publishing, ISBN: 978-1-942909-08-8.   135
• Type 2 is associated with ER in the inferior or inferolateral leads and is associated with a
moderate level of risk.
• Type 3 is associated with ER globally in the inferior, lateral, and right precordial leads, and
appears to be associated with the highest relative risk, though the absolute risk of sudden death
remains small.
• Type 4, or Brugada syndrome, is marked by J-wave/point elevation in the right precordial leads.

136  u  Section 5: Early Repolarization (ECG Pattern and the Syndrome)


CASE
Majid Haghjoo, MD 5.2
Patient History
An asymptomatic 25-year-old male athlete was referred for electrophysiologic evaluation. He had
no history of sudden cardiac death in his family. The following ECG was recorded.

Figure 5.2.1  Benign early repolarization ECG pattern. Note the prominent J waves, especially leads V4–V6, rapidly ascending ST-segment
elevations, and slurring (blue arrows).

Case 5.2  u  137


Discussion
Early repolarization (ER) has long been considered as a benign electrocardiographic phenomenon.
The first example of malignant variant of ER was reported 15 years ago.1 It has been demonstrated
that prognosis of a J wave with a rapidly ascending pattern is similar to persons without ER, whereas
horizontal pattern and presence of ER in inferior leads is associated with increased arrhythmic
risk.2

References
1. Kalla H, Yan GX, Marinchak R. Ventricular fibrillation in a patient with prominent J (Osborn) waves and ST
segment elevation in the inferior electrocardiographic leads: A Brugada syndrome variant? J. Cardiovasc.
Electrophysiol. 2000;11:95–98.
2. Tikkanen JT, Junttila MJ, Anttonen O, et al. Early repolarization: Electrocardiographic phenotypes associated with
favorable long-term outcome. Circulation. 2011;123:2666–2673.

138  u  Section 5: Early Repolarization (ECG Pattern and the Syndrome)


Juhani Junttila, MD, PhD CASE
5.3
Tuomas Kenttä, PhD
Heikki V. Huikuri, MD, PhD

Patient History
A previously healthy, 21-year-old male college athlete went through routine preparticipation
screening with 12-lead ECG (Figure 5.3.1). He had no history of palpitations or syncope and there
was no history of sudden cardiac death in the family.

Question
Does the young athlete have an increased risk for sudden death according to the ECG?

Figure 5.3.1

Case 5.3  u  139


Answer
According to current knowledge, it seems highly unlikely that the lateral ER pattern with rapidly
ascending ST segment presented in the ECG of our patient would increase his risk for sudden
cardiac death.

Discussion
Inferolateral early repolarization (ER) is very prevalent among young athletes, especially among
African-American athletes. The prevalence of inferolateral ER among college athletes is 30%
according to a study by the University of Miami. A majority of ER (~90%) is accompanied by rapidly
ascending ST segment after the J-point elevation. This pattern is associated with a benign outcome
in middle-aged individuals and can be also considered benign in asymptomatic young athletes.

References
1. Junttila MJ, Sager SJ, Freiser M, et al. Inferolateral early repolarization in athletes. J. Interv. Card. Electrophysiol.
2011;31:33–38.
2. Tikkanen JT, Junttila MJ, Anttonen O, et al. Early repolarization: Electrocardiographic phenotypes associated with
favorable long-term outcome. Circulation. 2011;123:2666–2673.

140  u  Section 5: Early Repolarization (ECG Pattern and the Syndrome)


Juhani Junttila, MD, PhD CASE
5.4
Tuomas Kenttä, PhD
Heikki V. Huikuri, MD, PhD

Patient History
A 45-year-old male comes to a routine exam. He is asymptomatic. He is a current smoker, has a
history of hypertension, and his cholesterol levels are elevated. He has a family history of coronary
disease. His 12-lead ECG is presented below (Figure 5.4.1).

Questions
1. Are there any additional risk markers of cardiac death and sudden cardiac death?
2. What would you do in regards of treatment?

Figure 5.4.1

Case 5.4  u  141


Answer
There is an inferior ER pattern with horizontal ST segment in the 12-lead ECG. The pattern is
associated with increased risk for sudden cardiac death during a coronary event.
Naturally, the cardiovascular risk factors should be treated as well as possible. Hypertension and
hypercholesterolemia should be treated with medication. Additionally, the patient should stop
smoking. An exercise ECG test would be a sensible next step in order to find asymptomatic
ischemia.

Discussion
Inferolateral early repolarization (ER), especially with horizontal/descending ST segment, has been
associated with increased risk for cardiac and arrhythmic death in the general population. In a
general population study, the incidence of sudden arrhythmic deaths increased among ER subjects
at the age of 55–65 years, which is also the age when first coronary events usually occur. Other
studies have also shown the association of inferolateral ER and sudden death due to coronary event.

References
1. Tikkanen JT, Anttonen O, Junttila MJ, et al. Long-term outcome associated with early repolarization on
electrocardiography. N. Engl. J. Med. 2009;361:2529–2537.
2. Tikkanen JT, Junttila MJ, Anttonen O, et al. Early repolarization: Electrocardiographic phenotypes associated with
favorable long-term outcome. Circulation. 2011;123:2666–2673.
3. Tikkanen JT, Wichmann V, Junttila MJ, et al. Association of early repolarization and sudden cardiac death during an
acute coronary event. Circ. Arrhythm Electrophysiol. 2012;5:714–718.

142  u  Section 5: Early Repolarization (ECG Pattern and the Syndrome)


Juhani Junttila, MD, PhD CASE
5.5
Tuomas Kenttä, PhD
Heikki V. Huikuri, MD, PhD

Patient History
A 35-year-old male was brought to the emergency department (ED) due to resuscitated ventricular
fibrillation. He was taking a shower when he collapsed to the floor, and his wife found him
unresponsive and not breathing in the bathroom. She started CPR and called the ambulance. When
the paramedics came, they found the patient in ventricular fibrillation and performed successful
cardioversion with a defibrillator. The patient woke up immediately and had no chest pain or dyspnea
apart from the discomfort in the chest after chest compressions. In the ED, his 12-lead ECG did not
show signs of myocardial infarction (Figure 5.5.1). He underwent extensive examinations including
angiography, echocardiogram, and cardiac MRI, which all turned out normal. During the stay in the
hospital, his ECG showed signs of inferior ST elevation infarction (Figure 5.5.2, paper speed 50
mm/s) and he was rushed to angiography again, but he was asymptomatic and coronaries were
normal. During the night, a short run of polymorphic VT was recorded in the telemetry.

Figure 5.5.1

Case 5.5  u  143


Figure 5.5.2

Questions
1. What is the patient’s diagnosis?
2. What would you do if the patient would continue to have runs of polymorphic VT?

Answer
The patient most likely has ER syndrome. All imaging examinations came out normal. The patient
presents ventricular arrhythmias and was resuscitated from ventricular fibrillation. ECG presents
significant inferior ER pattern with fluctuating ST segment morphology in repetitive ECG
recordings (arrow, Figure 5.5.2).
Naturally the patient needs to have an ICD, but if the ventricular arrhythmias continue,
quinidine can be used, and in the case of electrical storm isoproterenol infusion might be useful.

Discussion
According to a recent expert consensus paper on ventricular arrhythmias, early repolarization (ER)
syndrome can be diagnosed in patients with documented ventricular tachycardia or ventricular
fibrillation who present ER in two consecutive inferior or lateral leads or both without any other

144  u  Section 5: Early Repolarization (ECG Pattern and the Syndrome)


etiological explanation for the ventricular arrhythmia. One characteristic high-risk marker in ER
syndrome is the fluctuation of ST segment after the J-point elevation.

Reference
1. Pedersen CT, Kay GN, Kalman J, et al. EHRA/HRS/APHRS expert consensus on ventricular arrhythmias. Heart
Rhythm. 2014;11:e166–e196.

Case 5.5  u  145


CASE
5.6
Andrew D. Krahn, MD
Christian Steinberg, MD

Patient History
A 32-year-old healthy female experienced sudden cardiac arrest in her sleep due to ventricular
fibrillation (VF). She was successfully resuscitated with excellent neurologic recovery. Comprehensive
cardiac work-up did not reveal any structural heart disease or reversible cause and a secondary
prevention implantable cardioverter defibrillator (ICD) was implanted. Repeat electrocardiograms
(ECGs) demonstrated signs of varying degrees of inferolateral early repolarization. Follow-up was
complicated by multiple appropriate ICD shocks for recurrent nocturnal VF. Introduction of quinidine
200 mg twice daily resulted in a complete ECG normalization and prevention of further VF recurrence.

Figure 5.6.1  Early repolarization syndrome in the acute phase. A. ECG in the subacute phase. There is underlying sinus rhythm with
a marked early repolarization pattern. Presence of frequent, short-coupled ventricular ectopy with a bigeminal pattern (black arrows)
including one couplet. Eventually, a short-coupled premature ventricular contraction triggers ventricular fibrillation (blue arrow). B. ECG
recorded at intensive care unit during therapeutic hypothermia. Body temperature during the recording was 36°C. Sinus bradycardia at a
rate of 50 beats per minute (bpm) with marked P-wave flattening (dashed arrows) and prolonged PR interval. QRS complexes are wide.
Presence of diffuse J-point elevation up to 3 mm (black arrows) resulting in marked Osborn waves, which is unusual for that relatively mild
hypothermia. There is also presence of diffuse T-wave flattening associated with marked QT prolongation (blue arrows). Although all of
these findings are typical for hypothermia, they usually occur at lower body temperatures than in this case.

146  u  Section 5: Early Repolarization (ECG Pattern and the Syndrome)


Interpretation
Figure 5.6.1, panel A is a sinus rhythm with early repolarization pattern. Frequent, short-coupled
ventricular ectopy in a bigeminal pattern including one couplet. Eventually, short-coupled
ventricular ectopy triggers ventricular fibrillation. Panel B shows the ECG shows typical findings of
hypothermia with very prominent Osborn waves. An unusual feature is that the marked J-point
elevation occurs at a body temperature of 36°C. Other hypothermia-associated findings include
sinus bradycardia with marked P-wave flattening, PR prolongation, QRS widening and diffuse
T-wave flattening with QT prolongation.

Figure 5.6.2  Early repolarization syndrome controlled with quinidine. A. ECG several weeks after cardiac arrest. Sinus rhythm at a rate
of 60 bpm. Presence of inferolateral signs of early repolarization (type III early repolarization pattern) with J-point elevation up to 1.5 mm
and a predominantly horizontal ST-segment (black arrows). Notice the prominent terminal QRS notching. B. ECG after introduction of
quinidine. Complete J-point normalization and disappearance of all signs of early repolarization (dashed arrows).

Figure 5.6.2, panel A is a sinus rhythm with high-risk early repolarization pattern. Presence of
diffuse early repolarization (Antzelevitch type III) including the inferior, low lateral (V5–V6) and left
anterior leads (V3–V4). Other high-risk features include the horizontal ST-elevation of most leads
and the presence of combined J-point notching and slurring. Panel B shows the sinus rhythm with
absence of early repolarization signs.

Discussion
Early repolarization syndrome has been identified as a cause of unexplained cardiac arrest.1 The
ECG hallmark of early repolarization is J-point elevation ≥ 1 mm in at least two contiguous inferior
and/or lateral leads.2 ECG high-risk markers of ventricular arrhythmia include J-point elevation
≥ 2 mm, early repolarization in the inferior leads or generalized early repolarization and horizontal
or descending ST-elevation.3–6 Several of those high-risk features were present in this patient

Case 5.6  u  147


(Figures 5.6.1A and 5.6.2A). Ventricular arrhythmia typically occurs at rest or during sleep and is
usually initiated by short-coupled premature ventricular contractions, typically in the context of
accentuated early repolarization.7
J-point elevation during hypothermia is a well-known ECG finding that is traditionally
described as the Osborn wave.8 Although older studies postulated a strong correlation between
presence/degree of Osborn waves and body temperature, this was questioned in more recent
reports.9,10 In the presence of early repolarization, Osborn waves may be more prominent at lesser
degrees of hypothermia as shown in Figure 5.6.1B.
Recurrence of VF affects 18%–39% of patients over a median follow-up of 41–63 months and
quinidine was shown to be effective for prevention of ventricular arrhythmia.11–13 Quinidine blocks
the Ito channel and can normalize the characteristic ECG signs of early repolarization, as shown in
Figure 5.6.2B.14,15

References
1. Haïssaguerre M, Derval N, Sacher F, et al. Sudden cardiac arrest associated with early repolarization. N. Engl. J.
Med. 2008;358(19):2016–2023.
2. Priori SG, Wilde AA, Horie M, et al. HRS/EHRA/APHRS expert consensus statement on the diagnosis and
management of patients with inherited primary arrhythmia syndromes: Document endorsed by HRS, EHRA, and
APHRS in May 2013 and by ACCF, AHA, PACES, and AEPC in June 2013. Heart Rhythm. 2013;10(12):1932–1963.
3. Rosso R, Glikson E, Belhassen B, et al. Distinguishing “benign” from “malignant early repolarization”: The value of
the ST-segment morphology. Heart Rhythm. 2012;9(2):225–229.
4. Tikkanen JT, Anttonen O, Junttila MJ, et al. Long-term outcome associated with early repolarization on
electrocardiography. N. Engl. J. Med. 2009;361(26):2529–2537.
5. Tikkanen JT, Junttila MJ, Anttonen O, et al. Early repolarization: electrocardiographic phenotypes associated with
favorable long-term outcome. Circulation. 2011;123(23):2666–2673.
6. Haruta D, Matsuo K, Tsuneto A, et al. Incidence and prognostic value of early repolarization pattern in the 12-lead
electrocardiogram. Circulation. 2011;123(25):2931–2937.
7. Antzelevitch C, Yan GX. J-wave syndromes: Brugada and early repolarization syndromes. Heart Rhythm.
2015;12(8):1852–1866.
8. Osborn JJ. Experimental hypothermia; respiratory and blood pH changes in relation to cardiac function. Am. J.
Physiol. 1953;175(3):389–398.
9. Omar HR, Camporesi EM. The correlation between the amplitude of Osborn wave and core body temperature. Eur.
Heart J. Acute Cardiovasc. Care. 2015;4(4):373–377.
10. Vassallo SU, Delaney KA, Hoffman RS, et al. A prospective evaluation of the electrocardiographic manifestations of
hypothermia. Acad. Emerg. Med. 1999;6(11):1121–1126.
11. Haïssaguerre M, Sacher F, Nogami A, et al. Characteristics of recurrent ventricular fibrillation associated with
inferolateral early repolarization role of drug therapy. J. Am. Coll. Cardiol. 2009;53(7):612–619.
12. Champagne J, Geelen P, Philippon F, et al. Recurrent cardiac events in patients with idiopathic ventricular
fibrillation, excluding patients with the Brugada syndrome. BMC Med. 2005;3:1.
13. Knecht S, Sacher F, Wright M, et al. Long-term follow-up of idiopathic ventricular fibrillation ablation: A
multicenter study. J. Am. Coll. Cardiol. 2009;54(6):522–528.
14. Koncz I, Gurabi Z, Patocskai B, et al. Mechanisms underlying the development of the electrocardiographic and
arrhythmic manifestations of early repolarization syndrome. J. Mol. Cell. Cardiol. 2014;68:20–28.
15. Sacher F, Derval N, Horlitz M, et al. J wave elevation to monitor quinidine efficacy in early repolarization syndrome.
J. Electrocardiol. 2014;47(2):223–225.

148  u  Section 5: Early Repolarization (ECG Pattern and the Syndrome)


CASE
Melvin Scheinman, MD 5.7
Patient History
A 17-year-old female presents with atrial fibrillation and rapid rate. She undergoes direct current
cardioversion and returns with recurrent episodes of ventricular tachycardia (VT), as well as
episodes of ventricular fibrillation (VF) requiring direct current cardioversion.
Figure 5.7.1 shows atrial fibrillation (AF) with controlled ventricular response.

Questions
1. What are the most likely cardiac diagnoses to explain the arrhythmias in this patient?
2. What is optimal drug therapy for VT/VF storm in this patient?
3. What is optimal chronic therapy?

Figure 5.7.1

Figure 5.7.1 is recorded during the episode of AF and Figure 5.7.2 is recorded after conversion to
sinus rhythm.

Case 5.7  u  149


Figure 5.7.2

Figure 5.7.2 shows a 12-lead ECG after direct current conversion to sinus rhythm in a teenager
with AF and VF.

Questions
1. The patient presents with VF storm. What is the best acute treatment?
2. Outline the chronic therapy for this patient.

Discussion
No clear-cut diagnoses can be made from Figure 5.7.1, but J waves are best noted in the lateral leads.
The course pattern of the fibrillatory waves tends to obscure visualization of the J waves in the
inferior leads. In contrast, Figure 5.7.2 shows several noteworthy findings. Of note are the large J
waves in both the inferior and lateral leads. The J waves are followed by ST depression and flat or
inverted T waves. While J waves and elevated ST segments are common ECG findings (particularly
in athletes), the pattern of J waves inscribed for this patient as well as the clinical presentation is
diagnostic of the J-wave syndrome.
This syndrome may present with recurrent VT/VF and the J wave is seen to increase in size
prior to development of the arrhythmias. Some investigators (Antzelevitch) recommend melding
this entity with Brugada syndrome because of similar modes of arrhythmia initiation and treatment
as well as common genetic mutations. Of note, sodium channel blockers do not enhance the J wave
as seen in the Brugada syndrome. Genetic mutations affecting either sodium or calcium currents
have been described for those with the J-wave syndrome.

150  u  Section 5: Early Repolarization (ECG Pattern and the Syndrome)


Therapy for VT/VF storm involves use of infusions of isoproterenol, which is affective by
increasing the cardiac rate as well as an increased influx of calcium. Chronic therapy involves use of
oral quinidine, which blocks Ito as well as other potassium channels. At times it is necessary to use
agents that will increase the heart rate (i.e., aminophylline). The increase in heart rate is associated
with a decrease in the J wave and less propensity for arrhythmias. This patient was also treated with
an implantable cardioverter-defibrillator.

Case 5.7  u  151


SECTION 6A
Long QT Syndrome

CASE
6A.1
Li Zhang, MD
Peter R. Kowey, MD

Introduction
Torsades de pointes (TdP), or simply torsade(s), a French term that means “twisting of the spikes,” is
artfully adopted to describe a distinctive form of polymorphic ventricular tachycardia.1 TdP is
characterized by a gradual change in the amplitude and twisting of the QRS complexes around the
isoelectric line. The twisting morphology of TdP is better detected from 12-lead ECG recordings,
since it may not be shown in given or fixed single- or three-channel recordings used in ambulatory
ECGs. It comes in and lasts for a few seconds. TdP episodes cause a sudden drop in arterial blood
pressure, leading to presyncope and syncope. Although in most cases TdP terminates
spontaneously, it can degenerate into ventricular fibrillation (VF), causing cardiac arrest and sudden
death.
TdP is a hallmark tachyarrhythmia in long QT syndrome (LQTS), in either the inherited or
acquired forms.2 TdP also can occur in short QT syndrome (SQTS) or in subjects without QT
syndromes.3
The development of early after-depolarizations (EADs) may result in a tall U wave on the ECG.
Once an EAD reaches threshold, premature ventricular contractions (PVCs) can appear. TdP can be
triggered by a PVC if it lands on the preceding T wave, known as ‘R-on-T’ phenomenon. TdP can
also be initiated by a sequence of short-long-short RR intervals, so called “pause-dependent,” with
longer pauses associated with faster TdPs. Pause-dependent TdP is often observed in IKr channel-
dysfunction-related repolarization abnormalities, such as type-2 LQTS, drug-induced LQTS, and
hypokalemia-induced long QTU changes. Short-coupled TdP is another variant.
We would like to acknowledge Dr. G. Michael Vincent, Dr. Shuyan Li, Dr. Igor Splawski, and
Dr. Bortolo Martini for each of their contributions over the years to make this ECG collection
possible.

Patient History
An adult female of Caucasian-Hispanic descent is one of 24 affected members in a family with a
four-generation history of type-2 long QT syndrome (LQT2). Family screening for LQTS was
conducted 5 years prior to the LQT2 diagnosis (Figure 6A.1.1). She declined to participate at that
time. At age 38, she was diagnosed with breast cancer.

ECG Masters’ Collection: Favorite ECGs from Master Teachers Around the World © 2017 Mohammad Shenasa, Mark E. Josephson,
N. A. Mark Estes III, Ezra A. Amsterdam, Melvin Scheinman. Cardiotext Publishing, ISBN: 978-1-942909-08-8.   153
Figure 6A.1.1

A preoperative electrocardiogram (ECG) was taken (Figure 6A.1.2).4 The QT interval was
moderately prolonged (QTc 480 ms). Unfortunately, the surgeon did not recognize the QT
prolongation and prescribed erythromycin for a wound infection after the surgery.

Figure 6A.1.2

On the third day of erythromycin administration, she experienced cardiac arrest; at the time,
her QT was extremely prolonged (676 ms). Torsades de pointes (TdP) and ventricular fibrillation

154  u  Section 6A: Long QT Syndrome


were documented (Figure 6A.1.3). Luckily, she was resuscitated successfully without any serious
complications. A cardioverter defibrillator was implanted.

Figure 6A.1.3

Questions
1. How are affected family members with LQTS identified?
2. Why is avoiding QT-prolonging drugs important to patients with LQTS and medical
professionals who provide care to these patients?

Discussion
Inherited LQTS is caused by mutations of genes encoding or regulating cardiac ion channels. Of the
15 subtypes identified, LQT2 is among the most common genotypes.5 The majority of patients with
LQT2 present with gene-specific ECG patterns, which is considered a positive ECG phenotype.2 It is
very important for family members to participate in ECG screening if a blood relative is diagnosed
with inherited LQTS. A positive ECG phenotype is a strong indication that this person is likely a
gene carrier and therefore requires further evaluation.
Over 120 commonly prescribed drugs can cause QT prolongation. Most drugs that prolong QT
interval do so by blocking IKr potassium ion channels.6 LQT2 patients are born with deficiencies of
IKr channels. Taking IKr-blocking drugs is an added hazard that can place patients in a much higher
risk of developing TdP and sudden death. Therefore, avoiding QT-prolonging drugs in LQT2 is
essential.

Case 6A.1  u  155


References
1. Dessertenne F. La tachycardie ventriculaire a deux foyers opposes variables. Archives des maladies du coeur et des
vaisseaux (in French). 1966;59(2):263–272.
2. Kay GN, Plumb VJ, Arciniegas JG, et al. Torsade de pointes: The long-short initiating sequence and other clinical
features: Observations in 32 patients. J. Am. Coll. Cardiol. 1983;2(5):806–817.
3. Leenhardt A, Glaser E, Burguera M, et al. Short-coupled variant of torsade de pointes. A new electrocardiographic
entity in the spectrum of idiopathic ventricular tachyarrhythmias. Circulation. 1994;89(1):206–215.
4. Zarraga IG, Zhang L, Stump MR, et al. Nonsense-mediated mRNA decay caused by a frameshift mutation in a large
kindred of type 2 long QT syndrome. Heart Rhythm. 2011;8(8):1200–1206.
5. Zhang L, Timothy KW, Vincent GM, et al. Spectrum of ST-T-wave patterns and repolarization parameters in
congenital long-QT syndrome: ECG findings identify genotypes. Circulation. 2000;102(23):2849–2855.
6. Sanguinetti MC, Chen J, Fernandez D, et al. Physicochemical basis for binding and voltage-dependent block of
hERG channels by structurally diverse drugs. Novartis Found. Symp. 2005;266:159–166; discussion 166–170.

156  u  Section 6A: Long QT Syndrome


Gilles Lascault, MD CASE
6A.2
Olivier Piot, MD
Olivier Paziaud, MD

Patient History
The present ECG was recorded in a 29-year-old female patient, with congenital deaf dumbness and
a familial history of sudden death (sister with known long QT syndrome). This patient was
hospitalized in 2011 for non-documented presyncope where the ECGs showed a long QT syndrome.
Jervell and Lange-Nielsen (JLN) syndrome was diagnosed and a defibrillator was implanted. During
follow-up, no shocks and no ventricular tachycardias were stored in the device memory.
The last ECG recorded in September 2015 during routine follow-up: sinus rhythm at 60 bpm,
normal PR interval, narrow QRS, and a long QT interval. The QT duration is 520 ms and the
corrected QT interval is 520 m (according to Bazett’s formula), which is very abnormal and well
above the cut-off values. The morphology of the T wave is normal in some derivations and flattened
in leads II, aVF, V5, and V6.

Question
What is Bazett’s formula?

Figure 6A.2.1  Jervell Lange-Nielsen long QT syndrome.

Case 6A.2  u  157


Answer
Physiologically, QT interval changes with heart rate. It shortens when heart rate increases and
lengthens when it decreases. Therefore, Bazett’s formula was proposed to take into account those
variations of heart rate and to better differentiate between normal QTs and abnormal QTs (mostly
long QTs, rarely short QT intervals). What is important is the corrected QT interval (QTc).
According to Bazett’s formula: QTc (sec) = QT (sec)/√RR (sec).

Discussion
Corrected QT interval is considered abnormal when it is ≥ 440 ms in adult males and ≥ 460 ms in
adult females. In the setting of JLN syndrome, QTc is often ≥ 500 ms. JLN syndrome is attributed to
mutations of two genes, KCNQ1 (LQT1: 90% of cases) and KCNE1 (LQT5: 10%), which produce
potassium channel malfunction. It is inherited in an autosomal recessive manner.

REFERENCE
1. Schwartz PJ, Spazzolini C, Crotti L, et al. The Jervell and Lange-Nielsen syndrome: Natural history, molecular basis,
and clinical outcome. Circulation. 2006;113:783–790.

158  u  Section 6A: Long QT Syndrome


CASE
Robert J. Myerburg, MD 6A.3
Patient History
This 25-year-old female (AG) came to the surgical emergency department with a scalp laceration
after a syncopal event. She also provided a history of recurrent episodes of syncope during exercise,
beginning at the age of 5 years, and had never had a work-up for these events. She had been deaf
since birth, as was her 22-year-old sister, who did not have a history of syncope or other symptoms
of arrhythmias.
The ED staff obtained the following ECG, which was the first she had ever had.

Figure 6A.3.1  Used with permission from Junttila MJ, Castellanos A, Huikuri HV, et al. Risk markers of sudden cardiac death in standard
12-lead electrocardiograms. Ann. Med. 2012;44:717–732.

Because of the history of many episodes of exercise-induced syncope since she was 5 years of
age, a stress test was performed, with appropriated precautions.

Case 6A.3  u  159


Figure 6A.3.2

At 4 minutes into exercise, she was in sinus tachycardia with T-wave alternans (see arrows), and
1 minute later went into torsades de pointes (middle panel), becoming presyncopal. The test was
stopped immediately and she spontaneously converted in 44 seconds, as she was about to receive an
external shock. She was started on propranolol, 40 mg b.i.d., and a repeat stress test was normal.
Her QTc was unchanged on the drug. Because of the symptomatic LQT syndrome pattern, genetic
testing was performed and she was found to be homozygous for a variant in KCNQ1 - [R518X].
Subsequently, her three asymptomatic siblings had ECGs and stress tests, and underwent site-
specific testing for the same variant. The clinical correlates, phenotypes and genotypes of the three
siblings and the proband are listed below.

Table 6A.3.1
Age; Gender Symptoms ECG Hearing Clinical Arrhythmia Genotype
PROBAND: Recurrent Long Deaf Polymorphic VT; KCNQ1
25 y.o., syncope QT Exercise-induced [R518X]
Female Syncope and VT Homozygous

23 y.o., None Normal Normal None KCNQ1


Male [R518X]
Heterozygous

22 y.o., None Long Deaf Asymptomatic; KCNQ1


Female QT no ambient or [R518X]
exercise-induced Homozygous
arrhythmias

17 y.o., None Normal Normal None KCNQ1


Male [R518X]
Heterozygous

AG’s 22-year-old sister was also deaf, had a prolonged QTc, and was homozygous for the R518X variant in KCNQ1, but free of
symptoms of arrhythmias or syncope and had no arrhythmias during stress testing. Her two brothers were both heterozygous for the
R518X variant, with normal QTc intervals, normal hearing, and normal stress tests.

160  u  Section 6A: Long QT Syndrome


Question
Which of the following statements is most likely correct, in light of the genotypes and phenotypes
of the four siblings?
A. The fact that the two sisters are phenotypically identical, except for expression of arrhythmias
and syncope, suggests the presence of a modifier gene affecting long QT expression in the family
as the basis for their clinical difference.
B. The phenotype/genotype pattern in this family suggests a sex-linked recessive pattern of
inheritance.
C. The variable arrhythmia expression in the two sisters is most likely based upon the presence of a
modifier gene affecting transition to arrhythmogenesis in the Jervell Lange-Nielsen form of LQT
syndrome.
D. The genetic pattern in the two sisters excludes an autonomic nervous system component to
variable expression of arrhythmias.
E. The genetic pattern in the family allows a conclusion that the two heterozygous brothers have no
increased susceptibility to drug-induced arrhythmias.

Discussion, Interpretation and Answer


Looked at in isolation, AG, the proband in this family, meets all criteria for Jervell Lange-Nielsen
(JLN) syndrome, which is an autosomal recessive LQT disorder in which the affected individual is
homozygous for the genetic variant. The syndrome includes long QT, deafness, and torsades de
pointes.
However, the phenotype and genotype patterns in her three siblings introduce variables that
bring up considerations of subtle differences. The 22-year-old sister has an identical KCNQ1
genotype, and she shares the long QT and deafness components of JLN syndrome, but had no
history of the predictable exercise-associated arrhythmias and syncopal events that occurred in the
proband. This introduces the question of the presence of a modifier gene. In this case, genetic
variation is not affecting expression of prolonged QT interval, but could be affecting either the
expression of transition to arrhythmogenesis or autonomic responsiveness, since all of the syncopal
events in the proband occurred during exercise. There are data supporting the notion that
modification of either of these pathophysiologic pathways may be associated with variable
expression.
In regard to the brothers who had normal phenotypes and were heterozygous for the genotype,
the clinical pattern suggests an autosomal recessive state, typical for JLN. However, one cannot
completely exclude the possibility of weak expression in the heterozygote, which might be expressed
on ECG or clinically when exposed to drug or electrolyte effects, or autonomically based variability
of expression.
Considered in terms of probabilities, it is most likely that this is a typical JLN autosomal
recessive pattern, but with the added evidence of variable expression based on a modifier gene
affecting an arrhythmogenic pathway. The correct answer is “C,” because some basis for variable
expression of arrhythmias appears operative. Accordingly, a number of considerations of complex
pathophysiological mechanisms have to be kept in mind.

Case 6A.3  u  161


CASE
6A.4 Peter J. Schwartz, MD

Patient History
A 75-year-old female with history of syncopal episodes and a positive family history of cardiac
arrest during hypokalemia.
Figures 6A.4.1–6A.4.3 are from snapshots of a Holter recording of the same patient.

Figure 6A.4.1  Sinus bradycardia at 47 bpm.

162  u  Section 6A: Long QT Syndrome


Figure 6A.4.2  Sinus bradycardia at 41 bpm.

Case 6A.4  u  163


Figure 6A.4.3  Sinus bradycardia at 44 bpm.

The twelve-lead Holter recording showed sinus bradycardia with a normal PR interval and
markedly prolonged QT interval, especially in post extra-systolic beats (red asterisk in Figures
6A.4.1 and 6A.4.2).

164  u  Section 6A: Long QT Syndrome


CASE
Peter J. Schwartz, MD 6A.5
Patient History
A 16-year-old male patient with syncope and family history positive for sudden cardiac death.

Figure 6A.5.1  ECG at a rate of 53 bpm.

Case 6A.5  u  165


Figure 6A.5.2  ECG at a rate of 72 bpm.

166  u  Section 6A: Long QT Syndrome


Figure 6A.5.3  ECG at a rate of 69 bpm.

Twelve-lead Holter recording shows notched T waves in leads V4 –V6. After rapid heart rate
increments, notched T waves appear in leads V4 –V6 with QTc prolongation (QTc 590 ms).

Case 6A.5  u  167


CASE
6A.6 Peter J. Schwartz, MD

Patient History
An eleven-year-old female, with Jervell and Lange-Nielsen syndrome. Her personal history positive
for syncopal episodes during stress or physical activity.

Figure 6A.6.1  ECG at 62 bpm.

168  u  Section 6A: Long QT Syndrome


Figure 6A.6.2  ECG at 83 bpm.

In the basal ECG, positive T waves and QT prolongation are present. At cessation of exercise,
diphasic T waves appeared in leads V4 –V6 with marked QT prolongation.

Case 6A.6  u  169


CASE
6A.7 Peter J. Schwartz, MD

Patient History
A three-year-old male with Jervell and Lange-Nielsen syndrome (the parents are second-degree
cousins).
PR interval: 114 ms
QRS: 64 ms
QT/QTc: 572/574 ms

Figure 6A.7.1  ECG at 61 bpm.

The patient experiences many syncopal episodes triggered by play or emotional events,
sometimes associated with loss of urine. Biphasic T waves in leads V3–V6 and in inferior leads as
well as notched T waves in lead V2 with QTc > 550 ms are present.

170  u  Section 6A: Long QT Syndrome


CASE
Peter J. Schwartz, MD 6A.8
Patient History
A five-year-old female with long QT-syndrome caused by a de novo mutation in the Calmodulin
gene. Her personal history is positive for multiple episodes of ventricular fibrillation.

Figure 6A.8.1

T-wave alternans in a 12-lead Holter recording.

Case 6A.8  u  171


CASE
6A.9 Peter J. Schwartz, MD

Patient History
A five-year-old male patient with long QT syndrome type 2. Paternal history is positive for syncope.
PR interval: 124 ms
QRS: 72 ms
QT/QTc: 416/460 ms

Figure 6A.9.1  87 bpm.

Basal ECG shows typical biphasic T waves in lead V2 and notched T waves in leads V3–V4 and in
inferior leads.

172  u  Section 6A: Long QT Syndrome


SECTION 6B
Short QT Syndrome

CASE
6B.1
Li Zhang, MD
Peter R. Kowey, MD

Patient History
A 36-year-old Caucasian male with recurrent syncope was diagnosed with short QT syndrome
(SQTS).

Figure 6B.1.1  Polymorphic torsades de pointes (TdP) initiated by short-coupled premature ventricular contractions (PVCs).

ECG Masters’ Collection: Favorite ECGs from Master Teachers Around the World © 2017 Mohammad Shenasa, Mark E. Josephson,
N. A. Mark Estes III, Ezra A. Amsterdam, Melvin Scheinman. Cardiotext Publishing, ISBN: 978-1-942909-08-8.   173
Figure 6B.1.2  Upper panel: Frequent PVCs before and after TdP have extremely short RR coupling intervals (210 ms) so that the
premature beat lands on the peak of the T wave (R-on-T). Lower panel: This patient with SQTS also has paroxysmal atrial fibrillation.

Question
TdP is a hallmark of ventricular arrhythmia in long QT syndrome (LQTS). Is that also the case in
SQTS?

Discussion
The answer is, we really do not know. SQTS is an extremely rare genetic disorder associated with
life-threatening arrhythmias. PVCs with short coupling intervals in the presence of SQTS increases
the risk of developing TdP. In fact, cardiac arrest is often the first manifestation of the disease, with
a peak incidence in the first year of life, and 1.3% increased risk per year between 20 and 40 years
of age.1

Reference
1. Mazzanti A, Kanthan A, Monteforte N, et al. Novel insight into the natural history of short QT syndrome. J. Am.
Coll. Cardiol. 2014. 63(13):1300–1308.

174  u  Section 6B: Short QT Syndrome


SECTION 6C
Torsades de Pointes

CASE
N. A. Mark Estes III, MD 6C.1
Patient History
An electrocardiogram (ECG) of a 15-year-old female diagnosed since infancy with seizures with
recurrent syncope while rowing competitively.

Figure 6C.1.1

Questions
1. What is the ECG abnormality?
2. What is the likely cause of her exercise-related syncope?

ECG Masters’ Collection: Favorite ECGs from Master Teachers Around the World © 2017 Mohammad Shenasa, Mark E. Josephson,
N. A. Mark Estes III, Ezra A. Amsterdam, Melvin Scheinman. Cardiotext Publishing, ISBN: 978-1-942909-08-8.   175
Discussion
This tracing shows marked QT prolongation. Ambulatory recording during rowing documented
torsades de pointes as the cause of her recurrent exercise-induced loss of consciousness.
Beta-blocker therapy and restriction from competitive athletics has resulted in resolution of all
syncope and seizure-like activity.

176  u  Section 6C: Torsades de Pointes


CASE
Guy Fontaine, MD, PhD, HDR 6C.2
Patient History
Tracings are from a patient recorded during a Holter monitor. This patient has complete heart
block.

Figure 6C.2.1

Discussion
Note the bumps on the T wave of the last AV block beat, suggesting major trouble in repolarization,
as well as orientation of the first upward or downward beat, suggesting two initial opposite
preferential pathways. The bumps preceding the torsade are the result of major bradycardia produced

Case 6C.2  u  177


by the complete AV block. However, there is also a dispersion of refractory periods which may lead to
side-by-side phase 2 reentry of adjacent fibers. The next beat has a short coupling interval; therefore,
dispersion of refractory periods is no longer possible. Because of the extremely short intervals during
the torsade (250-300 bpm) the only mechanism is a reentrant longitudinal circus movement. This
fast tachycardia has two opposite exit sites invading the antero-superior and postero-inferior sulcia.
A phenomenon of Wenkebach on one of the two exits may explain the twisting of the QRS peaks
around the isoelectric line rather than a drifting vortex-like activation. This hypothesis is reinforced
by two consecutive beats of opposite directions at the same rate suggesting an abrupt Mobitz type
block on one of the two exit pathways. In our endocardial records during torsade in 9 patients no
early afterdepolarizations (EADs) have been observed.

References
1. Dessertenne F. [Ventricular tachycardia with 2 variable opposing foci]. Arch. Mal. Cœur. Viass. 1966;59:263–272.
2. Fabiato A, Coumel P. Torsades de Pointes, a quarter of a century later: A tribute to Dr. F Dessertenne. Cardiovasc.
Drugs Ther. 1991;5:167–169.
3. Takasugi N, Goto H, Takasugi M, et al. Prevalence of microvolt T-wave alternans in patients with long QT
syndrome and its association with torsade de pointes. Circ. Arrhythm. Electrophysiol. 2016;9:e003206.
4. Davis AM. Torsades de Pointes: 50 Years later, can we see it coming? Circ. Arrhythm. Electrophysiol. 2016;9:e003850.
5. Guoliang LI, Fontaine GH. GW26-e5376 Mechanism of torsades de pointes elucidated in human AV block. J Am
Coll Cardiol. 2015;66(16)(suppl):C178.

178  u  Section 6C: Torsades de Pointes


CASE
6C.3
Li Zhang, MD
Peter R. Kowey, MD

Patient History
A 40-year-old Caucasian female was admitted to the emergency department for recurrent syncope.
She had no family history of sudden death. On testing, she had no evidence of structural cardiac
disease.
Ambulatory electrocardiogram revealed that torsades de pointes (TdP) was initiated by short-
coupled premature ventricular contractions (PVCs) (Figure 6C.3.1) and lasted for over 10 seconds.

Figure 6C.3.1

Case 6C.3  u  179


Her QTc was borderline prolonged (468 ms) prior to TdP. Note that it was dramatically
shortened (QTc 338 ms) immediately post TdP. QTc returned to baseline (456 ms) a few days later
(Figure 6C.3.2).

Figure 6C.3.2

Question
Is the short-coupled PVC a trigger of TdP?

180  u  Section 6C: Torsades de Pointes


Discussion
Short-coupled PVCs can initiate TdP without the presence of QT prolongation. TdP initiated by
short-coupled PVCs is considered a unique variant.1 In these cases, the RR coupling interval rarely
exceeds 300 ms. R-on-T can trigger TdP and ventricular fibrillation, even in subjects without
structural heart disease.

Reference
1. Leenhardt A, Glaser E, Burguera M, et al. Short-coupled variant of torsade de pointes. A new electrocardiographic
entity in the spectrum of idiopathic ventricular tachyarrhythmias. Circulation. 1994;89(1):206–215.

Case 6C.3  u  181


CASE
6C.4
Andrew D. Krahn, MD
Christian Steinberg, MD

Patient History
A 31-year-old female of First Nations descent presented to the emergency department for
palpitations and multiple implantable cardioverter-defibrillator (ICD) shocks. The patient had
known type 1 long QT (LQT-1) syndrome and was homozygous for the V205M mutation of the
KCNQ1 gene. The patient had a secondary prevention ICD in situ and had experienced recurrent
appropriate shocks in the past. Other significant comorbidities included ethanol abuse and
intermittent cocaine use. Precipitants of ventricular arrhythmia in the past included substance
abuse, alcohol withdrawal, electrolyte imbalance, and nonadherence to beta-blocker use. The
patient was on nadolol 40 mg once daily. On this occasion, her symptoms were initiated by an acute
episode of heavy alcohol intake over several days.

Figure 6C.4.1  LQT-1 with T-wave alternans and torsades de pointes. (A) The electrocardiogram (ECG) shows sinus rhythm at a rate of
90 bpm with a marked QT prolongation (black arrows). The absolute QT interval is 560 ms and the corrected QT interval (QTc) is 684
ms (measured in lead V5). The absolute QT interval is measured according to the maximal slope technique and QTc is calculated using
Bazett’s formula. Note the striking T-wave alternans (blue arrows). T-wave alternans is more prominent in the precordial leads and lead
aVR, resulting in beat-to-beat changes of the T-wave polarity. (B) This ECG, recorded only a few minutes later, shows a run of torsades de
pointes (TdP). Hallmarks of (TdP) are periodic changes of the QRS-T amplitude and a twisting of the QRS axis around the electrical baseline
(arrows).

182  u  Section 6C: Torsades de Pointes


Interpretation
(A) Sinus rhythm at a rate of 90 bpm with marked QT prolongation (absolute QT interval of 560 ms
and corrected QT interval of 684 ms. Presence of giant T-U waves, and T-wave alternans most
evident in the precordial leads. (B) Torsades de pointes.

Discussion
Hereditary long-QT (LQT) syndrome occurs with a prevalence of approximately 1:2500 births.1
Gene defects responsible for LQT syndrome affect cardiac ion channels that are responsible for
repolarization. The pathophysiology of LQT syndrome is characterized by transmural dispersion of
repolarization that results in QT prolongation and predisposes to life-threatening ventricular
arrhythmia.2,3
To date, 15 different genotypes have been identified, but more than 90% of all cases are caused
by LQT-1-3.4 The V205M founder mutation of KCNQ1 has been described in two First Nations
communities of northern British Columbia and is associated with increased susceptibility to
ventricular arrhythmia.5,6
Torsades de pointes (TdP) is a specific form of polymorphic ventricular tachycardia,
representing the typical ventricular arrhythmia occurring in LQT patients. The ECG hallmarks of
TdP include periodic changes of the QRS-T amplitude and a rotating shift in QRS axis, resulting in
a characteristic spindle-shaped twisting around the electrical baseline.7 TdP is typically initiated by
late-coupled premature ventricular contractions (short-long sequence), often termed as pause-
dependent.7 The twisting of the QRS axis is the result of different circulating wavefronts with
opposed direction in the right and left ventricle.8
Several studies have suggested ECG markers predictive of TdP in LQT syndrome. The most
characteristic ECG marker is macroscopic T-wave alternans, which were present in this patient.9,10
Another study demonstrated a strong correlation between TdP and the presence of preceding giant
T-U waves (> 6 mm), which were also documented in the present case.11 One study suggested the
Tpeak–Tend interval as an additional marker. Topilski et al. showed that the Tpeak–Tend interval with a
cut-off value of 117 ms was a reliable predictor of TdP, with a positive and negative predictive value
of 93% and 99%, respectively.12

References
1. Schwartz PJ, Crotti L, Insolia R. Long-QT syndrome: From genetics to management. Circ. Arrhythm. Electrophysiol.
2012;5(4):868–877.
2. Antzelevitch C. Role of transmural dispersion of repolarization in the genesis of drug-induced torsades de pointes.
Heart Rhythm. 2005;2(Suppl 2):S9–S15.
3. Antzelevitch C. Ionic, molecular, and cellular bases of QT-interval prolongation and torsade de pointes. Europace.
2007;9(Suppl 4):iv4–iv15.
4. Obeyesekere MN, Antzelevitch C, Krahn AD. Management of ventricular arrhythmias in suspected
channelopathies. Circ. Arrhythm. Electrophysiol. 2015;8(1):221–231.
5. Arbour L, Rezazadeh S, Eldstrom J, et al. A KCNQ1 V205M missense mutation causes a high rate of long QT
syndrome in a First Nations community of northern British Columbia: A community-based approach to
understanding the impact. Genet. Med. 2008;10(7):545–550.
6. Jackson HA, McIntosh S, Whittome B, et al. LQTS in Northern BC: homozygosity for KCNQ1 V205M presents
with a more severe cardiac phenotype but with minimal impact on auditory function. Clin. Genet. 2014;86(1):85–90.
7. El-Sherif N, Turitto G. Torsade de pointes. Curr. Opin. Cardiol. 2003;18(1):6–13.

Case 6C.4  u  183


8. El-Sherif N, Chinushi M, Caref EB, et al. Electrophysiological mechanism of the characteristic electrocardiographic
morphology of torsade de pointes tachyarrhythmias in the long-QT syndrome: Detailed analysis of ventricular
tridimensional activation patterns. Circulation. 1997;96(12):4392–4399.
9. Habbab MA, El-Sherif N. TU alternans, long QTU, and torsade de pointes: Clinical and experimental observations.
Pacing Clin. Electrophysiol. 1992;15(6):916–931.
10. Schwartz PJ, Malliani A. Electrical alternation of the T-wave: Clinical and experimental evidence of its relationship
with the sympathetic nervous system and with the long Q-T syndrome. Am. Heart J. 1975;89(1):45–50.
11. Kirchhof P, Franz MR, Bardai A, et al. Giant T-U waves precede torsades de pointes in long QT syndrome:
A systematic electrocardiographic analysis in patients with acquired and congenital QT prolongation. J. Am. Coll.
Cardiol. 2009;54(2):143–149.
12. Topilski I, Rogowski O, Rosso R, et al. The morphology of the QT interval predicts torsade de pointes during
acquired bradyarrhythmias. J. Am. Coll. Cardiol. 2007;49(3):320–328.

184  u  Section 6C: Torsades de Pointes


CASE
6C.5
I. W. P. Obel, MBChB
Judith Daniels, RN, CRDS, CEPS

Patient History
A 79-year-old female presented to a general practitioner with chest discomfort and a flu-like illness.
Past history included hypertension (controlled with hydrochlorothiazide and captopril) and vaginal
thrush (ketoconazole). She was also receiving digoxin 0.25 mg daily. Allergic rhinitis and a flu-like
illness was diagnosed and treated with a nonsedating antihistamine.
She then experienced a number of syncopal episodes and presented to the emergency
department. Clinical examination there showed a well-perfused, normotensive patient. Bursts of an
irregular pulse were present.

Question
Immediate therapy should be:
1. Lidocaine followed by urgent coronary angiography
2. Correction of any electrolyte imbalance
3. Intravenous amiodarone and, in the absence of ischemia, referral for implantable cardioverter-
defibrillator (ICD) insertion
4. Intravenous magnesium and possible insertion of temporary pacemaker
5. Recent additional drug history

Figure 6C.5.1

Case 6C.5  u  185


Figure 6C.5.2

Figure 6C.5.3

Discussion, Interpretation, and Answers


Figure 6C.5.1 Interpretation
1. Sinus bradycardia with a normal PR interval, normal QRS complexes and T waves.
2. Prolonged QT (QTc = 0.59) and prominent U wave.
3. Nonsustained polymorphic ventricular preceded by ventricular ectopics with a long-short
interval (typical).

186  u  Section 6C: Torsades de Pointes


Figure 6C.5.1 Discussion
This shows QTc prolongation (QTc = 0.59), sinus rhythm, normal PR interval, QRS, and T-wave
configuration. A short run of polymorphic ventricular beats are preceded by ectopy with a long-
short sequence.

Figure 6C.5.2 Interpretation


Torsades de pointes

Figure 6C.5.2 Discussion


This pattern is of polymorphic ventricular arrhythmia preceded by a long-short sequence of ectopy.
This is repeated and diagnostic for torsades de pointes. However, this could also be called a
polymorphic VT.

Figure 6C.5.3 Interpretation


1. Sinus rhythm with normal QR interval (QTc = 0.434).
2. Diffuse T-wave inversions.

Treatment
Magnesium was unsuccessful and a temporary pacemaker was inserted (answer 4).

Course
Following temporary pacing, the inverted T waves reverted to normal within 2 days.

References
1. Dessertenne F. La tachiecardie ventriculaire a deux foyers opposes variables. Arch. Mal. Coer. 1966;59:263–272.
2. Tzivoni D, Banai S, Shuger C. Treatment of torsadae de pointes with magnesium sulphate. Circulation.
1988;77:392–397.
3. Leehart MD, Coumel P, Slama R. Torsades de Pointes. J. Cardiovasc. Electrophysiol. 1992;3:281–292.
4. Symanski JD, Gettes LS. Drug effects on the electrocardiogram. A review of their clinical importance. Drugs.
1993;46(2):219–248.

Case 6C.5  u  187


CASE
6C.6
I. W. P. Obel, MBChB
Judith Daniels, RN, CRDS, CEPS

Patient History
A 79-year-old female whose family origins were in Porto, Portugal, had presented to her general
practitioner with a 1-month history of occasional dizzy spells. Four days prior to admission, she had
the first of a number of syncopal episodes. The family history was positive for cardiac death.

Question
Your first therapeutic choice would be:
1. IV amiodarone and admission to rule out acute myocardial infarction
2. Single-chamber implantable cardioverter-defibrillator (ICD) and work-up for coronary disease
3. Permanent pacemaker
4. Oral amiodarone in a loading dose and full cardiac work-up to exclude acute ischemia and, in
particular, cardiac amyloidosis

Figure 6C.6.1

Interpretation
Polymorphic ventricular tachycardia and complete heart block.

188  u  Section 6C: Torsades de Pointes


Discussion
The pattern of the polymorphic ventricular tachycardia is typical of torsades de pointes, which
would be more in keeping with chronic heart block, as would the history of syncope over a fairly
prolonged period of time. Syncope related to cardiac amyloidosis (as suggested by her family history
and origins) is most often associated with postural hypotension, but malignant ventricular
arrhythmias may occur.

Case 6C.6  u  189


CASE
6C.7 Mohammad Shenasa, MD

Patient History
A 45-year-old male with history of nonischemic cardiomyopathy (with congestive heart failure) and
an ejection fraction of 25% awaiting an implantable cardioverter-defibrillator (ICD).

Figure 6C.7.1  With permission from Shenasa M, et al. Incessant ventricular tachycardia and fibrillation: Electrical storms. Card.
Electrophysiol. Clin. 2014;6:613–621.

Panels A–D show sinus rhythm with ventricular bigeminy. Episodes of nonsustained torsades de
pointes (TdP) are seen in panels A and B. Panels C and D illustrate conversion of the TdP
ventricular tachycardia to ventricular fibrillation.

Question
What is the more typical pattern of RR intervals in TdP?

Discussion, Interpretation, and Answer


Most episodes of TdP initiate in a RR sequence of short-long-short sequence that triggers the TdP.
Note that the intervals are exactly the same in the short-long-short RR intervals that triggers TdP;
however, the length of the TdP and subsequent ventricular arrhythmias are unpredictable.

190  u  Section 6C: Torsades de Pointes


Furthermore, the first two beats that trigger the TdP are exactly the same in morphology and
interval. The classical patterns of TdP (“twisting of a point”) is most evident in panel B, where the
ECG vector is upward and then downward as originally described by Francois Dessertenne in 1966.1

Reference
1. Dessertenne F. La tachycardie ventriculaire a deux foyers opposes variables. Arch. Mal. Coeur. Vaiss. 1966;59:
263–72.

Case 6C.7  u  191


SECTION 6D
Other Proarrhythmias

Begüm Yetiş Sayın, MD CASE


6D.1
Sercan Okutucu, MD
Ali Oto, MD

Patient History
A 21-year-old female was transferred to the emergency department by ambulance following a
generalized seizure at home. By the time of arrival, she was comatose with a poor respiratory effort.
Pupils were symmetrically dilated. Serum glucose was normal. Blood pressure was 70/40. Her ECG
(Figure 6D.1.1) is shown below:

Figure 6D.1.1

Discussion
This is a case of tricyclic antidepressant overdose. QRS duration is the cardiac parameter most often
followed in cases of tricyclic antidepressant overdose, because it has been shown that a QRS duration
of 100 ms or greater in sinus rhythm after a tricyclic antidepressant is predictive of seizures and
arrhythmias. Amplitude of the terminal R wave (3 mm or greater, shown with arrow) in aVR and the
ratio of the R wave to the S wave in aVR are predictors of arrhythmias in these patients.

ECG Masters’ Collection: Favorite ECGs from Master Teachers Around the World © 2017 Mohammad Shenasa, Mark E. Josephson,
N. A. Mark Estes III, Ezra A. Amsterdam, Melvin Scheinman. Cardiotext Publishing, ISBN: 978-1-942909-08-8.   193
CASE
6D.2
Magdi M. Saba, MD
David E. Ward, MD

Patient History
A 71-year-old female presented to the emergency department with tiredness and dizziness. She has
a history of a small membranous ventricular septal defect (VSD) and paroxysmal atrial fibrillation
on antiarrhythmic drug therapy.

Figure 6D.2.1  ECG on admission. Wide complex rhythm with right bundle branch block (RBBB) morphology and no obvious sinus
mechanism. Two premature beats, likely of atrial origin, are seen.

194  u  Section 6D: Other Proarrhythmias


Figure 6D.2.2  ECG obtained 8 hours later displays narrow complex junctional rhythm with retrograde conduction (best seen in lead V3).

Figure 6D.2.3  ECG obtained the next day shows a return to normal sinus rhythm. Flecainide (100 mg BD) was stopped on admission
with resultant normalization of ECG. Therefore, in retrospect, the initial ECG showed a junctional rhythm with marked RBBB delay due to
flecainide. This demonstrates a case of sodium-channel blocker toxicity.

Question
What can explain the T-wave inversion in the right precordial leads?

Case 6D.2  u  195


Answer
Cardiac memory. During RBBB, the T wave in these leads is opposite to the final QRS deflection,
preserving the ventricular gradient. Upon QRS normalization with resolution of the abnormal
depolarization, repolarization continues to follow the abnormal depolarization pattern for a period
directly proportional to the period of abnormal depolarization, but ultimately normalizes. As seen
in this case, the T-wave inversion is already less prominent on the third compared to the second
ECG, which are only hours apart.

196  u  Section 6D: Other Proarrhythmias


SECTION 7
Brugada Syndrome

CASE
7.1
Antoni Bayés de Luna, MD, PhD
Javier García-Niebla, RN

Figure 7.1.1

Patient History
The patient is a 30-year-old asymptomatic male. The preoperative ECG is shown (Figure 7.1.1).
Note the presence of r′ in V1–V2 with a small ST ascent in V1–V2 and a negative T wave in V1.
What is the explanation?

Questions
A. There are many causes that may explain r′ in V1–V2:
1. High location of electrodes in V1 and V2
2. Pectus excavatum

ECG Masters’ Collection: Favorite ECGs from Master Teachers Around the World © 2017 Mohammad Shenasa, Mark E. Josephson,
N. A. Mark Estes III, Ezra A. Amsterdam, Melvin Scheinman. Cardiotext Publishing, ISBN: 978-1-942909-08-8.   197
3. Wolff–Parkinson–White (WPW)
4. Athlete’s ECG
5. Right ventricular hypertrophy
6. Partial right bundle branch block
7. Brugada pattern type 2: Saddle-back type
8. Arrhythmogenic right ventricular dysplasia (ARVD)
9. Hyperkalemia
10. Lateral MI
B. Which of these causes best explains the r′ in V1 shown in the ECG of Figure 7.1.1?

Interpretation, Answer and Comments


The correct answer is 7, Brugada pattern.
A. The electrodes are located in the correct place: the fourth intercostal space (ICS). It was not
necessary to record the ECG in the second ICS. However, it is always recommended to do so, as
seen in Figure 7.1.2.

Figure 7.1.2  Electrocardiogram of a young male with non-vasovagal syncope. A. The ECG in the fourth ICS is suspicious of Brugada
pattern type 2 (V2). B. With the electrodes located in the second ICS, the ECG is typical of Brugada pattern type 2. The diagnosis was
confirmed by the ajmaline test, whereby the typical ECG pattern type 1 was observed.

B. Recently, the characteristics of r′, especially the slope of the descending arm, have been shown to
be useful in the differential diagnosis, especially between Brugada pattern type 2, athlete’s heart,
right ventricular hypertrophy, and incomplete bundle branch block.
a. Chevallier et al. reported that a wider angle between the two arms in V1–V2 (>58°) as a
diagnostic criteria for Brugada pattern type.

198  u  Section 7: Brugada Syndrome


b. According to Serra et al., a triangle base > 180 ms (4 mm) is very suggestive of Brugada pattern
type 2 (Se 85, Sp 96, PPV 95, NPP 88). In doubtful cases, the ajmaline test may confirm the
diagnosis.
In this case (Figure 7.1.3), the base measures 188 ms, indicating Brugada pattern, a
diagnosis that was supported by clinical family history and a positive ajmaline test. Athletes,
right ventricular hypertrophy (RVH), and partial right bundle branch block present a quicker
descending slope of r′ (see Figure 7.1.4).

Figure 7.1.3  In the r′ of V2 the base of the triangle may be measured to demonstrate if it is > or < 160 ms (4 mm).

Figure 7.1.4  A. A typical case of an athlete with r′ in V1 and V2. The slope of the descending arm is very quick (triangle base 40 ms).
B. An example of V2 in a case of Brugada pattern type 2. Note the clear upstroke of ST and the much longer triangle base (184 ms) when
compared to the athlete (40 ms) due to the slower descending arm of the triangle.

Case 7.1  u  199


C. Usually, only Brugada pattern type 2 and sometimes pectus excavatum, present ST elevation in
V1–V2. In Table 7.1.1, we see the ECG criteria for Brugada pattern type 2 according to the new
consensus paper published in 2012.1

Table 7.1.1
Type 2: Brugada pattern. ECG characteristics (modified from Bayés de Luna et al.)1

For this typical saddle-back pattern present in V1-V2, the following characteristics are present:
High take-off in r′ (which often does not coincide with J point) ≥ 2 mm
a.
Descending arm of r′ coincides with beginning of ST (is often not well seen).
b.
Minimum ST ascent ≥ 0.5 mm.
c.
ST is usually followed by a positive T wave in V2 (T peak > ST minimum > 0) and variable morphology in V1.
d.
The characteristics of the triangle formed by r′ allow different criteria useful for diagnosis to be defined (see
e.
text and Figures 7.1.3–7.1.5).
• β angle2
• Duration of the base of the triangle of r′ at 5 mm from the high take-off greater than 4 mm3
The duration of QRS is usually longer in BrP type 2 than in other cases with r′ in V1. There is a mismatch
f.
between V1 and V6 that may also be seen in ARVD and hyperkalemia (see Figure 7.5.5).
It is necessary to record the ECG in the second ICS (see Figure 7.1.2), and to perform an ajmaline test if there
g.
is any doubt.

D. The patient does not present pectus excavatum. Furthermore, the P wave in V1 is not negative
and the descending limb of r′ is very quick.
E. There is no delta wave and the PR interval is normal, thus excluding Wolff–Parkinson–White
syndrome.
F. The differential diagnosis between Brugada pattern type 2 and athletes is also based on the
characteristics of r′ (see Figures 7.1.4 and 7.1.5).

200  u  Section 7: Brugada Syndrome


Figure 7.1.5  Proposed diagnostic algorithm for patients with r′ in leads V1–V2 with the most important differential diagnosis
characteristics.

Baranchuk et al.4 published an algorithm that summarizes its criteria. It is recommended to


follow this algorithm to help in carrying out the correct diagnosis.
G. Cases of RVH usually correspond to an atrial septal defect and in some cases of mild pulmonary
hypertension. In these cases the base of triangle is narrow. In some occasions, auscultation of a
murmur and an abnormal echocardiogram may help in differential diagnosis.
H. In arrhythmogenic right ventricular dysplasia (ARVD) and hyperkalemia, the base of triangle
may be wider than 160 ms and may also be seen in both cases as a mismatch between V1 and V6
(Figure 7.1.5). However, in these cases the following characteristics may help the diagnosis
(Figure 7.1.5).5
a. Hyperkalemia. Positive T wave in V1–V2 > 7 mm.
b. ARVD. No ST elevation in V1–V2. Negative and usually symmetric T wave in V1–V4.
I. Finally, in rare cases of lateral myocardial infarction MI in V1 a final r′ may be present but
usually with the pattern Rr′ or rsr′. What is not seen is the rSr′ morphology.

Case 7.1  u  201


Conclusion
The presence of r′ in V1–V2 is a frequent finding that obliges us to perform a careful differential
diagnosis because there are many causes that may present this pattern, some of which are
potentially dangerous. Although an experienced cardiologist may reach the diagnosis easily, it is
recommended to follow Baranchuk’s algorithm,4,5 and consult Table 7.1.1.

References
1. Bayés de Luna A, Brugada J, Baranchuk A, et al. Current electrocardiographic criteria for diagnosis of Brugada
pattern: A consensus report. J Electrocardiol. 2012;45:433–442.
2. Chevallier S, Forclaz A, Tenkorang J, et al. New electrocardiographic criteria for discriminating between Brugada
types 2 and 3 patterns and incomplete right bundle branch block. J. Am. Coll. Cardiol. 2011;58:2290.
3. Serra G, Baranchuk A, Bayés de Luna A, et al. New electrocardiographic criteria to differentiate type-2 Brugada
pattern from the electrocardiogram of healthy athletes with r′ wave in leads V1/V2. Europace. 2014;16:1639–1645.
4. Baranchuk A, Enriquez A, Garcia-Niebla J, et al. Differential diagnosis of rSr′ pattern in leads V1-V2 comprehensive
review and proposed algorithm. Ann. Noninvasive Electrocardiol. 2015; 20 (1): 7–17.
5. Bayés de Luna A. Clinical Electrocardiography, Fourth Edition. Hoboken, NJ. Wiley-Blackwell. 2012:103.

202  u  Section 7: Brugada Syndrome


CASE
7.2
Pedro Brugada, MD, PhD
Sergio Richter, MD

Figure 7.2.1

Eighteen different genes are now known to be related to Brugada syndrome. The most
commonly found abnormalities in families with Brugada syndrome are related to mutations in the
cardiac sodium-channel gene SCN5A. However, mutations in genes encoding for potassium and
calcium cardiac channels can also cause the Brugada phenotype.

Question
1. Given the electrocardiogram (ECG) abnormalities, what are the most likely affected genes in the
patients in panels A and B?
1. Panel A calcium channel, panel B potassium channel.
2. Panel A sodium channel, panel B potassium channel.
3. Panel A sodium channel, panel B sodium channel.

Case 7.2  u  203


Answer
The correct answer is 2.

References
1. Brugada P. Brugada syndrome: More than 20 years of scientific excitement. J. Cardiol. 2016 Mar.;67(3):215–20.
2. Brugada P, Brugada J. Right bundle branch block, persistent ST-segment elevation and sudden cardiac death: A
distinct clinical and electrocardiographic syndrome. A multicenter report. J. Am. Coll. Cardiol. 1992;20:1391–1396.
3. Antzelevitch C. The Brugada syndrome: Diagnostic criteria and cellular mechanisms. Eur. Heart J. 2001;22:356–363.
4. Brugada J, Brugada R, Brugada P. Right bundle branch block, ST-segment elevation in lead V1 through V3 and
sudden cardiac death: A marker for sudden death in patients without demonstrable structural heart disease.
Circulation. 1998;97:457–460.
5. Brugada P, Brugada J, Roy D. Brugada syndrome 1992–2012: 20 years of scientific excitement, and more. Eur. Heart
J. 2013;34:3610–3615.

204  u  Section 7: Brugada Syndrome


CASE
7.3
Pedro Brugada, MD, PhD
Sergio Richter, MD

Patient History
The ECG on February 5th (precordial leads only) was recorded immediately after cardiopulmonary
resuscitation in a 40-year-old Japanese male. It shows the typical features of a type 1 ECG
compatible with Brugada syndrome. Within the following days, the ST elevation disappears. The
ECG is no longer a type 1, but a type 2 or type 3 Brugada ECG.

Figure 7.3.1

Question
Are there any features on the ECGs of February 7th–13th that would make you suspicious of
Brugada syndrome even if you did not have the ECG of February 5th available?

Case 7.3  u  205


Answer
The ECGs from February 7th–13th show an R′ that does not correspond to a simple incomplete
right bundle branch block. The R′ is too broad. The angle between the ascending limb of the S wave
and the descending limb of the R′ is too wide (see Figure 7.3.2). Although this feature is not
diagnostic of Brugada syndrome, it should make you suspicious about the diagnosis.

Figure 7.3.2  ECGs from February 7th–13th. Precordial leads V1 and V2.

References
1. Brugada P. Brugada syndrome: More than 20 years of scientific excitement. J. Cardiol. 2016 Mar.;67(3):215–20.
2. Brugada P, Brugada J. Right bundle branch block, persistent ST-segment elevation and sudden cardiac death: A
distinct clinical and electrocardiographic syndrome. A multicenter report. J. Am. Coll. Cardiol. 1992;20:1391–1396.
3. Antzelevitch C. The Brugada syndrome: Diagnostic criteria and cellular mechanisms. Eur. Heart J. 2001;22:356–363.
4. Brugada J, Brugada R, Brugada P. Right bundle branch block, ST-segment elevation in lead V1 through V3 and
sudden cardiac death: A marker for sudden death in patients without demonstrable structural heart disease.
Circulation. 1998;97:457–460.
5. Brugada P, Brugada J, Roy D. Brugada syndrome 1992–2012: 20 years of scientific excitement, and more. Eur. Heart
J. 2013;34:3610–3615.

206  u  Section 7: Brugada Syndrome


CASE
7.4
Pedro Brugada, MD, PhD
Sergio Richter, MD

Patient History
This 12-lead ECG of a young male with recurrent syncope is normal. Administration of a sodium
channel blocker (in this case ajmaline) unmasks the typical Brugada ECG pattern.

Figure 7.4.1

Discussion
Administration of a sodium channel blocker to unmask a possible Brugada syndrome is mandatory
in all patients with syncope of unknown origin as well as in patients with isolated atrial fibrillation
(atrial fibrillation may be the first manifestation of Brugada syndrome). Although ajmaline is
preferred for this purpose, flecainide, procainamide, and pilsicainide are also used. This
pharmacologic test can also be used to unmask family member carriers of the disease.

Case 7.4  u  207


Figure 7.4.2  Phenocopies: Not all ECGs that look like Brugada syndrome are Brugada syndrome. Two examples of phenocopies are
shown: A patient with pectus excavatum and a patient with a mediastinal tumor. Used with permission from Tarín et al. Pacing Clin.
Electrophysiol. 1999;22:1264.

References
1. Brugada P. Brugada syndrome: More than 20 years of scientific excitement. J. Cardiol. 2016 Mar.;67(3):215–20.
2. Brugada P, Brugada J. Right bundle branch block, persistent ST-segment elevation and sudden cardiac death: A
distinct clinical and electrocardiographic syndrome. A multicenter report. J. Am. Coll. Cardiol. 1992;20:1391–1396.
3. Antzelevitch C. The Brugada syndrome: Diagnostic criteria and cellular mechanisms. Eur. Heart J. 2001;22:356–363.
4. Brugada J, Brugada R, Brugada P. Right bundle branch block, ST-segment elevation in lead V1 through V3 and
sudden cardiac death: A marker for sudden death in patients without demonstrable structural heart disease.
Circulation. 1998;97:457–460.
5. Brugada P, Brugada J, Roy D. Brugada syndrome 1992–2012: 20 years of scientific excitement, and more. Eur.
Heart J. 2013;34:3610–3615.

208  u  Section 7: Brugada Syndrome


CASE
7.5
Alan Cheng, MD
Jane E. Crosson, MD

Patient History
A 1-year-old male with a family history of sudden death was diagnosed with flu-like symptoms. His
parents witnessed an episode of syncope. While awake and otherwise without complaints, this ECG
was obtained in the presence of a mild fever.

Figure 7.5.1

Discussion
This tracing demonstrates sinus rhythm with a rate of 120 bpm, normal for a 1-year-old boy. What
is most remarkable about this ECG is the marked ST segment elevation noted predominantly in
leads V1 and V2, strongly suggestive of Brugada syndrome.1 There is also mild PR interval
prolongation for this patient to 180 ms. Children with Brugada often have conduction delay, with
broad P waves, mild PR prolongation and mild QRS widening, a different pattern than adults. In a
young person, also consider the possibility of anomalous origin of the left coronary artery. In this
situation, ischemia or infarction is usually mediated by exercise. The presence of fever is a
provocative factor for the Brugada signature.

Reference
1. Arbelo E, Brugada J. Risk Stratification and Treatment of Brugada Syndrome. Curr. Cardiol. Rep. 2014;16:508.

Case 7.5  u  209


CASE
7.6 N. A. Mark Estes III, MD

Patient History
ECG of a 58-year-old female presenting with fever due to urosepsis.

Figure 7.6.1

Question
What is the ECG abnormality?

Discussion
The ECG demonstrates a Brugada type 1 pattern with ST segment elevation in leads V1–V3. This
type 1 pattern is the only ECG criterion that is diagnostic of the Brugada syndrome. The type 1 ECG
is characterized by a J elevation ≥2 mm (0.2 mV), a coved-type ST segment followed by a negative T
wave. ECG findings of types 2 and 3 Brugada ECG patterns are as follows:

Table 7.6.1
ST segment abnormalities in the different types of Brugada ECG patterns
Type 1 Type 2 Type 3
J-wave amplitude ≥ 2 mm ≥ 2 mm ≥ 2 mm

T wave Negative Positive or biphasis Positive

ST-T configuration Coved type Saddleback Saddleback

ST segment (terminal portion) Gradually descending Elevated ≥ 1 mm Elevated < 1 mm

210  u  Section 7: Brugada Syndrome


Patient History
ECG of a 58-year-old female shown in Figure 7.6.2 taken at the time of resolution of fever after
presenting with urosepsis. (Same as previous patient mentioned in Figure 7.6.1.)

Figure 7.6.2

Question
What ECG abnormality has resolved since the prior tracing?

Discussion
The 2 mm of ST segment elevation seen in the prior ECG demonstrates resolution of the Brugada
type 1 pattern with ST segment elevation in leads V1–V3 seen when the patient was febrile.

Case 7.6  u  211


CASE
7.7 Majid Haghjoo, MD

Patient History
A 32-year-old male presented with one episode of traumatic syncope. He had a history of nocturnal
palpitations. The following ECG (Figure 7.7.1) has been recorded from the patient.

Figure 7.7.1  Typical Brugada ECG pattern. This ECG clearly shows sinus rhythm with coved-type ST-segment elevation, J-point elevation,
and T-wave inversion in leads V1–V2 (blue arrow). There is also saddle-back ST-segment elevation and positive T wave in lead V3 (orange
arrow).

Discussion
Brugada syndrome should be considered in differential diagnosis of a young male with history of
syncope.1 This syndrome is characterized by typical coved-type ST-segment elevation and T-wave
inversion in at least two leads of V1–V3. ST-segment elevation is usually more prominent in lead V2.

Reference
1. Brugada P, Brugada J. Right bundle branch block, persistent ST segment elevation and sudden cardiac death: A
distinct clinical and electrocardiographic syndrome. A multicenter report. J. Am. Coll. Cardiol.
1992;20(6):1391–1396.

212  u  Section 7: Brugada Syndrome


CASE
7.8
Nicolas Derval, MD
Arnaud Denis, MD

Patient History
A 33-year-old male was referred to our institution for investigation of type 1 Brugada syndrome
fortuitously discovered on a routine ECG. The patient had no family history of structural heart
disease or sudden cardiac death and no personal medical history. He was completely asymptomatic
at the time of the initial examination.

Question
What is the diagnosis?

Figure 7.8.1

The 12-lead ECG at admission reveals a normal sinus rhythm with a significant ST-segment
elevation of more than 2 mm in leads V1 and V2, which is diagnostic of a type 1 Brugada syndrome.1
There is also a significant slurred J wave in the inferior leads diagnostic of early repolarization
(ER) pattern.1
At baseline, the maximal amplitude of the J wave was 5 mm (0.5 mV) in lead V1 and 4 mm
(0.4 mV) in lead III.

Case 7.8  u  213


Figure 7.8.2  ECG performed during Valsalva maneuvers.

We observe significant change in the J wave in amplitude and distribution. Immediately after
the end of maneuvers (*), note the dramatic change in J-wave distribution shifting from type 1
Brugada pattern associated with an inferior J wave to type 3 Brugada pattern (dotted arrows)
associated with a major inferior J wave (solid arrow). The J wave became maximal in the inferior
leads and extended all the way to the T wave (amplitude of 5 mm in lead III), while J wave decreased
in the anterior leads (disappeared in lead V1 and maximal maximal amplitude of 4 mm in lead V2).
After the end of the Valsalva maneuver, a gradual transition of the ECG back to its initial form
was observed.

214  u  Section 7: Brugada Syndrome


Figure 7.8.3  Dynamic change in the J wave spontaneously recorded by telemetry.

On day 1, at 10:10 am, the J wave is predominant in the anterior leads with type 1 Brugada
pattern. A short run of premature ventricular complex (PVC) originating from the right outflow
tract was recorded at the same time.
On day 1, at 10:23 am, we observed an asymptomatic episode of polymorphic nonsustained
ventricular tachycardia (VT). Again, the morphology of the first beat of the nonsustained
polymorphic VT was consistent with an origin from the right ventricular outflow tract.
On day 2, at 10:48 am, the J wave is predominant in the inferior leads with inferior ER pattern. A
short run of PVC originating from the inferior aspect of the left septum was recorded at the same
time.
On day 2, at 11:10 am, the J wave was almost normalized and only a discrete J-point elevation
was identified in the inferior leads. Of note, the heart rate was faster, suggesting a higher
sympathetic tone at that time.
We observed in this patient, the phenotypic association of type 1 Brugada syndrome and ER
with rapid dynamic changes in the J wave in location and amplitude. The changes were likely
associated with autonomic modulation (ER pattern predominant in the most vagal period, Brugada
pattern predominant in a more neutral period). It was also associated with ventricular arrhythmias
originated from the location of the J-wave abnormality.
This feature was a strong argument in favor of the malignancy of the J wave and finally led us to
propose subcutaneous ICD implantation to the patient.

Case 7.8  u  215


Reference
1. Priori SG, Wilde AA, Horie M, et al. HRS/EHRA/APHRS expert consensus statement on the diagnosis and
management of patients with inherited primary arrhythmia syndromes: Document endorsed by HRS, EHRA, and
APHRS in May 2013 and by ACCF, AHA, PACES, and AEPC in June 2013. Heart Rhythm. 2013;10(12):1932–1963.
doi:10.1016/j.hrthm.2013.05.014.

216  u  Section 7: Brugada Syndrome


CASE
7.9
Mohammad-Ali Jazayeri, MD
Mohammad-Reza Jazayeri, MD

Patient History
A 76-year-old male with a history of essential hypertension, hyperlipidemia, chronic kidney disease,
benign prostatic hyperplasia, and prior cerebrovascular accident presented with failure to thrive
and sepsis secondary to methicillin-sensitive Staphylococcus aureus (MSSA) bacteremia. His initial
ECG on arrival to the emergency department is shown in Figure 7.9.1.

Figure 7.9.1  ECG recorded on presentation. See text for further details.

Question
Which of the following best describes the abnormalities noted in this ECG?
A. Acute coronary syndrome
B. Brugada pattern/syndrome
C. QT interval prolongation
D. Both B and C
E. Early repolarization

Comment
The correct answer is D, both Brugada pattern/syndrome and QT interval prolongation. In this
patient presenting from a nursing home with a preceding history of poor oral intake for days to

Case 7.9  u  217


weeks, initial serum sodium was measured at greater than 169 mEq/L (upper limit of measurement).
Other pertinent laboratory results collected at the time of presentation are listed in Table 7.9.1.

Table 7.9.1  Initial Laboratory Values at the Time of Presentation. (Reference Ranges Are Presented in Brackets.)
Complete Blood Count: Basic Metabolic Panel: Arterial Blood Gas:
Hemoglobin 10.6 g/dL [13.1 – 17.5 g/dL] Potassium 3.2 mEq/L [135 – 148 mEq/L] pH 7.41 [7.35 – 7.45]
WBC count 19,600 [4.0 – 10.0 k/cm3] Chloride 134 mEq/L [100 – 108 mEq/L] pCO2 29 [35 – 45
Platelet count 464,000 [150 – 400 k/cm3] CO2 22 mEq/L [22 – 30 mEq/L] mm]
Cardiac troponin I 0.519 ng/mL [≤0.030 Serum creatinine 1.26 mg/dL [0.70 – 1.25 pO2 74 [75 – 85 mm]
ng/mL] mg/dL] CO2 29 [22 – 26
Normal hepatic function panel Ionized calcium 5.43 [4.40 – 5.20 mg/dL] mEq/L]
Albumin 2.2 g/dL [3.8 – 5.1 g/dL] Magnesium 2.4 mEq/L [1.3 – 2.0 mEq/L] O2 Sat 95%
Serum osmolality 354 mOsm/kg [285 – 305
mOsm/kg]

The presenting electrocardiogram (Figure 7.9.1) was suspicious for an atypical Brugada pattern,
featuring J-point elevation of approximately 4 mm, “coved” ST-segment, and negative T wave most
prominent in leads V3–V5, with a similar pattern also observed in the inferior leads, II and aVS.
There was suggestion of prior inferior myocardial infarction as well as a prolonged QT interval of
approximately 580 ms. Prior ECG reports from a local hospital within the preceding year made no
mention of ST-T abnormalities at baseline, and there was no prior record of hypernatremia either.
His ECG was also notable for the presence of low voltages, particularly in the limb leads, which was
consistent with his prior ECG and likely related to his increased anterior-posterior diameter.
His course was complicated at the outset by acute encephalopathy, suspected to be metabolic in
origin, in the setting of prior cerebrovascular accident and sepsis due to MSSA bacteremia. Given
his concerning initial ECG and myonecrosis, cardiology consultation was obtained. A formal
transthoracic echocardiogram demonstrated a large anterior and apical wall motion abnormality
consistent with takotsubo cardiomyopathy. His entrusted decision makers requested a limited care
plan, so a supportive approach with correction of his metabolic derangements and antibiotic
therapy was pursued, and urgent angiography was deferred. Resolution of his hypernatremia, as
illustrated in Table 7.9.2, with correction of the free water deficit, resulted in progressive
normalization of the ST-T changes noted in Figure 7.9.1, as illustrated in the ECGs obtained four
hours after presentation (Figure 7.9.2) and then 36 hours after presentation (Figure 7.9.3). The
patient was ultimately transferred out of the medical ICU and discharged to hospice care in a
skilled nursing facility in keeping with his wishes.

218  u  Section 7: Brugada Syndrome


Figure 7.9.2  ECG obtained 4 hours after presentation, showing a pattern similar to the initial ECG with perhaps slight improvement.
Concomitant serum sodium measurement was 168 mEq/L.

Figure 7.9.3  ECG recorded 36 hours after presentation showing normal sinus rhythm, resolution of previously noted type 1 Brugada
pattern, low limb lead voltage, loss of R-wave amplitude in leads V2-V6, and persistent QT interval prolongation at 570 ms.

Table 7.9.2  Serum Sodium Trend From the Time of Presentation.


Presentation (P) P + 4 hours P + 36 hours P + 60 hours P + 84 hours

>169 mEq/L 168 mEq/L 154 mEq/L 154 mEq/L 147 mEq/L

Case 7.9  u  219


Discussion
This case illustrates an interesting metabolic disturbance of hypernatremia associated with type 1
Brugada pattern on ECG. Descriptions of Brugada pattern in the setting of other electrolyte
disturbances including hypokalemia1,2 and hyperkalemia,3 as well as hypercalcemia,4 are well-
described. In the case presented, the major metabolic disturbance was marked hypernatremia.
Though mild hypokalemia was also present on presentation, it is unlikely to have been a significant
contributor to his ECG findings, as demonstrated later in the patient’s course when his free water
deficit had been corrected yet he continued to suffer from recurrent hypokalemia due to refeeding
syndrome with no similar ECG changes noted. Furthermore, the patient was being treated for sepsis
throughout the admission and was found to be bacteremic. He was not febrile, however, and
therefore fever-induced Brugada was considered to be an unlikely explanation for his condition.
Though echocardiographic findings were suggestive of takotsubo cardiomyopathy, the ECG pattern
is not typical for takotsubo, and the timeline of ECG normalization seems too short as compared to
what would be expected with takotsubo cardiomyopathy. It is, however apparent from the follow-up
ECG (Figure 7.9.3) that asignificant interval change in R-wave amplitude has occurred in an
anterolateral distribution, consistent with the echocardiographic wall motion abnormalities and
troponin elevation suggesting myocardial infarction. The list of etiologies capable of producing this
pattern continues to grow,5 but we believe this is the first reported case where severe hypernatremia
has been noted in association with this pattern.
Cases of hyponatremia have been reported with similar electrocardiographic findings,6,7 which
is a more intuitive metabolic disturbance for one to suspect upon viewing the initial ECG. In this
case the presenting ECG was atypical for the fact that it featured the type 1 Brugada pattern in only
the left precordial leads, though lateral manifestations have also been described.8 Despite
uncertainty regarding the precise underlying mechanism for Brugada pattern, a prevailing
hypothesis suggests that attenuation of the late sodium channel current in phase 0 of the cardiac
cycle action potential (AP) is implicated, producing a dampening of the rate of rise and amplitude of
the AP upstroke, as well as a widening of the QRS complex. This is thought to produce a transmural
voltage gradient in the right ventricle, which effectively prolongs the refractory period and increases
the risk of malignant arrhythmia triggered by extrasystoles.2
Contrary to cases of hyponatremia, it is difficult to reconcile the marked hypernatremia with a
reduced overall inward sodium current. One possibility is that because the hypernatremia likely
developed gradually in the setting of failure to thrive and significant dehydration, there may have
been some compensatory down regulation of sodium receptors (e.g., Nav1.5). This, coupled with a
recent small dose increase in his selective serotonin reuptake inhibitor and the presence of other
potentially implicated drugs, such as the tetracyclic antidepressant mirtazapine,4 may have
potentiated the blockade of a more limited number of available sodium channels at the membrane
level, giving rise to the observed pattern. Unfortunately, because of his overall trajectory and
clinical course, an opportunity for further provocative testing did not arise prior to his demise.

References
1. Gazzoni GF, Borges AP, Bergoli LC, et al. Brugada-like electrocardiographic changes induced by hypokalemia. Arq.
Brad. Cardiol. 2013;100(3):e35–e37.
2. Genaro NR, Anselm DD, Cervino N, et al. Brugada phenocopy clinical reproducibility demonstrated by recurrent
hypokalemia. Ann. Noninvasive Electrocardiol. 2014;19(4):387–390.

220  u  Section 7: Brugada Syndrome


3. Reingardiene D, Vilcinskaite J, Bilskiene D. Brugada-like electrocardiographic patterns induced by hyperkalemia.
Medicina (Kaunas). 2013;49(3):148–153.
4. Shimizu W. Acquired forms of the Brugada syndrome. J. Electrocardiol. 2005;38(4 Suppl):22–25.
5. Antzelevitch C, Brugada P, Borggrefe M, et al. Brugada syndrome: Report of the second consensus conference.
Circulation. 2005;111:659–670.
6. Alvarez PA, Vazquez Blanco M, Lerman J. Brugada type 1 electrocardiographic pattern induced by severe
hyponatremia. Cardiology. 2011;118(2):97–100.
7. Tamene A, Sattiraju S, Wang K, et al. Brugada-like electrocardiography pattern induced by severe hyponatraemia.
Europace. 2010;12(6):905–907.
8. Letsas KP, Weber R, Tsikrikas S, et al. Atypical Brugada ECG phenotype involving ST-segment elevation in lateral
leads. Hellenic J. Cardiol. 2010;51(6):563–564.

Case 7.9  u  221


CASE
7.10
Andrew D. Krahn, MD
Christian Steinberg, MD

Patient History
A 36-year-old male of Filipino–Chinese descent experienced ventricular fibrillation (VF) arrest
during sleep. He was successfully resuscitated and transferred to the intensive care unit of a tertiary
university center. The patient had no prior medical history, but his family history was positive for
unexplained cardiac death of a paternal uncle at age 60. Cardiac imaging did not reveal any
structural heart disease and coronary angiography was normal. Figure 7.10.1A shows his ECG after
resuscitation, which is compatible with a spontaneous type 1 Brugada pattern. The underlying
complete right bundle branch block (RBBB) was previously documented on a random ECG
(Figure 7.10.1B). Over the next 5 years, none of the repeat ECGs demonstrated a spontaneous type 1
pattern (Figure 7.10.1C).

Figure 7.10.1  Brugada syndrome with underlying RBBB. (A) ECG post-resuscitation from VF arrest. Sinus rhythm at a heart rate of
63 bpm. Presence of a type 1 Brugada pattern in leads V1-V2 (arrows) with J-point elevation up to 4 mm in lead V1. The typical coved
ST-segment associated with a negative T wave is shown. There is marked terminal QRS fragmentation in lead V2. Note the underlying
complete RBBB with a typical rsR′ pattern in lead V1. Presence of pseudo Q waves in the inferior leads. (B) Random ECG 3 years prior to
the VF arrest. Sinus rhythm at a heart rate of 88 bpm. Presence of a typical RBBB with QRS duration of 144 ms. Typical rsR’ pattern in
lead V1 and slurred S wave in leads V5-V6 (dashed arrows). Note the absence of J point abnormalities in leads V1-V2 (black arrows). Pseudo
Q waves in the inferior leads. (C) ECG during follow-up, 5 years after VF arrest. Note the striking similarity with the ECG prior preceding
the cardiac arrest. Again, absence of J-point abnormalities and no obvious Brugada pattern (black arrows). Presence of known RBBB.

222  u  Section 7: Brugada Syndrome


Interpretation
(A) Sinus rhythm at a heart rate of 63 bpm. Presence of a type 1 Brugada pattern in leads V1–V2
(arrows). Presence of an underlying complete right bundle branch block (RBBB).
(B) Sinus rhythm at a heart rate of 88 bpm. Presence of a complete RBBB with QRS duration of
144 ms. No J-point abnormalities.
(C) Sinus rhythm at a heart rate of 67 bpm. Presence of a complete RBBB with QRS duration of
140 ms. No J-point abnormalities.

Discussion
Brugada syndrome is a primary electrical disorder that is characterized by abnormal J-point
elevation and ST-changes. Most Brugada patients are asymptomatic, but a subset of patients
develops life-threatening ventricular arrhythmias.1 Nineteen different gene defects have been linked
to Brugada syndrome, but a genetic cause is only identified in 20%–30% of affected patients, with
SCN5A mutations representing the most common gene defects.2,3
Three different ECG Brugada patterns have been described, but only a spontaneous or
inducible type 1 pattern is diagnostic for Brugada syndrome.4 Type 1 Brugada pattern is
characterized by a J-point elevation ≥ 2 mm in leads V1 or V2 associated with a coved ST segment
and a negative T wave as demonstrated in Figure 7.10.1A.1 Ventricular arrhythmia usually occurs at
rest or during sleep and is characterized by polymorphic ventricular tachycardia or VF, typically
initiated by short-coupled premature ventricular contractions.3,5
Initial studies reported an increased arrhythmic risk for individuals with spontaneous type 1
ECG pattern compared to those with inducible or intermittent type 1 ECG pattern. However, this
concept was questioned by more recent studies demonstrating that the temporal ECG fluctuations
are more important than previously estimated. Studies using prolonged ECG monitoring showed
that only 12% of individuals with spontaneous type 1 pattern exhibit a persistent type 1 ECG (> 85%
of the time) and the majority will show nondiagnostic ECG patterns during follow-up. On the other
hand, at least 20% of individuals with initially inducible Brugada pattern will show a spontaneous
type 1 pattern over time.6,7
An interesting feature of the present Brugada case is the underlying RBBB. The prevalence of
RBBB in Brugada patients is reported to be 7.7%.8 Concomitant RBBB in Brugada syndrome can
mask an underlying Brugada pattern, rendering the ECG analysis more challenging.8,9 Isolated
RBBB was historically considered a benign finding, but a recent large-scale, population-based study
suggested that RBBB is an independent predictor for cardiac mortality.10 Interestingly, the presence
of RBBB in patients with idiopathic VF and early repolarization syndrome seems to be associated
with increased risk for arrhythmic events.11 The prognostic value of RBBB in Brugada syndrome is
unknown so far.

References
1. Antzelevitch C, Brugada P, Borggrefe M, et al. Brugada syndrome: Report of the second consensus conference:
Endorsed by the Heart Rhythm Society and the European Heart Rhythm Association. Circulation. 2005;111(5):
659–670.
2. Berne P, Brugada J. Brugada syndrome 2012. Circ. J. 2012;76(7):1563–1571.

Case 7.10  u  223


3. Obeyesekere MN, Antzelevitch C, Krahn AD. Management of ventricular arrhythmias in suspected
channelopathies. Circ. Arrhythm. Electrophysiol. 2015;8(1):221–231.
4. Priori SG, Wilde AA, Horie M, et al. HRS/EHRA/APHRS expert consensus statement on the diagnosis and
management of patients with inherited primary arrhythmia syndromes: Document endorsed by HRS, EHRA, and
APHRS in May 2013 and by ACCF, AHA, PACES, and AEPC in June 2013. Heart Rhythm. 2013;10(12):1932–1963.
5. Antzelevitch C, Yan GX. J-wave syndromes: Brugada and early repolarization syndromes. Heart Rhythm.
2015;12(8):1852–1866.
6. Cerrato N, Giustetto C, Gribaudo E, et al. Prevalence of type 1 Brugada electrocardiographic pattern evaluated by
twelve-lead twenty-four-hour holter monitoring. Am. J. Cardiol. 2015;115(1):52–56.
7. Veltmann C, Schimpf R, Echternach C, et al. A prospective study on spontaneous fluctuations between diagnostic
and non-diagnostic ECGs in Brugada syndrome: Implications for correct phenotyping and risk stratification. Eur.
Heart J. 2006;27(21):2544–2552.
8. Wada T, Nagase S, Morita H, et al. Incidence and clinical significance of Brugada syndrome masked by complete
right bundle-branch block. Circ. J. 2015;79(12):2568–2575.
9. Aizawa Y, Takatsuki S, Sano M, et al. Brugada syndrome behind complete right bundle-branch block. Circulation.
2013;128(10):1048–1054.
10. Bussink BE, Holst AG, Jespersen L, et al. Right bundle branch block: Prevalence, risk factors, and outcome in the
general population: results from the Copenhagen City Heart Study. Eur. Heart J. 2013;34(2):138–146.
11. Aizawa Y, Takatsuki S, Kimura T, et al. Ventricular fibrillation associated with complete right bundle branch block.
Heart Rhythm. 2013;10(7):1028–1035.

224  u  Section 7: Brugada Syndrome


CASE
Yuji Nakazato, MD, PhD 7.11
Patient History
A 49-year-old female patient was admitted with syncope. She was implanted with a dual chamber
pacemaker (DDD) due to sick sinus syndrome 10 months prior. Immediately after admission, she
experienced multifocal premature ventricular contractions followed by ventricular fibrillation (VF)
and was resuscitated in the intensive care unit.

Question
What is the cause of ST elevation (arrow) on the admission ECG (Figure 7.11.1B)?

Figure 7.11.1

Case 7.11  u  225


Figure 7.11.2

Discussion
In typical Brugada syndrome, coved-type ST elevation in precordial leads is widely accepted. ST
elevation is often known to be prominent before the development of VF; however, it is rare to be
observed during ventricular pacing. In Figure 7.11.1, marked bizarre ST elevation in leads V1–V3
(Figure 7.11.1B) compared to the baseline ECG (Figure 7.11.1A) can be seen, particularly in lead V2
(arrow). For the confirmation of ST elevation, pacing mode was changed from DDD to single-
chamber of the atrium (AAI) mode. Then, saddleback-type ST elevation was clearly demonstrated
in leads V1–V3, which improved by isoproterenol infusion.

References
1. Brugada P, Brugada J. Right bundle branch block, persistent ST segment elevation and sudden cardiac death:
A distinct clinical and electrocardiographic syndrome: A multicenter report. J. Am. Coll. Cardiol. 1992;20:1391–
1396.
2. Nakazato Y, Suzuki T, Yasuda M, et al. Manifestation of Brugada syndrome after pacemaker implantation in a
patient with sick sinus syndrome. J. Cardiovasc. Electrophysiol. 2004:15:1328–1330.

226  u  Section 7: Brugada Syndrome


SECTION 8A
Inappropriate Sinus Tachycardia

CASE
8A.1
Balaji Krishnan, MD
David G. Benditt, MD

Patient History
A 35-year-old female with no past cardiac or medical history of note presented with approximately
1 to 2 years of persistent palpitations with associated symptoms of worsening exercise intolerance.
The echocardiogram revealed normal ventricular function.

Figure 8A.1.1  A rhythm strip with a persistent sinus tachycardia.

ECG Masters’ Collection: Favorite ECGs from Master Teachers Around the World © 2017 Mohammad Shenasa, Mark E. Josephson,
N. A. Mark Estes III, Ezra A. Amsterdam, Melvin Scheinman. Cardiotext Publishing, ISBN: 978-1-942909-08-8.   227
Figure 8A.1.2  A 24-hour heart rate histogram. Heart rate is indicated on the ordinate and time of the day on the abscissa. Note the
absence of without diurnal heart rate changes, yet there is more variability than would be expected with a nonsinus atrial tachycardia.

Discussion
During initial evaluation, ambulatory ECG monitoring (AECG) conducted over 3–4 days was
interpreted as either an atrial or sinus tachycardia (Figure 8A.1.1). The hourly heart rate (HR)
histogram (Figure 8A.1.2) for one 24-hour period revealed absence of the usual diurnal HR changes
expected in healthy individuals; the mean HR was 105 bpm and showed only minor changes
throughout the recording period. A nonsinus atrial tachycardia would be expected to be even more
“fixed” in terms of HR. Electrophysiology study including autonomic interventions (e.g., carotid
massage, Valsalva maneuver) were undertaken and also supported a diagnosis of inappropriate sinus
tachycardia (IST).1
Initial treatment with a beta-blocker and calcium-channel blocker was ineffective (ivabradine
was not yet available),2,3 and radiofrequency sinus node modification was undertaken with a
decrease in mean HR to 80 bpm. The drugs were continued, and the patient felt better for 2 years
but symptoms recurred. On repeat AECG monitoring, her mean HR was approximately 100 bpm.
Repeat sinus node modification decreased her mean HR to 75 bpm. Beta-blocker and calcium-
channel blocker were continued.3 At 2-year follow-up, her exercise tolerance improved and mean
HR was approximately 80 bpm.
This work was supported in part by a grant from the Dr. Earl E. Bakken Family in support of
heart-brain research.

References
1. Olshansky B, Sullivan RM. Inappropriate sinus tachycardia. J. Am. Coll. Cardiol. 2013;61(8):793–801.
2. Raj SR. Highlights in clinical autonomic neurosciences: Treatment insights for postural tachycardia syndrome and
inappropriate sinus tachycardia. Auton. Neurosci. 2013;177(2):72–73.
3. Ptaszynski P, Kaczmarek K, Ruta J, et al. Metoprolol succinate vs. ivabradine in the treatment of inappropriate sinus
tachycardia in patients unresponsive to previous pharmacological therapy. Europace. 2013;15(1):116–121.

228  u  Section 8A: Inappropriate Sinus Tachycardia


SECTION 8B
Sinus Node Reentrant Tachycardia

CASE
8B.1
George D. Katritsis, MBChB, BSc
Demosthenes G. Katritsis, MD, PhD

Figure 8B.1.1

ECG Masters’ Collection: Favorite ECGs from Master Teachers Around the World © 2017 Mohammad Shenasa, Mark E. Josephson,
N. A. Mark Estes III, Ezra A. Amsterdam, Melvin Scheinman. Cardiotext Publishing, ISBN: 978-1-942909-08-8.   229
Patient History
Supraventricular tachycardia in a 68-year-old male with coronary artery disease who presented
with episodes of palpitations. An intravenous bolus of adenosine terminates the tachycardia.

Question
What is the most likely diagnosis of this case?

Interpretation
P waves are similar during tachycardia that is terminated by adenosine following prolongation of
the PR interval. This response and the similarity of P waves during tachycardia with those during
sinus rhythm suggest sinus nodal reentry. A cardioselective beta-blocker is the appropriate therapy.

230  u  Section 8B: Sinus Node Reentrant Tachycardia


SECTION 8C
Atrial Tachycardia/Atrial Flutter

CASE
8C.1
Marc Dubuc, MD
Jason Andrade, BSc, MD

Patient History
A 79-year-old male presented to the emergency department with a 16-hour history of irregular
palpitations.

Figure 8C.1.1

ECG Masters’ Collection: Favorite ECGs from Master Teachers Around the World © 2017 Mohammad Shenasa, Mark E. Josephson,
N. A. Mark Estes III, Ezra A. Amsterdam, Melvin Scheinman. Cardiotext Publishing, ISBN: 978-1-942909-08-8.   231
Discussion
This ECG (Figure 8C.1.1) demonstrates a regular atrial tachyarrhythmia consistent with atrial
flutter (low-amplitude F waves are seen in the inferior leads) at a cycle length of 240 ms (arrow).
Conduction to the ventricle is regularly irregular, alternating between a fusion of the
atrioventricular node and accessory pathway (*), and exclusive accessory pathway conduction (#).
This tracing requires differentiation between intermittent preexcitation versus frequent premature
ventricular contractions. Note that couplets of wide complex beats occur in fixed relationship to the
flutter interval, i.e., two flutter cycles.

232  u  Section 8C: Atrial Tachycardia/Atrial Flutter


CASE
8C.2
Marc Dubuc, MD
Jason Andrade, BSc, MD

Patient History
A 50-year-old male presented to hospital with a 72-hour history of recurrent rapid palpitations and
presyncope.

Figure 8C.2.1

Discussion
This ECG (Figure 8C.2.1) demonstrates incessant focal atrial tachycardia. At the beginning of the
tracing, a long R-P narrow complex supraventricular tachyarrhythmia at a rate of approximately 110
bpm is noted. After the eighth QRS complex, the tachyarrhythmia spontaneously terminates (last
atrial impulse is conducted to the ventricle). Following a brief pause, there is a spontaneous sinus
impulse (*) followed by tachyarrhythmia resumption at an atrial rate of approximately 200 bpm for
two impulses (arrows). Following a second pause, there is again a sinus impulse (second *) followed
by tachycardia resumption (note the stable coupling interval between the first beat of
tachyarrhythmia and the preceding sinus beat). In this latter case, the atrial rate is intermediate
(approximately 160 bpm); however, the origin of the tachyarrhythmia is stable; that is, there is
unchanged tachyarrhythmia P-wave morphology, which compared to the sinus P wave is relatively
narrow and prominent (high amplitude), suggesting a high septal or cristae origin.

Case 8C.2  u  233


CASE
8C.3 Jonathan Kalman, MBBS, PhD

Patient History
A 55-year-old female with incessant tachycardia arrived at the hospital after three failed attempts at
treatment by another institution.

Figure 8C.3.1

Discussion
Ablation of this focal atrial tachycardia was successfully carried out in the base of the left atrial
appendage. The P-wave morphology is broad, upright, and notched in lead V1 and in inferior leads,
consistent with a left superior pulmonary vein tachycardia origin. However, the deeply inverted
P wave in lead I strongly indicates a left atrial appendage origin. When tachycardia is incessant, the
left and right atrial appendages are a classic site of origin.

234  u  Section 8C: Atrial Tachycardia/Atrial Flutter


CASE
Jonathan Kalman, MBBS, PhD 8C.4
Patient History
A 31-year-old male with incessant atrial tachycardia (AT) with variable conduction to the ventricle.

Figure 8C.4.1

Discussion
The AT cycle length is 360 ms.
The P-wave morphology is broad, upright, and notched in lead V1 (red arrows) and in inferior
leads, suggesting a left superior pulmonary vein (LSPV) origin. The low-amplitude P wave in lead I
is also consistent with this site.
This AT was mapped distally within a small branch of the LSPV, where catheter pressure
terminated the tachycardia. Ablation was not performed at this distal site. Instead, the PV was
isolated in the antral region.

Case 8C.4  u  235


CASE
8C.5 Jonathan Kalman, MBBS, PhD

Patient History
An 18-year-old male presented with severe congestive heart failure, incessant tachycardia at 120
bpm, and an ejection fraction of 15%.

Figure 8C.5.1

Discussion
The tachycardia is inverted in lead V1 with late precordial transition. This suggests either the right
atrial appendage (RAA) or the tricuspid annulus. The RAA is a classic site for incessant
tachycardias and tachycardia-mediated cardiomyopathy. This focus was ablated in the lateral base
of the RAA. Within 4 months, the ejection fraction had returned to normal.

236  u  Section 8C: Atrial Tachycardia/Atrial Flutter


CASE
Jonathan Kalman, MBBS, PhD 8C.6
Patient History
A 52-year-old female presented with paroxysmal atrial tachycardia (AT).

Figure 8C.6.1

Discussion
This ECG (Figure 8C.6.1) shows an AT, which is negative/positive biphasic in lead V1 and in the
inferior leads. This is a nonspecific pattern and has been observed from sites on the right or left side
of the septum and in the noncoronary aortic cusp. This tachycardia was successfully ablated in the
right perinodal region using cryoablation to avoid AV nodal damage.

Case 8C.6  u  237


Patient History
A 28-year-old female presented with AT.

Figure 8C.6.2

Discussion
This ECG (Figure 8C.6.2) is another AT which is negative positive in lead V1 and the inferior leads.
Note that the spontaneous ventricular ectopics (asterisk) unmask the P-wave morphology (red
arrows). During sustained 1:1 tachycardia, the P wave cannot be clearly seen. On this occasion, the
focus was mapped to the left septum immediately adjacent to the right perinodal region. Ablation at
this site eliminated the tachycardia.
When this ECG pattern is observed, careful mapping on both sides of the septum and within
the aortic noncoronary cusp (NCC) is necessary.

Figure 8C.6.3  Composite figure of the two preceding figures showing the ECG similarity for tachycardia originating on either side of the
septum.

238  u  Section 8C: Atrial Tachycardia/Atrial Flutter


CASE
8C.7
Gilles Lascault, MD
Xavier Copie, MD

Patient History
A 53-year-old male patient, with a long history of symptomatic paroxysmal atrial fibrillation,
underwent radiofrequency ablation in 2010 (isolation of pulmonary veins). A second ablation was
performed in February 2014 for persistent atrial fibrillation: reisolation of the two left and the right
superior pulmonary veins, extensive defragmentation, roof and mitral isthmus lines. During
follow-up the patient still complained of regular palpitations. Repeat ECGs showed the same aspect
of atypical atrial flutter.
The ECG (Figure 8C.7.1) shows atrial flutter with an atrial cycle length of 280 ms. The F-wave
morphology is negative in leads I, II, and V4 –V6, positive in leads V1–V3, isoelectric in leads III and
aVF, and slightly negative in aVL. Note the sharp F waves in leads V1 and V2 (red arrows). The
ventricular response is irregular (between 2/1 and 5/1). The morphology of the atrial waves is not
typical of a right atrial isthmus-dependent flutter but rather that of a left atrial flutter. 3D color
mapping (Biosense Webster Carto) and ablation confirmed a clockwise perimitral flutter.

Figure 8C.7.1  Perimitral flutter.

Question
What is the ECG definition of atrial flutter?

Case 8C.7  u  239


Answer
In order to diagnose atrial flutter (right or left) on the standard ECG, a 12-lead ECG recording is
required. In atrial flutter we observe, in some derivations, no return of the atrial electrical activity
to the isoelectric line. Instead, the atrial electrical activity crosses the isoelectric line in an oblique
way (leads II, III, aVF in typical right atrial flutter). In other instances, there may be a return to the
isoelectric line (for example in V1 in typical atrial flutter). When there is a clear return to the
isoelectric line in all derivations and discrete P′ waves, we speak about atrial tachycardia which may
be focal or sometimes reentrant (in that case, the interval between 2 P′ waves is not electrically
silent but is occupied by very low-amplitude potentials, which are too small to be visible on the
ECG). These potentials will be recorded intracardially. As mentioned above, some ECG features can
help to anticipate the right or left location of the flutter circuit.

Discussion
Left atrial flutter is a frequent complication of atrial fibrillation ablation, particularly ablation
including linear lesions (i.e., mitral isthmus and roof lines). Right atrial flutter is also possible when
cavotricuspid isthmus has not been previously ablated. It is important to try to predict the location
of a flutter circuit, especially when a new radiofrequency ablation procedure is planned. Indeed, the
procedure strategy is markedly different between right atrial and left atrial flutter ablations. The
ECG pattern of left atrial flutter is very heterogeneous, especially after ablation, since many
mechanisms may be encountered: i.e., macroreentrant or microreentrant circuits. However, as
shown by Bocheyer et al.,1 it is sometimes possible to grossly anticipate the location of the circuit.
For example, flattened or low amplitude of F waves in inferior leads are in favor of a left atrial
flutter, as well as spike F waves in lead V1. This aspect was seen on the present ECG of a clockwise
perimitral flutter.

Reference
1. Bocheyer A, Yang Y, Cheng J, et al. Surface electrocardiographic characteristics of right and left atrial flutter.
Circulation. 2003;108:60–68.

240  u  Section 8C: Atrial Tachycardia/Atrial Flutter


CASE
8C.8
Gilles Lascault, MD
Antoine Lepillier, MD

Patient History
A 50-year-old male patient suffered from incessant atrial tachycardia associated with severe left
ventricular (LV) dysfunction (LV ejection fraction 30%). Coronary angiography was normal. The
initial diagnosis was likely an arrhythmic cardiomyopathy. A first radiofrequency ablation of this
tachycardia was performed in March 2014 with success. One year later, the patient complained of
recurrent palpitations. Repeat ECGs confirmed that the arrhythmia was intermittent.

Figure 8C.8.1  RA focus 1.

Figure 8C.8.1 shows a regular wide QRS tachycardia at a rate of 116 bpm with a left bundle
branch block (LBBB) morphology (QRS duration 130–140 ms) and left axis deviation. P′ waves are
clearly visible and are negative in inferior leads, and have a low amplitude in the other leads either
slightly positive or flattened. RP′ interval is long (440 ms), while P′R interval is short (130 ms).

Case 8C.8  u  241


Figure 8C.8.2  RA focus 2.

Figure 8C.8.2 was recorded in sinus rhythm. A blocked atrial premature beat (APB) is seen after
every two sinus beats. The morphology of each APB is constant: negative in leads II, III, and aVF,
probably positive in leads V1–V3 (spike on T waves) and rather negative in leads V4 –V6.

Question
Can P-wave morphology predict the location of focal atrial tachycardia?

Answer
It is possible to accurately predict the location of the focus and its inside the atria. Some algorithms
have been developed and may help, but they are quite complicated. In any case, they must be seen as
indicators.

Discussion
Preexisting LBBB in sinus rhythm immediately rules out ventricular tachycardia with very slow
retrograde conduction. The tachycardia is supraventricular, either junctional or atrial. It could be
paroxysmal junctional tachycardia with retrograde atrial activation through a slow-conducting
accessory pathway in the posteroseptal region or the fast-slow type of AV nodal reentrant
tachycardia. Analysis of Figure 8C.8.1 only cannot rule out those possibilities. It could also be an
atrial tachycardia originating from the posteroseptal region of the right atrium, with 1/1 AV
conduction. Figure 8C.8.2 gives some clues, showing APBs with the same morphology as the
tachycardia atrial beats. This strongly suggests an atrial focus giving rise to single premature,
nonsustained or sustained runs of tachycardia. A mapping and ablation procedure confirmed the
presence of an atrial focus close to the coronary ostium.

Reference
1. Kistler PM, Roberts-Thomson KC, Haqqani HM, et al. P-wave morphology in focal atrial tachycardia: Development
of an algorithm to predict the anatomic site of origin. J. Am. Coll. Cardiol. 2006;48:1010–1017.

242  u  Section 8C: Atrial Tachycardia/Atrial Flutter


CASE
Mohamed Magdy, MSc, L'AFSA, PhD, MD 8C.9
Patient History
A 60-year-old male presented with palpitation and chest pain. ECG upon arriving to the emergency
department is shown (Figure 8C.9.1).

Figure 8C.9.1

Note the two P waves (red arrows), followed by QRS 2:1 conduction with an atrial rate of
250 bpm.

Case 8C.9  u  243


Figure 8C.9.2

Figure 8C.9.3

244  u  Section 8C: Atrial Tachycardia/Atrial Flutter


Discussion
Transesophageal echocardiogram was performed to exclude the presence of atrial thrombus.
Intracardiac tracing showed 2:1 atrial flutter, and an ablation by cavotricuspid isthmus (CTI)
was performed.
This resulted in normal sinus rhythm (single P wave—red arrow) and the following ECG
(Figure 8C.9.4) was recorded.

Figure 8C.9.4

Case 8C.9  u  245


CASE
8C.10 John M. Miller, MD

Patient History
A 58-year-old female with a history of hypertrophic cardiomyopathy (HCM) had symptomatic atrial
fibrillation and underwent catheter ablation using wide-area antral pulmonary vein isolation. She
began to have recurrent palpitations that were found to be due to an organized atrial tachycardia/
flutter, for which she underwent another ablation procedure. After successful termination of left
atrial tachycardia during ablation, the ECG shown in Figure 8C.10.1A (standard gain) was obtained.
On careful inspection, some small deflections were observed that are better seen on the bottom
recording (Figure 8C.10.1B) of exactly the same ECG but at 4× normal gain. Here, the small
deflections are marked with black circles (or open circles where they probably occur but are
obscured by sinus P waves or QRS complexes).

Figure 8C.10.1

Question
What is the nature of these extra waves?

246  u  Section 8C: Atrial Tachycardia/Atrial Flutter


Discussion
The ECG shows sinus rhythm with a very long PR interval, right bundle branch block (RBBB), and
right axis deviation. The extra, small deflections highlighted in Figure 8C.10.1B are very narrow
and have a superior and leftward axis and are not well seen in the right precordial leads but are
inverted in the lateral precordial leads. Though difficult to diagnose as such on the ECG,
intracardiac recordings showed that these signals were due to an incessant atrial tachycardia that
was present in the previouslyisolated left pulmonary venous antrum. This is unusual in that the
isolated portion of the atrium does not usually have an independent rhythm (or only slow,
dissociated firing) and generally contains such a small muscle mass that even if it depolarized, it
would not have enough size to register on the regular ECG. In this case, an ongoing tachycardia was
present in the isolated segment, which had a large enough tissue volume (hypertrophied as part of
the patient’s HCM) that evidence of the tachycardia could be seen in Figure 8C.10A.

Lessons
Two dissociated rhythms in the same heart is not unusual, for instance sinus rhythm with complete
heart block and a junctional escape rhythm. It is very unusual, however, for two dissociated
rhythms to be occurring in the same chambers (both atria, as in this case, or both ventricles). This
can occur in transplanted hearts, in which the posterior left atrium with pulmonary vein
attachments remains with the donor heart anastomosed to it, or rarely in post-maze surgery
patients, and those who have undergone right ventricular disarticulation for treatment of
ventricular arrhythmias due to right ventricular cardiomyopathy/dysplasia.

Case 8C.10  u  247


CASE
8C.11 John M. Miller, MD

Patient History
A 42-year-old female had a 3-year history of palpitations. Physical examination was normal and
noninvasive evaluation showed a structurally normal heart. She had normal exertional capacity, but
noted irregular palpitations with increasing frequency when she took vigorous walks. An event
monitor was obtained from which a diagnosis of paroxysmal atrial fibrillation (AF) was made.
Neither beta- nor calcium-channel blockade were tolerated and when symptoms progressed (50% of
the time, associated with fatigue), she was referred for electrophysiologic evaluation. An ECG was
obtained at an outpatient consultation visit, as shown (Figure 8C.11.1).

Figure 8C.11.1

Question
What does the ECG show?

Discussion
The ECG (Figure 8C.11.1) shows an organized AT with discrete P waves that are all identical in
morphology, but with varying rate (black circles denote clear P waves, open circles indicate where
P waves are likely present but obscured by QRS complexes). The patient went on to have successful

248  u  Section 8C: Atrial Tachycardia/Atrial Flutter


catheter ablation of a focal tachycardia arising near the ostium of the left superior pulmonary vein;
her palpitations ceased entirely thereafter.

Lessons
The initial diagnosis of AF was made based from a monitor recording showing a rapid arrhythmia
with irregular RR intervals; perhaps the physician reviewing the monitor merely looked at the
irregularly irregular ventricular rate and assumed the fibrillation was the cause. Instead, it is an
organized AT with irregular, but otherwise identical P waves. This had therapeutic implications for
the patient; drug therapy may not have been different whether the rhythm was AF or AT, but an
organized AT is significantly easier to treat with catheter ablation than is AF. Further, the great
degree of irregularity strongly suggested a focal mechanism, since reentrant ATs are usually very
regular: wavefront propagation around a fixed anatomic circuit typically proceeds as rapidly as it
can, within the constraints of local refractoriness, and is rather regular. Significant alterations in
conduction velocity that present the wavefront to tissue that is still refractory from the prior AT
cycle will often result in tachycardia termination (this is part of the premise of using premature
extrastimuli or overdrive pacing to terminate reentrant arrhythmias). Thus, generally speaking, the
more irregular a tachycardia, the more likely it has a focal origin.

Case 8C.11  u  249


CASE Christopher E. Woods, MD, PhD

8C.12
Nitish Badhwar, MD
Melvin Scheinman, MD

Patient History
A 24-year-old male presented with palpitations at peak exercise. An exercise treadmill test was
preformed where he achieved 15 minutes on a Bruce Protocol, and had a peak heart rate of 193 bpm.
He failed to elicit any arrhythmias. Ambulatory ECG monitoring was done, revealing an arrhythmia
with peak exercise which is shown in Figure 8C.12.1, along with a baseline ECG. An
electrophysiologic study (EPS) was performed. A tachycardia was induced (Figure 8C.12.2) and
diagnostic maneuvers were performed (Figure 8C.12.3).

Figure 8C.12.1  A. Baseline ECG. B. Ambulatory monitoring demonstrating rapid SVT. Patient reported this only during peak exercise.

250  u  Section 8C: Atrial Tachycardia/Atrial Flutter


Figure 8C.12.2  A. Baseline intervals. HV is 50 ms as shown. B. Supraventricular tachycardia. The proximal electrodes are at the coronary
sinus os.

Case 8C.12  u  251


Figure 8C.12.3  A. Para-Hisian pacing during tachycardia at onset. B. Para-Hisian pacing during tachycardia at offset. See text for
discussion of arrowheads.

252  u  Section 8C: Atrial Tachycardia/Atrial Flutter


Questions
1. What is the diagnosis of the ECG in Figure 8C.12.1A?
2. What are the possible mechanisms of the supraventricular tachycardia (SVT) shown in Figure
8C.12.2B?
3. At the initiation of pacing during tachycardia (Figure 8C.12.3A), what diagnosis is excluded?
4. Does the offset of ventricular overdrive pacing in Figure 8C.12.3B during tachycardia help with
the diagnosis? Is this a VAAV or pseudo VAAV response, or something else?

Discussion
The baseline ECG demonstrates precordial peaked T waves and a short QT. This initially raised
concern for short QT syndrome. There was no family history of sudden cardiac death or atrial
fibrillation, no ventricular arrhythmias were seen, the patient had no syncope, and genetic testing,
although of limited value in short QT syndrome, was negative. Ambulatory monitoring shown in
Figure 8C.12.1 demonstrated a fast (mean heart rate at 250 bpm) SVT. The differential diagnosis at
this point includes atrioventricular nodal reentrant tachycardia (AVNRT), atrioventricular reentrant
tachycardia (AVRT), and atrial tachycardia. Electrophysiology testing demonstrated normal baseline
intervals (Figure 8C.12.2A) without preexcitation, and ventricular pacing showed a concentric atrial
pattern. A sustained tachycardia was demonstrated with isuprel (Figure 8C.12.2B). The tachycardia
is short R-P in nature with a clear wobble with the atrial cycle length oscillations driving the
ventricular cycle length changes. Along with spontaneous atrial initiation (not shown) and wobble,
atrial tachycardia was suspected. Ventricular overdrive pacing (VOD) was performed from the
distal his electrode during tachycardia. Figure 8C.12.3A shows the onset of pacing. Ventricular
pacing dissociates the atrium from the ventricle during tachycardia, excluding AVRT. Figure
8C.12.3B demonstrates offset of VOD with continued tachycardia. At first analysis, this appears to
be a pseudo VAAV response as the atrial electrogram associated with the last V pacing has a VA
interval too short for physiologic retrograde conduction (solid red arrow head). However, this atrial
electrogram, and the preceding atrial electrograms, are at the paced cycle length. The following
atrial electrogram (open arrow) is at the tachycardia cycle length. Because VOD is from the distal
his catheter during VOD, both ventricle and atrium were captured simultaneously. Therefore, this is
an example of dual-site pacing from a single catheter. The first return beat is an atrial signal,
excluding AVNRT, which would demonstrate a His-V signal first. This leaves atrial tachycardia as
the diagnosis. Mapping of the atrial tachycardia is shown in Figure 8C.12.4A and B. The earliest
pre-P wave atrial signal (Figure 8C.12.4A) was found at the coronary sinus os as demonstrated on
the 3D electroanatomic map (Figure 8C.12.4B). Ablation at the earliest site in the coronary sinus os
abolished the tachycardia, and it has not returned clinically in follow-up.

Case 8C.12  u  253


Figure 8C.12.4  A. Pre-P wave by –39 ms on distal ablator bipole at successful ablation site (AbD; red arrow). B. 3D electroanatomic map
of activation map with earliest site in red superimposed with successful ablation site (slow pathway ablation sites not shown).

This is an example of dual-site pacing with ventricular and atrial overdrive pacing
simultaneously, and a “pseudo pseudo VAAV” response. This technique can be useful in tachycardia
where VOD results in dissociation of atrial and ventricular signals and this fails to differentiate
between atrial tachycardia and AVNRT.1 In contrast, dual-site pacing may be diagnostic in
separating atrial tachycardia from AVNRT.

Reference
1. Saba S, Bhattarchaya S, Mezu U, et al. Simultaneous atrial and ventricular pacing can separate atrial tachycardia
from atrioventricular nodal reentrant tachycardia. Circulation. 2010;122:A10970.

254  u  Section 8C: Atrial Tachycardia/Atrial Flutter


SECTION 8D
Atrioventricular Nodal Reentrant Tachycardia

CASE
Bernard Belhassen, MD 8D.1
Patient History
This patient was a 67-year old male with no obvious heart disease and a history of recurrent PSVT.

Figure 8D.1.1

An irregular supraventricular tachyarrhythmia with narrow QRS complexes followed by a


regular wide QRS complex left bundle branch block (LBBB) tachycardia (258 bpm). The actual
diagnosis of both tachycardias is typical slow/fast atrioventricular nodal reentrant tachycardia with
various types of AV conduction: (1) 2:1 or Wenckebach AV block (narrow QRS) and (2) 1:1 AV
conduction (rate-dependent LBBB).

ECG Masters’ Collection: Favorite ECGs from Master Teachers Around the World © 2017 Mohammad Shenasa, Mark E. Josephson,
N. A. Mark Estes III, Ezra A. Amsterdam, Melvin Scheinman. Cardiotext Publishing, ISBN: 978-1-942909-08-8.   255
CASE
8D.2 Tamer S. Fahmy, MD, PhD

Patient History
A 42-year-old male patient complained of nondocumented palpitations that are mostly precipitated
by exercise. All investigations, including 48-hour Holter monitoring, showed no abnormality. His
general cardiologist recommended exercise testing. Figures 8D.2.1 and 8D.2.2 depict the first 2
minutes of the test.

Figure 8D.2.1  Initiation of tachycardia during initial phase of exercise testing shown on a 12-lead ECG during the first few minutes
of exercise with V5 as a continuous rhythm lead. The single asterisk shows the first beat of the tachycardia after a long P-R, with the
P seeming to have a different morphology than sinus morphology in the preceding three beats, indicating either a retrograde P or a
premature atrial contraction (PAC). After the fourth tachycardia beat (double asterisk), the QRS showed an incomplete RBBB, which then
normalized the following beat and returned to incomplete RBBB in the following two beats. Note that there is no or a slight increase in
tachycardia cycle length during RBBB, excluding right lateral APs. The triple asterisk shows slowing of tachycardia rate from 220 bpm
(cycle length: 280 ms) to 115 bpm (520 ms). See text for explanation.

256  u  Section 8D: Atrioventricular Nodal Reentrant Tachycardia


Figure 8D.2.2  Resetting of SVT during exercise testing is shown in a continuous 6-lead rhythm strip, showing maintenance of the
supraventricular tachycardia at a rate of 220 bpm. At the asterisk there is resetting of the tachycardia, which is preceded by a shorter RR
cycle and prolongation of the retrograde P wave. See text for further explanation.

Questions
1. What is the phenomenon that initiated the tachycardia at the asterisk?
2. Is the asterisk in Figure 8D.2.2 the same phenomenon that occurred at the last three beats in
Figure 8D.2.1? What is the diagnosis?

Discussion, Interpretation, and Answers


1. The tachycardia is initiated by a premature P wave (retrograde or a PAC) that is conducted with a
long AV time, indicating decrementation/block in the usual fast pathway. If blocked, it will result
in conduction over the slow pathway and back over the fast pathway/concealed septal pathway,
creating the circuit for initiation of the tachycardia.
2. After the fourth tachycardia beat, there is around a 20-ms decrease in cycle length, despite the
slight prolongation in the preceding PP interval. This exercise-induced autonomic variation with
the variable sympathetic output on the AV node with retrograde decrementation resulted in
delay of the P wave, whereas the faster antegrade conduction resulted in conduction of
incomplete recovery of the right bundle. After this aberrantly conducted beat, the opposite of the
previous phenomenon occurred, with shortening of the PP interval and prolongation of the RR,
resulting in a longer antegrade conduction time allowing full recovery of the right bundle. This
was followed by enhanced conduction over the AV node, resulting in RBBB for three successive
beats. However, in the last three beats, with a cycle length approaching 220 ms, there is an

Case 8D.2  u  257


antegrade conduction block (at the infra-Hisian level) with a retrograde P wave at end of the
QRS. This is another in the middle of the T wave, where the ventricular rate is around 440 ms.
In Figure 8D.2.2, the asterisk indicates resetting of the tachycardia, where the P wave
following the QRS of the last beat is decrementally delayed. Instead of terminating the
tachycardia, there is reinitiation of the tachycardia in the same way as before.
3. In view of the previous assumptions, this tachycardia is AVNRT. Although electrophysiology
confirmed the diagnosis, an alternative diagnosis considering another P wave at the beginning of
the QRS, in such case, the PP interval is too short for AV nodal effective refractory period and
the tachycardia may thus be atrial tachycardia.

258  u  Section 8D: Atrioventricular Nodal Reentrant Tachycardia


CASE
8D.3
Pieter Koopman, MD
Hein Heidbuchel, MD, PhD

Patient History
A 73-year-old female, admitted for dyspnea secondary to moderate to severe aortic regurgitation
due to aortic root dilatation, is scheduled for Bentall repair. She mentions intermittent palpitations
that are recorded during the hospitalization for surgery work-up.

Figure 8D.3.1

Question
Which arrhythmia explains this ECG (Figure 8D.3.1)?

Discussion
The patient has a supraventricular tachycardia with negative P waves in the inferior leads. During the
first beats, there is a long RP interval. This leads to a differential diagnosis of circus movement
tachycardia over a decrementally conductive accessory pathway (PJRT, permanent form of junctional
tachycardia), atypical fast-slow AV nodal reentrant tachycardia, or ectopic atrial tachycardia with
origin in the posteroseptal region. At closer look, the RR intervals lengthen for a single cycle between
the fourth and fifth beat, while the PP intervals remain stable at 525 ms. The lengthening of the RR
interval is due to a prolongation of the PR interval, indicating change of antegrade conduction from

Case 8D.3  u  259


the fast to the slow AV nodal pathway. This may occur in all three of the arrhythmias mentioned
above. However, the persistent stable PP interval would mean a compensatory decrease in the
conduction time over the retrograde pathway, i.e., the accessory pathway in case of PJRT or the
retrograde slow AV nodal pathway in case of AVNRT. Moreover, during the ensuing beats with the
same cycle length, this retrograde conduction remains much shorter which excludes retrograde
conduction over an accessory pathway (which would have the same RP intervals at the same cycle
length) and is very unlikely to continue to be fully compensatory during atypical AVNRT. Later, the
electrophysiological studies confirmed that the clinical arrhythmia was atrial tachycardia, with
intermittent conduction over the fast and slow AV nodal pathways.

Figure 8D.3.2  Explanatory ECG.

Reference
1. Roberts-Thomson KC, Kistler PM, Kalman JM. Focal atrial tachycardia I: Clinical features, diagnosis, mechanisms,
and anatomic location. Pacing Clin. Electrophysiol. 2006;29(6):643–652.

260  u  Section 8D: Atrioventricular Nodal Reentrant Tachycardia


CASE
8D.4
Pieter Koopman, MD
Hein Heidbuchel, MD, PhD

Patient History
A 39-year-old female with a history of paroxysmal palpitations fell off a chair that she was standing
on to grab some dinner plates in the kitchen cupboard. In an attempt to avoid falling on her 2-year-
old toddler, she fractured her hip. The anesthesiologist calls you postoperatively because of
relapsing tachycardia.

Figure 8D.4.1

Question
Can you identify the retrogradely conducting pathway?

Discussion
This case is an example of a dual AV nodal non-reentrant tachycardia. As is shown in Figure 8D.4.2
below, the ECG initially shows a narrow QRS tachycardia. The ladder diagram shows that there is
an uninterrupted underlying sinus rhythm. At the left side of the tracing, each P wave gives rise to
two ventricular complexes (“two-for-one” response or “double firing”) due to AV conduction first
through the fast AV nodal pathway (FP), and subsequently through the slow AV nodal pathway (SP).
Both pathways are conducting simultaneously antegradely.

Case 8D.4  u  261


In the middle part of the tracing, block occurs in the FP while AV conduction continues
through the SP, with markedly prolonged PR interval. Antegrade block in the FP is presumably due
to retrograde penetration of the FP following antegrade SP conduction (Figure 8D.4.2 ladder
diagram). However, no retrograde V-A conduction is present, so the concealed retrograde
conduction in the FP does not reach the atria. The P waves marked by the arrows are not retrograde
P waves, but are still sinus P waves. Antegrade block in the FP is maintained because of
perpetuating concealed retrograde penetration.
When looking more closely at the last part of the tracing, decremental (Wenckebach)
conduction in the SP can be noticed, as the PR interval slightly prolongs and the P waves seem to
get closer to the previous QRS complex. At the last arrow, AV conduction is blocked in the AV node,
both in the FP and SP. Given the absence of retrograde FP penetration, the next sinus impulse
conducts over the FP (with a short PR interval). Also, the two-for-one conduction is absent, likely
due to concealed retrograde penetration of the SP this time. The PR interval is noticeably shorter
than the PR interval at the beginning of the tracing, related to loss of antegrade SP conduction, due
to partial loss of electrotonic inhibition of the FP by the SP, and/or loss of retrograde concealed
conduction in the FP.
Therefore, no retrograde V-A conduction is seen in this tracing.

Figure 8D.4.2  Explanatory ECG.

Reference
1. Peiker C, Pott C, Eckardt L, et al. Dual atrioventricular nodal non-re-entrant tachycardia. Europace. 2016;18(3):
332–339.

262  u  Section 8D: Atrioventricular Nodal Reentrant Tachycardia


CASE
Henry H. Hsia, MD 8D.5
Patient History
A 58-year-old male with a history of nonischemic dilated cardiomyopathy at NYHA class III–IV
heart failure was being evaluated for cardiac transplantation. An echocardiogram revealed a
severely depressed left ventricular ejection fraction (LVEF) of ~25%. He presented with a syncopal
event and telemetry demonstrated sinus tachycardia with frequent PVCs and nonsustained
ventricular tachycardia. He was referred for an ICD implantation.

Question
Is this sinus tachycardia?

Interpretation and Discussion


A 12-lead ECG (Figure 8D.5.1) showed what appeared to be a sinus tachycardia at 120 bpm with P
waves in front of the QRS. Left axis deviation was present with poor precordial R-wave transition.
Of note, the P-wave morphology was atypical for sinus site-of-origin with an unusually short PR
interval (80–100 ms).

Figure 8D.5.1

Case 8D.5  u  263


A prior ECG (Figure 8D.5.2) showed a sinus tachycardia at ~100 bpm, with a distinctly different
P-wave morphology and a longer PR interval at 200 ms. The much shorter PR interval and atypical
P-wave morphology raised the concern of a supraventricular tachycardia (SVT) of nonsinus origin,
such as an ectopic atrial tachycardia or other forms of reentrant SVTs.

Figure 8D.5.2

A telemetry recording showed a conversion of patient’s presenting SVT (left panel) to sinus
tachycardia (right panel) (Figure 8D.5.3). The SVT stopped abruptly with PVCs, which was most
consistent with a reentrant mechanism that may be terminated by premature stimulation. SVT
termination by ventricular ectopy argues against an arrhythmia of atrial origin. The presence of a
P before QRS (with a very short P-R) also makes the diagnosis of orthodromic AV reentrant
tachycardia (AVRT) unlikely.

264  u  Section 8D: Atrioventricular Nodal Reentrant Tachycardia


Figure 8D.5.3

An electrophysiology study demonstrated inducible “slow-fast” incessant AV nodal reentrant


tachyardia (AVNRT) with “lower common pathway” (LCP) delay and was successfully ablated. The
ladder diagram shows the relative timing of R and P during AVNRT (Figure 8D.5.4). Due to delay of
ventricular activation, the retrograde P waves precede the QRS and may be mistaken as sinus/atrial
tachycardia. The patient had a “tachycardia-induced” cardiomyopathy. A follow-up echocardiogram
6 months later showed a significant improvement of LVEF to ~45%.

Figure 8D.5.4

Case 8D.5  u  265


CASE
8D.6
George D. Katritsis, MBChB, BSc
Demosthenes G. Katritsis, MD, PhD

Patient History
Supraventricular tachycardia in a 50-year-old male who complained of frequent episodes of
palpitations and was referred for atrial fibrillation ablation.

Figure 8D.6.1

Interpretation
There is atrial fibrillation evident in the coronary sinus intracardiac recordings (CS), but in the
presence of atrioventricular nodal reentrant tachycardia (AVNRT), evident in the His intracardiac
recordings (His), with a cycle length of 294 ms. Ablation of AVNRT resulted in abolition of the
atrial fibrillation episodes as well.

266  u  Section 8D: Atrioventricular Nodal Reentrant Tachycardia


CASE
8D.7
I. W. P. Obel, MBChB
Judith Daniels, RN, CRDS, CEPS

Patient History
A 42-year-old male presented with repeated episodes of palpitations over a 3-year period. The
episodes had a sudden onset and were terminated by either cardioversion or verapamil. The patient
has no structural heart disease.

Questions
1. What is the rhythm?
2. What would be the most appropriate treatment?
All ECGs are from the same tachycardia episode.

Figure 8D.7.1

Case 8D.7  u  267


Interpretation
The first part of the ECG (Figure 8D.7.1) shows 2:1 AV block. There are two beats that conduct with
aberration. The tachycardia conducts with 1:1 with left bundle branch block (LBBB) in the last third
of the ECG.

Figure 8D.7.2

268  u  Section 8D: Atrioventricular Nodal Reentrant Tachycardia


Interpretation
Figure 8D.7.2 shows narrow QRS, with normal axis and 2:1 AV nodal conduction. The atrial rate is
172 bpm (350 ms) and the ventricular rate is 86 bpm (700 ms). The P-wave morphology is negative
in the inferior leads showing a low-high activation.
There are P waves at the end of the QRS (pseudo R-wave), which can be seen clearly in lead V1
when compared with a sinus beat, proving that there is 2:1 AV conduction.

Figure 8D.7.3

Case 8D.7  u  269


Interpretation
In this ECG (Figure 8D.7.3), there is resolution of the LBBB. There is no change in the rate with the
narrow complex tachycardia.

Figure 8D.7.4  HRA, high right atrium; HBED, His bundle electrogram distal.

Interpretation
Figure 8D.7.4 shows an atrial electrogram (red arrow) and the His spike before the A (blue arrow) on
the HBED recording. This proves that the level of block is below the His.

Discussion
The tachycardia starts with 2:1 AV nodal conduction. This excludes AV reentry tachycardia (AVRT)
as the AV node is part of the tachycardia circuit in accessory mediated tachycardias.
The differential diagnoses are an atrial tachycardia or AV nodal reentry tachycardia (AVNRT).
This was shown to be typical AVNRT—antegrade down the slow pathway and retrograde via the
fast pathway.
Ashman’s phenomenon is seen on Figure 8D.7.1 and Figure 8D.7.3.
This manifests as a beat that conducts with either right bundle branch block (RBBB) or LBBB
following a long-short sequence. This occurs because one of the bundles is still refractory due to the
preceding long interval.

270  u  Section 8D: Atrioventricular Nodal Reentrant Tachycardia


There is 1:1 conduction with LBBB seen at the end of Figure 8D.7.1. RBBB aberration is more
common in normal hearts and LBBB is more common in diseased hearts. The patient was
successfully ablated.

Answers
1. The rhythm is AVNRT with intermittent 2:1 block at the start followed by 1:1 conduction with
LBBB.
2. Slow pathway ablation.

References
1. Josephson M. Clinical Cardiac Electrophysiology. Techniques and Interpretations. 4th ed. Philadelphia, PA.
Lippincott; 2008.
2. Issa Z, Miller J, Zipes D. Clinical Arrhythmology and Electrophysiology, A Companion to Braunwald’s Heart Disease.
New York, NY: Saunders; 2009.

Case 8D.7  u  271


CASE
8D.8
Mohammad Shenasa, MD
Hossein Shenasa, MD, MS

Patient History
A 62-year-old male referred due to recurrent palpitations and lightheadedness.
Heart rate: 200 bpm

Figure 8D.8.1A 

See the magnified leads aVR and II below in Figures 8D.8.1B and 8D.8.1C.

Question
What is the diagnosis?
1. Atrioventricular nodal reentrant tachycardia (AVNRT)
2. Atrioventricular reentrant tachycardia (AVRT)
3. Atrial tachycardia
4. Atrial flutter

Answer
1. AVNRT. No obvious P waves can be identified. The retrograde P wave at the very end of the QRS
(see blue arrows on Figures 8D.8.1B and 8D.8.1C) suggests AVNRT.

272  u  Section 8D: Atrioventricular Nodal Reentrant Tachycardia


Figure 8D.8.1B

Figure 8D.8.1C

Same patient as Figure 8D.8.1A with enlarged leads aVR and II. The blue arrows denote a
retrograde P wave at the very end of the QRS in leads aVR and II, suggesting typical (common-type)
AVNRT (i.e., anterograde slow and retrograde fast pathways—see Figure 8D.8.2). However, this is
usually seen in lead I.

Figure 8D.8.2  Schematic representation of common (typical) form of AVNRT with slow anterograde and fast retrograde pathways.
Reprinted with permission from Shenasa et al. Card Electrophysiol Clin. 2014;6:483–509.

Case 8D.8  u  273


Heart rate: 200 bpm

Figure 8D.8.3 

Intravenous adenosine (6 mg) was administered and the tachycardia was terminated. The first
beat in sinus rhythm (SR) followed by a blocked P wave (blue arrow). Then, a few beats in SR with a
spontaneous conversion to atrial fibrillation (AF). This phenomenon is recognized after
administration of adenosine.
Heart rate: 158 bpm

Figure 8D.8.4 

274  u  Section 8D: Atrioventricular Nodal Reentrant Tachycardia


Same patient as in Figure 8D.8.1A now present with AF and rapid ventricular response. This
episode of AF converted spontaneously from AVNRT to AF.

Figure 8D.8.5

Same patient as previous ECGs.

Question
What is the diagnosis of Figure 8D.8.5?
1. AVNRT
2. AVRT
3. Atrial flutter with 2:1 AV block
4. AF

Answer
3. Atrial flutter with 2:1 AV block. Red arrows denote flutter waves.

Case 8D.8  u  275


Heart rate: 69 bpm
PR interval: 164 ms
QRS: 88 ms
QT/QTc: 426/456 ms
The PR interval is essentially within normal limits. There is no evidence of preexcitation.

Figure 8D.8.6  Normal sinus rhythm.

Discussion
Supraventricular reentrant tachycardias may be a driver for AF. Once the supraventricular
tachycardia is eliminated, the AF is also often eliminated.

276  u  Section 8D: Atrioventricular Nodal Reentrant Tachycardia


CASE
8D.9
Mohammad Ali Sadr-Ameli, MD
Mohammad Shenasa, MD

Patient History
A 42-year-old male with several episodes of narrow QRS tachycardia.

Figure 8D.9.1

This ECG shows an atrioventricular nodal reentrant tachycardia (AVNRT) with cycle length of
480 ms and 2:1 AV block.
A very short ventriculoatrial (VA) conduction time of <60 ms in His bundle electrogram (HBE)
of intracardiac tracing is in favor of AVNRT with 2:1 AV block.

Case 8D.9  u  277


Discussion
AVNRT with 2:1 AV block is relatively rare and is often seen in the electrophysiology laboratory in
the induction of the tachycardia. This phenomenon illustrates that the lower common pathway in
AVNRT is independent of the His bundle. The tachycardia circuit is confined into the compact AV
node.

Reference
1. Willems S, Shenasa M, Borggrefe M, et al. Atrioventricular nodal reentry tachycardia; electrophysiologic
comparisons in patients with and without 2:1 intra-His block. Clin. Cardiol. 1993;16:883–888.

278  u  Section 8D: Atrioventricular Nodal Reentrant Tachycardia


CASE
Mohammad Shenasa, MD 8D.10
Patient History
A 12-year-old male presented with multiple episodes of rapid palpitations. A 12-lead
electrocardiogram is obtained and shown in Figure 8D.10.1.
Heart rate: 220 bpm

Figure 8D.10.1 

The ECG shows narrow QRS complex tachycardia. Note the significant QRS alternans in leads
II, aVR, and V3–V6.

Question
The most likely diagnosis is:
1. Atrioventricular nodal reentrant tachycardia
2. Atrioventricular reentrant tachycardia (orthodromic tachycardia)
3. Atrial tachycardia
4. Atrial flutter

Case 8D.10  u  279


Answer
No obvious retrograde P wave is detectable; however, considering the tachycardia rate of above
200 bpm and presence of QRS alternans, the most likely diagnosis will be atrioventricular reentrant
tachycardia using the normal AV nodal pathway anterogradely and a concealed accessory pathway
retrogradely. The QRS alternans are, however, nonspecific and a rate-related phenomenon.
Green et al. reported that the presence of QRS alternans during sustained narrow QRS
supraventricular tachycardia is indicative of retrograde atrioventricular pathway in the
tachycardia circuit.1

Reference
1. Green M, Heddle B, Dassen W, et al. Value of QRS alternation in determining the site of origin of narrow QRS
supraventricular tachycardia. Circulation. 1983:68(2):368–373.

280  u  Section 8D: Atrioventricular Nodal Reentrant Tachycardia


CASE
Mohammad Shenasa, MD 8D.11
Patient History
A 25-year-old male admitted for further workup due to recurrent palpitations. The
electrocardiogram in the emergency department (Figure 8D.11.1) is shown below.

Figure 8D.11.1  Narrow QRS complex tachycardia at a rate of 225 bpm. The retrograde P wave is outside the QRS (see red arrows).

Figure 8D.11.2  Obtained after conversion of supraventricular tachycardia, shows sinus rhythm of 85 bpm with manifest preexcitation
(delta wave—see red arrows). This is probably a left lateral accessory pathway.

Question
The most likely diagnosis in this case is:
1. Atrioventricular nodal reentrant tachycardia (AVNRT)
2. Atrioventricular reentrant tachycardia (AVRT) using normal pathway anterogradely and
accessory pathway retrogradely
3. Atrial tachycardias
4. Atrial flutter

Case 8D.11  u  281


Answer
2. AVRT. In Figure 8D.11.1, the retrograde P waves are outside of the QRS and Figure 8D.11.2 shows
ventricular preexcitation.
The algorithm in Figure 8D.11.3 provides clues for the differential diagnosis of narrow complex
tachycardias.

Figure 8D.11.3  Abbreviations: AVNRT, atrioventricular nodal reentrant tachycardias; AVRT, atrioventricular reentrant tachycardias; ST,
sinus tachycardia; IAST, inappropriate sinus tachycardia; SNRT, sinus node reentrant tachycardia. Used with permission from Shenasa et al.
Card. Electrophysiol. Clin. 2014;6:483–509.

282  u  Section 8D: Atrioventricular Nodal Reentrant Tachycardia


SECTION 8E
Atrioventricular Reentrant Tachycardia

CASE
8E.1
George D. Katritsis, MBChB, BSc
Demosthenes G. Katritsis, MD, PhD

Patient History
Wide QRS complex tachycardias (Figure 8E.1.1A) recorded in a 62-year-old female with a history of
frequent palpitations and tachycardia episodes. On the right (Figure 8E.1.1B), there is an ECG
during sinus rhythm.

Figure 8E.1.1

ECG Masters’ Collection: Favorite ECGs from Master Teachers Around the World © 2017 Mohammad Shenasa, Mark E. Josephson,
N. A. Mark Estes III, Ezra A. Amsterdam, Melvin Scheinman. Cardiotext Publishing, ISBN: 978-1-942909-08-8.   283
Interpretation
This is antidromic atrioventricular reentrant tachycardia (Figure 8E.1.1A) with left bundle branch
block pattern and left axis deviation due to an atriofascicular Mahaim accessory pathway. Note that
during sinus rhythm (Figure 8E.1.1B), the ECG is unremarkable. Successful pathway ablation
resulted in abolition of the episodes of the arrhythmia.

284  u  Section 8E: Atrioventricular Reentrant Tachycardia


CASE
8E.2
George D. Katritsis, MBChB, BSc
Demosthenes G. Katritsis, MD, PhD

Patient History
Narrow-QRS tachycardia, recorded in a 47-year-old female who presented with a long history of
persistent episodes of arrhythmia.

Figure 8E.2.1

Case 8E.2  u  285


Interpretation
This is a long R-P tachycardia with prominent, negative P waves in the inferior leads. Pacing
maneuvers at electrophysiology study established the diagnosis of orthodromic atrioventricular
reentrant tachycardia (AVRT) due to a septal accessory pathway with decremental properties, which
was successfully ablated.

Question
What would be a differential diagnosis?

Answer
It had to be differentiated from atypical AVNRT and atrial tachycardia.

286  u  Section 8E: Atrioventricular Reentrant Tachycardia


SECTION 8F
Atrial Fibrillation

CASE
8F.1
Adrian Baranchuk, MD
Antoni Bayés de Luna, MD, PhD

Patient History
A 68-year-old male with a prior myocardial infarction diagnosed 15 years ago. Ejection fraction was
40%. Previously, he presented an episode of atrial fibrillation (AF) requiring cardioversion. A post-
cardioversion ECG was recorded (Figure 8F.1.1).

Figure 8F.1.1

Question
What marker likely indicates a new episode of AF in less than 1 to 2 years?
1. QS from V1 to V3
2. Deep and negative T wave from V2 to V4
3. P-wave morphology
4. Low ejection fraction

ECG Masters’ Collection: Favorite ECGs from Master Teachers Around the World © 2017 Mohammad Shenasa, Mark E. Josephson,
N. A. Mark Estes III, Ezra A. Amsterdam, Melvin Scheinman. Cardiotext Publishing, ISBN: 978-1-942909-08-8.   287
Interpretation, Answer, and Comments
The correct answer is 3.
The P-wave duration is ≥ 120 ms and the P-wave morphology is biphasic (±) in leads II, III, and
aVF. This criterion corresponds to advanced interatrial block (IAB), according to a recent consensus
document,1 although these diagnostic criteria were published many years ago.2 In partial IAB the P
wave is also ≥ 120 ms and is usually bimodal, but with no biphasic (±) morphology in the inferior
leads. This can be clearly seen in the current case, especially when amplifying glasses (or
semiautomatic calipers) are used (Figure 8F.1.2).

Figure 8F.1.2  A. Note the ECG pattern of advanced IAB. P-wave duration of at least 160 ms and definitively ± morphology in leads II, III,
and VF. B. The P wave in leads II, III, VF of the same patient in an ECG recorded 8 years before (P < 120 ms).

In 1988, Bayés de Luna et al.3 demonstrated (Figure 8F.1.3) that patients with advanced
interatrial block with a P wave that is ≥ 160 ms who present an important structural heart disease
and ambient arrhythmias during Holter monitoring (Figure 8F.1.4) were at higher risk of presenting
atrial fibrillation/flutter in a short follow-up. This risk is much higher compared to that of patients
with similar clinical characteristics, including same size of the left atrium, who present partial IAB
(97.2% vs. 25% at the 3-year follow-up).

288  u  Section 8F: Atrial Fibrillation


Figure 8F.1.3  Life table analysis of the probability of remaining free of supraventricular tachyarrhythmias in patients with advanced
interatrial block (IAB) and RALA, and controls.3 RALA = retrograde activation of left atrium.

Figure 8F.1.4  Patients with advanced IAB who are older than 65 years, present P-wave duration greater than 160 ms, show advanced
structural HD (high CHA2DS2-V), and have ambient arrhythmias, are at high risk of AF in a short follow-up.

In recent years, some papers have emphasized the importance of diagnosing this new
arrhythmological syndrome, calling it Bayés’ syndrome.4–6
Several papers from Baranchuk’s group and others have recently been published, confirming the
frequent association of advanced IAB and atrial fibrillation.7–10

Case 8F.1  u  289


Therefore, patients with this type of advanced IAB (Figure 8F.1.4) may need to be
anticoagulated, even in the absence of known AF. Patients with this type of ECG that have IAB
should be anticoagulated. Even if AF is not present, and the patient remains in sinus rhythm at
present, they are still at a risk of AF in the future. In many cases, AF is not the final cause of stroke.
Consequently, the presence of advanced IAB and the characteristics shown in Figure 8F.1.4, as
mentioned previously, is most likely a big risk factor by itself for AF/stroke and may warrant
anticoagulation.11
The pattern of advanced IAB appears as an isolated finding or is only associated to one risk
factor in a global population cohort with a P-wave duration around <160 ms and no advanced
structural heart disease. This pattern may represent a risk factor for AF, although with a much
lower predictor value during long-term follow-up (REGICOR cohort: unpublished data).

Conclusion
The ECG pattern of advanced IAB should be recognized because in association with the clinical
characteristics outlined in Figure 8F.1.4, it represents an important risk factor for atrial fibrillation/
flutter during a short-term follow-up.
Based on this data, it is recommended to always use amplified glasses (or calipers) to better
identify the characteristics of the P-wave morphology. Many cases of advanced IAB are visible only
with those specific tools (Figure 8F.1.2).
The other 3 markers previously mentioned (see Questions) may suggest extensive myocardial
infarction and even heart failure, and obviously a higher possibility of new-onset AF. However,
these last relationships are not as clear as those observed in patients with advanced IAB and the
clinical characteristics outlined in Figure 8F.1.4.

References
1. Bayés de Luna A, Fort de Ribot R, Trilla E, et al. Electrocardiographic and vectorcardiographic study of interatrial
conduction disturbances with left atrial retrograde activation. J. Electrocardiol. 1985;18:1.
2. Bayés de Luna A, Platonov P, Garcia-Cosio F, et al. Interatrial blocks. A separate entity from left atrial enlargement.
A consensus report. J. Electrocardiol. 2012;45:445.
3. Bayés de Luna A, Cladellas M, Oter R, et al. Interatrial conduction block and retrograde activation of the left atrium
and paroxysmal supraventricular tachyarrhythmias. Eur. Heart J. 1988;9:1112.
4. Conde D, Baranchuk A. What a cardiologist must know about Bayés’ syndrome. Rev. Arg. Cardiol. 2014;82:220–222.
5. Conde D, Baranchuk A. Bloqueo interauricular como sustrato anatómico-eléctrico de arritmias supraventriculares:
Síndrome de Bayés. Arch. Cardiol. Mex. 2014;84(1):32–40.
6. Bacharova L, Wagner GS. The time for naming the interatrial block syndrome: Bayés’ syndrome. J. Electrocardiol.
2015;48:133–134.
7. Enriquez A, Sarrias A, Villuendas R, et al. New-onset atrial fibrillation after cavotricuspid isthmus ablation:
Identification of advanced interatrial block is key. Europace. 2015;17(8):1289–1293.
8. Enriquez A, Conde D, Hopman W, et al. Advanced interatrial block is associated with recurrence of atrial
fibrillation post pharmacological cardioversion. Cardiovasc. Ther. 2014;32(2):52–56.
9. Sadiq Ali F, Enriquez A, Conde D, et al. Advanced interatrial block is a predictor of new onset atrial fibrillation in
patients with severe heart failure and cardiac resynchronization therapy. Ann. Noninvasive Electrophysiol.
2015;20(6):586–591.
10. Enriquez A, Conde D, Femenia F, et al. Relation of interatrial block to new-onset atrial fibrillation in patients with
Chagas cardiomyopathy and implantable cardioverter defibrillators. Am. J. Cardiol. 2014;113(10):1740–1743.
11. Bayés de Luna A, Baranchuk A, Platonov P. Is it time to start anticoagulation in patients at high risk of stroke but
without documented atrial fibrillation? (submitted).

290  u  Section 8F: Atrial Fibrillation


CASE
Robert Lemery, MD 8F.2
Patient History
A 72-year-old female presented with atrial fibrillation (AF) with rapid ventricular rates. During the
month prior to her admission, she had developed AF and was anticoagulated and treated with
digitalis 0.125 mg p.o. daily, and sotalol 240 mg p.o. daily. She was admitted with symptoms of
shortness of breath and rapid palpitations. She underwent cardioversion.

Question
The electrocardiogram following cardioversion is most likely due to:
1. Digitalis use
2. Sotalol
3. Sudden conversion of AV node reentry to sinus rhythm
4. Acute coronary syndrome

Answer
2

Discussion
The admission ECG (Figure 8F.2.1) shows AF with a rapid ventricular rate of 114/min. The post-
cardioversion ECG (Figure 8F.2.2) shows sinus rhythm with a markedly prolonged QT interval of
approximately 608 ms and a QTc of 562 ms. Within the next several hours, the patient showed
torsades de pointes (Figure 8F.2.3). As characterized by Dessertenne,1 bradycardia, prolonged QT
interval and ventricular tachycardia (VT) showing a reversal of peaks or twisting of the points, are
the electrocardiographic hallmarks of torsades de pointes.
In this patient, the other significant feature is the association of restoration of sinus rhythm and
proarrhythmic risks.2 While rapid ventricular rates during AF prior to cardioversion prevented the
full manifestation of polymorphic VT, the (sudden) onset of sinus bradycardia in the context of a
class 3 agent such as sotalol creates a particularly dynamic state that increases the risk of
polymorphic VT.
The patient underwent permanent cardiac pacing (Figure 8F.2.4) and received treatment with
amiodarone. Recurrence of AF resulted in follow-up with discontinuation of amiodarone, and a rate
approach was followed. There were no untoward events during annual follow-up in the pacemaker
clinic.

Case 8F.2  u  291


Figure 8F.2.1

Figure 8F.2.2

292  u  Section 8F: Atrial Fibrillation


Figure 8F.2.3

Figure 8F.2.4

Case 8F.2  u  293


References
1. Dessertenne F. La tachycardie ventriculaire a deux foyers opposes variables. Arch. Mal. Coeur. 1966;59:263–272.
2. Prystowsky EN. Management of atrial fibrillation: Therapeutic options and clinical decisions. Am. J. Cardiol.
2000;85:3–11.

294  u  Section 8F: Atrial Fibrillation


SECTION 8G
Junctional Rhythms

CASE
Fred Morady, MD 8G.1
Patient History
This ECG (Figure 8G.1.1) was recorded in a 42-year-old female one day after aortic valve
replacement. What is the rhythm?

Figure 8G.1.1

Discussion
This is a junctional rhythm with alternating short and long RP intervals. The most likely
explanation for the alternating RP intervals is the presence of retrograde dual atrioventricular nodal
pathways and conduction from the ventricle to the atrium alternating between the fast pathway and
slow pathway. Sinus rhythm with alternating PR intervals is ruled out by the inverted P waves in the
inferior leads. An irregular ectopic atrial rhythm with alternating PR intervals is highly unlikely
because the cycle length of the QRS complexes is perfectly regular.

ECG Masters’ Collection: Favorite ECGs from Master Teachers Around the World © 2017 Mohammad Shenasa, Mark E. Josephson,
N. A. Mark Estes III, Ezra A. Amsterdam, Melvin Scheinman. Cardiotext Publishing, ISBN: 978-1-942909-08-8.   295
Note that the morphology of the P wave changes with the RP interval in the inferior leads.
There is a small terminal positive deflection in the P wave in the inferior leads with the shorter of
the two RP intervals. The variability in P-wave morphology is explained by the different locations of
the fast and slow pathways relative to the compact atrioventricular node. The region of the fast
pathway is anterior and superior relative to the compact atrioventricular node while the slow
pathway is posterior and inferior. This is often manifest by a change in retrograde atrial activation
sequence when retrograde conduction switches from one to the other of the atrioventricular nodal
pathways. This can result in a slight change in P-wave morphology in the electrocardiogram.

296  u  Section 8G: Junctional Rhythms


SECTION 9A
Ventricular Tachycardia/Fibrillation

CASE
9A.1
Amit Noheria, MBBS, SM
Samuel J. Asirvatham, MD

Patient History
A middle-aged male with remote history of myocardial infarction presented with mild palpitations,
dyspnea, and fatigue. He is on amiodarone and his ECG is shown (Figure 9A.1.1).

Figure 9A.1.1

Questions
1. What is this rhythm, and what information can you infer about the arrhythmia?
2. Why is the QRS morphology changing? What is the location of his myocardial infarction?

ECG Masters’ Collection: Favorite ECGs from Master Teachers Around the World © 2017 Mohammad Shenasa, Mark E. Josephson,
N. A. Mark Estes III, Ezra A. Amsterdam, Melvin Scheinman. Cardiotext Publishing, ISBN: 978-1-942909-08-8.   297
Discussion, Interpretation, and Answers
The ECG rhythm strips show evidence of sinus P waves marching through the tracing
(Figure 9A.1.2, red arrows) with a dissociated and faster ventricular rate (102 bpm) suggesting
ventricular tachycardia (VT). There is a repetitive pattern of interspersed fusion beats between
conducted sinus and VT (F) and purely conducted sinus beats (C). These “capture beats”
demonstrate a narrow QRS complex with a sharp initial deflection in lead V1.

Figure 9A.1.2

In general, the QRS complex in VT is negative in ECG leads corresponding to the site from
where activation spreads out to the bulk of the ventricle (inferior leads corresponding to the inferior
wall in this case). However, for a VT from an endocardial site in a structurally normal heart, there
will be a small initial r wave, representing transmural activation in direction of the ECG lead, before
the vector becomes predominantly negative. Presence of QS complexes (without initial r wave)
suggests either an epicardial site with the entire activation wavefront (including local transmural
depolarization) moving away from the corresponding ECG lead, or, as in this case, point to the
presence of transmural infarction. Presence of an inferior infarction is further confirmed by q
waves in the conducted beats in leads II and III (arrowheads).
The VT morphology demonstrates an “Rs” configuration in leads V1–V3 transitioning to “rS” in
leads V4 –V6 suggesting spread of activation from a site in the left ventricle (atypical right bundle
branch block morphology in lead V1), closer to the apex than the base (predominantly negative in
leads V5 and V6). It has a superior axis (upright in aVR and aVL; “QS” in inferior leads), further
localizing to the inferior apical wall. The QRS width during VT is relatively short (approximately
120 ms) suggesting the depolarization wavefront spreading centrifugally from the septum to
activate both ventricles at the same time. A narrow QRS also suggests activation spreading from the

298  u  Section 9A: Ventricular Tachycardia/Fibrillation


endocardium with early access to fascicular conduction system, as opposed to spreading from the
epicardial surface that has a slurred initial component (pseudodelta wave) and overall wider QRS
complex. Put together, the VT morphology suggests activation spreading from the left ventricular
apical inferoseptal region to the rest of the ventricles.
On careful measurements, other interesting phenomena can be noted in Figure 9A.1.2. First, the
PR interval for the sinus capture beats is longer for F and shorter for C (blue arrows). The
prolongation of the PR interval for F is explained by concealed retrograde penetrance of VT into the
AV node. Therefore, the AV node is relatively refractory and antegrade AV nodal conduction
encounters more decrement with PR prolongation. Second, the conducted complexes F and C reset
the tachycardia so that the subsequent VT beat (asterisk) is slightly advanced. Both focal and
reentrant mechanisms of tachycardia can be reset with premature beats. Reset with a fused QRS
complex, however, proves reentry. Comparison with conducted QRS complexes during normal sinus
rhythm (not available) can help determine if the complex C is purely a conducted sinus beat or
demonstrates some evidence of fusion with the VT. Regardless, a stable monomorphic VT in the
setting of structural heart disease has overwhelmingly high odds of being a reentrant arrhythmia.

Case 9A.1  u  299


CASE
9A.2 Bernard Belhassen, MD

Figure 9A.2.1

Identical QRS morphology achieved during rapid atrial pacing (Figure 9A.2.1 Left) and left
posterior fascicular ventricular tachycardia (VT)—also called “Belhassen VT”—(Figure 9A.2.1
Right) in a young patient with a normal heart. In both instances, an identical pattern of complete
right bundle branch block and left axis deviation is observed. Atrial pacing is initiated during sinus
rhythm showing a normal QRS configuration and is associated with left bundle branch block
aberration on the first paced beat. This tracing suggests that a tachycardia with right bundle branch
block-left axis deviation in a patient with a normal heart and normal baseline electrocardiogram
may have (albeit exceptionally) a supraventricular origin.

300  u  Section 9A: Ventricular Tachycardia/Fibrillation


CASE
Tamer S. Fahmy, MD, PhD 9A.3
Patient History
A 44-year-old female patient presented with rapid palpitations. Clinical and echocardiographic
examination reveals a structurally normal heart. Her basic ECG shows no abnormality. ECG during
tachycardia is shown in Figure 9A.3.1. Patient was admitted to electrophysiology lab for diagnosis
and radiofrequency ablation of the tachycardia (Figures 9A.3.2, 9A.3.3, and 9A.3.4).

Figure 9A.3.1  Surface ECG of the tachycardia. A 12-lead surface ECG showing wide complex tachycardia having right bundle branch
block (RBBB) morphology. Tachycardia apparently seems irregular due to multiple narrow complexes that appear as captured beats. The
normalized beats have no terminal delay on as seen in V1, giving the impression of a significant irregularity. Note that the extreme right
axis, monophasic R in V1, predominantly negative V6, and dominant R in aVR is in favor of VT.

Case 9A.3  u  301


Figure 9A.3.2  Intracardiac interpretation of the tachycardia. Intracardiac recordings show His bundle electrode (proximal and distal),
coronary sinus (proximal and distal) and RV apex. As shown, there is gradual conduction delay over three beats, then it normalizes. There
is a minor delay in the HH timing due to variable AV nodal conduction time, thus giving time for the right bundle to recover in the normal
beats. Note that in the aberrant beats there is delay in the RV activation, especially the apical activation to approximate the LV activation,
giving the irregularity seen in the RVa electrode. Also note the infra-Hisian delay results in earlier activation of the A wave, resulting in
pseudo q wave on the surface ECG in the aberrant beats [rather than the usual pseudo S].

302  u  Section 9A: Ventricular Tachycardia/Fibrillation


Figure 9A.3.3  Surface ECG of another morphology of the tachycardia. Intracardiac recording showing normalization of the RBBB in the
same session, with fixed HH interval of 440 ms.

Case 9A.3  u  303


Figure 9A.3.4  Intracardiac findings for tachycardia 2. Intracardiac recording shows concentric AV activation, with early RV activation,
consistent with typical AV nodal reentrant tachycardia.

Questions
1. Using the differentiation algorithms, what is the diagnosis of the wide complex tachycardia?
2. What is the cause for the irregularity of the tachycardia?

Discussion, Interpretation, and Answers


Although the right bundle branch block (RBBB) is not wider than 160 ms, the morphologic criteria
of the RBBB favors ventricular tachycardia (VT). The monophasic R wave in V1, the predominant S
in V6, and R in aVR together with the pseudo-capture beats favors the diagnosis of VT; however, the
presence of two successive capture beats in VT is very unusual, unless there is simultaneous
supraventricular arrhythmia (atrial fibrillation/atrial flutter).
Two interesting phenomena appear in the intracardiac recordings: the mechanism of pseudo
capture beats and the earlier appearance of the A wave in the aberrant beats. The HH variation with
its later entrance in the right bundle gives it more time to recover. The variation of the autonomic
input to the AV node during tachycardia is responsible for that phenomenon, with more rapid
conduction resulting in aberration. Interestingly, the earlier appearance of the A wave in relation to
the QRS and its disappearance in the nonaberrant beats disrupts the appearance of VA association,
giving the impression of VA dissociation.

304  u  Section 9A: Ventricular Tachycardia/Fibrillation


CASE
Robert Frank, MD 9A.4
Patient History
A 62-year-old patient with an old inferior myocardial infarction complains of near syncopal
episodes. His ECG (Figure 9A.4.1) shows atrial fibrillation and a right bundle branch block, and
episodes of ventricular fibrillation (VF).

Figure 9A.4.1

Question
What is the electrophysiological mechanism of VF onset?

Answer
VF is induced by short-coupled PVCs, but not every short-coupled PVC induces VF. Induction
occurs each time after a long RR interval (long-short sequence), usually described in patients with a
long QT and torsades de pointes. This sequence of VF onset has been also described in patients
with an ICD and a low pacing rate. The underlying mechanism proposed is a different change in

Case 9A.4  u  305


refractory periods between myocardium and Purkinje in response to an abrupt change in cycle
length, allowing reentry to occur.

References
1. Denker S, Lehmann M, Mahmud R, et al. Divergence between refractoriness of His-Purkinje system and ventricular
muscle with abrupt changes in cycle length. Circulation. 1983;68:1212–1221.
2. Sweeney MO, Ruetz LL, Belk P, et al. Bradycardia pacing-induced short-long-short sequences at the onset of
ventricular tachyarrhythmias: A possible mechanism of proarrhythmia? J. Am. Coll. Cardiol. 2007;50(7):614–622.

306  u  Section 9A: Ventricular Tachycardia/Fibrillation


CASE
Jonathan Kalman, MBBS, PhD 9A.5
Patient History
A 30-year-old male presented with wide complex tachycardia at 200 bpm with a blood pressure of
90/60.

Figure 9A.5.1

The ECG revealed:


1. Bicuspid aortic valve with marked calcification
2. Mean atrioventricular (AV) gradient of 25 mmHg (moderate aortic stenosis)
3. Moderate aortic regurgitation
4. Normal left ventricle size and systolic function
5. Mild concentric left ventricular hypertrophy (1.4 cm wall thickness)

Case 9A.5  u  307


Figure 9A.5.2  Panel A: Resting 12-lead ECG shows right bundle branch block (RBBB)/RAD and first-degree AV block. Panel B: The
bottom panel shows RBBB/RAD tachycardia with similar but not identical morphology as the sinus rhythm ECG. There are retrograde P
waves (asterisk) in some beats but not others.

308  u  Section 9A: Ventricular Tachycardia/Fibrillation


Figure 9A.5.3  Left bundle branch block (LBBB) ventricular tachycardia (VT) with a cycle length of 360 ms (165 bpm). Later on after
medical therapy with flecainide, the patient went into a LBBB tachycardia with a cycle length of 360 ms.

Case 9A.5  u  309


Figure 9A.5.4  RBBB tachycardia with a cycle length of 360 ms (165 bpm). The LBBB tachycardia terminated spontaneously, and soon
after the RBBB tachycardia recurred with the identical cycle length. In the electrophysiology lab, both of these arrhythmias could be
induced. The LBBB tachycardia was induced with programmed stimulation from the right ventricle and was confirmed to be bundle branch
reentry using the RBB in the antegrade direction and the LBB in the retrograde direction. The RBBB tachycardia was induced with atrial
programmed electrical stimulation and was demonstrated to be bundle branch reentry using the LBB in the antegrade direction and the
RBB in the retrograde direction. Ablation of the right bundle eliminated both tachycardias.

310  u  Section 9A: Ventricular Tachycardia/Fibrillation


CASE
9A.6
Andrew D. Krahn, MD
Christian Steinberg, MD

Patient History
A 50-year-old male with known coronary artery disease and normal left ventricular function
presented to emergency for recurrent chest pain and dyspnea. A remote non-ST segment elevation
myocardial infarction (NSTEMI) had been treated with angioplasty and stenting of the right
coronary artery. His initial ECG (Figure 9A.6.1A) was unremarkable and the first set of troponin
was negative. Given his history of previous angioplasty and his ongoing chest pain, a coronary
angiogram was performed that demonstrated a patent stent and no other flow-limiting lesion. The
ECG of Figure 9A.6.1B was recorded after the coronary angiogram. The patient was completely
asymptomatic. Echocardiogram and cardiac magnetic resonance imaging (MRI) showed a normal
biventricular function and no evidence of myocardial scarring. Exercise treadmill testing resulted in
complete suppression of the ECG findings displayed in Figure 9A.6.1B.

Case 9A.6  u  311


Figure 9A.6.1  Intermittent accelerated idioventricular rhythm with likely RV outflow tract (RVOT) origin. A. Initial ECG at emergency.
Normal sinus rhythm at a rate of 65 bpm. Isolated, nonspecific QRS fragmentation in lead III, otherwise normal ECG. Intrinsic sinus beats
have a normal QRS duration. B. There are three different types of QRS complexes: wide QRS complexes with left bundle branch block
pattern and inferior axis at a rate of 75 bpm (black arrows); narrow QRS complexes at a rate of 65 bpm (blue arrows); and QRS complexes
with intermediate width and similar morphology compared to the wide complexes (dark red arrows). There is AV-dissociation (dashed
arrows) with more R than P, suggesting an underlying ventricular rhythm. The QRS complexes with intermediate width represent fusion
beats between conducted sinus beats and idioventricular rhythm. Note the slightly shorter PR interval of fused beats. The most likely
diagnosis in this asymptomatic patient is intermittent accelerated idioventricular rhythm with likely origin from the posteroseptal RVOT
(precordial transition at V4 and V5, leads I and aVL positive).

312  u  Section 9A: Ventricular Tachycardia/Fibrillation


Interpretation
(A) Normal sinus rhythm at a rate of 65 bpm. Isolated, nonspecific QRS fragmentation in lead III,
otherwise normal ECG.
(B) Intermittent accelerated idioventricular rhythm at a rate of 75 bpm. Underlying sinus
rhythm at a rate of 65 bpm, with presence of fusion beats. The idioventricular rhythm most likely
originates from the RVOT.

Discussion
The term accelerated idioventricular rhythm (AIVR) describes an ectopic rhythm originating from
the His-Purkinje system or the ventricular myocardium.1 The underlying mechanism is
automaticity, and AIVR usually manifests during increased vagal tone.2 This ectopic rhythm is
typically monomorphic with rates between 50 and 120 bpm and creates no or minimal symptoms.1
AIVR has traditionally been reported in the context of acute myocardial infarction, but can also
occur in the context of drug intoxication, chronic cardiomyopathies, electrolyte disorders, or
pregnancy.1,3,4 The presence of AIVR in the setting of acute myocardial infarction with ST-segment
elevation gained widespread interest in the era of thrombolysis because it was initially thought to be
a marker of successful reperfusion. However, more recent studies have demonstrated that AIVR is
not reliably associated with reperfusion, but rather is a marker of extensive myocardial damage and
abnormal myocardial microcirculation.3,5 In the present case, intermittent increase of vagal tone
appears to be the sole explanation for his AIVR in the absence of active ischemia or other reversible
causes.
The ECG suggested an RVOT origin based on widely accepted morphology criteria for outflow
tract arrhythmia.6–9

References
1. Riera AR, Barros RB, de Sousa FD, et al. Accelerated idioventricular rhythm: History and chronology of the main
discoveries. Indian Pacing Electrophysiol. J. 2010;10(1):40–48.
2. Castellanos A, Jr., Lemberg L, Arcebal AG. Mechanisms of slow ventricular tachycardias in acute myocardial
infarction. Dis. Chest. 1969;56(6):470–476.
3. Bonnemeier H, Ortak J, Wiegand UK, et al. Accelerated idioventricular rhythm in the post-thrombolytic era:
Incidence, prognostic implications, and modulating mechanisms after direct percutaneous coronary intervention.
Ann. Noninvasive Electrocardiol. 2005;10(2):179–187.
4. Navarro V, Nathan PE, Rosero H, et al. Accelerated idioventricular rhythm in pregnancy: A case report. Angiology.
1993;44(6):506–508.
5. Terkelsen CJ, Sorensen JT, Kaltoft AK, et al. Prevalence and significance of accelerated idioventricular rhythm in
patients with ST-elevation myocardial infarction treated with primary percutaneous coronary intervention. Am. J.
Cardiol. 2009;104(12):1641–1646.
6. Betensky BP, Park RE, Marchlinski FE, et al. The V(2) transition ratio: A new electrocardiographic criterion for
distinguishing left from right ventricular outflow tract tachycardia origin. J. Am. Coll. Cardiol. 2011;57(22):
2255–2262.
7. Dixit S, Gerstenfeld EP, Callans DJ, et al. Electrocardiographic patterns of superior right ventricular outflow tract
tachycardias: Distinguishing septal and free-wall sites of origin. J. Cardiovasc. Electrophysiol. 2003;14(1):1–7.
8. Hutchinson MD, Garcia FC. An organized approach to the localization, mapping, and ablation of outflow tract
ventricular arrhythmias. J. Cardiovasc. Electrophysiol. 2013;24(10):1189–1197.
9. Yoshida N, Yamada T, McElderry HT, et al. A novel electrocardiographic criterion for differentiating a left from
right ventricular outflow tract tachycardia origin: The V2S/V3R index. J. Cardiovasc. Electrophysiol. 2014;25(7):
747–753.

Case 9A.6  u  313


CASE
9A.7 Robert Lemery, MD

Patient History
A 52-year-old male with a history of coronary artery disease (CAD), coronary artery bypass graft
surgery, and left ventricular dysfunction (ejection fraction of 28%, inferior aneurysm) underwent
primary prevention implantation of an implantable cardioverter-defibrillator (ICD). The baseline
ECG showed sinus bradycardia with a remote inferior myocardial infarction. The patient was
followed in the heart failure clinic; during follow-up, his ECG showed atrial pacing with ventricular
sensing and an increase of the PR interval to 250 ms (Figure 9A.7.1). Eight years following ICD
implantation, the patient was admitted with VT storm. He required multiple ICD shocks and was
treated with amiodarone.

Figure 9A.7.1

314  u  Section 9A: Ventricular Tachycardia/Fibrillation


The patient presented two years later with two shocks from his ICD. The patient underwent VT
ablation; multiple morphologies were also induced during mapping, showing a narrower
morphology (Figure 9A.7.2) and another very wide complex morphology associated with left bundle
branch block morphology (Figure 9A.7.3). The patient had a ventricular aneurysm, which has been
associated with pleiomorphic VT (i.e., >1 morphologically distinct VT during the same episode of
VT, while the QRS does not continuously change) Figure 9A.7.4.2,3 This patient had different
morphologies of induced VT, rather than pleiomorphic VT. During VT ablation, using activation
and pacemapping, homogenization of the extensive inferior wall scar was associated with no
inducible VT post ablation. During > 2 years of follow-up in ICD clinic, the patient has not had
recurrence of VT, while his follow-up ECG continued to show atrial pacing with prolonged AV
conduction.

Question
Pleiomorphic ventricular tachycardia refers to:
1. Ventricular tachycardia associated with atrial fibrillation
2. The change of morphology of ventricular tachycardia during continuous recording, showing two
distinct morphologies
3. Ventricular tachycardia associated with ventricular fibrillation
4. Ventricular tachycardia showing multiple morphologies at different times, but without distinctly
showing a change from one morphology to another morphology

Answer
2

Figure 9A.7.2

Case 9A.7  u  315


Figure 9A.7.3

Figure 9A.7.4

316  u  Section 9A: Ventricular Tachycardia/Fibrillation


References
1. Josephson ME, Horowitz LN, Farshidi A, et al. Recurrent sustained ventricular tachycardia. 4. Pleomorphism.
Circulation. 1979;59(3):459–468.
2. Liu E, Josephson ME. Pleomorphic ventricular tachycardia and risk of sudden cardiac death. Circ. Arrhythm.
Electrophysiol. 2011;4:2–4.

Case 9A.7  u  317


CASE
9A.8
James E. Ip, MD
Bruce B. Lerman, MD

Patient History
A 59-year-old female without a previous cardiac history developed sudden and intermittent
palpitations associated with lightheadedness. During one of her episodes, she was found to have
heart rates in the 160 seconds. Her ECG is shown in Figure 9A.8.1 (initiation) and Figure 9A.8.2
(termination). Her echocardiogram showed a structurally normal heart.

Figure 9A.8.1  12-lead ECG showing initiation of ventricular tachycardia. Note the presence of dissociated P waves (arrows).

Figure 9A.8.2  12-lead ECG showing termination of wide complex tachycardia. Note the capture beat (arrow) revealing diagnosis of
ventricular tachycardia.

318  u  Section 9A: Ventricular Tachycardia/Fibrillation


Questions
What type of arrhythmias is this? Where is the site of origin?

Discussion
The presence of A-V dissociation and capture beats indicates that this is ventricular tachycardia
rather than supraventricular tachycardia with aberrant conduction. The ventricular tachycardia
(VT) has a left bundle branch block morphology and an inferior axis, suggesting an outflow tract
origin.
In localizing outflow tract arrhythmias, it important to remember that the posterior wall of the
right ventricular outflow tract (RVOT) is adjacent and immediately anterior to the right coronary
cusp (RCC) and part of the left coronary cusp (LCC).1 A precordial transition after lead V3 suggests
a right-sided origin, whereas an early transition (≤ lead V2) with a right bundle branch block (RBBB)
morphology usually indicates a left-sided focus. However, outflow tract arrhythmias on the
posterior aspect of the RVOT or the anterior aspect of the left ventricular outflow tract (LVOT) may
have similar ECG characteristics. This case shown here has a precordial transition in V3, indicating
that the arrhythmia focus could originate from either the RVOT or LVOT.
The RVOT wraps around the anterior aspect of the LVOT.2,3 The plane of the pulmonic valve is
approximately 1–2 cm more superior to that of the aortic valve (Figure 9A.8.2). Therefore, the crest
of the posterior RVOT myocardial wall is adjacent to the aortic sinuses, generally the RCC and a
portion of the LCC. Because the LVOT is positioned more posteriorly in the chest compared to the
“septal” RVOT, arrhythmias from this region are associated with larger anterior forces and therefore
an earlier precordial transition. Thus, idiopathic LVOT PVCs with a precordial transition in lead V3
show an earlier transition compared to sinus rhythm. This is confirmed by computing the
percentage R wave during the PVC/VT in lead V2 (R/R+S)VT divided by the percentage R wave in
sinus rhythm (R/R+S)SR. A V2 transition ratio ≥ 0.60 predicts an LVOT origin with a sensitivity of
95%, specificity of 100%, positive predictive value of 100% and negative predictive value of 95%.4
The case shown has a PVC precordial transition that occurs later than sinus. The V2 transition
ratio is 0.5, suggesting that the origin is from the RVOT rather than LVOT.5 The VT was mapped
and localized to the posteroseptal region of the RVOT. It was adenosine-sensitive, indicating that
the mechanism was due to triggered activity. The arrhythmia was eliminated with radiofrequency
ablation.

References
1. Lerman BB. Outflow tract ventricular arrhythmias: An update. Trends Cardiovasc Med. 2015;25(6):550–558.
2. Asirvatham SJ. Correlative anatomy for the invasive electrophysiologist: Outflow tract and supravalular arrhythmia.
J. Cardiovasc. Electrophysiol. 2009;20:955–968.
3. Ho SY. Anatomic insights for catheter ablation of ventricular tachycardia. Heart Rhythm. 2009;6:S77–S80.
4. Betensky BP, Park RE, Marchlinski FE, et al. The V(2) transition ratio: A new electrocardiographic criterion for
distinguishing left from right ventricular outflow tract tachycardia origin. J Am Coll Cardiol. 2011;57:2255–2262.
5. Yoshida N, Yamada T, McElderry HT, et al. A novel electrocardiographic criterion for differentiating a left from
right ventricular outflow tract tachycardia origin: the V2S/V3R index. J Cardiovasc Electrophysiol 2014;25:747–759.

Case 9A.8  u  319


CASE
9A.9
Dan Blendea, MD, PhD
Moussa Mansour, MD

Patient History
A 72-year-old female developed ventricular tachycardia after cardiac catheterization.
The patient presented with profound sinus bradycardia with heart rates in the 20–30 bpm
range. Cardiac catheterization showed a stenosis of the right coronary artery, which was stented.
A temporary pacing wire was placed before the intervention given her bradycardia. Soon after the
catheterization, the patient developed ventricular tachycardia (Figure 9A.9.1). She received a 150 J
shock externally that was successful in terminating the arrhythmia. Post-shock, the patient is
complaining of chest soreness. She received 150 mg of amiodarone bolus but remains with frequent
runs of nonsustained ventricular tachycardia post shock. The rhythm strip from the event is shown
in Figure 9A.9.1.

Figure 9A.9.1  Rhythm strip showing the onset of the ventricular tachycardia that converted to sinus after a 150 J external shock.

320  u  Section 9A: Ventricular Tachycardia/Fibrillation


Question
What should be the next step in this patient’s management?
1. More amiodarone boluses
2. Lidocaine boluses
3. Take her to the catheterization laboratory and recheck the RCA stent for possible occlusion
4. None of the above

Figure 9A.9.2  The morphology of the PVCs and first beat of the ventricular tachycardia (marked with red arrows) are very similar to the
paced complex.

Discussion, Interpretation, and Answers


The correct answer is 4. The morphology of the PVCs (Figure 9A.9.2, arrows) and first beat of the
ventricular tachycardia are very similar to the paced complex, suggesting that the ectopy is induced
by mechanical stimulation by the temporary pacing wire. After repositioning the temporary pacing
wire, there was no more ventricular tachycardia. The patient received a permanent pacemaker.
During the pacemaker implant, there were multiple runs of ventricular tachycardia during the
implant of the right ventricular lead.

Case 9A.9  u  321


CASE
9A.10 John M. Miller, MD

Patient History
A 32-year-old male arrived at the emergency department for evaluation of persistent fatigue. He had
a viral illness some weeks earlier and had not recovered his energy. On routine vital signs, his pulse
was rapid and irregular, and an ECG (Figure 9A.10.1) was obtained as shown.

Figure 9A.10.1

Question
What does his ECG illustrate and how might it relate to the cause of his fatigue?

Discussion
The ECG shows a very irregular wide complex tachycardia, which at first glance appears to be atrial
fibrillation with left bundle branch block aberration. This would be unusual in an otherwise healthy
young male (right bundle branch block being the more common type of aberration in young people)
and should raise suspicion that the arrhythmia is not aberrantly conducted (instead, conduction to
the ventricles using an accessory pathway, or ventricular tachycardia [VT]). On closer inspection, it is
evident that the atrial rhythm is not fibrillation, but discrete P waves are visible (most are inverted,

322  u  Section 9A: Ventricular Tachycardia/Fibrillation


suggesting retrograde conduction; P = sinus P wave, P’ = retrograde P waves, ? = uncertain atrial
activity). Application of algorithms to differentiate among potential causes of wide QRS tachycardia
in this case yield mixed results, as indicated in the table (SVT = supraventricular tachycardia):

Table 9A.10.1
Algorithm Finding in This Case Implication
Wellens 1
QRS duration 160 ms VT

Kindwall 2
QRS onset to S wave nadir in V1–V2 >60 ms VT

Brugada 3
RS present in precordial leads, RS interval <100 ms SVT

Vereckei (Vi/Vt) 4
Vi/Vt > or < 1 depending on which complex used in V5 VT or SVT

Vereckei (aVR) 5
Q wave in aVR = 40 ms VT or SVT

Pava 6
Time to peak in lead 2 = 50 ms VT or SVT

The patient was hospitalized and treated with beta- and angiotensin receptor blockers after
echocardiography showed global hypokinesis with an ejection fraction of 34%. Cardiac magnetic
resonance imaging showed neither scar nor discrete abnormality of the right ventricle to suggest
dysplasia/cardiomyopathy. Over the next several days, there was no decrease in the arrhythmia and
he was taken to the electrophysiology laboratory, where the diagnosis of VT was confirmed. The
cause was an erratically discharging focus on the lateral right ventricular free wall; catheter tip
trauma (“bump”) at the focus transiently terminated VT that then resumed. It was then quickly
eliminated with radiofrequency energy. The patient had a good recovery of his left ventricular
function to normal as well as relief from his fatigue.

Lessons
Most wide complex tachycardias that are grossly irregular on ECG are atrial fibrillation with
aberrant conduction or preexcitation. This case illustrates that, on occasion, VT can also be quite
irregular. Most VTs are reentrant and have a very regular cadence, varying by < 10 ms over the
course of several minutes or more. Focal VTs can show more irregularity but even some reentrant
VTs may also be irregular, especially when influenced by antiarrhythmic drugs (sodium channel
blocking agents).

References
1. Wellens HJ, Bar FW, Lie KI. The value of the electrocardiogram in the differential diagnosis of a tachycardia with a
widened QRS complex. Am. J. Med. 1978;64(1):27–33.
2. Kindwall KE, Brown J, Josephson ME. Electrocardiographic criteria for ventricular tachycardia in wide complex left
bundle branch block morphology tachycardias. Am. J. Cardiol. 1988;61(15):1279–1283.
3. Brugada P, Brugada J, Mont L, et al. A new approach to the differential diagnosis of a regular tachycardia with a
wide QRS complex. Circulation. 1991;83(5):1649–1659.
4. Vereckei A, Duray G, Szenasi G, et al. Application of a new algorithm in the differential diagnosis of wide QRS
complex tachycardia. Eur. Heart J. 2007;28(5):589–600.
5. Vereckei A, Duray G, Szenasi G, et al. New algorithm using only lead aVR for differential diagnosis of wide QRS
complex tachycardia. Heart Rhythm. 2008;5(1):89–98.
6. Pava LF, Perafan P, Badiel M, et al. R-wave peak time at DII: A new criterion for differentiating between wide
complex QRS tachycardias. Heart Rhythm. 2010;7(7):922–926.

Case 9A.10  u  323


CASE
9A.11 John M. Miller, MD

Patient History
A 55-year-old male with hypertensive nephropathy, who had been on dialysis for 3 years, was
referred for palpitations and an abnormal resting ECG. He had no known history of heart disease
but had palpitations for the last 6 months. ECGs in the past had shown left ventricular hypertrophy
but were otherwise normal.

Figure 9A.11.1  Resting ECG.

Question
What does this resting ECG (Figure 9A.11.1) illustrate as to the cause of the patient’s palpitations?

Discussion
The ECG (Figure 9A.11.1) shows what appears to be sinus rhythm with preexcitation using a left
posterior accessory pathway for ventricular activation (short PR interval, prominent R waves in right
precordial leads, positive in lead 1, negative in inferior leads). There are some complexes that do not
appear to have as much (if any) preexcitation (#s 4, 8, 14), during which conduction over the
pathway is less vigorous or fails entirely for no apparent reason (no change in atrial rate or
prematurity). On closer inspection, each wide QRS complex is not preceded by a P wave (see #18

324  u  Section 9A: Ventricular Tachycardia/Fibrillation


and to an extent, #17). This cannot be the case with preexcitation, so something else besides atrial
activity transmitted over an accessory pathway must be causing the wide QRS complexes. The
answer is ventricular tachycardia (VT), arising from the inferobasal free wall. The few narrow
complexes appear rather normal (no infarct pattern); the VT comes in short spurts lasting several
cycles and is rather slow (cycle length 580 ms). All of this suggests a focal VT occurring in the
absence of significant structural heart disease (i.e., no scar). This was proven to be the case at an
electrophysiologic study; his palpitations were due to idiopathic ventricular tachycardia that had
recently developed. The VT focus was very basal in the left ventricle, explaining why the QRS
pattern was consistent with preexcitation (an accessory pathway would insert into ventricular
myocardium at a very similar location after crossing the mitral annulus).

Lessons
The entire ECG must be evaluated, not just a few complexes; the correct diagnosis was made by
looking carefully at the last few complexes on the tracing. In addition, the narrower QRS complexes
were not easily explained by a premature atrial complex that could block in an accessory pathway.
VT and preexcited rhythms can look very similar due to the similarity of site of earliest ventricular
activation (basal ventricular myocardium at the atrioventricular annulus). The fact that the patient
had prior ECGs showing only left ventricular hypertrophy (no preexcitation) would be unusual for a
patient with preexcitation, but not impossible; preexcitation may rarely come and go.

Case 9A.11  u  325


CASE
9A.12
Andrés Ricardo Pérez-Riera, MD, PhD
Raimundo Barbosa-Barros, MD

Patient History
A 56-year-old female with known coronary artery disease (CAD) and past history of myocardial
infarction presented to the emergency department with palpitations followed by an episode of
syncope.
The initial ECG, illustrated in Figure 9A.12.1, revealed a sustained wide QRS tachycardia (heart
rate ≥100 bpm and QRS duration ≥120 ms). Figure 9A.12.2 shows an explanation of probable focus.

Figure 9A.12.1  ECG diagnosis. Sustained wide QRS tachycardia; heart rate of 214 bpm, QRS axis with extreme right axis deviation
between –90° and ±180° in the right upper quadrant “northwest quadrant” also known as No Man’s Land, RBBB pattern with
monophasic R in V1; predominantly negative QRS complexes in the inferior and lateral precordial leads, and pure R in aVR and aVL. This
QRS morphology suggests ventricular tachycardia (VT) with a probable focus in the apical region of the left ventricle (LV).

326  u  Section 9A: Ventricular Tachycardia/Fibrillation


Figure 9A.12.2  The presence of wide tachycardia with QRS axis on right superior quadrant (between –90° and ±180°) “northwest
quadrant” also known as No Man’s Land is strongly suggestive of VT. When RBBB-like pattern is associated, the focus of VT is in the apex
of LV.

The ECG immediately after electrical cardioversion is illustrated in Figure 9A.12.3. A previous
ECG taken 1 year earlier showed a similar pattern.
An echocardiogram confirmed the presence of anteroapical LV aneurysm with fibrosis and
apical dyskinesia.
A previous cardiac catheterization showed a complete LAD occlusion in the absence of
reperfusion therapy.

Case 9A.12  u  327


Figure 9A.12.3  ECG after cardioversion showing sinus rhythm; SAQRS –40º, qR pattern in I and aVL, SIII >SII suggesting left anterior
fascicular block. Slight ST segment elevation is present in aVL and I; old electrically inactive anteroapical area is manifested by low voltage
QRS in V4–V6) and QRS fragmentation (f-QRS) in V4 and V5. f-QRS is defined as additional notches within the QRS complex. In patients
with CAD f-QRS is associated with myocardial scar that can be detected with single-photon emission computed tomography. f-QRS is a
predictor of mortality and arrhythmic events in patients with reduced left ventricular function.

Questions
1. Which is the possible diagnosis?
2. Which is the appropriate approach?

Discussion
The diagnosis of sustained wide QRS tachycardia (defined by heart rate ≥100 bpm, and QRSd ≥120
ms) remains a challenge. The differentiation between supraventricular tachycardia with aberration
(SVT-A) and ventricular tachycardia (VT) has important therapeutic and prognostic implications.

Conceptual Definitions
Wide QRS Tachycardia (WQRST)
A name given to any ECG arrhythmic event with heart rates ≥100 bpm and QRS duration ≥120 ms.

Ventricular Tachycardia (VT)


A wide QRS tachycardia with at least ≥3 consecutive QRS complexes with a heart rate of ≥100 bpm
originating below the His bundle, i.e., in the ventricular chambers.

328  u  Section 9A: Ventricular Tachycardia/Fibrillation


Supraventricular Tachycardia with Aberration (SVT-A)
Defined by ≥3 consecutive wide QRS complexes with a heart rate of ≥100 bpm originating proximal
to the His bundle bifurcation.
In the presence of a WQRST, the following arrhythmias should be considered in the differential
diagnosis.
I. Regular or minimally irregular wide QRS complex tachycardia
a. SVT-A due to bundle branch block (15%–30% of the cases)
i. Preexisting or fixed bundle branch block
ii. Functional or tachycardia-dependent bundle branch block
b. SVT using an accessory preexcitation pathway for antegrade conduction with a macro-
reentry circuit (preexcited SVT, 1%–5% of the cases). The supraventricular impulse reaches
the ventricles through the anomalous pathway and returns retrogradely to the atria through
the normal AV junction resulting in a wide QRS complex tachycardia. This SVT entity is
known as atrioventricular reentrant tachycardia (AVRT)
c. SVT with nonspecific IV conduction delays caused by drugs, electrolyte abnormalities or
hypothermia that result in QRS prolongation
d. VT8: 80% of the cases and 95% of the cases in patients with structural heart disease. The
onset of VT may be markedly irregular during the first 30 seconds
e. Paced ventricular rhythms at tachycardia rates
II. Markedly or grossly irregular wide QRS tachycardia
a. Preexcited atrial fibrillation (AF) with antegrade conduction through an accessory pathway
b. AF with rapid heart rate response and typical bundle branch block
c. Preexcited atrial flutter with anterograde conduction through the accessory pathway
d. Atrial flutter with bundle branch block

Diagnosis
VT with a probable focus in the apical region of the left ventricle. Monomorphic VT has a similar
QRS configuration from beat to beat, originating from a single focus with identical QRS complexes.
Monomorphic VT results from a single abnormal focus or reentrant pathway and has regular,
identical-appearing QRS complexes. Some variability in QRS morphology at initiations is not
uncommon, followed by stabilization of the QRS morphology. RBBB and LBBB-like VT
configurations are terms used to describe the dominant deflection in V1, with a dominant R wave
described as “RBBB-like” and dominant S wave as “LBBB-like” configuration.5 While virtually all
VT or PVCs with “RBBB-like” pattern arise in the LV only, VTs or PVCs with “LBBB-like”
morphology can arise in either the LV or the RV. In the presence of prior infarction, VTs with
“LBBB-like” pattern virtually always arise on or adjacent to the LV septum. In patients without
structural heart disease, QRS complexes tend to be smooth and tall. With scarring of any etiology,
the QRS complexes have lower amplitudes and are broader. Notching of the QRS is a sign of scar.
QS complexes, other than in aVR, suggest the wavefront is moving away from the recording site, but
does not necessarily mean scar/infarct; however, qR or QR complexes in anatomically adjacent sites
typically is a sign of infarction. Patients without structural heart disease usually exhibit a
single-VT morphology, while in patient with significant structural heart disease multiple VTs are
common.3

Case 9A.12  u  329


Table 9A.12.1 shows electrocardiographic QRS morphology criteria favoring VT over SVT-A.

Table 9A.12.1
Authors Morphology Criteria Favoring VT
Wellens RBBB-like Monophasic R in V1 (such as the present case)
(Wellens 1978)
qR, QS, RS in V1

rS, QS, qR in V6

R/S < 1 in V6 (S > R or QS in V6)

Left axis deviation

QRS width > 140 ms

Kindwall LBBB-like R in V1 or V2 > 30 ms


(Kindwall 1988)
Any Q wave in V6

Onset of QRS to nadir of S ≥ 60 ms in V1 or V2

Notching of downstroke of S in V1 or V2

Akhtar LBBB-like Positive QRS concordance across the precordium


(Akhtar 1988)
Extreme left axis deviation (–90° to ± 180°)

Right axis deviation

QRS > 160 ms

Brugada RBBB-like Absence of RS complex in all precordial leads


(Brugada 1991)
R/S interval > 100 ms in ≥ one precordial

Wellens’ morphologic criteria in leads V1 or V6. Initial R wave in lead aVR

Vereckei Initial r or q wave > 40 ms in aVR


(Vereckei 2007, Notch on descending limb of negative onset, predominantly negative QRS in lead
2008) aVR Vi/Vt ≤ 1

References
1. Akhtar M, Shenasa M, Jazayeri M, et al. Wide QRS complex tachycardia. Reappraisal of a common clinical problem.
Ann. Intern. Med. 1988;109(11):905–912. doi:10.7326/0003-4819-109-11-905.
2. Brugada P, Brugada J, Mont L, et al. A new approach to the differential diagnosis of a regular tachycardia with a
wide QRS complex. Circulation. 1991;83(5):1649–1659. doi: 10.1161/01.CIR.83.5.1649.
3. Josephson ME. Clinical Cardiac Electrophysiology: Technique and Interpretation. 4th ed. Philadelphia, PA, USA:
Walters Kluwer/Lippicott Williams& Wilkins. 2008.
4. Kindwall KE, Brown J, Josephson ME. Electrocardiographic criteria for ventricular tachycardia in wide complex left
bundle branch block morphology tachycardias. Am. J. Cardiol. 1988;61(15):1279–1283.
5. Miller JM, Marchlinski FE, Buxton AE, et al. Relationship between the 12-lead electrocardiogram during ventricular
tachycardia and endocardial site of origin in patients with coronary artery disease. Circulation. 1988;77(4):759–766.
doi: 10.1161/01.CIR.77.4.759.
6. Vereckei A, Duray G, Szénási G, et al. Application of a new algorithm in the differential diagnosis of wide QRS
complex tachycardia. Eur. Heart J. 2007;28(5):589–600. doi: 10.1093/eurheartj/ehl473.
7. Vereckei A, Duray G, Szénási G, et al. New algorithm using only lead aVR for differential diagnosis of wide QRS
complex tachycardia. Heart Rhythm. 2008;5(1):89–98. doi: 10.1016/j.hrthm.2007.09.020.
8. Wellens HJ. Electrophysiology: Ventricular tachycardia: Diagnosis of broad QRS complex tachycardia. Heart.
2001;86(5):579–585.

330  u  Section 9A: Ventricular Tachycardia/Fibrillation


CASE
Philip Podrid, MD 9A.13
Patient History
An 82-year-old male without any cardiac history presents to ophthalmology clinic for an evaluation
prior to a cataract extraction. His blood pressure is normal, but his heart rate is noted to be rapid.
He is sent to the emergency department, where an ECG is obtained (Figure 9A.13.1). He is treated
with adenosine, IV beta-blocker and IV verapamil. However, the tachycardia continued and he was
successfully cardioverted. He was admitted to the hospital and shortly after admission palpitations
recurred and another ECG was obtained (Figure 9A.13.2).

Figure 9A.13.1

Case 9A.13  u  331


Figure 9A.13.2

Questions
1. What is the etiology of the rhythm in Figure 9A.13.1 and Figure 9A.13.2?
2. What features are useful for establishing the etiology of the rhythm?

Diagnosis
Ventricular tachycardia.

Discussion
In Figure 9A.13.1, the rhythm is regular with a rate of 210 bpm. The QRS complexes are wide (0.12
seconds) and they do not have a morphology that is typical for a left or right bundle branch block.
The axis is normal between 0° and +90° (positive QRS complex in leads I and aVF). There are no
obvious P waves seen, but there are subtle changes in the ST-T waves (^), which may represent
superimposed P waves or changes in ventricular repolarization and also subtle changes in the QRS
complex morphology. These features are consistent with a ventricular tachycardia. Ventricular
tachycardia results from a focus within the ventricular myocardium and the impulse activating the
ventricles does not travel along the normal His-Purkinje system, but rather there is direct
myocardial activation. There may be changes in the direction of myocardial activation, accounting
for the changes in QRS complex morphology and in the ST-T waves. With any supraventricular
rhythm (sinus, atrial or AV nodal) conduction to the ventricles is through a fixed pathway (AV node

332  u  Section 9A: Ventricular Tachycardia/Fibrillation


or His-Purkinje system) and hence all the QRS complexes and ST-T waves are uniform in
morphology.
In Figure 9A.13.2, the rhythm is regular with a rate of 210 bpm. The QRS complex axis and
morphology are identical to the QRS complexes seen in Figure 9A.13.1. Subtle changes in QRS
complex morphology (+) and in the ST-T waves (^) are again seen. In addition, there is clear evidence
of P waves (v), especially in lead V1, that are dissociated from the QRS complexes, i.e., they are not
seen with each QRS complex and they have a variable relationship to the QRS complexes. As with
Figure 9A.13.1, this is ventricular tachycardia. The most important finding is AV dissociation,
which in a wide complex tachycardia is diagnostic of ventricular tachycardia.

Case 9A.13  u  333


CASE
9A.14
Magdi M. Saba, MD
David E. Ward, MD

Patient History
A 55-year-old male presents to the emergency department with repetitive bursts of wide complex
tachycardia and presyncope.

Figure 9A.14.1

Questions
What is the most likely mechanism? What condition must be excluded?
The QRS morphology (left bundle branch block pattern, left superior axis) of the wide-complex
rhythm suggested an origin from the right ventricular body, rather than the outflow tract. The
visible atrioventricular (AV) dissociation (arrows) rules out AV reentrant tachycardia (AVRT)
involving an atriofascicular pathway and confirms a ventricular arrhythmia.

334  u  Section 9A: Ventricular Tachycardia/Fibrillation


Figure 9A.14.2

The patient had a normal echocardiogram and cardiac magnetic resonance imaging, ruling out
arrhythmogenic right ventricular cardiomyopathy (ARVC).

Discussion
During the electrophysiology study, the repetitive ventricular tachycardia (VT) was mapped to a
region consistent with the free wall insertion of the moderator band. Radiofrequency ablation at
this site resulted in total abolition of the VT and ectopy.

Case 9A.14  u  335


CASE
9A.15 Mohammad Ali Sadr-Ameli, MD

Figure 9A.15.1

The ECG has a wide QRS tachycardia, with a right bundle branch block (RBBB) pattern. There is a
superior axis at a rate of 180 bpm.
RBBB pattern with R/S ratio <1 in lead V6.
The ECG has a positive QRS complex in aVR.
A positive monophasic R wave as well as atrioventricular dissociation is present in lead V1.
Sustained monomorphic ventricular tachycardia originating from the left ventricle.

336  u  Section 9A: Ventricular Tachycardia/Fibrillation


CASE
Mohammad Shenasa, MD 9A.16
Patient History
A 55-year-old male with remote anterior wall myocardial infarction and palpitations. The patient
underwent 2-lead 24-hour Holter monitoring, which is shown in Figure 9A.16.1, Panels A–D.
He had an episode of near-syncope (Figures 9A.16.1A–C) while in the waiting room to take his
monitor off and received a chest thump (Figure 9A.16.1D). Interestingly, the tachycardia terminated
and the patient was admitted for cardiac workup and an implantable cardioverter-defibrillator.

Figures 9A.16.1.A

Case 9A.16  u  337


Figures 9A.16.1.B

Figures 9A.16.1.C

338  u  Section 9A: Ventricular Tachycardia/Fibrillation


Figures 9A.16.1.D  Used with permission from Shenasa et al. Card. Electrophysiol. Clin. 2014;6:613–621.

Discussion
Analysis of his Holter monitor showed:
Panel A: Sinus rhythm (SR) with premature ventricular complexes in the pattern of ventricular
bigeminy
Panel B: Initiation of sustained monomorphic ventricular tachycardia (VT) after a three-beat
VT with a short–long–short sequence (SLS)
Panel C: Sustained monomorphic VT continues
Panel D: Termination of VT with chest thump and resumption of SR

References
1. Roelke M, Garan H, McGovern BA, et al. Analysis of the initiation of spontaneous monomorphic tachycardias by
stored intracardiac electrograms. J. Am. Coll. Cardiol.1994;23(1):117–122.
2. Denker S, Lehmann M, Mahmud R, et al. Facilitation of ventricular tachycardia induction with abrupt changes in
ventricular cycle length. Am. J. Cardiol. 1984;53(4):508–515.

Case 9A.16  u  339


CASE
9A.17
Hossein Shenasa, MD, MS
Mohammad Shenasa, MD

Patient History
A 27-year-old male was admitted to the emergency department due to palpitations and
near-syncope.
Resting ECG showed sinus rhythm and was within normal limits. Cardiac enzymes were
negative; therefore, he underwent a treadmill testing (Bruce protocol).

Figure 9A.17.1

The ECG in Figure 9A.17.1 was taken during stage one treadmill testing, where the developed
wide complex tachycardia at the rate of 193 bpm. The blood pressure was 88/48 mmHg. There were
occasional sinus beats with narrow complex beats (red arrows).

340  u  Section 9A: Ventricular Tachycardia/Fibrillation


Figure 9A.17.2

The patient continued to be in tachycardia. The heart rate was 190 bpm; the QRS duration was
126 ms.

Question 1
What is the diagnosis of this ECG?
1. Ventricular tachycardia
2. Preexcited tachycardia
3. Atrial fibrillation with conduction over accessory pathway
Due to the relatively narrow QRS duration and the patient’s age and morphology, it was felt that
this could be a right ventricular (RV) outflow tract tachycardia, and 2.5 mg of intravenous
metoprolol was administered by the on-call emergency department cardiologist. The tachycardia
rate decreased to 181 bpm (Figure 9A.17.3) and then converted to sinus rhythm at the rate of 78 bpm
(Figure 9A.17.4). The QRS duration in Figure 9A.17.4 was 88 ms and was unremarkable. There was
no evidence of ventricular preexcitation.

Case 9A.17  u  341


Figure 9A.17.3

Figure 9A.17.4

The patient’s echocardiogram was normal. Electrophysiology studies demonstrated ventricular


tachycardia, originating from high RV outflow region.

342  u  Section 9A: Ventricular Tachycardia/Fibrillation


Question 2
Is this a variant of:
1. RV dysplasia VT
2. RV outflow VT
3. Malignant form of RV outflow VT that has recently been reported

Answer 2
All of the above are possible, but it is most likely number 3.

References
1. Iwai S, Cantillon DJ, Kim RJ, et al. Right and left ventricular outflow tract tachycardias: Evidence for a common
electrophysiologic mechanism. J. Cardiovasc. Electrophysiol. 2006;17:1052–1058.
2. Hoffmayer K, Machado ON, Marcus GM, et al. Electrocardiographic comparison of ventricular arrhythmias in
patients with arrhythmogenic right ventricular cardiomyopathy and right ventricular outflow tract tachycardia. J.
Am. Coll. Cardiol. 2011;58(8):831–838.
3. Hawwani HM, Morton JB, Kalman JM. Using the 12-lead ECG to localize the origin of atrial and ventricular
tachycardias: Part 2—Ventricular tachycardia. J. Cardiovasc. Electrophysiol. 2009;20:825–832.
4. Mehrotra A, Dixit S. Electrocardiogram characteristics of outflow tract ventricular tachycardia. Card.
Electrophysiol. Clin. 2014;6: 553–565.
5. Noda T, Shimizu W, Taguchi A, et al. Malignant entity and idiopathic ventricular fibrillation and polymorphic
ventricular tachycardia initiated by premature extrasystoles originating from the right ventricular outflow tract. J.
Am. Coll. Cardiol. 2005;46:1288–1294.
6. Shimizu W. Arrhythmias originating from the right ventricular outflow tract: How to distinguish “malignant” from
“benign”? Heart Rhythm. 2009;6:1507–11.

Case 9A.17  u  343


CASE
9A.18
Mohammad Shenasa, MD
Mark E. Josephson, MD

Patient History
A 54-year-old male arrived to the emergency department with recurrent palpitations and near-
syncope. The patient had a known history of myocardial infarction, recurrent ventricular
tachycardia (VT), a dual-chamber implantable cardioverter-defibrillator (ICD) placement, and
hypertension. At the time of presentation he was not on antiarrhythmic agents. Rhythm strips were
recorded upon arrival and are shown below. The patient had signed against medical advice;
therefore, a 12-lead electrocardiogram is not available.

Figure 9A.18.1  Sinus rhythm at 76 bpm with normal PR, QRS, and QT intervals.

344  u  Section 9A: Ventricular Tachycardia/Fibrillation


Figure 9A.18.2  Demonstrate initiation of rapid nonsustained VT (tachycardia 1). The red line denotes the coupling interval between the
last sinus beat to the first beat of the tachycardia.

Figure 9A.18.3  Initiation of a sustained monomorphic wide complex tachycardia (tachycardia 2) (186 bpm), preceded by a rapid
nonsustained VT (tachycardia 1).

Case 9A.18  u  345


Figure 9A.18.4  Initiation of a sustained wide complex tachycardia (tachycardia 3) with a rate of 198 bpm preceded by rapid
nonsustained VT (tachycardia 1).

Figure 9A.18.5  Initiation of a sustained monomorphic wide complex tachycardia (tachycardia 3) with spontaneous conversion to another
wide complex tachycardia (tachycardia 2) with a different morphology with the same heart rate of 198 bpm.

Figure 9A.18.6  Spontaneous termination of tachycardia 2. Note there is a P wave at the end (red arrow) that is most likely a
coincidence. Also note the four pacing (stimulus artifacts) due to an attempt of antitachycardia pacing by the ICD. This attempt arrived at
the time the tachycardia was terminated; however, it was a committed attempt by the device.

346  u  Section 9A: Ventricular Tachycardia/Fibrillation


Questions
1. What is the definition of this short nonsustained wide complex tachycardia (tachycardia 1) in
Figures 9A.18.2–9A.18.5?
2. What is the sustained tachycardia (tachycardia 2) after the short run of fast tachycardia
(tachycardia 1) in Figures 9A.18.3, 9A.18.5, and 9A.18.6?
3. What is the tachycardia (tachycardia 3) in Figures 9A.18.4 and 9A.18.5?

Answers and Discussion

Figure 9A.18.7

1. Tachycardia 1 in Figure 9A.18.7 is a short-coupled rapid nonsustained VT. This is a specific form
of VT.1 The coupling interval between the last sinus beat and this tachycardia is very similar
(red line).

Case 9A.18  u  347


Figure 9A.18.8

2. The short-coupled nonsustained VT initiates a wide complex monomorphic tachycardia at a rate


of 186 bpm and is most likely VT. Note 2:1 ventriculoatrial block (red arrows), which is highly
specific for VT.2

Figure 9A.18.9

3. The wide complex tachycardia (tachycardia 3) in Figure 9A.18.9 after a short-coupled


nonsustained VT are also VT at a rate of 198 bpm. These strips also demonstrate 2:1
ventriculoatrial conduction block (red arrows).

348  u  Section 9A: Ventricular Tachycardia/Fibrillation


In Figure 9A.18.6, note that tachycardia 2 is terminated spontaneously with the presence of a
P wave and is part of the 2:1 ventriculoatrial block and is coincident with termination of the
tachycardia. Note four pacing spikes are present after termination of the tachycardia due to a
committed antitachycardia program of the ICD, which arrived after the tachycardia termination.
The mechanisms of the change in tachycardia morphology and rate remain speculative, but it may
be due to a presence of different exits from the same circuit with a shared isthmus.3,4

References
1. Leenhardt A, Glaser E, Burguera M, et al. Short-coupled variant of torsade de pointes. A new electrocardiographic
entity in the spectrum of idiopathic ventricular tachyarrhythmias. Circulation. 1994;89:206–215.
2. Akhtar M, Shenasa M, Jazayeri M, et al. Wide QRS complex tachycardia reappraisal of a common clinical problem.
Ann. Intern. Med. 1988;109(11):905–912.
3. Stevenson WG. Current treatment of ventricular arrhythmias: State of the art. Heart Rhythm. 2013;10(12):
1919–1926.
4. Anter E, Josephson ME. Ventricular tachycardia in ischemic heart disease. In Cardiac Electrophysiology: From Cell
to Beside. Zipes D, Jalife J, eds. Philadelphia, PA: Elsevier; 2014.

Case 9A.18  u  349


CASE
9A.19
Paolo China, MD
Sakis Themistoclakis, MD

An 80-year-old male with type 2 diabetes, mild chronic kidney disease, and history of myocardial
infarction with severe systolic dysfunction, and a dual-chamber implantable cardioverter-
defibrillator (ICD) for secondary prevention was admitted to the emergency department for
palpitations and shortness of breath. The patient also suffered from recurrent paroxysmal atrial
fibrillation and had an ischemic stroke several years ago. He recovered well after the stroke and was
taking the following medical therapy: clopidogrel, ramipril, furosemide, amiodarone, bisoprolol,
potassium kanreonate, atorvastatin, warfarin, and insulin. At admission, the arrhythmias were well
tolerated, and the first ECG obtained showed a wide QRS-complex tachycardia (Figure 9A.19.1). The
ICD did not give any therapy; however, because of symptoms and suspecting a ventricular
tachycardia, the patient was cardioverted with direct current shock. The ECG after successful
cardioversion is shown in Figure 9A.19.2. The ICD detected the arrhythmias as a supraventricular
tachycardia (SVT) and for this reason and for the slow rate did not delivered any therapy. The strips
with electrogram (EGM) during tachycardia are showed in Figures 9A.19.3 and 9A.19.4.

Figure 9A.19.1  First 12-leads resting ECG (paper speed 25 mm/s).

350  u  Section 9A: Ventricular Tachycardia/Fibrillation


Figure 9A.19.2  12-lead ECG obtained after cardioversion (paper speed 25 mm/s).

Figure 9A.19.3  EGM obtained from ICD interrogation referring to the last tachycardia.

Figure 9A.19.4  EGM obtained from ICD interrogation referring to the last tachycardia.

Questions
Who was right: the ICD or the cardiologist?

ECG
The ECG in Figure 9A.19.1 showed a wide complex tachycardia with a heart rate of 125 bpm. It was
possible to recognize many P waves at a faster rate (about 167 bpm) and regular interval (black
arrows) in lead V1 that confirms the presence of supraventricular tachycardia (SVT). The

Case 9A.19  u  351


differential diagnosis needs to be made between SVT conducted to the ventricles with bundle
branch block aberrancy and a ventricular tachycardia (VT) concomitant with SVT.
No fusion beats were observed in the ECG despite the low rate of the tachycardia. The RR
interval was regular. A left axis deviation in the frontal plane and an initial R wave in lead aVR were
observed. The QRS duration was 200 ms with a negative concordance of QRS complexes in the
precordial leads. The QRS morphology showed a left bundle branch block (LBBB) pattern with an
initial R wave in lead V1 at 40 ms, notching of downstroke of S wave and interval between the
beginning of QRS and the nadir of S wave of 120 ms in lead V2. A QS morphology in lead V6 could
also be observed.
The 12-lead ECG in Figure 9A.19.2 showed a dual-chamber paced rhythm with QRS fusion after
sinus rhythm restoration.

Discussion, Interpretation, and Answer


According to the ICD interrogation, the tachycardia was regular with an atrial cycle length (CL) of
360 ms and a ventricular CL of 480 ms with a misleading cyclical regular relationship between
them. The ventricular rate was slower than the programmed window of therapy. Since there were
more atrial than ventricular beats detected, the device considered the arrhythmia as an SVT.
However, looking at the atrial and ventricular EGM, it was evident that the atrial activity is
completely independent of ventricular activity (atrioventricular [AV] dissociation) (Figures 9A.19.3
and 9A.19.4). In fact, in Figure 9A.19.3, a single atrial premature contraction occurred with an AA
interval of 310 ms without affecting ventricular tachycardia CL (tCL) (480 ms). Similarly, in Figure
9A.19.4, the atrial tCL shortened without any changing in ventricular tCL.
The AV dissociation is one of the most important ECG criteria for the diagnosis of VT with a
specificity of 100% but with a sensitivity of 21%. AV dissociation can be difficult to recognize
because the P waves are often difficult to identify on ECG during a wide complex tachycardia.
However, in this case, the AV dissociation can be clearly visible on the EGM detected by the ICD
(arrowheads in Figure 9A.19.1 show regular fast P waves in lead V1 fused in the QRS during the VT).
At device interrogation, the atrial arrhythmia was found to begin at a slightly shorter tCL many
days before the ventricular tachycardia. With the EGM, the AV conduction was visible (Figure
9A.19.5).

352  u  Section 9A: Ventricular Tachycardia/Fibrillation


Figure 9A.19.5  EGM obtained from ICD interrogation referring to tachycardia many days before admission.

Moreover, the 12-lead ECG is diagnostic for VT for the following criteria.
1. QRS complex duration: A QRS duration greater than 160 ms with LBBB pattern suggests the
diagnosis of VT.
2. Concordant precordial pattern: The QRS complexes in precordial leads are entirely negative
(“concordant”) and are diagnostic for VT. Indeed, a negative concordance is nearly always visible
in VT and never in SVT. This pattern suggests VT with a specificity of more than 90%, however
this criterion has low sensitivity being present in <20% of all VTs.
3. Morphology of QRS complexes in leads V1 and V6: In patients with LBBB morphology, the
presence of broad R wave in lead V1, slurred or notched downstroke of the S wave, delayed nadir
of S wave in leads V1 and V2, and the presence of Q or QS wave in lead V6 were strong predictors
of VT.1,2
4. The interval from the onset of the R wave to the deepest part of the S wave longer than 100 ms is
not observed in any SVT with aberrant conduction and is specific for the diagnosis of VT. This
criterion is part of the Brugada algorithm that includes the evaluation for AV dissociation and
the morphological QRS criteria in leads V1 and V6 as previously described. This algorithm has a
sensitivity and specificity of 98.7% and 96.5% in the diagnosis of VT, respectively.3
5. The presence of an initial R wave (Rs complex) in aVR suggests VT. This criteria is included in
the aVR ‘Vereckei’ algorithm that has a 91.5% overall accuracy in the diagnosis of VTs.4
In conclusion, the patient had both SVT and a sustained VT at the same time. The device
recognized only the first arrhythmia and didn’t deliver any therapy. The clinician made the right
decision based on the 12-lead ECG that was confirmed at the EGM analysis. The electrical
cardioversion interrupted both arrhythmias.

Case 9A.19  u  353


References
1. Wellens HJ. Electrophysiology: Ventricular tachycardia: Diagnosis of broad QRS complex tachycardia. Heart.
2001;86:579–585.
2. Steurer G, Gursoy S, Frey B, et al. The differential diagnosis on the electrocardiogram between ventricular
tachycardia and preexcited tachycardia. Clin. Cardiol. 1994;17:306–308.
3. Brugada P, Brugada J, Mont L, et al. A new approach to the differential diagnosis of a regular tachycardia with a
wide QRS complex. Circulation. 1991;83:1649–1659.
4. Vereckei A, Duray G, Szénási G, et al. New algorithm using only lead aVR for differential diagnosis of wide QRS
complex tachycardia. Heart Rhythm. 2008;5:89–98.

354  u  Section 9A: Ventricular Tachycardia/Fibrillation


SECTION 9B
Supraventricular Tachycardia (SVT) with Bundle
Branch Block (BBB)

CASE
9B.1
Mohammad-Ali Jazayeri, MD
Mohammad-Reza Jazayeri, MD

Patient History
A 22-year-old male patient underwent electrophysiological studies (EPS) for evaluation of recurrent
palpitations. Figure 9B.1.1 is a segment of tachycardia in this patient, which shows cycle length (CL)
and QRS alternans.

Figure 9B.1.1  QRS complex and cycle length alternans. The tracings from top to bottom are ECG leads 1, 2, and V1 followed by time
lines (T). See text for further discussion.

Question
What is the most likely type of rhythm shown in Figure 9B.1.1?
1. Atypical (slow/slow) form of AVNRT
2. Bigeminal rhythm with aberrant conduction
3. Orthodromic reentrant tachycardia (ORT)
4. Sinus rhythm with interpolated ectopic beats

ECG Masters’ Collection: Favorite ECGs from Master Teachers Around the World © 2017 Mohammad Shenasa, Mark E. Josephson,
N. A. Mark Estes III, Ezra A. Amsterdam, Melvin Scheinman. Cardiotext Publishing, ISBN: 978-1-942909-08-8.   355
Comments
The correct answer is 3. The two striking features shown in Figure 9B.1.1 are the alternating CL
and QRS morphology. Neither of these two features is commonly observed during supraventricular
tachycardia (SVT). It has been reported in one series as occurring in 10% of ECGs showing SVT.1
Alternating QRS morphology is even less common. As depicted in Figure 9B.1.1, the CL difference
is subtle at 20 ms. Needless to say, on a regular ECG at a paper speed of 25 mm/s, this alternation is
unperceivable and most likely remains unnoticed. In addition to the CL alternans, the QRS
morphology also alternates between normal complexes (#1, 3, 5, 7) and those (#2, 4, 6) exhibiting
left anterior fascicular block (LAFB). Moreover, the complexes #2 and 4 also exhibit right bundle
branch block (RBBB) along with LAFB. Of note, all the complexes showing LAFB consistently
precede the longer CLs. The lack of such correlation between the CL alternans and RBBB is evident
in the cycle preceded by the complex #6, which only exhibits LAFB. Figure 9B.1.2 is a duplicate of
Figure 9B.1.1, which also provides the intracardiac tracings for further analysis. The ventriculoatrial
(VA) time during complexes showing LAFB (with or without RBBB) is 285 ms, 20 ms longer than
that during other complexes. The VA prolongation of 15–35 ms during ORT with LAFB is
consistent with a left free-wall accessory pathway being the retrograde limb of the reentrant
circuit.2 The presence of RBBB has no further impact on this phenomenon.

Figure 9B.1.2  Intracardiac ECGs reveal the underlying electrophysiological characteristics. Tracings from top to bottom are: ECG leads 1,
2, V1, high right atrial (HRA), proximal and distal coronary sinus (CSp and CSd), His bundle (HB) electrograms followed by time lines (T).
The intervals are in milliseconds (ms). Note that the ventriculoatrial (VA) intervals for the QRS complexes #2, 4, and 6 with LAFB are 145
ms, 20 ms longer than that for the complexes #3, 5, and 7 with narrow QRS complexes. Also note the presence of a RBBB in complexes
#2 and 4 and how its disappearance in complex #6 has no bearing on the VA. Asterisks in CSp denote a separation of the local ventricular
and atrial electrograms due to VA lengthening in response to LAFB.

356  u  Section 9B: Supraventricular Tachycardia with Bundle Branch Block


Discussion
In a normal His-Purkinje system (HPS), like any other non-pacemaker tissues, the resting
membrane potential (phase 4) of the transmembrane action potential (TAP) is between –85 and
–95 mV and remains constant until the cell is excited (Figure 9B.1.3). Once the cell is excited, the
TAP becomes less negative until it reaches the threshold potential (~ –70 mV). At this point, a
sequence of events takes place that gives rise to the TAP. The fast inward sodium current, through
fast sodium channels, produces the initial part (phase 0) of TAP. The level of membrane potential at
the time of phase 0 is a major determinant of the sodium channel availability. In general, a
maximum number of sodium channels are available and thus maximal fast inward sodium current
is achievable when the membrane potential is around –85 and –95 mV. Otherwise, in less negative
membrane potentials, the number of sodium channels and therefore, the amplitude and maximal
rate of phase 0 upstroke (dV/dTmax also known as Vmax) are diminished. Conduction of the impulses
across the HPS depends upon a number of interrelated factors including dV/dTmax, the amplitude of
the action potential, the level of membrane potential, and the level of threshold potential. It may be
safe to say that once the membrane potential falls below –70 mV, conduction delay becomes
apparent, and when it reaches –50 mV or less, a complete conduction block develops.3,4

Figure 9B.1.3  Schematic representation of a normal Purkinje fiber TAP (phases 0-4) and of the responses elicited by premature
stimulation at selected times during repolarization. Note that the amplitude and Vmax of the responses are related to the level of
membrane potential at the time of stimulation. The earliest responses (a and b) arise at such low levels of membrane potential and are
consequently so small and slow rising that they cannot propagate. Subsequent responses (c to d) show progressive increase in amplitude,
rising velocity, and duration until it completely normalizes (e). (From Singer D, Ten Eick RE. Am. J. Cardiol. 1971;28:381–401; modified
with permission.)

Aberrant conduction or functional block (FB) occurs when a supraventricular impulse arrives
during the refractory period (phase 3) of the HPS. Figure 9B.1.3 shows the normal TAP responses to
the premature impulses arriving at the HPS, from early to late phase 3, resulting in different
magnitude of conduction disturbances. FB during arrival of an appropriately timed atrial impulse
most commonly occurs in the proximal bundle branches. Nonetheless, under different
circumstances, FB may also develop in the main His bundle, the distal bundle branches (unilateral
or bilateral), or the left anterior and posterior fascicles. This type of conduction disturbance is
primarily provoked by: (1) long-to-short CL changes, (2) rate acceleration, or (3) retrograde
concealment.5 Because of its relevance to the case presented, only the long-to-short changes will be
discussed here. For an FB to occur in the HPS, the functional refractory period of the

Case 9B.1  u  357


atrioventricular (AV) node must be equal to or shorter than the relative (or effective) refractory
period of the HPS. In the majority of individuals, there is a critical range of premature coupling
intervals that result in FB. Since the action potential duration and thus the refractoriness of the
HPS are CL dependent, FB is more likely to occur at a relatively long CL preceding the premature
impulses. It has been shown that an abrupt CL variation has a dynamic influence on the HPS
refractoriness. A sudden CL prolongation (i.e., short-to-long) lengthens the refractoriness of the
HPS and conversely, an abrupt CL shortening (i.e., long-to-short) shortens the HPS refractoriness.
Atrial bigeminal rhythms generally occur when two supraventricular pacemakers (including
sinus beats, junctional, or ectopic atrial beats) generate impulses in an alternative fashion. The
resultant rhythm typically exhibits alternating short and long CLs. For all practical purposes, the
impulse with a shorter preceding CL behaves as a premature beat (PB). Depending upon several
factors, the PB may conduct with aberrancy or FB along the HPS. Among these factors, the degree
of prematurity of the PB and the length of the long CL should be considered as the most important
ones.5 Figure 9B.1.4 is the schematic representation of the TAPs of the normal HPS during CL
alternans leading to FB. FB occurs in the right bundle more frequently than its left counterpart and
this has been attributed to the longer refractoriness of the former.

Figure 9B.1.4  Development of FB during cycle length alternans. Schematic TAPs in the HPS and simultaneous ECG leads II and V1 during
cycle length (CL) alternation are depicted. Note that the QRS complexes and their corresponding APs during the first two beats at CL1
are normal. The third QRS complex is also normal, but the AP duration is prolonged (a + n > a) due to an abrupt CL prolongation (CL2 >
CL1). The fourth QRS complex at CL1 exhibits RBBB and LAFB with the corresponding AP, which occurs when the preceding beat has not
completed its phase 3 and the membrane potential is at –60 mV resulting in markedly diminished AP amplitude and Vmax.

358  u  Section 9B: Supraventricular Tachycardia with Bundle Branch Block


As shown in Figure 9B.1.5, the CL of ORT circuit is the sum of AH, HV, and VA intervals.
Prolongation of any of these components would result in ORT CL prolongation if the other intervals
remain constant. In this case, the VA prolongation in response to LAFB in QRS complexes #2, 4,
and 6 leads to CL alternans. This phenomenon of CL alternans is obviously not unique to ORT and
may occur in other types of SVT. Usually, in the typical form of AVNRT, the CL alternans is due to
alternating conduction over two (or more) pathways with two (or more) sets of conduction time in
the antegrade (i.e., AH) pathway(s) (Figure 9B.1.6 and 9B.1.7A). However, in the atypical form of
AVNRT, the AH or the retrograde (HA) intervals or both may show alternation
(Figure 9B.1.7).

Figure 9B.1.5  Cycle length (CL) alternans in ORT. The ORT circuit is depicted here in the laddergrams. The three components of the
circuit (i.e., AH, HV, and VA intervals) comprise the ORT CL. Panel A shows a regular ORT without any CL variation. Panel B exhibits CL
alternans as a result of alternating long and short AV nodal (AH) intervals. This is probably the most common type of CL alternans. Panel
C represents the situation encountered in this case (9B.1) with CL alternans caused by alternating VA interval.

Case 9B.1  u  359


Figure 9B.1.6  Cycle length (CL) alternans in AV nodal reentrant tachycardia (AVNRT). Both panels show 12-lead ECGs of typical AVNRT
in a patient. Panel A depicts CL alternation between 445 and 395 ms. Panel B demonstrates grouped beating consisting of three CLs, the
short (#1), intermediate (#2), and long (#3) constantly recurring in that order until AVNRT is terminated.

360  u  Section 9B: Supraventricular Tachycardia with Bundle Branch Block


Figure 9B.1.7  Different situations in AV nodal reentrant tachycardia (AVNRT) resulting in cycle length (CL) alternans. Panels A-C exhibit
CL alternans in an AVNRT circuit caused by alternating AH intervals (panel A), HA intervals (panel B), and both intervals (panel C). Of
note, the situation depicted in panel A is usually encountered in the typical form of AVNRT and as shown in the laddergram, the HH
interval changes precede the AA interval changes of the same magnitude. Conversely, panel B shows atypical AVNRT, in which AA interval
changes precede the HH interval changes of the same magnitude. Panel C represents a mixed form of AVNRT in which variations of
AA and HH intervals do not match. Panel D demonstrates a hypothetical situation with constant AA and HH intervals, but with V-V CL
alternans due to the alternating conduction times across the lower AV node, below the lower turn-around point, or the HPS. The intervals
are shown as a, b, c, and d.

The interesting and distinguishing feature in this case is an intimate and mutual relationship
between the CL alternans and LAFB, in which one sets the stage for the development of the other.
In other words, the long-to-short CL variation facilitates the development of LAFB, and that in turn,
by prolonging the VA interval, gives rise to the CL prolongation. Once developed, this dyad may
persist uninterrupted until one of the two elements (most likely the FB) is resolved. Nonetheless, the
combination of these two features can only occur in patients with ORT and alternating FB,
ipsilateral to the AP location. It is conceivable that a similar situation may also occur in patients
with ORT, in which a right- or left-sided AP (including posteroseptal AP) is associated with

Case 9B.1  u  361


ipsilateral functional bundle branch block (FBBB), alternating with narrow QRS complexes. The
rarity of this situation is probably due to the fact that once FBBB develops, the retrograde
penetration of the same bundle branch via its counterpart (concealed transseptal conduction)6
promotes the maintenance of the same FBBB for several cycles and therefore, precludes the
alternans of the QRS complexes and their corresponding CLs.

References
1. Barker PS, Johnston FD, and Wilson FN. Auricular paroxysmal tachycardia with alternation of cycle length. Am.
Heart J. 1943;25:799–811.
2. Jazayeri MR, Caceres J, Tchou P, et al. Electrophysiologic characteristics of sudden QRS axis deviation during
orthodromic tachycardia. Role of functional fascicular block in localization of accessory pathway. J. Clin. Invest.
1989;83:952–959.
3. Singer DH, Ten Eick RE. Aberrancy: Electrophysiologic aspects. Am. J. Cardiol. 1971;28:381–401.
4. Singer DH, Baumgarten CM, Ten Eick RE. Cellular electrophysiology of ventricular and other dysrhythmias: Studies
on diseased and ischemic heart. Prog. Cardiovasc. Dis. 1981;24:97–156.
5. Jazayeri M, Sra J, Akhtar M. Wide QRS complexes. Electrophysiologic basis of a common electrocardiographic
diagnosis. J. Cardiovasc. Electrophysiol. 1992;3:365–393.
6. Jazayeri M. Concealed conduction and allied concepts. Card. Electrophysiol. Clin. 2014;6:377–418.

362  u  Section 9B: Supraventricular Tachycardia with Bundle Branch Block


CASE
9B.2
George D. Katritsis, MBChB, BSc
Demosthenes G. Katritsis, MD, PhD

Patient History
Wide-QRS tachycardia, recorded in a 37-year-old female who presented with episodes of
paroxysmal arrhythmias during which both wide- and narrow-QRS tachycardias were recorded.
ECG during sinus rhythm was normal.

Figure 9B.2.1

Case 9B.2  u  363


Interpretation
This could be ventricular tachycardia, antidromic atrioventricular tachycardia, or atrioventricular
nodal reentrant tachycardia (AVNRT) conducted with aberration. Electrophysiology study was
necessary to establish a diagnosis of atypical AVNRT, as documented by prolonged RP intervals
most prominent in leads II and V2, conducted with left bundle branch block aberration. Anatomical,
slow pathway ablation resulted in abolition of the tachycardias.

364  u  Section 9B: Supraventricular Tachycardia with Bundle Branch Block


CASE
Melvin Scheinman, MD 9B.3
Patient History
A 23-year-old male presented with Ebstein’s anomaly and mild palpitations.

Figure 9B.3.1  A normal PR interval with definite evidence of preexcitation reflective of posterior septal AP; consider the possibility of
Mahaim with right-sided AP or atriofascicular or Mahaim tract.

Magnetic Resonance Imaging


• Right ventricle
• Septal leaflet apically displaced > 2 cm from the level of the mitral valve annulus
• Anterior and posterior tricuspid valve leaflets redundant and dysplastic
• Atrialization of the RV
• No tricuspid stenosis, 2+ tricuspid regurgitation
• No restricted motion of the valve leaflets

Case 9B.3  u  365


• Right atrium
• Dilated
• Atrial septal aneurysm of 2.5 cm with probable patent foramen ovale
• Left ventricle
• Mild systolic dysfunction left atrium
• Left atrium
• Normal

Further Studies/Therapy
• None required; minimal symptoms
• Exercise stress test
• Invasive electrophysiology studies are indicated
• Start therapy with beta-blockers

Exercise Stress Test


• Bruce 10:31 min
• Maximal heart rate of 171 bpm (350 ms)
• No symptoms
• Preexcitation without change from baseline to maximal heart rate
• No ischemia

Figure 9B.3.2  A wide complex tachycardia with typical RBBB pattern. The RBBB pattern is that of the type usually seen with Ebstein’s
anomaly in sinus rhythm. It is brought out by orthodromic activation of the atrioventricular node–His axis during tachycardia.

366  u  Section 9B: Supraventricular Tachycardia with Bundle Branch Block


Most Likely Cause of This Tachycardia
• Atrial flutter
• Ventricular tachycardia
• Antidromic atrioventricular reentrant tachycardia (AVRT)
• Orthodromic AVRT

Figure 9B.3.3  Atrial fibrillation with rapid ventricular response, which was induced during the study. WPW: Wolff-Parkinson-White
syndrome.

Case 9B.3  u  367


Figure 9B.3.4  A 12-lead tracing after ablation, but this time the typical RBBB pattern is shown which was originally masked by the
right-sided posterior AP (Figure 9B.3.1).

Figure 9B.3.5  Right heart in Ebstein’s anomaly.

368  u  Section 9B: Supraventricular Tachycardia with Bundle Branch Block


Ebstein’s ECG/EP Patterns
• The atrialized right ventricle is morphologically and electrically a ventricle but mechanically an
atrium.
• Intra-atrial conduction disturbances.
• Prolonged atrioventricular conduction.
• Bizarre right bundle branch block (RBBB) with a “second” QRS attached to the preceding
“normal” complex.
• Normal location of atrioventricular node, His bundle, and tricuspid annulus.
Why is there minimal pre-excitation and normal P-R in Ebstein’s anomaly:
• Abnormal intra-atrial conduction delays activation of the right-sided AP.
• The APs are often long and slowly conducting tracts.

RBBB and Ebstein’s Anomaly


• Absence of RBBB pattern: strongly predicts an accessory pathway (AP).
• 98% sensitivity, 92% specificity for diagnosis of an AP.
• Positive predictive value 91% (0.77, 0.97 CI 95%), negative predictive value 98%
(0.85, 0.99 CI 95%).

Figure 9B.3.6

Case 9B.3  u  369


Discussion
In a patient with Ebstein’s anomaly, recording an ECG without a RBBB pattern should immediately
raise the issue of a right-sided AP, which serves to mask the RBBB pattern. Patients with Ebstein’s
anomaly show the highest incidence of right free wall or posterior septal AP. In addition, they
commonly have two pathways and may have an atriofascicular pathway as well. The RBBB pattern
is due to late activation of the atrialized portion of the right ventricle. An additional feature of the
patient with Ebstein’s anomaly and APs is the frequent occurrence of a normal PR interval due to
delayed interatrial conduction. Finally, catheter ablation may prove very difficult for two reasons:
(1) difficulty in discerning the true right-sided annulus together with (2) marked fractionation of
potentials over the atrialized ventricle. At times, a right coronary artery injection may help
delineate the true annulus, since the right coronary artery courses over the true annulus rather than
over the displaced valve.

370  u  Section 9B: Supraventricular Tachycardia with Bundle Branch Block


SECTION 9C
Preexcited Tachycardia

CASE
Jonathan Kalman, MBBS, PhD 9C.1
Patient History
A 57-year-old male with recurrent palpitations and presyncope.

Figure 9C.1.1

ECG Masters’ Collection: Favorite ECGs from Master Teachers Around the World © 2017 Mohammad Shenasa, Mark E. Josephson,
N. A. Mark Estes III, Ezra A. Amsterdam, Melvin Scheinman. Cardiotext Publishing, ISBN: 978-1-942909-08-8.   371
Discussion
There are bursts of wide complex tachycardia on this Holter monitor trace. Analysis of the sinus
beats suggests the presence of preexcitation. There are bursts of atrial tachycardia with increasing
preexcitation.

372  u  Section 9C: Preexcited Tachycardia


SECTION 9D
Idioventricular Rhythm

CASE
9D.1
Marc Dubuc, MD
Jason G. Andrade, BSc, MD

Patient History
A 100-year-old male presented to hospital with oppressive retrosternal chest pain and shortness of
breath that began 45 minutes prior to presentation.

Figure 9D.1.1

Discussion
This ECG demonstrates an accelerated idioventricular rhythm (wide complex rhythm at a rate of
approximately 78 bpm), which is nearly isorhythmic with the sinus rate (arrows highlight the sinus
P waves). The third to last beat on the tracing (*) demonstrates a sinus rhythm beat with capture of
the His-Purkinje system. During this normally conducted beat, there is evidence of significant
anterior ST segment elevation, suggesting acute anterior myocardial ischemia is the cause of the
accelerated ventricular rhythm.

ECG Masters’ Collection: Favorite ECGs from Master Teachers Around the World © 2017 Mohammad Shenasa, Mark E. Josephson,
N. A. Mark Estes III, Ezra A. Amsterdam, Melvin Scheinman. Cardiotext Publishing, ISBN: 978-1-942909-08-8.   373
SECTION 10
Ischemia and Infarction

Jonathan Bui, MD CASE


10.1
Femi Philip, MD
Ezra A. Amsterdam, MD

Patient History
A 60-year-old male presented to the emergency department with severe chest pressure for several
hours. His ECGs are shown in Figures 10.1.1 and 10.1.2. Based on the ECGs, which showed a true
posterior MI, he was taken emergently to the cardiac catheterization laboratory, where he
underwent successful percutaneous coronary intervention (PCI) of a totally occluded coronary
artery. The patient’s troponin peaked at 6.94 ng/mL (ref 0.04).

Figure 10.1.1

ECG Masters’ Collection: Favorite ECGs from Master Teachers Around the World © 2017 Mohammad Shenasa, Mark E. Josephson,
N. A. Mark Estes III, Ezra A. Amsterdam, Melvin Scheinman. Cardiotext Publishing, ISBN: 978-1-942909-08-8.   375
Figure 10.1.2

In the coronary reperfusion era, recognition of true posterior ST-segment elevation myocardial
infarction (STEMI) is both crucial and challenging. Overlooking this diagnosis represents a missed
opportunity for acute coronary reperfusion by PCI or thrombolytic therapy with the associated
benefits of these approaches on mortality and morbidity.

Question 1
Which findings on the initial ECG (Figure 10.1.1) led to emergency coronary angiography?

Answer 1
The initial ECG (Figure 10.1.1) shows anterior ST-segment depression of 1–2 mm in leads V2–V4. In a
patient with symptoms compatible with myocardial ischemia, this finding is consistent with either
anterior ischemia or posterior ST-segment elevation. The latter conclusion is based on evidence that
anterior ST depression may be reciprocal to posterior ST elevation, reflecting a true posterior STEMI.1
The next step in the evaluation was urgent acquisition of a repeat ECG with the electrodes for leads
V4 –V6 relocated (at the level of V6): to the left posterior axillary line (V7), the tip of the left scapula (V8),
and just left of the spinal column (V9), respectively. As shown in Figure 10.1.2, there is marked ST
elevation in V7–V9, confirming the diagnosis of true posterior STEMI and thereby providing the
indication for acute coronary reperfusion.

Question 2
What evidence of potential true posterior MI is not present on the initial ECG (Figure 10.1.1)?

Answer 2
Tall R waves in leads V1 and V2, which may be the reciprocal of posterior Q waves, are not present
in leads V1 and V2 since there were no pathologic Q waves in leads V7–V9. The differential diagnosis

376  u  Section 10: Ischemia and Infarction


of tall R waves in leads V1 and V2 includes right ventricular enlargement, right bundle branch block,
Wolff–Parkinson–White syndrome, and lead misplacement.

Question 3
Which coronary artery is most likely occluded in this patient?

Answer 3
Studies in humans have demonstrated that occlusion of the right coronary artery is associated with
ST-segment elevation in leads II, III, and aVF, whereas left circumflex coronary artery occlusion
commonly results in ST-segment elevation in the lateral precordial leads and V7–V9.2,3 However,
distribution of the coronary arteries can differ and right coronary occlusion may also be associated
with true posterior MI. In this patient, posterior MI was due to a total proximal occlusion of the left
circumflex artery (Figure 10.1.3). He received PCI to this vessel with excellent results, as shown in
Figure 10.1.4.

Figure 10.1.3

Case 10.1  u  377


Figure 10.1.4

References
1. Amsterdam E, Wenger N, Brindis R, et al. 2014 AHA/ACC guideline for the management of patients with
non-ST-elevation acute coronary syndromes. Circulation. 2014;130:e344–e426.
2. Kulkarni A, Brown R, Ayoubi M, et al. Clinical use of posterior electrocardiographic leads: A prospective
electrocardiographic analysis during coronary occlusion. Am. Heart J. 1996;131:736–741.
3. Casas R, Marriott H, Glancy L. Value of leads V7-V9 in diagnosing posterior wall acute myocardial infarction and
other causes of tall R waves in V1-V2. Am. J. Cardiol. 1997;80:508–509.

378  u  Section 10: Ischemia and Infarction


Jonathan Bui, MD CASE
10.2
Femi Philip, MD
Ezra A. Amsterdam, MD

Patient History
A 67-year-old male presented to the emergency department with severe epigastric pain radiating to
the left chest associated with nausea, vomiting, and diaphoresis. There was no relief with antacids.
The patient’s ECG (Figure 10.2.1) revealed a paced cardiac rhythm and showed marked ST segment
elevation in leads II, III, and aVF, as well as less ST elevation in leads V4 –V6 and marked ST
depression in leads I and aVL. These findings were consistent with a possible inferior ST-segment
elevation MI (STEMI) and the patient underwent coronary angiography with revascularization of
the culprit occluded coronary artery by percutaneous coronary intervention (PCI).

Figure 10.2.1

Left ventriculography showed inferior wall hypokinesis. Troponin I was elevated. An ECG one
week following the procedure (Figure 10.2.2) revealed considerable resolution of the ST elevations
seen in Figure 10.2.1 during the acute process. Thus, the entire clinical presentation confirmed
acute myocardial infarction (MI) with total coronary artery occlusion. Note that the patient had a
ventricular pacemaker at the time of presentation (Figure 10.2.1) and that he has an atrioventricular
(AV) sequential pacer post-MI (Figure 10.2.2).

Case 10.2  u  379


Figure 10.2.2

Question 1
Which coronary artery was the culprit vessel in this patient?

Answer 1
The patient had an inferior STEMI. It was statistically likely that the culprit vessel was the right
coronary artery, which is true for 85%–90% of inferior STEMIs. However, ST elevation in the
inferior and lateral leads is consistent with a left circumflex artery occlusion, which coronary
angiography demonstrated. The challenge of recognizing STEMI in patients with paced rhythms is
similar to that in patients with left bundle branch block (LBBB), i.e., the inherent repolarization
abnormalities in paced rhythm and LBBB confound the standard criteria for diagnosing ST
elevation MI.
This patient’s inferior ST elevations were striking with “reciprocal” ST depression in leads I and
aVL. These findings together with the patient’s symptoms prompted emergency coronary
angiography.

Question 2
Are there criteria for the diagnosis of STEMI in patients with electrically paced cardiac rhythm?

Answer 2
Criteria have been suggested for diagnosis in the setting of pacemaker rhythm:1,2 (1) ≥5 mm ST
elevation in leads with negative QRS complexes; (2) ≥1 mm ST elevations in leads with positive QRS
complexes; (3) ≥1 mm ST depression in V1–V3 (if these leads are positive). The accuracy of these
criteria is limited: sensitivity is poor but the specificity is better in one report: 82%–94%.2 As in

380  u  Section 10: Ischemia and Infarction


patients with LBBB, management of these patients should be based on the entire clinical
presentation and should not depend solely on the foregoing ECG criteria for STEMI.

References
1. Klimczak A, Wranicz JK, Cygankiewicz I, Chudzik M, Goch JH, Baranowski R. Electorcardiographic diagnosis of
acute coronary syndromes in patients with left bundle branch block or paced rhythm. Cardiol. J. 2007;14(2):
207–213.
2. Karumbaiah K and Omar B. ST-elevation myocardial infarction in the presence of biventricular paced rhythm.
J. Emerg. Med. 2013;45:e35–e40.

Case 10.2  u  381


Antoni Bayés de Luna, MD
CASE Diego Goldwasser, MD

10.3 Guillem Pons-Lladó, MD


Francesc Carreras Costa, MD

Patient History
This is an ECG (Figure 10.3.1) from a 57-year-old male who presented with an acute coronary
syndrome (ACS) with ST elevation 6 months prior. The x-ray showed that the heart was in a normal
position. The “cath lab” was activated 4 hours after the onset of pain. A stent was deployed in the
middle segment of an occluded left circumflex artery (LCX) (Figure 10.3.2).

Figure 10.3.1  12-lead ECG showing sinus rhythm, normal PR and QT intervals, and normal QRS morphology in the frontal plane. Note a
tall and slurred R wave in lead V1, Rs from leads V2–V5 with small QRS in lead V6, and a positive tall T wave is present in leads
V1–V5 with slight ST ascent and asymmetric T wave, and qRs pattern in lead V6.

382  u  Section 10: Ischemia and Infarction


Figure 10.3.2  Coronary angiography showing LCX occlusion at middle segment (right). No right coronary artery occlusion is seen (left).

Question
What is the location of the myocardial infarction (MI)? Select the correct choice
1. There is no MI. This is a normal ECG with extreme levorotation.
2. Posterior wall. Segment 4 of Cerqueira classification.7
3. Lateral wall. Segments of lateral wall of Cerqueira classification.7

Interpretation, Answer, and Comments


The correct answer is 3. The ECG shows a normal P wave, PR and QT intervals, and QRS complex.
There is no abnormal Q wave in the frontal plane, but there is a low voltage R wave is present in lead
I and VL. In the horizontal plane there is a slurred tall R wave in V1 and Rs from V2–V5 with small
QRS in V6, and a positive tall T wave is present in V1–V4.
This ECG may not be explained by extreme leftward rotation. The heart at x-ray presents
normal position and the ECG of extreme leftward rotation may present unique R in V2 but not in
V1. Furthermore, in extreme left rotation the QRS axis in the frontal plane is around 0° but not
present with a S1, S2, or S3 pattern.1
Using the classic nomenclature and considering the patient history, the patient would be
diagnosed with posterior MI. According to Perloff,2 the diagnosis of posterior MI may be made
when there is in V1 R/S>1, that represents a mirror pattern of posterior Q wave (Figure 10.3.3).

Figure 10.3.3  Original drawing of true posterior MI with the QRS morphology according to Perloff (1).

Nowadays thanks to correlation between ECG and contrast-enhanced cardiovascular magnetic


resonance,3–6,8 it has been demonstrated that the prominent R in V1 corresponds to lateral (and not
posterior wall) MI due to occlusion of LCX (Figure 10.3.2). In fact, the so-called “posterior wall”
corresponds to an inferobasal segment of the inferior wall.7 The low voltage of the R wave in leads I
and VL, although without pathological Q wave, suggest lateral wall involvement. Therefore, the
correct answer is 3.
Figure 10.3.4 shows a cardiovascular magnetic resonance study where delayed contrast
enhancement (i.e., myocardial necrosis) is present at the lateral wall (B and C), while the inferobasal
segment and the rest of the inferior wall (A) are, in fact, free of necrosis.

Case 10.3  u  383


Figure 10.3.4  Contrast-enhanced cardiovascular magnetic resonance showing: A. Long axis view with no inferior wall involvement, B.
Short axis view: clear non-transmural extensive lateral involvement, and C. A 4-chamber view that shows the longitudinal extension of
lateral infarction.

On the contrary, Figure 10.3.5 shows a typical example of inferolateral MI (QR in II, III, aVF,
and R/S >1 in V1). See the correlation with contrast-enhanced cardiovascular magnetic resonance.
In old terminology, this would be diagnosed as inferoposterior MI.

Figure 10.3.5  Electrocardiographic and cardiovascular magnetic resonance images of an inferolateral infarction. Q waves of necrosis
are present in leads II, III, and aVF (inferior necrosis); an R ≥ S morphology is present in lead V1 (lateral necrosis). Contrast-enhanced
cardiovascular magnetic resonance (four-chamber, long- and short-axis views) clearly shows an inferolateral necrosis.

Figure 10.3.6 explains that in cases of MI of lateral wall the necrosis vector (yellow) faces lead V1
and records a prominent R in this lead. In case of necrosis of the inferobasal segment (former
“posterior wall”), the blue vector faces leads V3 and V4 and the surface ECG cannot record any
characteristic pattern because this lead already presents an RS pattern in normal conditions.4,7

384  u  Section 10: Ischemia and Infarction


Figure 10.3.6  Schematic drawing of the transverse plane of the thorax viewed from below. The infarction vector produced by
involvement of the wall formerly termed posterior (blue arrow) is directed toward leads V3 and V4, while the infarction vector generated by
the lateral wall (yellow arrow) is directed toward leads V1 and V2.

Figure 10.3.7 shows the three typical patterns of R wave in lead V1 (exclusive R wave often with
slurs, R>S, and low voltage r/s >0.5 compared with rS seen in case of inferior MI.

Figure 10.3.7  A comparison between different types of lateral MI showing a tall R wave in V1 with R/S > 1 or at least > 0.5 if S wave is
very small.

Conclusions
For decades, a prominent R wave (R, Rr, RS) in a right precordial lead (V1) has been considered a key
criterion for the diagnosis of posterior MI. This conclusion has been accepted as a dogma since the
paper of Perloff published more than 50 years ago.2 Now it is time to consider the end of this
dogma.9 After ECG cardiovascular magnetic resonance correlations performed 10 years ago, various
papers3-6,8 argued that, in the absence of conditions that modified QRS shape such as right
ventricular hypertrophy, complete right bundle-branch block, or Wolff–Parkinson–White
syndrome, the appearance of a prominent R wave in V1 in the course of ACSs indicates a lateral MI.
This lateral MI is usually larger and more transmurally extended than when a prominent R wave in
the right precordial leads is absent; therefore, this conclusion should be considered as a standard of
scientific guidelines.9

Case 10.3  u  385


References
1. Bayés de Luna A. Clinical Electrocardiography. West Sussex, UK: Wiley-Blackwell; 2012.
2. Perloff JK. The recognition of strictly posterior myocardial infarction by conventional scalar electrocardiography.
Circulation. 1964;30:706–718.
3. Bayés de Luna A, Cino JM, Pujadas S, et al. Concordance of electrocardiographic patterns and healed myocardial
infarction location detected by cardiovascular magnetic resonance. Am. J. Cardiol. 2006;97:443–451.
4. Bayés de Luna A, Wagner G, Birnbaum Y, et al. International Society for Holter and Noninvasive
Electrocardiography. A new terminology for left ventricular walls and location of myocardial infarcts that present Q
wave based on the standard of cardiac magnetic resonance imaging: A statement for healthcare professionals from a
committee appointed by the International Society for Holter and Noninvasive Electrocardiography. Circulation.
2006;114:1755–1760.
5. Rovai D, Di Bella G, Rossi G, et al. Q-wave prediction of myocardial infarct location, size and transmural extent at
magnetic resonance imagining. Coron. Artery Dis. 2007;18:381–389.
6. Van Der Weg K, Bekkers SC, Winkens B, et al. Evaluation of the electrocardiogram in identifying and quantifying
lateral involvement in nonanterior wall infarction using cardiovascular magnetic resonance imaging. J.
Electrocardiol. 2012;45(5):478–484.
7. Cerqueira MD, Weissman NJ, Dilsizian V, et al. American heart Association Writing Group on Myocardial
Segmentation and Registration for Cardiac Imaging. Standardized myocardial segmentation and nomenclature for
tomographic imaging of the heart. A statement for healthcare professionals from the Cardiac Imaging Committee of
the Council on Clinical Cardiology of the American Heart Association. Circulation. 2002;105:539–542.
8. Goldwasser D, Senthilkumar A, Bayés de Luna A, et al. Lateral MI explains the presence of prominent R wave (R ≥
S) in V1. Ann. Noninvasive Electrocardiol. 2015;20(6):570–577.
9. Bayés de Luna A, Rovai D, Pons LLado G, et al. The end of an electrocardiographic dogma: A prominent R wave in
V1 is caused by a lateral not posterior myocardial infarction-new evidence based on contrast-enhanced cardiac
magnetic resonance-electrocardiogram correlations. Eur. Heart J. 2015;36:959–964.

386  u  Section 10: Ischemia and Infarction


CASE
David J. Callans, MD 10.4
Patient History
A 66-year-old male with a medical history of diabetes, hypertension, and coronary heart disease
(recent PCI to the proximal LAD) presents with acute onset chest pain and dyspnea. The ECG on
presentation and a prior ECG from one month before are shown:

Figure 10.4.1  ECG on presentation.

Figure 10.4.2  ECG from one month prior.

Case 10.4  u  387


Discussion
Despite development of right bundle branch block, changes consistent with acute anterior
infarction are evident (ST elevation in anterior precordial leads and aVR, reciprocal ST depression
in inferior and lateral precordial leads). Coronary angiography on presentation demonstrated
thrombotic occlusion of the prior stent with extension of the thrombus into the left main coronary
artery.

388  u  Section 10: Ischemia and Infarction


CASE
10.5
Andrei G. Dan, MD, PhD
Catalin A. Buzea, MD, PhD

Patient History
A 62-year-old hypertensive smoker developed sudden chest pain during an admission in the
gastroenterology department. The present tracing was obtained shortly after symptom onset. ECG
one year before was normal. A coronary angiogram was done. The patient died on the day of
admission to hospital.

Figure 10.5.1  Anterior ST-elevation acute myocardial infarction with acute right bundle branch block.

Questions
1. What is cause of the ECG abnormalities?
2. Where is the culprit lesion?

Discussion, Interpretation, and Answers


The rhythm is sinus tachycardia 107 bpm. Several premature beats (x) preceded by barely visible
P waves with a very short PR interval (black arrow) are late junctional extrasystoles (as their
morphology is similar to the normal beats) associated with normal P waves. The most striking
abnormality is that of the QRS complexes. The morphology is of right bundle branch block—
monophasic type, with a small q wave preceding the R waves in V1 and V2. The ST segment is
markedly elevated in all precordial leads with the appearance of “tombstoning,”1 most apparent in
V4 –V6 (red arrow). This elevation is of “primary” type as the secondary repolarization alteration

Case 10.5  u  389


seen in usual right bundle branch block morphology is opposite to QRS complex polarity. The
appearance is of an acute myocardial infarction involving an important part of the anterior left
ventricle wall. The presence of acute right bundle branch block morphology and ST elevation > 2.0
mm in V1 and V2 are highly suggestive for left anterior descending artery obstruction at the origin,
before the first septal branch (specificity and sensitivity close to 100%).2 The tombstoning aspect
suggests lack of protection through collaterals or ischemic preconditioning.1 The low voltage in the
frontal plane is also compatible with an important infarction area.
The coronary arteriography revealed 90% LAD obstruction at the origin; a stent was placed, but
the patient developed shock followed shortly by ventricular fibrillation.

References
1. Birnbaum Y, Herz I, Sclarovsky S, et al. Prognostic significance of the admission electrocardiogram in acute
myocardial infarction. J. Am. Coll. Cardiol. 1996;27(5):1128–1132. doi:10.1016/0735-1097(96)00003-4.
2. Engelen DJ, Gorgels AP, Cheriex EC, et al. Value of the electrocardiogram in localizing the occlusion site in the
left anterior descending coronary artery in acute anterior myocardial infarction. J. Am. Coll. Cardiol.
1999;34(2):389–395.

390  u  Section 10: Ischemia and Infarction


CASE
10.6
Arnaud Denis, MD
Mélèze Hocini, MD

Patient History
A 61-year-old male with medical history of smoking and dyslipidemia was referred to our
institution for an anterior ST segment elevation myocardial infarction. Primary percutaneous
intervention was performed 12 hours after the onset of symptoms and a coronary stent was placed
in the proximal left anterior descending artery. Left ventricular ejection fraction was 30%. Ten days
after, he was referred to the intensive care unit for aborted sudden cardiac death.

Figure 10.6.1  ECG at admission.

Question
What was the mechanism of the aborted sudden cardiac death?

Discussion
The basic rhythm is sinus at 60 bpm, QTc is normal (440 ms). The ECG is consistent with a Q-wave
infarction of the anterior and inferior walls (Q waves in V1–V4, DIII, and AVF). There is a persistent

Case 10.6  u  391


ST elevation in the anterior and inferior leads, which usually indicates failed reperfusion treatment
(no-reflow phenomenon related to microvascular damage).
Two short-coupled premature ventricular complexes (PVCs) (solid arrow) are present. These
PVCs, which come from the apical and septal wall of the left ventricle, both occurred on the peak of
the T wave with a coupling interval of 280 ms. This is then followed by a sinus beat that conducts to
the ventricle with PR prolongation (PR = 240 ms). The PR prolongation is explained by the
concealed conduction of the PVC in the AV node but not in the atrium.

Figure 10.6.2

1. Same morphology of the PVC described in ECG 1.


2. The same PVC with a short coupling interval initiates a run of nonsustained polymorphic
ventricular tachycardia.

392  u  Section 10: Ischemia and Infarction


Figure 10.6.3

There is a PVC (solid arrow) with a slightly different morphology than PVC described in
previous ECG (QRS is now negative in DIII). There is a retrograde P wave (dotted arrow) (negative
in the inferior leads) coming from the PVC because the coupling interval is slightly longer (300 ms)
and the AV node is no longer in a refractory period. There is a compensatory pause after the PVC
that indicates that the sinus node has been depolarized by the retrograde conduction of the PVC.
This is then followed by a second PVC (with a short coupling interval) that initiates ventricular
fibrillation precipitated by short–long–short sequence.
This patient had recurrent ventricular fibrillation episodes even though he received
anitarrhythmic drugs and sedation. A repeat coronary angiogram did not show intrastent
thrombosis. Catheter ablation of the triggering ectopy was successfully performed. A Purkinje
potential was observed at the successful site of the PVC ablation. Recurrent episodes of ventricular
fibrillation were triggered by PVCs arising from partially injured Purkinje fibers due to ischemia
and/or reperfusion. An ICD was implanted and no therapy was required after one year of follow-up.

Case 10.6  u  393


CASE Begüm Yetiş Sayın, MD

10.7
Sercan Okutucu, MD
Ali Oto, MD

Patient History
Electrocardiogram of a 74-year-old male with chest pain. Angiography revealed ostial narrowing of
left main coronary artery. Note the ST elevation in aVR and V1 and associated ST depressions in the
anterior and inferior leads.

Figure 10.7.1

Discussion
Left main coronary artery (LMCA) disease is another clinical situation in which prompt diagnosis
by the clinician can help initiate life-saving invasive therapy. The typical electrocardiographic
finding in patients with preserved flow through the LMCA is widespread ST segment depression
maximally in leads V4 –V6, with inverted T waves and ST segment elevation in lead aVR and
frequently in V1. ST elevation in lead aVR, when accompanied by either anterior ST elevation or
widespread ST segment depression, may indicate LMCA occlusion.

394  u  Section 10: Ischemia and Infarction


CASE
10.8
Carlos Alberto Pastore, MD, PhD
Nelson Samesima, MD, PhD

Patient History
A 54-year-old male with complaints of high-intensity precordial pain radiating to his back and left
upper limb, lasting about one hour, was admitted to the emergency department.

Medical History
The patient was hypertensive, a smoker for the last 40 years (two packs of cigarettes/day),
dyslipidemic, and had a cerebral vascular accident 5 years before. He had no family history of heart
disease and no further complaints.

Medications
The patient was currently taking acetylsalicylic acid 100 mg, hydrochlorothiazide 25 mg, enalapril
40 mg, carvedilol 25 mg, and simvastatin 40 mg.

Physical Examination
Patient was alert, oriented, hydrated, afebrile, tachypneic, tachycardic, sweating, with a normal
blood pressure. Cardiovascular and respiratory systems had no abnormality. No abnormality was
found in abdomen or lower limbs.

Complementary Exams
Laboratory data: CKMB 469,000 ng/mL; troponin >150 ng/mL.

Figure 10.8.1

Case 10.8  u  395


Etiological Diagnosis: Myocardial Ischemia
Sinus rhythm with normal P-wave morphology. However, the PR interval is relatively short and
cannot rule out a high junctional rhythm with positive P waves. (Figure 10.8.1). Extensive ST
segment elevation in anterior wall (V1–V6, I, and aVL), with mirror image in inferior wall (II, III, and
aVF); QRS with R wave from V1–V3. Lesion location (subtotal occlusion). Diagnosis: AMI with ST
elevation.

Management and Evolution


Patient underwent chemical thrombolysis with streptokinase without reperfusion criteria and was
referred to a rescue angioplasty. The coronary angiogram showed 95% lesion in the proximal third
of left anterior descending artery and left coronary trunk, with suggestion of ulceration in the
medial third. Patient evolved to a stable condition, in Killip class 1.

Postcatheterization Exams

Figure 10.8.2

Sinus rhythm, anterior MI with residue of ST segment elevation and diffuse alteration of
ventricular repolarization (Figure 10.8.2).

Echocardiography
Septum 13 mm; posterior wall 12 mm; LVDD 52 cm; left atrium 41 cm; ejection fraction 0.34—mild
mitral insufficiency; akynesia in mid-apical, medial, and basal anteroseptal wall of the left ventricle.

Discussion
The lesion blockade consists in a distortion of the terminal portion of the QRS complex in two or
more consecutive leads, rise of the J point above the middle portion of the R wave, or disappearance
of the S wave in leads with RS configuration. This is due to changes of conduction velocity in the

396  u  Section 10: Ischemia and Infarction


Purkinje fibers caused by severe ischemia. These fibers are less sensitive to ischemia than the
contractile cells. Lesion blockade is associated with increased in-hospital and 1-year mortality and it
predicts ventricular dysfunction. In a study with 112 patients of the Heart Institute of the University
of São Paulo Medical School hospital who underwent thrombolytic therapy with hemodynamic
study, there were 16 events among the 42 patients who had lesion blockade (higher mortality,
complex ventricular arrhythmias, and evolution to congestive heart failure), while among the 70
patients who did not present with that abnormality on ECG, there were only five events.
The ST-segment and T-wave abnormalities caused by an acute myocardial infarction are
significant for risk stratification for new events and prognostic evaluation. In 1995, Sclarovsky1
analyzed the predictive value of the ECG in acute Q-wave myocardial infarctions and the
recurrence of ischemia; they suggested that patients with postinfarction ST segment and inverted
T-wave resolution had a smaller infarction area, although with a greater risk for recurrence of
ischemia, when compared to patients with ST segment elevation and positive T wave. In 1997,
Kusniec et al.2 demonstrated that patients with postinfarction isoelectric ST segment and inverted
T waves had greater incidence of patency of the culprit vessel (TIMI 3 flow) on angiography, in
addition to a better ejection fraction, when compared to other ECG patterns.

Importance of the Diagnosis


The electrocardiographic recognition of a lesion location is associated with worse clinical evolution.
Identification of the ECG pattern and LV dysfunction during follow-up of such patients is important
to help determine indication for an implantable cardioverter-defibrillator (ICE) as primary
prevention in the management of patients with left ventricular ejection fraction (LVEF) below 30%,
in order to prevent sudden cardiac death.

References
1. Sclarovsky S. Electrocardiography of Acute Myocardial Ischemic Syndromes. London, U.K.: Martin Dunitz; 1999:95.
2. Kusniec J, Solodky A, Strasberg B, et al. The relationship between the electrocardiographic pattern with TIMI flow
class and ejection fraction in patients with a first acute anterior wall myocardial infarction. Eur. Heart J.
1997;18(3):420–425.

Case 10.8  u  397


CASE
10.9
Andrés Ricardo Pérez-Riera, MD, PhD
Luiz Carlos de Abreu, PhD

Patient History
A 47-year-old Caucasian male was admitted to the emergency department with 40 minutes of
retrosternal pain without radiation accompanied by profuse sweating and emesis.

Physical Examination
Clinically unstable patient with pale, cold skin; thready pulse, heart rate 85 bpm; and unobtainable
blood pressure.
Central venous catheter was placed, supplemental oxygen therapy started and ECG monitoring
initiated. The admission ECG is illustrated in Figure 10.9.1.

Figure 10.9.1  Electrocardiographic diagnosis. Sinus rhythm with premature atrial complexes, heart rate 83 bpm, prolonged QRS duration
(160 ms), qR pattern V1–V3: complete right bundle branch block (RBBB) associated with upward, convex ST segment elevation followed by
negative T wave, indicative of acute anterior STEMI. R/S pattern from V4–V6 with a wide final S wave.

398  u  Section 10: Ischemia and Infarction


Diagnosis
Acute ST segment elevation myocardial infarction (STEMI) due total occlusion of the left main
coronary artery (LMCA) without collateral circulation complicated with cardiogenic shock and
intraventricular conduction disturbance.

Management
The patient was immediately transferred to the cardiac catheterization laboratory where
angiography revealed total occlusion of the left main coronary artery (LMCA) with thrombus
(Figure 10.9.2A). After administrating abciximab and following aspiration of the coronary
thrombus, a stent was implanted resulting in TIMI 2 flow (Figure 10.9.2B). After the procedure, the
hemodynamic status immediately stabilized.

Figure 10.9.2  A. Angiography at admission showing absence of coronary tree due to total occlusion of the LMCA without collateral
circulation. B. Angiography performed immediately after abciximab and thrombus aspiration followed by stent placement in the LMCA.
Coronary tree was totally re-established.

One week later, coronary angiography confirmed full patency of the LMCA.
The patient was discharged on the fifteenth hospital day with left ventricular ejection fraction
by echo of 47%.

Discussion
An acute coronary syndrome (ACS) due to total LMCA occlusion is a catastrophic event associated
with poor prognosis and early high likelihood of very early mortality. Most of these patients die before
reaching the hospital. There is conflicting literature regarding the electrocardiographic features of
this unusual scenario. Differing ECG patterns have been described in several small series.
Patients with total LMCA occlusion who develop collateral circulation have more favorable left
ventricular function, although the collateral circulation is usually insufficient to prevent angina pain.1
In cases of acute total occlusion of the LMCA, the ECG is characterized by the dominant
presence of ST segment elevation (STSE). The few patients with ACS with a total LMCA without
STSE have adequate collateral circulation.1,2–4 Early recognition and immediate revascularization,
preferably by primary percutaneous coronary intervention, is the recommended management
strategy for this high risk group of patients.5–7 Fiol et al., 8 in a small series of cases, showed that the

Case 10.9  u  399


most typical STEMI pattern of total occlusion of the LMCA without collateral circulation
manifests as:
• STSE in the precordial leads from V2 to V6 and in aVL, with ST segment depression in the
inferior leads similar to the STEMI pattern of left anterior descending occlusion, proximal to the
first septal perforator branch and the first diagonal branch.
• Absence of STSE in aVR and V1. The ST injury vector is pointed to aVL because of circumflex
artery involvement that results in ST depression in these leads, decreasing STSE (Figure 10.9.3A).
• Intraventricular conduction disturbance, including complete RBBB frequently associated with
left fascicular blocks.

Figure 10.9.3A  Total obstruction of the LMCA without collateral circulation. Injury vector pointed to near –30°; consequently, there is
significant STSE in aVL, minimal in aVR. As a consequence of the superior and leftward direction of the injury vector in the frontal plane
(between –45º and –30º), the following ECG changes are seen: STSE in aVL, isoelectric ST segment in aVR (because the injury vector is
perpendicular to this lead), ST segment depression in the inferior leads (III > II).

In our experience, patients with this ECG pattern treated with reperfusion therapy, or in whom
it was not successful, all died in cardiogenic shock.
ACS due to partial occlusion of the LMCA (Figure 10.9.3B) generally results in global
circumferential ischemia characterized by diffuse ST segment depression in seven or more
precordial leads and STSE in aVR and V1 (ST elevation aVR ≥ V1).9–11 Moreover, ST deviation in
V6 ≥ ST deviation in V1 has greater sensitivity to predict partial LMCA occlusion than ST elevation
in aVR ≥ V1.12 Partial occlusion of the LMCA usually shows transient ST depression with negative
T waves from V4 to V5.1

400  u  Section 10: Ischemia and Infarction


Figure 10.9.3B  Partial obstruction of the LMCA. Injury vector pointed to aVR; consequently, STSE is registered in this lead.
Concomitantly, ST depression is seen in I, II, and aVL.

References
1. Slunga L, Eirksson P, Osterman G. Complete occlusion of the left main coronary artery: clinical and angiographic
observations in five cases. J. Intern. Med. 1989; 225(2):123–127. http://www.unboundmedicine.com/medline/
citation/2921593/Complete_occlusion_of_the_left_main_coronary_artery:_clinical_and_angiographic_
observations_in_five_cases
2. Nikus KC, Eskola MJ, Sclarovsky S. Electrocardiographic presentations of left main or severe triple vessel disease in
acute coronary syndromes—An overview. J. Electrocardiol. 2006;39(4 Suppl): S68–S72. doi: 10.1016/j.
jelectrocard.2006.05.023.
3. Nikus KC. Acute total occlusion of the left main coronary artery with emphasis on electrocardiographic
manifestations. Timely Top Med. Cardiovasc. Dis. 2007;11:E22.
4. Nikus KC. Electrocardiographic presentations of acute total occlusion of the left main coronary artery.
J. Electrocardiol. 2012;45(5):491–493. doi: 10.1016/j.jelectrocard.2012.06.014.
5. Udayakumaran K, Subban V, Pakshirajan B, et al. Primary percutaneous thrombus aspiration alone as definitive
intervention for left main coronary artery occlusion presenting as acute anterior wall ST elevation myocardial
infarction. Heart Lung Circ. 2014;23(2):166–170. doi: 10.1016/j.hlc.2013.07.018.
6. Kramer MC, Verouden NC, Li X, et al. Thrombus aspiration alone during primary percutaneous coronary
intervention as definitive treatment in acute ST-elevation myocardial infarction. Catheter Cardiovasc. Interv.
2012;79(6):860–867. doi: 10.1002/ccd.23214.
7. Russo JJ, Dzavík V, Cairns JA, et al. An international survey of clinical practice during primary percutaneous
coronary intervention for ST-elevation myocardial infarction with a focus on aspiration thrombectomy.
EuroIntervention. 2013;8(10):1143–1148. doi: 10.4244/EIJV8I10A177.
8. Fiol M, Carrillo A, Rodríguez A, et al. ECG changes of STEMI in patients with complete occlusion of the left main
trunk without collateral circulation: Differential diagnosis and clinical considerations. J. Electrocardiol.
2012;45(5):487–490. doi: 10.1016/j.jelectrocard.2012.05.001.

Case 10.9  u  401


9. Yamaji H, Iwasaki K, Kusachi S, et al. Prediction of acute left main coronary artery obstruction by 12-lead. J. Am.
Coll. Cardiol. 2001;38(5):1348–1354. doi:10.1016/S0735-1097(01)01563-7.
10. Kosuge M, Kimura K, Ishikawa T, et al. Predictors of left main or three-vessel disease in patients who have acute
coronary syndromes with non-ST-segment elevation. Am. J. Cardiol. 2005;95(11):1366–1369.
doi: 10.1016/j.amjcard.2005.01.085.
11. Bayés de Luna A, Fiol-Sala M, Eds. Electrocardiography in Ischemic Heart Disease. Oxford, U.K.: Blackwell/Futura.
2008:234.
12. Mahajan N, Hollander G, Thekkoott D, et al. Prediction of left main coronary artery obstruction by 12-lead
electrocardiography: ST segment deviation in lead V6 greater than or equal to ST segment deviation in lead V1.
Ann. Noninvasive Electrocardiol. 2006;11(2):102–112. doi: 10.1111/j.1542-474X.2006.00090.x.

402  u  Section 10: Ischemia and Infarction


CASE
Andrés Ricardo Pérez-Riera, MD, PhD 10.10
Patient History
A 78-year-old Caucasian female was admitted to the emergency department with typical clinical-
electrocardiographic pictures of ST-segment elevation acute coronary syndrome (STE-ACS). A
standard 12-lead ECG was performed at admission within the first 12 hours of chest pain (Figure
10.10.1). Immediately, the patient was submitted to primary percutaneous transluminal coronary
angioplasty (PTCA) with successful stent placement in proximal right coronary artery (RCA).
Additionally, the coronary angiography shows critical proximal obstruction of the left anterior
descending (LAD) artery before the first septal perforator (S1). The strategy was to perform an
elective repeat coronary angioplasty.
A few days later, the patient returned with a typical episode of effort induced precordial pain
(Figure 10.10.2).

Figure 10.10.1  Standard 12-lead ECG performed at admission within the first 12 hours of chest precordial pain onset.

ECG Diagnosis
Sinus tachycardia (heart rate 125 bpm), left axis deviation (QRS axis –30°: minimal degree of left
anterior fascicular block?), pathological QS waves in III and aVF followed by ST segment elevation
≥1 mm upwardly convex (inferior subepicardial injury). ST segment elevation in lead III greater
than in lead II is suggestive of RCA occlusion. Additionally, ST segment depression is

Case 10.10  u  403


observed in V2. These represent reciprocal ST changes (mirror image) in leads remote from the site
of an acute infarct anteroseptal wall from V1 to V4 (marked ST-segment depression in V2).

Conclusion
Sinus tachycardia and inferior acute myocardial infarction as a consequence of RCA occlusion.

Figure 10.10.2  Performed immediately after effort-induced episode of typical angina pectoris.

ECG Diagnosis
Sinus rhythm, heart rate 94 bpm, extreme QRS left axis deviation in the frontal plane (QRS axis
-70°), QRS duration 115 ms, qR pattern in I and aVL, rS pattern in inferior leads, SIII>SII, SIII>15
mm and rS pattern in left precordial leads V5 and V6: Rosenbaum’s type IV left anterior fascicular
block (LAFB). Additionally, inverted T wave in the left leads I, aVL, V5, and V6 are observed
(ischemic T waves?).
QRS duration < 120 ms (close to 115 ms). qR pattern in V2 lead, embryonic initial q wave in V2
followed by very tall R wave in V2 (R wave >15 mm): prominent anterior QRS forces (PAF), absence
of initial q wave in the left precordial leads: left septal fascicular block (LSFB).

Conclusion
Bifascicular (left fascicular) block: LAFB associated with LSFB.

404  u  Section 10: Ischemia and Infarction


Figure 10.10.3  Angiography before and after stent placement.

Figure 10.10.4  ECG before stent placement.

Case 10.10  u  405


An ECG was conducted on the third day after a successful PTCA of the LAD (Figure 10.10.5).

Figure 10.10.5  Performed on the third day after successful stent placement in the proximal LAD.

ECG Diagnosis
Extreme left axis QRS deviation in the frontal plane, QRS axis –65°, qR pattern on I and aVL, rS
pattern in inferior leads, SIII > SII, RS in left precordial leads: LAFB.
V2 prominent QRS anterior forces (LSFB) disappeared.
Negative T wave is observed in anterolateral wall suggestive of anterolateral ischemia.

Conclusion
LAFB associated with anterolateral ischemia. LSFB disappeared.

Discussion
This case report is irrefutably a typical transient or intermittent LSFB consequence of severe
proximal obstruction of LAD associated with LAFB: bifascicular left bundle block.
The sudden onset and intermittent prominent anterior forces (PAF) in the right and/or middle
precordial leads is indicative of critical proximal obstruction of the LAD secondary to left septal
block (LSFB).
The LSF is irrigated exclusively by the septal perforating branches of the LAD. Consequently,
critical lesions of the LAD before the first septal perforating branch (S1) constitute the main cause
of LSFB in developed countries, and it is a major determinant of high R-wave amplitude in right or
intermediate precordial leads during acute myocardial ischemia.1,2 The LSB is occasionally exercise-
induced, transient, or intermittent, and sometimes it causes giant R waves.
Intermittent LSFB secondary to a critical lesion of LAD was described recently during exercise
testing.4 The appearance of LSFB in critical LAD lesions favors a proximal lesion of LAD and
therefore, a worse prognosis.

406  u  Section 10: Ischemia and Infarction


References
1. Pérez Riera AR, Ferreira C, Ferreira Filho C, et al. Electrovectorcardiographic diagnosis of left septal fascicular
block: Anatomic and clinical considerations. Ann. Noninvasive Electrocardiol. 2011;16(2):196–207.
2. Riera AR, Kaiser E, Levine P, et al. Kearns-Sayre syndrome: Electro-vectorcardiographic evolution for left septal
fascicular block of the his bundle. J. Electrocardiol. 2008;41(6):675–678.
3. Riera AR, Ferreira C, Ferreira Filho C, et al. Wellens syndrome associated with prominent anterior QRS forces: An
expression of left septal fascicular block? J. Electrocardiol. 2008;41(6):671–674.
4. Uchida AH, Moffa PJ, Riera AR, et al. Exercise-induced left septal fascicular block: An expression of severe
myocardial ischemia. Indian Pacing Electrophysiol. J. 2006;6(2):135–138.

Case 10.10  u  407


CASE
10.11 Philip Podrid, MD

Patient History
A 57-year-old male presents to the emergency department after several hours of substernal chest
pressure that radiated to his shoulders and was associated with nausea and diaphoresis. He initially
thought that this was his typical GERD. However, the symptoms persisted despite antacids and
proton-pump inhibitors, and the chest discomfort was becoming worse. Upon admission, an ECG
was obtained (Figure 10.11.1). Shortly thereafter telemetry showed a change in the rhythm and QRS
complexes and a second ECG was obtained (Figure 10.11.2).

Figure 10.11.1

408  u  Section 10: Ischemia and Infarction


Figure 10.11.2

Diagnosis
Figure 10.11.1 shows sinus versus atrial tachycardia, acute inferior and anterolateral ST elevation
myocardial infarction (STEMI), early transition.
Figure 10.11.2 shows sinus tachycardia, complete (third-degree) AV block, escape ventricular
rhythm, acute inferior, and anterolateral STEMI.

Questions
1. What abnormality is present in Figure 10.11.1?
2. What is the etiology for the QRS complexes in Figure 10.11.2?
3. What accounts for the ST changes seen in Figure 10.11.2?

In Figure 10.11.1, there is a regular rhythm at a rate of 130 bpm. There are P waves seen in lead
V1 and V2 only. It is therefore uncertain if this is a sinus or atrial tachycardia. The PR interval (0.20
seconds) is constant. The QRS complex width is normal (0.08 seconds) and there is a normal
morphology and axis between 0° and +90° (positive QRS complex in leads I and aVF). There is a tall
R wave in lead V2 (→) which is termed early transition or counterclockwise rotation of the electrical
axis in the horizontal plane. This is established by imagining the heart as if viewed from under the
diaphragm. When there is counterclockwise rotation, left ventricular forces are shifted anteriorly
and are seen prominently in the right precordial leads (particularly V2). The QT/QTc intervals are
normal (260/380 ms). There is J point and ST segment elevation in leads II, III, and aVF as well as
leads V3–V6 (↑). This is an acute inferior and anterolateral wall ST elevation myocardial infarction
(STEMI). There are ST segment depressions in leads I and aVL (↓) which are reciprocal changes

Case 10.11  u  409


associated with the inferior wall myocardial infarction. There are Q waves in these leads (^)
suggesting that the acute MI is several hours old and there is now infarcted myocardium.
In Figure 10.11.2, there is a regular rhythm at a rate of 85 bpm. There are P waves seen
(especially in lead II) (+). When seen the P intervals are constant at a rate of 130 bpm. The P waves
are positive in leads II, aVF, and V4 –V6. Hence there is a sinus tachycardia. The PR intervals are
variable and there is no relationship between atrial and ventricular complexes; this is AV
dissociation. As the atrial rate is faster than the rate of the QRS complexes, this is complete (third-
degree) AV block. The QRS complexes are wide (0.14 seconds) and they have a morphology
resembling a LBBB with a QS morphology in lead V1 (←) and a broad R wave in leads I, V5, and V6
(→). However, there is marked variability in the QRS complex morphology (↓), which is consistent
with a ventricular complex and not supraventricular. Ventricular tachycardia results from a focus
within the ventricular myocardium and the impulse activating the ventricles does not travel along
the normal His-Purkinje system, but rather there is direct myocardial activation. There may be
changes in the direction of myocardial activation, accounting for the changes in QRS complex
morphology. With any supraventricular rhythm (sinus, atrial, or AV nodal), conduction to the
ventricles is through a fixed pathway (AV node or His-Purkinje system) and hence all the QRS
complexes are uniform in morphology. J point and ST segment elevation (↑) is seen in leads II, III,
aVF, and V3–V6. There is also ST segment depression in leads I and aVL (^). The ST segment
changes are in the same leads and are identical to the ST segment changes seen in Figure 10.11.1. In
addition there is variability in the amplitude of the ST segment elevation, another feature consistent
with a ventricular complex. Therefore this is a sinus tachycardia with complete (third-degree) AV
block and an escape ventricular rhythm. An acute ST elevation myocardial infarction can still be
diagnosed even though the QRS complexes are ventricular. In general, abnormalities of the
ventricular myocardium cannot be reliably diagnosed with a ventricular complex, a LBBB, or a
ventricular paced rhythm as there is direct myocardial activation and the impulse does not conduct
through the normal His-Purkinje system. However, it has been reported that an acute STEMI can
be diagnosed in the presence of a LBBB or right ventricular paced rhythm if there are certain
abnormalities present:
1. ST segment elevation of 1 mm or more that is in the same direction (concordant) as the QRS
complex in any lead.
2. ST segment depression of 1 mm or more in any lead from V1 to V3.
3. ST segment elevation of 5 mm or more that is discordant with the QRS complex (i.e., associated
with a QS or rS complex).
Although these criteria have been reported for a LBBB or a right ventricular paced rhythm, it is
probable that they can be applied to other situations in which there is direct myocardial activation,
i.e., a ventricular complex. As seen in this ECG, there is >1 mm ST-segment elevation in leads II, III,
aVF, and V3–V6 (↑), indicating an inferior wall and anterolateral wall STEMI. Importantly, the
ST-segment changes are the same that are present with the supraventricular rhythm as seen in
Figure 10.11.1.

410  u  Section 10: Ischemia and Infarction


CASE
10.12
Luca Santini, MD, PhD
Massimo Santini, MD

Patient History
A 52-year-old male patient, smoker, hypercholesterolemic, with family history of coronary artery
disease and no previous cardiac events. Patient arrived in our emergency department for typical
ischemic chest pain for 45 minutes.

Figure 10.12.1

Question
Is it possible to diagnose the site of ischemic attack by ECG analysis?

Discussion, Interpretation, and Answer


The ECG reveals a typical anterior myocardial infarction (STEMI) with extension as can be seen by
the presence of prominent ST segment elevation in all the precordial leads. Patient underwent
primary percutaneous coronary intervention, which demonstrated an acute occlusion of the left
anterior descending artery and was immediately treated by direct stenting.

Case 10.12  u  411


SECTION 11
Electrolyte Disturbances, Pharmacological and
Recreational Agents

Lily Chen, MD
Benjamin Stripe, MD
Jonathan Bui, MD CASE
Femi Philip, MD
Ezra A. Amsterdam, MD 11.1
Patient History
A 73-year-old female was admitted to the emergency department for diabetic ketoacidosis (DKA):
glucose 580 mg/dL, CO2, 8 mEq/L, anion gap 29, potassium 7.6 mEq/L, creatinine 3.24 mg/dL
(baseline 1.28). Medications included lisinopril. Multiple factors contributed to the patient’s
hyperkalemia including DKA, renal insufficiency, and ACE inhibitor.
Figure 11.1.1 shows admission ECG; Figure 11.1.2 is a prior normal ECG obtained 2 weeks
before admission. Figure 11.1.1 demonstrates peaked T waves in leads II, V5, and V6 (arrowheads),
marked ST elevation in V1 and V2 (stars), and Q waves in V1, V2 (arrows). Troponin I peaked at
1.36 ng/ml (ref 0.04). The patient’s metabolic abnormalities responded to standard therapy and her
ECG changes resolved, as shown in subsequent ECG (Figure 11.1.3).

Figure 11.1.1

ECG Masters’ Collection: Favorite ECGs from Master Teachers Around the World © 2017 Mohammad Shenasa, Mark E. Josephson,
N. A. Mark Estes III, Ezra A. Amsterdam, Melvin Scheinman. Cardiotext Publishing, ISBN: 978-1-942909-08-8.   413
Figure 11.1.2

Figure 11.1.3

414  u  Section 11: Electrolyte Disturbances, Pharmacological and Recreational Agents


Question
Are there any other abnormalities on the ECG?

Answer
There are several abnormalities in addition to the peaked T waves, a classic sign of hyperkalemia,
including ST elevation, which has been recognized as a finding with hyperkalemia since the mid-
20th century.1 Additional abnormalities include: Absent P waves, junctional escape rhythm
(ventricular rate 55/min), increased QRS duration (~110 ms) and Q waves (arrows) in leads V1 and
V2 (q in V3 is very diminutive). There are also tiny excursions before the QRS complexes in the
inferior leads, which may be artifact or retrograde P waves originating from the junctional rhythm.
Peaked T waves, flattened P waves, bradycardia, increased QRS duration, and sinus node arrest are
well-established effects of hyperkalemia. ST elevation in the precordial leads with hyperkalemia was
first reported over a half century ago and should be included in the differential diagnosis of acute
ST elevation myocardial infarction. None of the foregoing findings were present shortly before her
admission (Figure 11.1.2) or in her ECG after reversal of her metabolic abnormalities (Figure 11.1.3).

Question
Have transient Q waves been reported to occur with hyperkalemia?

Answer
Yes. Reversible Q waves have been reported previously in association with hyperkalemia and have
been attributed to a ventricular conduction abnormality induced by elevated potassium.2 In
addition, severely deranged metabolic conditions, such as DKA, have been associated with elevated
cardiac troponin.

References
1. Wang K, Asinger RW, Marriott HJ. ST-segment elevation in conditions other than acute myocardial infarction. N.
Engl. J. Med. 2003;349:2128–2135.
2. Arnsdorf M. Electrocardiogram in hyperkalemia. Arch. Intern. Med. 1976;136;1161–1163.

Case 11.1  u  415


CASE
11.2 Haran Burri, MD

Patient History
An 80-year-old female patient was admitted due to malaise. She had previously been implanted
with a single-chamber pacemaker for atrial fibrillation and bradycardia. The blood tests revealed
renal failure with hyperkalemia at 8.2 mmol/L. Her ECG recorded upon admission (Figure 11.2.1) is
shown.

Figure 11.2.1  ECG recorded upon admission.

Question
What does this ECG show?

Discussion
The tracing shows ventricular tachycardia (note the very broad QRS complexes due to slowed
intraventricular conduction secondary to the hyperkalemia). The ventricular rhythm is undersensed
by the pacemaker, with delivery of asynchronous pacing at the programmed baseline rate of 50
bpm.

416  u  Section 11: Electrolyte Disturbances, Pharmacological and Recreational Agents


The undersensing by the pacemaker may be explained by two factors:
1. Perpendicular orientation of the pacemaker lead dipole with respect to the vector of the QRS
complexes, leading to low electrogram amplitude.
2. Hyperkalemia leading to changes in the electrogram signal (with a lower slew rate). The filters of
the pacemaker attenuate low-frequency components (e.g., due to repolarization), and may
therefore have reduced the amplitude of the resulting electrograms.
It is unclear if there is noncapture, as all the spikes (apart possibly from the second one) fall in the
ventricular myocardial refractory period.
The patient was immediately administered calcium gluconate, and the hyperkalemia was
corrected, with interruption of the ventricular tachycardia and resumption of a paced rhythm.

Case 11.2  u  417


CASE
11.3 Johan Saenen, MD, PhD
Hein Heidbuchel, MD, PhD

Patient History
A 44-year-old female was resuscitated on the hematologic isolation ward, where she was
hospitalised for agranulocytosis secondary to thiamazol therapy for hyperthyroidism. She was
treated with claritromycine, itraconazol, propranolol among other drugs. Her serum K+ was
3.1 mEq/L and Ca++ was 8.1 mEq/L.

Figure 11.3.1

Question
Do you need signal averaging to see what happens on her ECG?

Discussion
The ECG shows a sinus rhythm at about 80 bpm, with normal QRS complexes but with extremely
prolonged QT intervals of 630 ms (QTc 840 ms), which led to torsades de pointes polymorphic
ventricular tachycardia and ventricular fibrillation, from which the patient was resuscitated.
Although she had normal repolarization on prehospital ECGs, the extreme QT prolongation could
be attributed to a multitude of factors that prolong the QT interval, as mentioned in the
introduction. Apart from the prolonged QT interval, the ECG shows prominent alternation of the
morphology of the T waves, which could be called “macro-voltage T-wave alternans” in this case.
Instability of repolarization is a hallmark of arrhythmogenic risk in many conditions (not only

418  u  Section 11: Electrolyte Disturbances, Pharmacological and Recreational Agents


long-QT syndromes, but also post-infarction or in patients with heart failure), and is for instance
looked for in evaluation of “micro-voltage T-wave alternans” using signal-averaging techniques
(usually during light exercise). This ECG under extreme conditions exemplifies for the naked eye
the electrical instability that comes with T-wave alternans.

Reference
1. Saenen JB, Vrints CJ. Molecular aspects of the congenital and acquired long QT syndrome: Clinical implications.
J. Mol. Cell. Cardiol. 2008;44(4):633–646.

Case 11.3  u  419


CASE
11.4 Mohammad-Ali Jazayeri, MD
Mohammad-Reza Jazayeri, MD

History
A 64-year-old male with a past medical history significant for cerebrovascular accident with
residual right hemiparesis and dysphagia was brought in by ambulance to the emergency
department following a cardiac arrest at a restaurant after being left unattended by his family for a
few minutes. His presenting ECG on arrival is shown in Figure 11.4.1 and initial cardiac troponin I
(cTnI) measurement was undetectable (<0.010 ng/mL).

Figure 11.4.1  ECG recorded on presentation.

Question
Which of the following can be excluded based on the ECG pattern observed?
1. Ventricular tachycardia.
2. Supraventricular tachycardia with aberrant conduction.
3. Wide QRS-complex tachycardia due to metabolic abnormality.
4. None of the above.

Comment
The correct answer is 4, none of the above. The patient was transported to the emergency
department by ambulance following out-of-hospital pulseless electrical activity cardiac arrest at a
restaurant where he was having a meal with his family. He received 40 minutes of cardiopulmonary
resuscitation on the scene and en route, as well as an additional 25 minutes of resuscitation on

420  u  Section 11: Electrolyte Disturbances, Pharmacological and Recreational Agents


arrival to the emergency department. During the latter period he underwent endotracheal
intubation, at which point it was noted he had a large piece of meat between his vocal folds. His
ECG was concerning for ongoing ischemia with what appears at first glance to be wide complex
tachycardia in a right bundle branch block configuration along with diffuse ST-segment depression.
There is notching that suggests possible p waves most prominent in precordial lead V1 and in
inferior lead II. In addition, the narrow upstroke of the QRS complex observed in lateral leads V4 –V6
argues against a ventricular origin. Laboratory values at the time of the ECG shown in Figure 11.4.1
are recorded in Table 11.1.1.
Table 11.4.1  Initial Laboratory Measurements at the Time of Presentation. Reference Ranges Are Presented in Brackets.
cTnI <0.010 ng/mL [≤0.030 ng/mL] Venous Blood Gas: Basic Metabolic Panel:
Lactate >14.8 mmol/L [0.7–2.1 mmol/L) pH 6.94 [7.32–7.42] Sodium 145 mEq/L [135–148 mEq/L]
Hemoglobin 12.8 g/dL [13.1–17.5 g/dL] pCO2 96 [41–51 mm] Potassium 4.2 mEq/L [3.5–5.3 mEq/L]
pO2 52 [25–40 mm] CO2 15 mEq/L [22–30 mEq/L]
CO2 20 [24–28 mEq/L] Creatinine 1.51 mg/dL [0.70–1.25 mg/dL]
O2 Sat 60% Ionized calcium 4.46 [4.40–5.20 mg/dL]

While still in the emergency department, his subsequent ECG was recorded as shown in Figure
11.4.2. Prior to this recording his blood gas was reassessed following the initiation of mechanical
ventilation and was remarkable for a pH of 6.98 from 6.94. Due to continued concern for an
ischemic process with an ECG 30 minutes later (Figure 11.4.2) demonstrating ST-segment elevation
in the anterior precordial leads (V1 and V2) and diffuse ST-segment depression elsewhere, including
inferior (II, III, aVF) and lateral (V4 –V6) leads, a STAT echocardiogram was ordered. In the interim,
a second cTnI level was drawn, which was markedly elevated at 2.042 ng/mL. The echocardiogram
demonstrated hyperdynamic systolic performance, concentric left ventricular hypertrophy, no
segmental wall motion abnormality or valvular regurgitation, and normal right ventricular systolic
performance.

Figure 11.4.2  ECG recorded 30 minutes after presentation. Concerning for ST-segment elevation in the anterior precordial leads (V1 and
V2) and diffuse ST-segment depression elsewhere, including inferior (II, III, aVF) and lateral (V4–V6) leads. P waves are more evident in lead
III as compared to the initial ECG.

Case 11.4  u  421


A noncontrast computed tomography scan of the head following intubation demonstrated findings
consistent with anoxic brain injury. Given the clinical context and the echocardiographic findings
showing no definitive evidence of new focal or global dyskinesis, the patient was not taken to the
cardiac catheterization laboratory and was instead admitted to the medical intensive care unit for
continued mechanical ventilation and supportive care. The next ECG 2.5 hours after presentation
(Figure 11.4.3) was recorded with a pH of 7.3, demonstrating resolution of the wide complexes
previously described. A final ECG recorded the following morning (21 hours after presentation)
showed further normalization (Figure 11.4.4). The overall trend of cTnI measurements during this
span is shown in Table 11.4.2. The patient was supported overnight in the medical ICU, but
ultimately, given his poor prognosis, he was transitioned to comfort care.

Figure 11.4.3  ECG recorded 2.5 hours after presentation demonstrating resolution of the wide complexes noted earlier, as well as
clearer identification of atrial and ventricular activities in what appears to be an underlying sinus tachycardia.

Figure 11.4.4  ECG recorded 21 hours after presentation showing further normalization of the ECG in the setting of normal pH.
Nonspecific ST-T changes persist in the anterior precordial leads (V1–V3). The QT interval is prolonged at approximately 580 ms.

422  u  Section 11: Electrolyte Disturbances, Pharmacological and Recreational Agents


Table 11.4.2  Serial Cardiac Troponin I Measurements Recorded From the Time of Presentation
Presentation (P) P + 2 hours P + 6 hours P + 9 hours P + 13 hours
<0.010 ng/mL 2.042 ng/mL 15.847 ng/mL 39.376 ng/mL 69.944 ng/mL

Discussion
This case illustrates an interesting electrocardiographic manifestation of profound acidosis following
cardiac arrest, namely the morphologic appearance of wide complex tachycardia on ECG, which
subsequently evolved to show more classic ischemic features and ultimately resolved with correction of
primary respiratory acidosis secondary to airway obstruction. Given the concerning presentation with
out-of-hospital arrest requiring prolonged resuscitation, ECG suggesting wide complex tachycardia, and
an initially undetectable cTnI level, which subsequently became significantly elevated, there was great
concern for an acute coronary syndrome. Transthoracic echocardiography was reassuring in that there
was no evidence of focal wall motion abnormality or global hypokinesis to suggest a role for urgent
angiography, and despite the marked myonecrosis, the patient was felt to have suffered a type 2
myocardial infarction in the setting of marked acidosis, hypoxemia, and central nervous system injury.1
The electrocardiographic findings on the initial ECG are quite striking with an almost
sinusoidal waveform in the setting of respiratory and metabolic acidosis with presenting pH of 6.94
that subsequently resolved with correction of acidemia. No other major metabolic disturbance was
identified to explain the findings. There were no offending medications. Oxygenation was already
adequate at the time of the initial ECG (Figure 11.4.1), suggesting against hypoxemia as the primary
mechanism. When confronted with a sinusoidal pattern ECG, severe hyperkalemia is a major
concern, as illustrated in a case by Pluijmen et al.2 Other differential considerations include drug
toxicity, particularly sodium channel blocker toxicity,3 and, less likely, tricyclic antidepressant
overdose. Earlier investigations of the effects of acidemia on the human electrocardiogram, as well
as in isolated rat hearts, suggest prominent T waves, QT interval prolongation, and PR interval
prolongation as potential manifestations,4–7 but review of the English-language literature is
unrevealing for other cases of primary respiratory and/or metabolic acidosis presenting with similar
findings. Despite the uncertainty surrounding the exact mechanism underlying the presented ECG
abnormalities, similar cases should give pause to readers when considering activation of the cardiac
catheterization laboratory in patients with serious metabolic disturbances.

References
1. Thygesen K, Alpert JS, Jaffe AS, et al. The Writing Group on behalf of the Joint ESC/ACCF/AHA/WHF Task Force
for the Universal Definition of Myocardial Infarction. ESC/ACCF/AHA/WHF Expert Consensus Document: Third
universal definition of myocardial infarction. Circulation. 2012;126:2020–2035.
2. Pluijmen MJHM, Hersbach FMRJ. Sine-wave pattern arrhythmia and sudden paralysis that result from severe
hyperkalemia. Circulation. 2007;116:e2–e4.
3. Auzinger GM, Scheinkestel CD. Successful extracorporeal life support in a case of severe flecainide intoxication.
Crit. Care Med. 2001;29:887–890.
4. Aberra A, Komukai K, Howarth FC, et al. The effect of acidosis on the ECG of the rat heart. Exp. Physiol.
2001;86(1):27–31.
5. Barker PS, Shrader EL, Ronzoni E. The effects of alkalosis and of acidosis upon the human electrocardiogram. Am.
Heart J. 1939;17(2):169–186.
6. Martell R, Buchanan N, Cane R. The effect of blood pH on the electrocardiogram. Crit. Care Med. 1979;7(1):24–26.
7. Nadler CS, Bellet S, Lanning M. Influence of the serum potassium and other electrolytes on the electrocardiogram
in diabetic acidosis. Am. J. Med. 1948;5(6):838–848.

Case 11.4  u  423


CASE
11.5 Luca Santini, MD, PhD
Massimo Santini, MD

Patient History
A 41-year-old male with a parathyroid adenoma who presented to the emergency department in a
critically unstable condition with a serum calcium of 6.1 mmol/L. He suffered a ventricular
fibrillation (VF) arrest not long after this ECG was taken.

Figure 11.5.1

Question
What are the ECG features of a severe hypercalcemia?

Discussion, Interpretation, and Answer


The main ECG abnormality seen with hypercalcemia is a marked shortening of the QT interval.
Moreover, ventricular irritability and VF arrest may be provoked by extreme hypercalcemia. In this
ECG, bizarre-looking QRS complexes with a short QT interval are visible. Moreover, Osborn waves
(J waves; a notching of the terminal QRS) are seen in lead V1 (arrow).

424  u  Section 11: Electrolyte Disturbances, Pharmacological and Recreational Agents


SECTION 12
Paced Rhythms and Device Troubleshooting

CASE
Amit Noheria, MBBS, SM
Samuel J. Asirvatham, MD 12.1
Patient History
A 43-year-old male with mild global left ventricular systolic dysfunction and sinus node
dysfunction underwent dual-chamber pacemaker implantation. An ECG (Figure 12.1.1) was
obtained at the time of pacemaker interrogation next day. ECG prior to pacemaker implantation is
shown in Figure 12.1.2.

Figure 12.1.1

ECG Masters’ Collection: Favorite ECGs from Master Teachers Around the World © 2017 Mohammad Shenasa, Mark E. Josephson,
N. A. Mark Estes III, Ezra A. Amsterdam, Melvin Scheinman. Cardiotext Publishing, ISBN: 978-1-942909-08-8.   425
Figure 12.1.2

Questions
1. What is the rhythm shown in Figure 12.1.1?
2. What can you say about the pacemaker based on the QRS morphology?

Discussion, Interpretation, and Answers


This is a ventricular paced rhythm at 90 bpm. The dissociated sinus rhythm at 60 bpm can be seen
marching through the tracing (Figure 12.1.3, black arrows). No obvious pacing artifact is noticeable.

Figure 12.1.3

426  u  Section 12: Paced Rhythms and Device Troubleshooting


The QRS morphology is similar to ventricular preexcitation during sinus rhythm in patients with
an anteroseptal accessory pathway.1 The red arrow (Figure 12.1.3) points to the onset of the QRS
complex (“delta wave”) followed by the blue arrow showing a change to a sharper component. This
suggests activation of the ventricular myocardium (red arrow) preceding and subsequently fusing
with ventricular excitation through the His-Purkinje system (blue arrow). The pacing stimulus, in
this case at the site of proximal right bundle branch, recruits the local ventricular myocardium
immediately and inscribes the delta wave. It also stimulates the proximal right bundle from where
there is a finite delay for excitation to propagate through the insulated conduction system before
exiting from the distal Purkinje network.
The delta wave is negative in lead V1 (left bundle branch block morphology) suggesting right
ventricular or septal site of initial activation. Positive initial activation in leads V4−V6 suggests basal
rather than an apical site. The overall QRS is relatively narrow with the frontal plane axis
horizontally to the left (positive aVL, negative aVR). This localizes to the septal tricuspid valve
region in anatomic proximity to the His-bundle/proximal right bundle branch. Preexcitation over
an accessory pathway inserting to the ventricle at the septal atrioventricular annulus results in
quick transition of delta wave from negative to positive from lead V1 to V2. In this case, the delta
wave is negative in lead V2 (red arrow) and becomes positive only by lead V4, due to stimulation
further away from the annulus and approaching the septal right ventricular outflow tract
myocardium. Chest radiographs showing the right bundle pacing electrode (arrows) are shown in
Figure 12.1.4 (asterisk—electrode in the right atrial appendage).

Figure 12.1.4

A differential diagnosis for Figures 12.1.1 and 12.1.3 is an accelerated idioventricular rhythm from the
interventricular septum. Activation from a septal site in a structurally normal heart also results in a
relatively narrow QRS due to centrifugal simultaneous depolarization of both ventricles; however, the
QRS should not be notched (leads V1 and V2). Notching of QRS (Figure 12.1.3, blue arrows) reflects an
abrupt change in the direction of the depolarizing wavefront on account of the activation from the
conduction system catching up and fusing with the delta wave. A similar phenomenon resulting in
abrupt changes in direction of the wavefront and notching in QRS could also occur on account of

Case 12.1  u  427


regions of conduction block (e.g., myocardial infarcts or surgical suture lines). Also notable in this
ECG is the slight variation in the degree of contribution of the delta wave (leads aVL and V4). This is
probably due to small changes in the timing and field of capture of the insulated conduction system
relative to the local myocardium with respirophasic movement of the pacing electrode.
There has been a resurgence of interest in pacing the proximal conduction system to preserve left
ventricular electrical synchrony as opposed to the adverse hemodynamic effects from dyssynchrony
related to right ventricular apical pacing.2 Figure 12.1.5 is an anatomic section through the
membranous septum with illustration of the course of the proximal ventricular conduction system.
Though referred to as “His-bundle pacing,” placement of an active fixation electrode at the bundle of
His is technically difficult due to its short course through the thin membranous septum before
splitting into the bundle branches. Further, the fixation screw of a pacing electrode deployed at the
true anatomic bundle of His could perforate into the left ventricle, get dislodged, or have problems
with oversensing/pacing the atrium. Often the proximal right bundle branch is the actual site of
pacing. Pacing the conduction system from this site with adequate output can result in a narrow QRS
complex even in cases with underlying left bundle branch block.3

Figure 12.1.5

428  u  Section 12: Paced Rhythms and Device Troubleshooting


References
1. Arruda MS, McClelland JH, Wang X, et al. Development and validation of an ECG algorithm for identifying
accessory pathway ablation site in Wolff-Parkinson-White syndrome. J. Cardiovasc. Electrophysiol. 1998;9(1):2–12.
2. Sharma PS, Dandamudi G, Naperkowski A, et al. Permanent His-bundle pacing is feasible, safe, and superior to
right ventricular pacing in routine clinical practice. Heart Rhythm. 2015;12(2):305–312.
3. Lustgarten DL, Crespo EM, Arkhipova-Jenkins I, et al. His-bundle pacing versus biventricular pacing in cardiac
resynchronization therapy patients: A crossover design comparison. Heart Rhythm. 2015;12(7):1548–1557.

Case 12.1  u  429


CASE
12.2 Haran Burri, MD

Patient History
An 82-year-old female implanted with a pacemaker 8 years prior for AV block presented to the
emergency department due to dizziness and dyspnea. The ECG recorded upon her admission
(Figure 12.2.1) is shown.

Figure 12.2.1  ECG recorded upon the patient’s admission.

Question
What is the rhythm?

Discussion
The issue with this ECG recording is its scale, which was accidentally set to 4 cm/mV (see the
calibration mark on the bottom left of the tracing). The same ECG is shown reduced to 25% in the
vertical axis (corresponding to a scale of 1 cm/mV, see Figure 12.2.2), showing sinus rhythm with
complete AV block, and a ventricular paced rhythm alternating with a junctional rhythm. The

430  u  Section 12: Paced Rhythms and Device Troubleshooting


patient had not been seen for a pacemaker follow-up for a year, and the battery had reached end of
life, with delivery of erratic pacing spikes by the device. This example shows the importance of
systematic ECG interpretation, which includes analyzing calibration.

Figure 12.2.2  ECG reduced to 25% in the vertical axis and corresponding to a usual scale of 1 cm/mV, with recognizable P waves
(asterisks), complete AV block, paced QRS complexes (blue arrows), and junctional beats (green arrows).

Case 12.2  u  431


CASE Santosh K. Padala, MD

12.3 Gautham Kalahasty, MD


Kenneth A. Ellenbogen, MD

Patient History
A 68-year-old male with history of coronary artery disease and ventricular tachycardia underwent
implantation of dual-chamber implantable-cardioverter defibrillator (Guidant, Ventak Prizm 2DR)
5 years ago. He was admitted to the hospital with symptoms of nausea, palpitations, presyncope,
and low blood pressure. The rhythm strip while on telemetry is shown in the Figure 12.3.1. Device
interrogation was done which showed the following parameters:

Mode = DDDR AP = 41%, VP = 26%


Lower Rate Limit = 65 bpm RA Lead
Sensing = 2.0 mV
Maximum Tracking Rate = 120 bpm Amplitude = 0.4 V @ 0.4 ms
Impedance = 650 ohms

Maximum Sensor Rate = 125 bpm RV Lead


Sensing = 7.0 mV
Amplitude = 0.6 V @ 0.4 ms
Dynamic AV Delay = on Impedance = 600 ohms
Maximum Delay = 270 ms Shock Impedance = 45 ohms
Minimum Delay = 160 ms

AV search Hysteresis = On

Dynamic PVARP = on Programmed Settings


Maximum PVARP = 300 ms
Minimum PVARP = 260 ms Threshold
RA Lead = 3.5 V @ 0.4 ms
PVARP After PVC = 400 ms RV Lead = 2.0 V @ 0.4 ms

Figure 12.3.1  Rhythm strip.

432  u  Section 12: Paced Rhythms and Device Troubleshooting


Questions
A. What is the diagnosis based on the rhythm strip (Figure 12.3.1)?
1. Pacemaker-mediated tachycardia (PMT)
2. Atrial lead malfunction resulting in loss of capture
3. Repetitive nonreentrant ventriculo-atrial synchrony
4. AV search hysteresis
5. Both 3 and 4
B. How do you treat this condition?
1. Increase the post-ventricular atrial refractory period (PVARP)
2. Atrial lead revision
3. Shorten the AV delay
4. Shorten the PVARP
5. Both 3 and 4

Answers
A. 5
B. 5

Discussion
The rhythm strip demonstrates both repetitive nonreentrant ventriculo-atrial synchrony (RNRVAS)
and AV search hysteresis (Figure 12.3.2). The first seven beats show atrial-paced, ventricular-sensed
(AP-VS) complexes. This is followed by a PVC with retrograde atrial conduction. Note the inverted P
waves in lead II (dashed arrows). The device does not sense the retrograde P as it falls in the PVARP
that has extended to 400 ms following a PVC. This is followed by an atrial pacing spike that does not
capture the atrium because the pacing spike falls into the absolute refractory period of the atrium. The
paced AV interval expires resulting in a VP with subsequent retrograde atrial activation and the process
repeats. This repetitive process of functional atrial undersensing due to retrograde atrial activation
falling within the PVARP along with functional atrial noncapture due to the pacing stimulus falling in
the absolute refractory period of the atrium is termed as RNRVAS. It may also be called AV
desynchronization arrhythmia or pseudo atrial exit block. RNRVAS is a less common form of endless
loop tachycardia resulting from ventriculo-atrial conduction; the more common form being PMT.

Figure 12.3.2  Annotated rhythm strip demonstrating repetitive nonreentrant ventriculo-atrial synchrony (RNRVAS) and AV search
hysteresis.

Case 12.3  u  433


Although the upper pacing rate is not seen with RNRVAS, the hemodynamics are similar to PMT.
Patients typically present with symptoms suggestive of loss of AV synchrony; fatigue, palpitations,
dizziness, hypotension, dyspnea, and syncope. RNRVAS can be stored as automatic mode-switching
events because two atrial events [retrograde atrial activation and AP] are recorded by the device for
each VP signal. RNRVAS can be prevented by shortening the PVARP (although this increases the
risk of PMT), shortening the AV delay, reducing the lower rate limit or sensor-driven rate, using
noncompetitive atrial pacing algorithm (manufacturer specific, NCAP delays AP by 300 ms when
retrograde atrial activation falls in the PVARP) or by synchronous AP on PVC detection (functional
noncapture of retrograde atrial impulse).
The other finding demonstrated on this rhythm strip is AV search hysteresis. Dynamic AV delay
and AV search hysteresis were turned on in this patient’s device. In the beats prior to the PVC, the
paced AV delay is approximately 190 ms. Post PVC, the paced AV delay extends out to 300 ms
despite the maximum dynamic AV delay set at 270 ms. This is due to AV search hysteresis
algorithm designed to promote intrinsic AV conduction and reduce deleterious effects of chronic
right ventricular pacing. As there was no intrinsic R wave sensed by the end of the extended AV
delay (300 ms in this case), VP occurred. In the present case, turning off the AV search hysteresis
will shorten the AV delay, and may potentially prevent RNRVAS.

434  u  Section 12: Paced Rhythms and Device Troubleshooting


Santosh K. Padala, MD CASE
Gautham Kalahasty, MD
Kenneth A. Ellenbogen, MD 12.4
Patient History
A 91-year-old female with a history of hypertension, end-stage renal disease on hemodialysis, and
sick sinus syndrome for which she underwent a dual-chamber permanent pacemaker (St. Jude
Medical, Accent DR RF 2210 Pacemaker) implant 1 year ago, was admitted to the hospital with
sepsis. The device was programmed to DDD mode with a base rate of 70 bpm, sensed AV delay of
170 ms, and paced AV delay of 200 ms. She was atrially paced 60% and ventricular paced 66%.
Sensing, pacing threshold, and impedance on right atrial and ventricular leads were within normal
limits. ECG during hospitalization is shown in Figure 12.4.1.

Figure 12.4.1  Baseline ECG.

Question 1
What does the ECG show?
1. Undersensing
2. Intermittent failure to pace
3. Fusion beat
4. Pseudo-fusion beat
5. Pseudo-pseudo fusion beat

Case 12.4  u  435


Answer 1
The correct answer is 5.

Question 2
During PVCs, why is there no ventricular pacing following atrial pacing despite expiration of the AV
interval?
1. PVCs are sensed in the post-atrial ventricular blanking period (PAVB)
2. PVCs are sensed in the cross-talk sensing window
3. PVCs are sensed beyond the PAVB and cross-talk sensing window
4. Failure to pace ventricles intermittently

Answer 2
The correct answer is 3.

Discussion
The ECG (Figure 12.4.2) shows AV sequential pacing with frequent pseudo-pseudo-fusion beats in a
bigeminal pattern.

Figure 12.4.2  Annotated electrocardiogram demonstrating pseudo-pseudo-fusion beats.

During ventricular pacing, different QRS morphologies can be observed depending on the degree of
ventricular capture by the pacing stimulus and intrinsic AV conduction. When the ventricles are
captured solely by the pacing stimulus, a fully paced QRS complex is seen (Figure 12.4.2, bold
arrows). In such cases, each pacing stimulus is seen preceding the QRS complex. When the
ventricles are captured partly by the pacing stimulus and partly by intrinsic conduction, the QRS

436  u  Section 12: Paced Rhythms and Device Troubleshooting


morphology will be “in between” that of an intrinsic and purely paced complex and is referred to as
a fusion beat. The presence of a fusion beat confirms capture from the pacing site.1
In biventricular devices, sometimes a pacing spike can be seen after the initiation of an R wave
and can give a false impression of loss of sensing or capture. This finding is observed when a
triggered pacing algorithm is turned on. The trigger for pacing in the left ventricle and/or right
ventricle is a sensed signal (such as a PVC or conducted atrial arrhythmia) in either chamber. The
resultant QRS complex is actually a fusion between the sensed beat and the paced beat. The
rationale for triggered pacing is to provide at least some ventricular resynchronization to conducted
complexes or PVCs when the ventricular response exceeds the programmed lower pacing rate.1
However, triggered pacing is not as effective as pure biventricular pacing in improving clinical
outcomes.
If the pacing stimulus is seen after the onset of QRS complex and if the morphology of QRS
complex is identical to the intrinsic complex, then it is referred to as a pseudo-fusion beat. In such
cases, capture of the ventricular myocardium cannot be assessed.1 During pseudo-fusion beats
pacing and sensing occur in the same chamber. Pseudo-pseudo-fusion beats occur when timing of an
atrial pacing spike coincides with the early part of the QRS complex, commonly with a PVC (Figure
12.4.2, dashed arrows).1 This beat can be confused for a ventricular pseudo-fusion beat, but on
careful measurement of the intervals, it coincides with an atrial spike. During pseudo-pseudo-fusion
beats sensing and pacing occur in different chambers. In the present case, following the AV
dual-chamber paced beat, an atrial spike occurs after expiration of VA interval, after the onset of
the PVC QRS complex (lead II rhythm strip). This is because the sensing of a ventricular
electrogram is delayed compared to the onset of the QRS complex on the surface ECG leading to an
atrial spike when VA interval expires.
The AV interval is a programmable interval initiated by a sensed or a paced atrial event followed
by ventricular pacing after the interval expires. The AV interval has an initial PAVB followed by a
cross-talk sensing window. If the PVC were sensed during the PAVB period, we would have seen a
ventricular pacing spike after expiration of the AV interval. If the PVC was sensed during the cross-
talk sensing window, an earlier ventricular spike with an AV interval of 110 ms would have been
seen, commonly referred to as “safety pacing.” However, the ventricular pacing spike in these two
scenarios would not have captured the ventricles, as it would have fallen in the post-PVC ventricular
refractory period. In the present case, following an atrial spike just after the PVC, there is no
ventricular spike as the PVC is sensed beyond the PAVB period and cross-talk sensing window, thus
inhibiting ventricular output. The sensed PVC initiates the VA interval and results in an atrial spike
after VA interval expires.

Reference
1. Karoly K. Evaluation, troubleshooting, and management of pacing system malfunctions. In: Kenneth AE and Karoly
K. Cardiac Pacing and ICDs, 6th ed. ch. 7. 2014.

Case 12.4  u  437


CASE Santosh K. Padala, MD

12.5
Gautham Kalahasty, MD
Kenneth A. Ellenbogen, MD

Patient History
A 72-year-old female with a history of persistent atrial fibrillation, sick sinus syndrome for which
she underwent a dual-chamber permanent pacemaker implant (Boston Scientific, Ingenio K173)
three years ago was admitted for dofetilide loading. Figure 12.5.1 shows a 12-lead ECG performed
after two doses of dofetilide. Device interrogation showed the following parameters.

Mode = DDD RA Lead


Sensing = 1.2–3.3 mV
ATR Mode Switch = 150 bpm = DDI Amplitude = 1.0 V @ 0.4 ms
Impedance = 662 ohms
Lower Rate Limit = 60 bpm

Maximum Tracking Rate = 130 bpm RV Lead


Sensing = 14.4 mV
Paced AV Delay = 220–300 ms Amplitude = 0.6 V @ 0.4 ms
Sensed AV Delay = 220–300 ms Impedance = 1081 ohms

PVARP = 240–280 ms Programmed Settings

AP = 3%, VP = 25% Sensing


RA = 0.15 mV
% AT/AF = 97%
RV = 2.5 mV

Threshold
RA Lead= 0.4 V @ 0.5 ms
RV Lead= 0.6 V @ 0.5 ms

Figure 12.5.1  Baseline paced electrocardiogram.

438  u  Section 12: Paced Rhythms and Device Troubleshooting


Question
The QRS morphology shown in ECG (Figure 12.5.1) is due to?
1. Pure ventricular pacing
2. Loss of ventricular capture
3. Fusion beat
4. Pseudo-fusion beat
5. Pseudo-pseudo-fusion beat

Answer
The correct answer is 4.

Discussion
Figure 12.5.2 shows AV sequential paced rhythm with ventricular pseudo-fusion complexes. The
differential diagnosis for RBBB QRS morphology in this ECG are pure RV pacing, a ventricular
fusion beat, and a ventricular pseudo-fusion beat/intrinsic QRS complex. RV pacing typically results
in LBBB QRS morphology. RV apical pacing, however, can result in RBBB QRS morphology. In such
cases, the pacing spike is seen prior to the onset of the QRS morphology and not after the QRS
onset as shown in this case.

Figure 12.5.2  Annotated electrocardiogram demonstrating ventricular pseudo-fusion beats.

A pacing spike seen after the onset of QRS complex is most consistent with fusion or pseudo-fusion
beats. If the pacing spike is seen after QRS onset, and the morphology of QRS is “in between” that
of an intrinsic and purely paced complex, it is referred to as a fusion beat. The presence of a fusion
beat confirms capture from the pacing site. The typical RBBB morphology in the present case is

Case 12.5  u  439


unlikely to be a fusion beat. If the pacing spike is seen after QRS onset, and the morphology of QRS
complex is “identical” to the intrinsic complex, then it is referred to as a pseudo-fusion beat. Figure
12.5.3 shows the 12-lead nonpaced ECG of the same patient showing typical RBBB morphology that
is identical to the pseudo-fusion beat during pacing. In this case, due to RBBB, the left ventricle is
depolarized earlier than the right ventricle via the left bundle branch resulting in initiation of the
QRS complex on the surface ECG. The depolarization then spreads myocyte to myocyte towards
the right ventricle, resulting in a delay of sensing of the QRS complex by the RV pace/sense lead.
During this functional sensing delay, if the AV interval times out, a ventricular spike is delivered
during the QRS complex. The ventricular spike does not result in a paced QRS complex on surface
ECG as ventricles have already started to depolarize. However, capture of the ventricular
myocardium cannot be assessed during a pseudo-fusion beat.

Figure 12.5.3  Baseline nonpaced electrocardiogram.

440  u  Section 12: Paced Rhythms and Device Troubleshooting


CASE
Andrew E. Epstein, MD
Michael P. Riley, MD, PhD 12.6
Patient History
A 62-year-old male with a nonischemic dilated cardiomyopathy, left ventricular ejection fraction
(LVEF) 10%, and a biventricular implantable cardioverter-defibrillator (ICD) who had received an
appropriate ICD shock for ventricular tachycardia presented 4 days later with the following ECG
and was referred for ablation. The device was programmed to pace from 60 to 140 bpm, and to
detect ventricular tachycardia at 180 bpm.

Figure 12.6.1

Discussion
Careful inspection of the tracing shows two pacing stimulus artifacts preceding each QRS. The first
is pacing from the LV lead but not captured due to the threshold being above the programmed
output. The second artifact is from the RV lead. This is pacemaker-mediated tachycardia (PMT)
with LV noncapture that was acutely managed by programming to the VVI temporary pacing mode
(magnet application has no effect on the bradycardia pacing functions of ICDs):

Case 12.6  u  441


Figure 12.6.2

The problem of PMT was managed by reprogramming the PMT detection rate to 120 bpm. The
tracing also highlights the point that PMT can occur below the maximum tracking rate when
retrograde (VA) conduction is slow.

442  u  Section 12: Paced Rhythms and Device Troubleshooting


CASE
Andrew E. Epstein, MD
Michael P. Riley, MD, PhD 12.7
Patient History
A 57-year-old male admitted with heart failure. He has a nonischemic cardiomyopathy with a left
ventricular ejection fraction (LVEF) 20% and a biventricular implantable cardioverter-defibrillator
(ICD). His ECG is shown (Figure 12.7.1). Is there biventricular pacing?

Figure 12.7.1

Discussion
While there is right bundle branch block (RBBB) in lead V1 and a Q in lead I, criteria usually
accepted to indicate biventricular pacing, this is right ventricular-only pacing. The LV pacing
threshold was 4.0 V at a 1.0 ms pulse width and the output programmed at 3.4 V. The underlying
native QRS morphology was RBBB and the QRS when paced from the right ventricle (RV) alone
identical to the above. The following ECG (Figure 12.7.2) shows pure LV pacing after the first two
beats with the LV output increased above threshold.

Case 12.7  u  443


Figure 12.7.2

The patient was managed by programming the device AAI in view of the worsening of heart failure
with RV pacing.

References
1. Ammann P, Sticherling C, Kalusche D, et al. An electrocardiogram-based algorithm to detect loss of left ventricular
capture during cardiac resynchronization therapy. Ann. Intern. Med. 2005;142:968–973.
2. Van Stipdonk A, Wijers S, Meine M, et al. ECG patterns in cardiac resynchronization therapy. J. Atrial Fib.
2015;7:33–38.

444  u  Section 12: Paced Rhythms and Device Troubleshooting


CASE
Andrew E. Epstein, MD
Michael P. Riley, MD, PhD 12.8
Patient History
The patient had a single-chamber implantable cardioverter-defibrillator (ICD) implanted for
primary prevention. Amiodarone was prescribed for atrial fibrillation and flutter. He presented with
fatigue.

Figure 12.8.1

Discussion
The ECG (Figure 12.8.1) shows marked sinus bradycardia with asynchronous ventricular pacing at
60 bpm. AV conduction is intact (see two conducted beats). This made the diagnosis of the
pacemaker syndrome likely. After upgrading the ICD to a dual-chamber system, the symptoms
resolved.

Case 12.8  u  445


CASE
12.9 Andrew E. Epstein, MD
Michael P. Riley, MD, PhD

Patient History
This ECG (Figure 12.9.1) was recorded the first day following dual-chamber pacemaker
implantation for complete heart block.

Figure 12.9.1

Discussion
The ECG (Figure 12.9.1) shows AV pacing throughout (very small ventricular pacing artifacts).
There is malfunction of the atrial lead with noncapture and nonsensing. Every third ventricular
paced complex causes a retrograde P wave. Atrial lead revision was required.

446  u  Section 12: Paced Rhythms and Device Troubleshooting


CASE
Andrew E. Epstein, MD
Michael P. Riley, MD, PhD 12.10
Patient History
A dual-chamber implantable cardioverter-defibrillator was implanted 4 years prior to this ECG
(Figure 12.10.1). The device was programmed as follows: DDDR 60–120 bpm, paced and sensed AV
delays both 220–400 ms, PVARP 230–250 ms.

Figure 12.10.1

Discussion
The underlying rhythm is sinus with first-degree AV block (390–400 ms) and frequent premature
ventricular complexes (PVCs). There is normal pacemaker function. Pacing stimuli are identified by
blue arrows at bottom of the tracing. A redundant ECG lead was removed between rhythm strips V1
and V5. Notable findings include: fusion of conduction and ventricular pacing (second, sixth, and
ninth QRS complexes), simultaneous atrial pacing and a PVC had occurs during the atrial blanking
period resulting in subsequent ventricular pacing with noncapture (seventh QRS complex), a
nontracked sinus beat that conducts with a very long PR (560 ms) at the same time as scheduled
atrial pacing occurs (eighth QRS complex)—this conducted QRS also occurs during atrial blanking
resulting in ventricular pacing with noncapture.

Case 12.10  u  447


CASE
12.11 Nishant Verma, MD, MPH
Bradley P. Knight, MD

Patient History
A 58-year-old female with a dilated cardiomyopathy, heart failure, and LBBB had previously
undergone implantation of a defibrillator with cardiac resynchronization therapy (CRT-D) using a
bipolar lead placed in the coronary sinus. She had a known chronically elevated left ventricular (LV)
pacing threshold and was referred for device evaluation. The following intracardiac electrogram
(EGM) (Figure 12.11.1) was obtained during LV lead pulse-width threshold testing. What is the
explanation for the change in QRS morphology seen during LV threshold testing?

Figure 12.11.1  Intracardiac EGM during LV lead pacing threshold test.

During pulse-width threshold testing, an abrupt change in QRS morphology is seen in ECG lead II
(dashed versus solid circle) at 4.5 V @0.8 ms. The RV tip to ring EGM shows a simultaneous change
in morphology (dashed versus solid arrow) and evidence of latency (curved arrow).
The initial concern when examining the intracardiac tracing was the presence of anodal
capture. Anodal capture may occur at high pacing outputs. In this case, the LV lead configuration
was from the LV tip to the right ventricular (RV) ring. At high outputs in this configuration, RV
capture may occur due to anodal stimulation. RV stimulation may occur either in isolation or with
concomitant LV stimulation during both biventricular and LV pacing alone. This phenomenon is
almost always seen at outputs above LV capture threshold and may lead to confusion and
misinterpretation of the LV capture threshold. In addition, there has been concern for adverse
clinical effects including lack of response to cardiac resynchronization and worsening of heart
failure symptoms.1,2 The intracardiac EGMs will often show an abrupt change in the QRS

448  u  Section 12: Paced Rhythms and Device Troubleshooting


morphology as well as latency on the RV EGM when RV capture is lost and LV capture persists.
Both of these features were seen on this intracardiac tracing.
The most reliable way of troubleshooting these types of cases and diagnosing anodal capture is
by analyzing the 12-lead ECG during threshold testing. During biventricular pacing with anodal
capture, the ECG may show a decrease in QRS duration by as much as 20 ms as well as an increase
in the QRS amplitude of leads I and aVL.3 In this instance, the 12-lead ECG revealed something else
entirely (Figure 12.11.2). LV pacing at high output reveals a QRS with a right bundle branch (RBB)
morphology (dashed arrows), consistent with LV capture. However, as the pulse width was
decreased and LV capture was lost, it unmasked the presence of left atrial (LA) capture (solid
arrows), associated with 1:1 AV conduction and an underlying LBBB. Because the time from LV
capture to RV activation was similar to the time from LA capture to RV activation, the intracardiac
EGMs gave the appearance of latency and a change in the QRS morphology mimicking anodal
capture.
A review of the patient’s history revealed that the coronary sinus (CS) lead placement was
extremely difficult. The CS lead remained in a basal location, likely predisposing to atrial capture.
Rather than revise the lead, the 12-lead ECG was used to find a configuration that did not result in
atrial capture and the LV lead output was programmed just above threshold.

Figure 12.11.2  12-lead ECG obtained during LV lead pacing threshold test.

References
1. Dendy KF, Powell BD, Cha YM, et al. Anodal stimulation: An underrecognized cause of nonresponders to cardiac
resynchronization therapy. Indian Pacing Electrophysiol. J. 2011;11(3):64–72.
2. Abu Sham’a R, Kuperstein R, Barsheshet A, et al. The effects of anodal stimulation on electrocardiogram, left
ventricular dyssynchrony, and acute haemodynamics in patients with biventricular pacemakers. Europace.
2011;13(7):997–1003.
3. Bulava A, Ansalone G, Ricci R, et al. Triple-site pacing in patients with biventricular device-incidence of the
phenomenon and cardiac resynchronization benefit. J. Interv. Card. Electrophysiol. 2004;10(1):37–45.

Case 12.11  u  449


CASE
12.12
Dan Blendea, MD, PhD
Moussa Mansour, MD

Patient History
An 89-year-old patient was implanted a dual-chamber pacemaker for sinus node dysfunction. Post
implant, a right bundle branch block (RBBB)-like pattern was noted (Figure 12.12.1).

Figure 12.12.1  Atrioventricular sequential pacing. The paced QRS has a RBBB-like morphology in lead V1.

Questions
1. Should the lead be repositioned?
2. Why did the RBBB-like pattern occur?

Discussion, Interpretation, and Answers


In patients with ventricular paced rhythm, an RBBB-like pattern may suggest inadvertent left
ventricular (LV) lead placement. However, a RBBB-like pattern can occur even if the lead is indeed
in the right ventricle as intended.1
Almehairi and colleagues analyzed 943 patients who underwent pacer implants. The prevalence
of RBBB-like pattern was 8.1% (n = 77). Lowering the electrocardiographic leads V1 and V2 to the
fifth intercostal space resulted in transition to a LBBB-like pattern with a QS wave in V1 in 14 of 26
patients (sensitivity, 53%; specificity, 100%), whereas displacement to the sixth intercostal space
resulted in a QS pattern in all patients (sensitivity and specificity, 100%). In all patients in the

450  u  Section 12: Paced Rhythms and Device Troubleshooting


control group, the ECG depicted an RBBB configuration with leads V1 and V2 in the standard
position, as well as at the fifth and sixth ICS.6

Answers
1. Not necessarily, because a lead correctly positioned in the right ventricle can generate a RBBB-
like pattern.
2. The RBBB-like pattern can be explained by the relatively low position of the ventricles in the
thorax in this patient. This alters the usual relationship between the ventricles and precordial
leads V1 and V2. In this particular patient the low position of the heart generated during the
initial part of the ventricular depolarization an activation wavefront traveling toward the V1 and
V2 electrodes (Figures 12.12.2 and 12.12.3) and therefore generating R waves in V1 (Figure
12.12.1) By lowering V1 and V2 with one intercostal interspace the pattern in V1 changed to a
LBBB-like pattern (Figure 12.12.4).

Figure 12.12.2  PA chest x-ray showing that the apex of the heart and tip of the right ventricular lead are situated relatively low in the
thorax.

Case 12.12  u  451


Figure 12.12.3  CT of the chest—coronal view—revealing the position of the apex of the heart situated very low in relationship to the
fourth intercostal space (where lead V1 is positioned). The resultant depolarization vector is oriented from the apex toward the base of the
heart generating a positive deflection in lead V1.

Figure 12.12.4  Lowering the precordial leads with one interspace associated with disappearance of the RBBB-like pattern of the paced
QRS.

Reference
1. Almehairi M, Enriquez A, Redfearn D, et al. Right bundle branch block-like pattern during ventricular
pacing: A surface electrocardiographic mapping technique to locate the ventricular lead. Can. J. Cardiol.
2015;31(8):1019–1024.

452  u  Section 12: Paced Rhythms and Device Troubleshooting


CASE
Yuji Nakazato, MD, PhD 12.13
Patient History
A 97-year-old female was admitted with syncope. Although she was quite elderly, her quality of life
in daily activity was good. ECG on admission (Figure 12.13.1) showed intermittent complete
atrioventricular block and a single-chamber pacemaker (VVI) was implanted.

Figure 12.13.1

Question
Where is the pacing site?

Discussion
A ventricular pacing lead has been conventionally placed in the apex. However, long term apical
pacing deteriorates ventricular function due to dyssynchronous motion between the right and left
ventricle. Eventually, QRS width became wider because of left ventricular contraction delay. If septal
pacing could be performed, time delay would be shorter than the apical pacing and QRS width
becomes narrow as a result. In this case, successful septal pacing was done and narrow QRS pacing
could be obtained.

Case 12.13  u  453


CASE
12.14 I. W. P. Obel, MCChB
Judith Daniels, RN, CRDS, CEPS

Patient History
A 32-year-old African American male flight attendant had a dual-chamber pacemaker (DDD)
implanted for AV nodal disease with syncope.

Programmed Settings
DDD, lower rate limit (LRL) 60 bpm (1000 ms) and an AV interval of 240 ms.
This baseline ECG (Figure 12.14.1) was taken a few hours later.

Figure 12.14.1

Question
What is the reason for the abnormally low pacing rate?

454  u  Section 12: Paced Rhythms and Device Troubleshooting


Figure 12.14.2

Figure 12.14.3

Case 12.14  u  455


Figure 12.14.4   Pacemaker tracing.

Discussion, Interpretation, and Answers


Figures 12.14.1 and 12.14.2 show atrial and ventricular pacing rate of 43 bpm (1380 ms).
The paced beats are narrow, suggesting fusion with intrinsic activation. Early repolarization is
seen. The rate is due to oversensing of the T wave.
A DDD pacemaker timing cycle consists of an LRL, several refractory periods on both channels,
and an upper rate limit. When an event is sensed on either channel outside a refractory period, the
timing is set.
The LRL is divided into AV interval (A) and a ventriculoatrial (VA) interval (B). At the end of
the VA interval atrial pacing would be expected. The VA interval is 760 ms.
By measuring back from a beat on the rhythm strip, it is possible to see what was sensed.
Measuring the VA interval from the atrial pacing spike (C) or the LRL from the ventricular spike
(D) shows that the T wave was sensed as an R wave on the ventricular channel.
This pacemaker uses an auto adjusting sensitivity as used by implantable cardioverter
defibrillators (ICDs). The programmed sensitivity is 0.3 mV. Following a paced beat, sensing starts
at 4.5 times the programmed sensitivity with an increasing sensitivity to 0.3 mV. This, together with
the large T wave caused by early repolarization, facilitated T-wave oversensing. The sensitivity was
reprogrammed to 5.6 mV (less sensitive), which corrected the rhythm.
Alternate programmable features that could lead to a lower rate below the LRL were ruled out.
Circadian/night rate was not programmed. The night rate would drop the lower rate at a specified
time for sleep. If the pacemaker time is incorrect (as with travel) this could happen during the day.
Hysteresis would allow the intrinsic rate to drop below the lower rate but when the hysteresis rate
was reached, pacing would be at the LRL. Atrial based timing, although allowing a lower rate, would
be minimally below the LRL.

456  u  Section 12: Paced Rhythms and Device Troubleshooting


Figure 12.14.3 shows baseline ECG with early repolarization.
Figure 12.14.4 shows the oversensing on the ventricular channel. R-wave oversensing is noted on
the atrial channel (Ab). Since it occurs in the blanking period it does not reset the timing.
(AP = atrial pace; Ab = atrial blanking; VP = ventricular pace; VS = ventricular sense)

Reference
1. Ellenbogen K, Kay GN, Lau C, et al. Clinical Cardiac Pacing, Defibrillation, and Resynchronization Therapy. 4th ed.
Philadelphia, PA: Elsevier; 2011.

Case 12.14  u  457


CASE
12.15 Mohammad Ali Sadr-Ameli, MD

Patient History
A 63-year-old male with acute coronary syndrome was brought to the catheter laboratory for
angiography and percutaneous coronary intervention on the left circumflex coronary artery. A
temporary pacemaker was inserted due to bradycardia.

Figure 12.15.1

R-T phenomenon and induction of nonsustained ventricular tachycardia.

458  u  Section 12: Paced Rhythms and Device Troubleshooting


SECTION 13A
Arrhythmogenic Right Ventricular Dysplasia/Cardiomyopathy
(ARVD/C)

CASE
N. A. Mark Estes III, MD 13A.1
Patient History
ECG of a 16-year-old female patient presenting recurrent syncope while running.

Figure 13A.1.1 

Questions
1. What is the ECG abnormality?
2. What cardiovascular condition is likely to cause the ECG abnormality and exercise-induced
syncope?

ECG Masters’ Collection: Favorite ECGs from Master Teachers Around the World © 2017 Mohammad Shenasa, Mark E. Josephson,
N. A. Mark Estes III, Ezra A. Amsterdam, Melvin Scheinman. Cardiotext Publishing, ISBN: 978-1-942909-08-8.   459
Discussion
The ECG demonstrates T-wave inversions in leads V1–V4. ARVD/C would most commonly be
associated with these ECG findings and exercise-related syncope. Echocardiogram confirmed
marked enlargement of the right ventricle with areas of dyskinesis at the apex and outflow tract.
Magnetic resonance imaging demonstrated fibrofatty infiltration the RV diagnostic of ARVD/C.

460  u  Section 13A: Arrhythmogenic Right Ventricular Dysplasia/Cardiomyopathy


CASE
13A.2
Richard N. Hauer, MD, PhD
Judith A. Groeneweg, MD, PhD

Patient History
A patient was referred because of sustained monomorphic ventricular tachycardia (VT) with left
bundle branch block (LBBB) morphology and superior axis. The 12-lead ECG during sinus rhythm
is shown in Figure 13A.2.1, and magnification of recordings indicated by arrows in Figure 13A.2.2.

Figure 13A.2.1  12-lead ECG of normal sinus rhythm. Terminal activation duration (TAD), indicated by vertical lines in lead V2, is with 60
ms slightly prolonged. Epsilon waves and obvious repolarization abnormalities are absent. Arrows indicate deflections at the end (lead III)
or after (leads II and aVF) of QRS complex.

Figure 13A.2.2  Magnification of leads III and II, and aVF and II, respectively, showing separation of indicated deflections from QRS in aVF
and II.

Case 13A.2  u  461


Questions
1. Is the diagnosis of arrhythmogenic right ventricular dysplasia/cardiomyopathy (ARVD/C)
according to revised 2010 consensus-based Task Force Criteria (TFC) justified because of the
arrhythmia and ECG during sinus rhythm?
2. What is the interpretation of deflections indicated by arrows?

Discussion, Interpretation, and Answers


Diagnosis of ARVD/C is based on major and minor criteria of the international consensus-based
TFC. Either two major, one major with two minor, or four minor criteria are needed, after exclusion
of alternative explanations, such as sarcoid and other diseases.1
The VT with LBBB morphology counts for one major criterion because of the morphology with
superior axis. The ECG during sinus rhythm is apparently normal, although with some remarkable
features. The TAD is with 60 ms marginally prolonged, counting for maximally one minor
criterion. Signals indicated by arrows give the impression of “early repolarization” in lead III,
whereas in leads II and aVF separation from QRS and steep terminal upstroke suggest local inferior
activation delay, similar to epsilon waves in leads V1–V3, which are absent in this case. Endocardial
ventricular activation mapping of sinus rhythm demonstrates low-amplitude high dV/dt signals up
to 150 ms after QRS onset and clearly after QRS termination in the inferior subtricuspid area
(Figure 13A.2.3). This observation is in accordance with local inferior activation delay giving rise to
late signals in the surface ECG, mimicking epsilon waves. Magnetic resonance imaging (MRI)
confirmed right ventricular dilatation and a subtricuspid aneurysm as major TFC (Figure 13A.2.4).
In addition, the pathogenic c.235C>T mutation in PKP2 was identified, which is also a major
criterion. With three major and one minor criteria, the diagnosis of advanced severe ARVD/C is
confirmed, despite the apparently nearly normal ECG.

Figure 13A.2.3  Recordings during endocardial activation catheter mapping of sinus rhythm. From above ECG leads I–III and V1, bipolar
electrograms from high right atrium (HRA), subtricuspid area from distal MAP1,2 and proximal MAP3,4 electrodes, and corresponding
unipolar recordings MAP1 and MAP2. Vertical line indicates onset QRS complex. At 150 ms after onset QRS, and after QRS termination, a
low amplitude (0.30 mV), high dV/dt signal is recorded. This signal originates from a site adjacent to the second electrode since exclusively
visible in unipolar MAP2, and not in MAP1.

462  u  Section 13A: Arrhythmogenic Right Ventricular Dysplasia/Cardiomyopathy


Figure 13A.2.4  Cardiac MRI showing subtricuspid aneurysm.

Answers
1. Combination of VT with LBBB morphology with superior axis and prolonged TAD gives only
one major and one minor TFC, which is insufficient for diagnosis. Presence of major structural
disease and/or the pathogenic mutation were needed for TFC diagnosis fulfillment.
2. Deflections appeared to be due to the equivalent of epsilon waves in the right precordial leads
V1–V3, which were absent in this case. These deflections in the inferior leads are not included in
the 2010 revised TFC and thus will officially not contribute to ARVD/C diagnosis. This case
supports meticulous analysis of inferior leads in patients with a suspicion for ARVD/C. Finally
our case challenge the concept of “early repolarization,” since the cause of these signals is most
likely a depolarization abnormality.

Reference
1. Marcus FI, McKenna WJ, Sherrill D, et al. Diagnosis of arrhythmogenic right ventricular cardiomyopathy/dysplasia:
Proposed modification of the task force criteria. Circulation. 2010;121:1533–1541.

Case 13A.2  u  463


CASE
13A.3 Richard N. Hauer, MD, PhD

Patient History
ECG during sinus rhythm from an adult patient known to have ventricular tachycardia episodes.
The terminal activation duration (TAD) is defined as the longest interval between the nadir of the S
wave to the end of all depolarization deflections in leads V1–V3. In this case, the TAD is markedly
prolonged at 110 ms, indicated in lead V2.1

Figure 13A.3.1  12-Lead ECG of normal sinus rhythm. Depolarization and repolarization are markedly disturbed. TAD in lead V2 is 110
ms. Arrows point to late activity signals.

Questions
1. What is the interpretation of deflections indicated by arrows in leads V1–V3?
2. What is the measured value of the width of the QRS complex?
3. What is the interpretation of deflections indicated by arrows in leads II, III, and aVF?

464  u  Section 13A: Arrhythmogenic Right Ventricular Dysplasia/Cardiomyopathy


Discussion, Interpretation, and Answers
Diagnosis of arrhythmogenic right ventricular dysplasia/cardiomyopathy (ARVD/C) is based on
major and minor criteria of the international consensus-based 2010 revised Task Force Criteria
(TFC). Either two major, one major with two minor, or four minor criteria are needed, after
exclusion of alternative explanations, such as sarcoid and other diseases.2
Prolonged TAD (TAD ≥ 55 ms) is a minor TFC. Thus, TAD in this case (110 ms) means markedly
prolonged. The arrows in leads V1–V3 point to a signal suggesting an epsilon wave. An epsilon wave
is a major TFC and thus contributes more to ARVD/C diagnosis than prolonged TAD. Negative T
waves in V1–V3 and beyond, is also a major TFC. In this case there are inverted T waves up to lead
V5. Thus, if the arrows point to an epsilon wave, and if the ECG is not related to an alternative
diagnosis, the presented ECG fulfills with two major TFC already the definite ARVD/C diagnosis.
However, are the indicated signals in leads V1–V3 epsilon waves? The epsilon wave was originally
defined as “tiny signals that consistently occurred after the end of each QRS complex.”3,4 In
agreement with this definition, an epsilon wave can be defined as a distinct deflection after the end
of the QRS complex, i.e., after the QRS complex had returned to the isoelectric line.1 However, if
high-amplitude signals in the QRS complex are followed by a series of low-amplitude signals, as is
visible in this case, the end of the QRS complex and thus the width of the QRS complex are
questionable. In addition, visibility of low-amplitude signals are dependent on filter setting and
magnification. With these limitations, the interpretation of the indicated signals in leads V1–V3 as
epsilon waves remains equivocal.4 However, epsilon wave always means a severe activation delay,
which is correct in this case, since epsilon waves are always associated with markedly prolonged
TAD. The arrows in leads II, III, and aVF point to low-amplitude signals due to activation delay in
inferior parts of the ventricles. Inverted T waves in these leads are also indicative of inferior wall
involvement. This ECG is very characteristic for severe and extensive ventricular disease in ARVD/C.

Answers
1. The indicated depolarization signals in leads V1–V3 are apparently separated from QRS and thus
may be interpreted as epsilon waves. However, this separation is less clear with magnification.
Important activation delay is already indicated by TAD being 110 ms.
2. QRS width is very questionable since the end of QRS complex is not well defined.
3. Deflections in leads II, III, and aVF are due to activation delay in inferior parts of the ventricles,
which is not included in the revised TFC.

References
1. Cox MG, Nelen MR, Wilde AA, et al. Activation delay and VT parameters in arrhythmogenic right ventricular
dysplasia/cardiomyopathy: Toward improvement of diagnostic ECG criteria. J. Cardiovasc. Electrophysiol.
2008;19:775–781.
2. Marcus FI, McKenna WJ, Sherrill D, et al. Diagnosis of arrhythmogenic right ventricular cardiomyopathy/dysplasia:
Proposed modification of the task force criteria. Circulation. 2010;121:1533–1541.
3. Fontaine G, Guiraudon G, Frank R, et al. Simulation studies and epicardial mapping in ventricular tachycardia:
Study of mechanisms and selection for surgery. In: Kulbertus HE, ed. Reentrant Arrhythmias. Philadelphia, PA:
Springer; 1977:334–350.
4. Platonov PG, Calkins H, Hauer RH, et al. High interobserver variability in the assessment of epsilon waves:
Implications for diagnosis of arrhythmogenic right ventricular cardiomyopathy/dysplasia. Heart Rhythm.
2015;15:S1547–S5271.

Case 13A.3  u  465


CASE
13A.4 Richard N. Hauer, MD, PhD

Patient History
Both Figures 13A.4.1 and 13A.4.2 are from two patients that show negative T waves in the
precordial leads and maximum QRS voltage in the standard leads at 0.6 mV.

Figure 13A.4.1  12-lead ECG of sinus rhythm. Ventricular depolarization and repolarization abnormalities. A very tiny deflection at
150 ms after QRS onset is indicated by arrow.

Figure 13A.4.2  12-lead ECG of normal sinus rhythm. Ventricular repolarization abnormalities.

466  u  Section 13A: Arrhythmogenic Right Ventricular Dysplasia/Cardiomyopathy


Questions
1. The P waves and QRS width in both ECGs are very different. What is the explanation?
2. The arrow in lead V2 points to a very tiny deflection. What differentiates this very tiny deflection
from artefact?
3. What is the ECG diagnosis derived from Figures 13A.4.1 and 13A.4.2, respectively? To facilitate
interpretation of Figure 13A.4.2, an ECG of the patient’s sister is included in Figure 13A.4.3.

Figure 13A.4.3  12-lead ECG of sinus rhythm. Low-voltage ECG with repolarization abnormalities.

Discussion, Interpretation, and Answers


Figure 13A.4.1 shows sinus rhythm with enlargement of the terminal negative component of the P
wave (larger than 1 mm2) and positive terminal component of the P wave in aVL suggesting left
atrial enlargement. The QRS width is 0.12 seconds in the absence of right- or left-bundle branch
block, thus indicating intraventricular conduction delay. The maximum QRS voltage in the
standard leads is only 0.6 mV. The T wave in all precordial and inferior leads is inverted. This
combination of left atrial enlargement with low ventricular voltages with specific ventricular
depolarization and repolarization abnormalities suggests a form of dilated cardiomyopathy. The
arrow points to a very tiny deflection in lead V2. This nearly invisible deflection, recorded at 150 ms
after ORS onset, is not an artefact because of reproducible recording in subsequent cycles and
synchrony with a hump-like positive contour in lead V1. The signal in lead V2 is in that lead
separated from the QRS complex conform the definition of epsilon wave (separation from QRS in
only one of leads V1–V3 is sufficient). Due to epsilon waves and negative T waves in all precordial

Case 13A.4  u  467


leads (although less than 0.1 mV in V6), the patient has two major revised task force criteria for
arrhythmogenic right ventricular dysplasia/cardiomyopathy (ARVD/C) diagnosis. The extension of
negative T waves to all precordial and inferior leads, the low voltage, and the left atrial enlargement
indicate biventricular disease in a very advanced stage of ARVD/C. This patient, known to have
ventricular tachycardia with left bundle branch block (LBBB) morphology (minor criterion),
deteriorated several years after this ECG recording to end-stage disease with serious hemodynamic
sequelae necessitating a cardiac transplant procedure. In addition, she was known to have a
pathogenic PKP2 mutation, revealing an additional major ARVD/C criterion.
Figure 13A.4.2 in contrast shows normal P waves and QRS width. However, the maximum QRS
voltage is also only 0.6 mV and also inverted T waves are recorded in all precordial leads, and in
addition in leads I, II, and aVL. Symmetric negativity of all negative T waves and the high initial
positivity of the QRS complex in lead V1 may suggest coronary artery disease and posterior
myocardial infarction, respectively. However, coronary angiography did not reveal evidence of
coronary artery disease. This ECG may be due to a form of dilated cardiomyopathy, though in a less
advanced stage. However, this patient had VT with LBBB morphology and a widened right ventricle
with dyskinesia. This patient fulfills TFC for ARVD/C if imaging findings are included. Figure
13A.4.3 from the patient’s sister shows the typical feature of low voltage (maximum QRS voltage in
lead II only 0.3 mV), and in addition, negative T waves in leads V5 and V6, suggesting left ventricular
disease. However, the left ventricular ejection fraction was 62% and cine-angiography and
echocardiography did not show evidence of structural disease. Nevertheless, similarity in the ECG
abnormalities in both sisters suggests the possibility of a genetic disorder. The pathogenic
c.40_42delAGA (p.Arg14del) founder mutation (founder mutation in The Netherlands) in the
phospholamban (PLN) gene was found in both sisters.1 This mutation is associated with a
biventricular form of ARVD/C. Low QRS voltages (in the sister even without identifiable structural
disease) and negative T waves in left precordial leads are characteristic features.

Answers
1. Figure 13A.4.1 is from a patient with more advanced ventricular disease with hemodynamic
sequelae giving rise to left atrial enlargement, and more electrical activation delay. These features
are not found in Figure 13A.4.2.
2. The arrow in lead V2 points to a very tiny epsilon wave, illustrating the need of meticulous
observation.
3. Figure 13A.4.1 is from a patient nearly in the end stage of ARVD/C, whereas in Figure 13A.4.2
the low QRS voltage, the left lateral negative T waves, and the familial occurrence (with very low
QRS voltage in Figure 13A.4.3 from the sister) of these manifestations are suggestive of the
typical expression of the phospholamban founder mutation.

Reference
1. Van der Zwaag PA, van Rijsingen IA, Asimaki A, et al. Phospholamban R14del mutation in patients diagnosed with
dilated cardiomyopathy or arrhythmogenic right ventricular cardiomyopathy: Evidence supporting the concept of
arrhythmogenic cardiomyopathy. Eur. J. Heart Fail. 2012;14:1199–1207.

468  u  Section 13A: Arrhythmogenic Right Ventricular Dysplasia/Cardiomyopathy


CASE
Charles Jazra, MD
Simon Abou Jaoude, MD 13A.5
Patient History
The patient was a 21-year-old male who had a first syncopal episode while swimming. His family
history included sudden death in his father at 49 years old.

Figure 13A.5.1 

The ECG (Figure 13A.5.1) shows sinus rhythm with incomplete right bundle branch block
pattern. Presence of a notch at the end of the QRS suggests an “epsilon wave” (red arrow).1 The
workup confirmed the diagnosis of arrhythmogenic right ventricular dysplasia (ARVD/C).

Discussion
The epsilon wave is a small positive deflection (“blip”) buried in the end of the QRS complex.

Case 13A.5  u  469


It is the characteristic finding in ARVD/C.
The ECG changes in ARVD/C include:1
• Epsilon wave (most specific finding, seen in 30% of patients)
• T-wave inversions in V1–V3 (85% of patients)
• Prolonged S-wave upstroke of 55 ms in V1–V3 (95% of patients)
• Localized QRS widening of 110 ms in V1–V3 (incomplete or complete right bundle branch block)

Reference
1. Jaoude SA, Leclercq JF, Coumel P. Progressive ECG changes in arrhythmogenic right ventricular disease: Evidence
for an evolving disease. Eur. Heart J. 1996;17:1717–1722.

470  u  Section 13A: Arrhythmogenic Right Ventricular Dysplasia/Cardiomyopathy


CASE
Frank I. Marcus, MD 13A.6
Patient History
A 3-lead ECG (V1 V2 and V3) is taken from a 31-year-old patient with recurrent palpitations. He had
no symptoms of heart disease until the age of 24, at which time he developed ventricular
tachycardia. Note that the ECG was recorded at twice normal amplitude.

Figure 13A.6.1 

This ECG (Figure 13A.6.1) shows a sinus rhythm at 70 bpm with incomplete right bundle branch
block pattern and a notch at the end of the QRS, known as an epsilon wave (red arrows). There is a
QRS axis of +90°, a PR interval of 0.166 seconds, and QRS of 130 ms.

Question
What is the diagnostic significance of the presence of an epsilon wave in V1–V3?

Answer
This ECG shows the presence of an epsilon wave. This is highly suggestive of ARVD/C.

Case 13A.6  u  471


CASE
13A.7 Frank I. Marcus, MD

Patient History
A 21-year-old male presented with several syncope episodes and frequent palpitations. He was a
competitive cyclist during his teenage years and at age 16 had an onset of palpitations during
exercise that lasted 5–10 minutes. His resting ECG (not shown) showed T-wave inversion in leads
V1–V4. At age 17, he had palpitations at rest. He was hospitalized due to prolonged ventricular
tachycardia (VT) at 180 bpm and was treated with cardioversion.

Figure 13A.7.1 

Question
What is the most likely diagnosis of this arrhythmia?

Answer
Considering the patient’s age and presentation, the most likely diagnosis is VT.
Patients with ARVC/D can tolerate rapid VT because the left ventricle has normal
hemodynamic function. This patient’s right ventricular angiogram showed outpouching in the
posterior aspect of the pulmonary infundibulum (right ventricular outflow tract [RVOT]). Open-
heart surgery with incision in the RVOT area was performed for treatment of his VT. Unfortunately,
he had reccurrence of VT after the procedure.

472  u  Section 13A: Arrhythmogenic Right Ventricular Dysplasia/Cardiomyopathy


CASE
Carlos Alberto Pastore, MD, PhD
Horácio Gomes Pereira Filho, MD 13A.8
Patient History
Electrical cardioversion in 1999, after palpitations while playing soccer. New cardioversion in 2003,
at rest.
Medications: Amiodarone, carvedilol. Physical examination: normal. Laboratory data: normal,
negative serology for Chagas disease. Echocardiography: right ventricle with important dilatation.
24 hour Holter-ECG: frequent PVCs: 8796 (7.7%). Cardiac catheterization: normal.
Electrophysiological study: no arrhythmia induction.

Figure 13A.8.1 

Figure 13A.8.2  QRS aspect on ECG.

Case 13A.8  u  473


Figure 13A.8.3  QRS Loop aspect on VCG between ARV, RBB, and Normal.

Other Exams
Signal-Averaged Electrocardiogram (SAECG)
Filtered QRS: 151 ms; <40 mV: 67; last 40 ms: 6.9.

Magnetic Resonance Imaging


Diastolic dimensions: 52 mm; systolic dimensions: 31 mm; LVEF: 64%; dilated right chamber: RVEF:
37%; RV outflow tract with dyskinesia.

Conclusion
Arrhythmogenic right ventricular dysplasia/cardiomyopathy (ARVD/C)

Discussion
ARVD/C is characterized by fatty or fibrous-fatty infiltration of the myocardium, in the right
ventricle inflow and/or outflow tracts, and/or the RV apex. In its typical clinical manifestation, 40%
of the affected individuals are asymptomatic. Symptoms consist of dizziness, palpitations, syncope,
atypical precordial pain, dyspnea, and sudden cardiac death. ARVD/C has a familial autosomal
dominant inheritance.
ARVD/C is a significant cause of sudden cardiac death among young adults, affecting 11% of the
global population, 22% of athletes, with an estimated prevalence of 1:3000 to 1:5000 individuals.
In 2010, a task force1 established new criteria for the diagnosis of ARVD/C: three major and one
minor criteria, or two major and two minor criteria should be met. The criteria are the following:

474  u  Section 13A: Arrhythmogenic Right Ventricular Dysplasia/Cardiomyopathy


Major Criteria
MRI: right ventricular akinesis, dyskinesis or dyssynchrony, RVEF ≤40%.
ECG: Epsilon waves, inverted T waves (V1-V3), patients >14 years old without right bundle branch
block (RBBB).

Minor Criteria
ECG: inverted T waves (V1-V3), >14 years old with RBBB.
SAECG: late potentials (with QRS <110 ms); filtered QRS ≥114 ms); QRS duration <40 mV ≥38 ms;
root mean squared last 40 ms ≤20 mV.

When should we consider ARVD/C as the diagnosis? In young individuals with premature
ventricular complexes or ventricular tachycardia (VT) with LBBB morphology, negative QRS in the
inferior wall.

Reference
1. Marcus FI, McKenna WJ, Sherrill D, et al. Diagnosis of arrhythmogenic right ventricular cardiomyopathy/dysplasia:
Proposed modification of the task force criteria. Circulation. 2010;121(13):1533–1541.

Case 13A.8  u  475


CASE
13A.9 Melvin Scheinman, MD

Figure 13A.9.1 shows a wide complex tachycardia with left bundle branch block (LBBB) pattern.
What is the most likely diagnosis?

Figure 13A.9.1  Spontaneous wide complex tachycardia after minor surgical procedure (43-year-old male).

Figure 13A.9.2 shows a 12-lead ECG from the same patient. What is the most likely cardiac
diagnosis?

Figure 13A.9.2  43-year-old male presented with wide complex tachycardia and cardioverted.

476  u  Section 13A: Arrhythmogenic Right Ventricular Dysplasia/Cardiomyopathy


Discussion
Figure 13A.9.1 shows a wide complex regular tachycardia with an LBBB pattern. Note the
concordant negative precordial pattern as well as the right axis deviation and the R wave in aVR,
all of which point to the diagnosis of ventricular tachycardia (VT). The late precordial transition
and inferior axis point to a superior right ventricular free wall focus. Figure 13A.9.2 shows the
12-lead ECG after restoration of sinus rhythm. Note the incomplete right bundle branch block
(RBBB) pattern as well as a clear cut epsilon wave in lead V2 as well as the deep T-wave inversion
in leads V1 and V2 . The finding of a epsilon wave is strong evidence for the diagnosis of
arrhythmogenic right ventricular cardiomyopathy (ARVD/C). It may also be seen in patients
with granulomatous infiltration of the right ventricle such as sarcoidosis or giant cell
myocarditis.
Further evaluation should include a cardiac magnetic resonance study which is seen in
Figures 13A.9.3 and 13A.9.4. The patient shows a definite aneurysmal bulge over the right
ventricular out flow tract region as well as evidence of delayed enhancement, which is also a
major criteria for ARVD/C. The finding of two major criteria places the patient in the definite
category for ARVD/C. Since he presented with VT an automatic implantable cardioverter-
defibrillator is indicated.

Figure 13A.9.3  Right ventricular outflow tract aneurysm.

Case 13A.9  u  477


Figure 13A.9.4  Delayed enhancement.

478  u  Section 13A: Arrhythmogenic Right Ventricular Dysplasia/Cardiomyopathy


SECTION 13B
Hypertrophic Cardiomyopathy (HCM)

CASE
N. A. Mark Estes III, MD 13B.1
Patient History
ECG of a 61-year-old male with intermittent atrial fibrillation.

Figure 13B.1

Question
What is the ECG abnormality?

Discussion
This ECG demonstrates left ventricular hypertrophy and diffuse T-wave inversions. The patient had
multiple normal echocardiograms and a normal cardiac magnetic resonance imaging (MRI) with
gadolinium. Five years after these ECG changes were noted, septal thickening of 1.6 cm developed
on the echocardiogram. Repeat MRI confirmed septal thickening and new gadolinium
enhancement in the midseptum, diagnostic of hypertrophic cardiomyopathy (HCM). ECG changes
can develop prior to imaging changes in HCM.

ECG Masters’ Collection: Favorite ECGs from Master Teachers Around the World © 2017 Mohammad Shenasa, Mark E. Josephson,
N. A. Mark Estes III, Ezra A. Amsterdam, Melvin Scheinman. Cardiotext Publishing, ISBN: 978-1-942909-08-8.   479
CASE
13B.2 Robert Lemery, MD

Patient History
A 58-year-old male was seen in consultation on the surgical service. The patient, an avid marathon
runner, had gotten up during the night to eat and have a glass of water. While he was standing in
his kitchen, he had syncope and found himself on the floor. He got up, went back to bed, and a few
hours later got up not feeling well, and had diaphoresis and another syncopal event. The paramedics
were called; on arrival at the hospital, the patient had free fluid in the abdomen, and underwent
emergency surgery for a lacerated spleen.

Figure 13B.2.1

Question
The most striking findings on the electrocardiogram include
1. An epsilon wave
2. A pseudo-delta wave
3. Abnormalities of repolarization
4. Interatrial block

Answer
3

480  u  Section 13B: Hypertrophic Cardiomyopathy (HCM)


The admission ECG (Figure 13B.2.1) shows sinus rhythm with repolarization changes consistent
with early repolarization.1 However, the patient also had marked T-wave changes, more
prominent on a subsequent ECG (Figure 13B.2.2), which could be secondary to various
abnormalities, including ischemia, electrolyte disorders, cardiomyopathy, and post-tachycardia.
2D echocardiograms showed hypertrophic cardiomyopathy of the apical form; the septum
measured up to 1.4 cm, without obstruction, and with apical hypertrophy and severe LA dilation
(100.7 mL).

Figure 13B.2.2

We could not exclude that the patient may have had cardiogenic syncope associated with a
ventricular tachy-arrhythmia, likely associated with repolarization abnormality. The patient
underwent defibrillator implantation without receiving any shocks for VT during four years of
follow-up. Subsequent ECGs over time have shown various repolarization abnormalities. Figure
13B.2.3 shows another patient with the apical form of cardiomyopathy (apical wall thickness of 17
mm, septum 12 mm) who has not had a rhythm disorder.

Case 13B.2  u  481


Figure 13B.2.3

References
1. Haïssaguerre M, Derval N, Sacher F, et al. Sudden cardiac arrest associated with early repolarization. N. Engl. J.
Med. 2008;358:2016–2023.
2. Dumont CA, Monserrat L, Soler R, et al. Interpretation of electrocardiographic abnormaities in hypertrophic
cardiomyopathy with cardiac magnetic resonance. Eur. Heart J. 2006;27:1725–1731.

482  u  Section 13B: Hypertrophic Cardiomyopathy (HCM)


CASE
Robert J. Myerburg, MD 13B.3
Patient History
The patient is a 48-year-old Caucasian male of European descent who has had a renal transplant
and is referred because of an abnormal ECG in the absence of hypertension or known heart disease.
He denied any symptoms of ischemic heart disease, heart failure, or cardiac arrhythmias.
A nuclear stress test was negative for evidence of transient or fixed perfusion defects, but an
echocardiogram demonstrated the apical variant of hypertrophic cardiomyopathy, with a maximum
wall thickness of 1.9 cm in the distal septum. A cardiac MRI revealed minimal delayed
hyperenhancement in the mid-distal septum (<3%).

Figure 13B.3.1  Used with permission from Junttila MJ, Castellanos A, Huikuri HV, et al. Risk markers of sudden cardiac death in standard
12-lead electrocardiograms. Ann. Med. 2012;44:717–732.

Question
Which of the following statements is correct?
1. This patient is at high risk for life-threatening arrhythmias and he should receive an ICD.
2. The electrocardiographic pattern is not typical for the apical variant of hypertrophic
cardiomyopathy.
3. This electrocardiographic pattern is a common, nonspecific pattern in renal transplant patients,
even in the absence of hypertension.

Case 13B.3  u  483


4. In contrast to the obstructive and mid-myocardial forms of hypertrophic cardiomyopathy, apical
hypertrophic cardiomyopathy rarely has a genetic basis.
5. Apical ACM is no longer considered an entity that dominates in Asian populations.

Discussion, Interpretation, and Answer


The correct answer is 5. Early citations regarding this form of HCM suggested that it was dominant
in Asian populations, but is now recognized that it is worldwide distribution although the relative
incidences are not clear. This variant of HCM generally has a lower risk of sudden death than
obstructive or mid-myocardial forms, although such patients should be followed over the years for
any progression of disease or expression of arrhythmias.
Finally, the ECG pattern demonstrated in this patient is typical for the apical variant of
hypertrophic cardiomyopathy, although there may be overlap with other patterns in more severe
cases.

484  u  Section 13B: Hypertrophic Cardiomyopathy (HCM)


CASE
Carlos Alberto Pastore, MD, PhD
Horácio Gomes Pereira Filho, MD 13B.4
Patient History
A 46-year-old male with complaints of tiredness and palpitations at major effort. He denies familial
or personal previous history of disease. No use of medications.

Physical Examination
Alert, oriented, hydrated, afebrile, eupneic, BP 120/80 mmHg, HR 82 bpm. No cardiovascular or
respiratory abnormalities. No abnormality either in abdomen or lower limbs.

Complementary Exam

Figure 13B.4.1  ECG/VCG (at 18 years old).

ECG
Tall R waves in DI, DII, DIII, aVF, V3 to V6. Tall S waves in V1 and V2. Q waves in DII, DIII, and aVF.

Case 13B.4  u  485


VCG
QRS loop with counterclockwise rotation in the transverse plane, directed forward and leftward.
Note the high voltages of QRS loops in both the transverse and the frontal planes.

Echocardiography (at 24 years old)


Severe obstructive, asymmetric septal hypertrophy, septum 3.2 cm; posterior wall 1.4 cm; septum/
wall ratio = 2.2 cm.

Echocardiography (at 40 years old)


Septum and posterior wall 0.9 cm; LVDD 10.2 cm; LVSD 9.2 cm; left atrium 5.2 cm; ejection fraction
26%; severe dilated cardiomyopathy. Diffuse left and right ventricles hypokinesia.

Cardiopulmonary Exercise Test


Severely limited physical capacity, consistent with cardiac limitation.

Holter-ECG
Nonsustained ventricular tachycardia.

Coronary Cineangiography (at 40 years old)


Marked hypertensive pressures of right chambers; normal coronary arteries; dilated left ventricle,
with diffuse 4+/4 hypokinesia, ejection fraction 18%.

Signal-averaged Electrocardiogram (SAECG)


Negative.

Gated-SPECT (at 25 years old)


Left ventricle hypertrophy, apical hypokinesia; left ventricle ejection fraction 76%.

Gated-SPECT (at 38 years old)


Remarkable diffuse hypokinesia. Anterior- and inferior-apical akinesia; left and right ventricle
ejection fractions: 22% and 17%, respectively.

Diagnosis
Myocardial hypertrophy (hypertrophic cardiomyopathy)

Evolution
Patient was treated with digoxin, spironolactone, furosemide, valsartan, amiodarone, carvedilol,
which kept him asymptomatic for a long time. After one year, he progressed to dilatation of the
heart chambers and congestive heart failure (CHF). The following year, he was admitted to hospital
due to decompensated CHF, then his clinical condition worsened and he was referred to evaluation

486  u  Section 13B: Hypertrophic Cardiomyopathy (HCM)


for cardiac transplantation. Admitted again due to another decompensated CHF, he progressed to
refractory cardiogenic shock and eventually died.

Discussion
Hypertrophic cardiomyopathy (HCM) is caused by a mutation of the sarcomeric proteins, which
may result in myocardial hypertrophy, myocyte disarray and fibrosis. It is characterized by
increased thickness of ventricular wall, with no dilatation, in the absence of any heart or systemic
condition that might lead to hypertrophy. The left ventricle cavity can be normal or decreased in
volume, and the involvement can be either symmetrical (concentric) or asymmetrical. HCM can be
further divided, hemodynamically, into obstructive and nonobstructive forms. Its prevalence in the
overall adult population is around 1:500. Annual mortality is about 2%–3% in adults, and 6% in
children. It has a familial (autosomal dominant) origin in 55% of the cases, and is usually
asymptomatic. The abnormal stiffness of the left ventricle during diastole results in poor
ventricular filling, leading to increased left atrial, pulmonary venous and capillary pressures, which
causes dyspnea, precordial pain, and sometimes syncope. Among its complications are the
obstruction of the left ventricle tract, diastolic dysfunction, ischemia, sudden death and left
ventricle failure (the latter occurring in less than 5% of patients). Risk factors for sudden death from
HCM include ventricular fibrillation, both nonsustained and sustained ventricular tachycardia,
family history of sudden death, syncope, ventricular thickness >30 mm, BP fall >25 mm in the
exercise test among patients <50 years of age. The ECG of patients can be normal in 25% of patients.
The most common findings are: evidence of left ventricle enlargement, with QRS of large
amplitudes in intermediate precordial leads; giant negative T waves are common in the apical
hypertrophic cardiomyopathy, as it was reported by Japanese investigators; deep Q waves in inferior
and/or lateral leads; leftward deviation of the QRS axis; short PR interval; and ventricular
arrhythmias, such as ventricular tachycardia.

Significance of the Diagnosis


A detailed observation of the electrocardiogram can give us additional information to help in the
diagnosis of HCM.

Reference
1. Priori SG, Zipes DP. Sudden Cardiac Death: A Handbook for Clinical Practice. Malden, MA: Blackwell; 2005.

Case 13B.4  u  487


CASE Mohammad Shenasa, MD

13B.5 Shahriar Heidary, MD


Hossein Shenasa, MD, MS

Patient History
A 91-year-old male with history of hypertension and an abnormal ECG is referred for occasional
episodes of chest pain.

Vital Signs
Heart rate: 66 bpm
Blood pressure 134/36 mmHg
Current medications include:
Verapamil 100 mg extended-release daily
Metropolol succinate 50 mg daily
Simvastatin 40 mg daily

Figure 13B.5.1  Initial ECG obtained at presentation (2005).

Figure 13B.5.1 shows sinus bradycardia, left ventricular hypertrophy (LVH) with significant
symmetrical T-wave inversion in leads V4 –V6, biphasic in leads V2 and V3, and T-wave inversion in
leads I–III and aVF.

488  u  Section 13B: Hypertrophic Cardiomyopathy (HCM)


Heart rate: 66 bpm
PR interval: 162 ms
QRS: 97 ms
QT/QTc: 443/464

Figure 13B.5.2   Follow-up ECG obtained 5 years after initial tracing (2010).

There is profound LVH and symmetrical deep inverted T waves in all precordial and anterior lateral
leads and less in inferior leads.
Holter monitor tracing (Figures 13B.5.3) obtained on January 2012 showed sinus bradycardia
and a short run of slow ventricular tachycardia at the rate of 110 bpm.

Figure 13B.5.3

Case 13B.5  u  489


Figure 13B.5.4

Question
What is the diagnosis?
1. Acute coronary syndrome
2. Hypertrophic cardiomyopathy
3. Apical hypertrophic cardiomyopathy
4. Severe left ventricular hypertrophy due to hypertension

Answer
The correct answer is 3. Apical hypertrophic cardiomyopathy is the most likely diagnosis, as also
suggested by the ECG. Cardiac magnetic resonance imaging is also a very useful test to confirm the
diagnosis of apical hypertrophic cardiomyopathy.
Note the spontaneous variability of ST-T wave changes between Figures 13B.5.1 and 13B.5.2.

Echocardiogram
Left atrial enlargement and left ventricle; normal systolic and diastolic chamber size. LVEF equal to
more than 70%. Apical hypertrophic cardiomyopathy. No resting left ventricular outflow gradient.
Short-and long-axis transthoracic echocardiograms are shown in Videos 13B.5.1 and 13B.5.2.

490  u  Section 13B: Hypertrophic Cardiomyopathy (HCM)


Heart rate: 84 bpm
PR interval: 172 ms
QRS: 90 ms
QT/QTc: 416/492

Figure 13B.5.5  ECG obtained 3 years after Holter monitoring (2015).

ST-T wave abnormalities are less pronounced compared to the ECG of 2010, probably due to
regression of LVH after aggressive antihypertensive therapy; however, the ECG pattern of apical
hypertrophic cardiomyopathy persists.
Cardiac catheterization revealed patent coronary arteries.

Discussion
The electrocardiogram is suggestive of left ventricular apical hypertrophy (Figure 13B.5.1) with
significant ST-T wave abnormalities, which suggests apical hypertrophic cardiomyopathy. The most
recent ECG (Figure 13B.5.2), shows ST-T wave abnormalities that are less pronounced compared to
the ECG in 2010.

Prognosis
Compared to hypertrophic cardiomyopathy with LV outflow obstruction, apical hypertrophic
cardiomyopathy has a more benign course, as shown in this case with the ECGs from 2005 to 2015.
Except for one short episode of nonsustained VT, shown in Figure 13B.5.3, there have been no other
arrhythmic events. Thus, there is not enough evidence to recommend ICD therapy for this patient.
“Apical hypertrophic cardiomyopathy in North American patients is not associated with sudden
cardiac death and has a benign prognosis in terms of cardiovascular mortality”.4

References
1. Maron BJ, Haas TS, Kitner C, Lesser JR. Onset of apical hypertrophic cardiomyopathy in adulthood. Am. J. Cardiol.
2011;108(12):1783–1787.
2. Authors/Task Force, et al. 2014 ESC guidelines on diagnosis and management of hypertrophic cardiomyopathy: The
Task Force for the Diagnosis and Management of Hypertrophic Cardiomyopathy of the European Society of
Cardiology (ESC). Eur. Heart J. 2014;35(39):2733–2779.
3. Maron BJ, Ommen SR, Semsarian C, Spirito P, Olivotto I, Maron, MS. Hypertrophic cardiomyopathy: present and
future, with translation into contemporary cardiovascular medicine. J. Am. Coll. Cardiol. 2014;64(1):83–99.
4. Eriksson M, Sonnenberg B, Woo A, et al. Long-term outcome in patients with apical hypertrophic cardiomyopathy.
J. Am. Coll. Cardiol. 2002;39(4):638–645.

Case 13B.5  u  491


Video Legends
Video 13B.5.1 Short axis shows concentric LVH
Video 13B.5.2 Long axis 3 chamber view shows concentric LVH and pronounced apical
hypertrophy consistent with apical hypertrophic cardiomyopathy

492  u  Section 13B: Hypertrophic Cardiomyopathy (HCM)


Mohammad Shenasa, MD CASE
Shahriar Heidary, MD
Hossein Shenasa, MD, MS 13B.6
Patient History
A 73-year-old male initially presented with rapid palpitations and near-syncope.

Heart rate: 58 bpm


PR interval: 244 ms
QRS duration: 131 ms
QT/QTc: 376/369 ms

Figure 13B.6.1

Figure 13B.6.1 shows first-degree AV block right bundle branch block morphology, right axis
deviation, and QRS fractionation (see arrows).
Due to the rapid nonsustained VT, right bundle branch block morphology, and syncope, the
patient underwent diagnostic electrophysiological studies that revealed easily inducible ventricular
tachycardia which degenerated to ventricular fibrillation. Patient underwent a single-lead ICD
implantation.
Interrogation of his device also demonstrated sinus rhythm, episodes of nonsustained
ventricular tachycardia, with no ICD therapies noted.
The echocardiogram in 2014 showed mild left atrial enlargement and mild concentric left
ventricular hypertrophy with normal left ventricular ejection fraction (65%–70%). The diastolic
filling pattern is consistent with impaired relaxation. Short- and long-axis echocardiograms are
shown in Videos 13B.6.1 and 13B.6.2.

Case 13B.6  u  493


Heart rate: 73 bpm
PR interval: 215 ms
QRS duration: 150 ms
QT/QTc: 376/586 ms

Figure 13B.6.2

Figure 13B.6.2 shows first-degree AV block, right bundle branch block morphology and QRS
fractionation.

Question
This electrocardiogram is consistent with:
1. Right bundle branch block morphology
2. Brugada variant
3. Hypertrophic cardiomyopathy variant
4. Left ventricular hypertrophy

Answer
The correct answers are 2 and 3.

Discussion
The patient presentation and ECG fits both Brugada as well as mild HCM. QRS fractionation in
pericordial leads has been reported in both conditions and is also an independent marker for
ventricular tachyarrhythmias and SCD.1 The ECG is not suggestive of a classical Brugada-type ECG
with ST segment elevation in leads V1 to V4. Similarly, the echocardiogram was also suggestive of
HCM with mild LVH without significant septal hypertrophy.

Reference
1. Shenasa M, Shenasa H. Electrocardiographic Markers of Sudden Cardiac Death in Different Substrates. In: Shenasa M,
Josephson ME, Estes NA, eds. ECG Handbook of Contemporary Challenges. Minneapolis, MN, Cardiotext; 2015:83-105.

Video Legends
Video 13B.6.1 Short axis of mild left ventricular hypertrophy
Video 13B.6.2 Long axis 3 chamber view of hypertrophic cardiomyopathy

494  u  Section 13B: Hypertrophic Cardiomyopathy (HCM)


SECTION 13C
Dilated Cardiomyopathy (DCM)

CASE
N. A. Mark Estes III, MD 13C.1
History
ECG of a 55-year-old male with a dilated cardiomyopathy and Class II heart failure.

Figure 13C.1.1

Questions
What is the ECG abnormality? How is this ECG related to the patient’s cardiomyopathy?

Discussion
The ECG shows a long R-P tachycardia with inverted P waves in leads II, III, and aVF. Ambulatory
monitoring showed persistent tachycardia with rates that varied between 96 and 152 bpm. An
electrophysiology study demonstrated that the patient had the permanent form of junctional
reciprocating tachycardia (PJRT) with a decremental bypass tract conducting retrograde from the
right ventricular apex to the os of the coronary sinus. This arrhythmia is commonly associated with
a tachymyopathy. Ablation of the atrial insertion at the coronary sinus os resulted in termination of
the tachycardia, maintenance of normal sinus rhythm, and normalization of left ventricular function.

ECG Masters’ Collection: Favorite ECGs from Master Teachers Around the World © 2017 Mohammad Shenasa, Mark E. Josephson,
N. A. Mark Estes III, Ezra A. Amsterdam, Melvin Scheinman. Cardiotext Publishing, ISBN: 978-1-942909-08-8.   495
CASE
13C.2
Nishant Verma, MD, MPH
Bradley P. Knight, MD

Patient History
A 70-year-old male with a nonischemic dilated cardiomyopathy and advanced heart failure is
admitted to the hospital with worsening congestive heart failure symptoms. He is treated with
guideline directed medical therapy. An echocardiogram showed severe global left ventricular
dysfunction and dyssynchrony, but no pericardial effusion. A 12-lead ECG is shown
(Figure 13C.2.1).

Figure 13C.2.1  12-lead ECG showing alternation in the QRS morphology in a patient with heart failure.

Question
What is the explanation for the alternating changes in the QRS complex?

Discussion
The differential diagnosis for beat-to-beat changes in the QRS morphologies during sinus rhythm is
the alternating change in ventricular activation caused by an intraventricular conduction delay,
ventricular preexcitation, ventricular ectopy, ventricular pacing, or electrical alternans associated
with cardiac tamponade. In this example, there is a left bundle branch block (LBBB) pattern with an
alternating change in the QRS morphology with and without left axis deviation (LAD). The
presence of a fixed and normal PR interval excludes alternating ventricular preexcitation from
activation over an accessory pathway, ventricular pacing, or late-coupled ventricular ectopy. The
absence of a history of a large pericardial effusion rules out electrical alternans due to movement of
the heart in the pericardial space. Therefore, the ECG is consistent with LBBB and alternating LAD.

496  u  Section 13C: Dilated Cardiomyopathy (DCM)


Alternating RBBB and LBBB is not uncommon. It is a sign of advanced conduction disease and
is an indication for a permanent pacemaker. Block in the left bundle is usually associated with a
normal axis without alternation, but in some patients can be associated with LAD. The presence of
LAD suggests more advanced conduction disease and is thought to be related to more severe heart
disease and a worse prognosis. Note that in the present example, the QRS duration increases
slightly when the axis is leftward. Progression from a normal axis to a leftward axis in an individual
patient has been reported.1 However, some studies have shown that LAD is not associated with a
further reduction in systolic function in patients with LBBB.2
The presence of alternating LAD in an individual patient with LBBB is rare but has been
reported.3 The mechanism of the changing axis is not known. Potential explanations include
predivisional LBBB with alternating left anterior fascicular block or alternation of the earliest site of
ventricular activation from the right ventricular apex when the axis is leftward (as occurs with right
ventricular apical pacing), to the right ventricular septum (as occurs with right ventricular septal
pacing).

References
1. Patanè S, Marte F, Dattilo G, Sturiale M. Acute myocardial infarction and left bundle branch block with changing
axis deviation. Int. J. Cardiol. 2012;154(3);e47–e49.
2. Das MK, Cheriparambil K, Bedi A, et al. Prolonged QRS duration (QRS ≥ 170 ms) and left axis deviation in the
presence of left bundle branch block: A marker of poor left ventricular systolic function? Am. Heart J.
2001;142(5):756–759.
3. Ranginani A, Doshi V, Denes P. Left bundle branch block with changing QRS morphology. Pacing Clin.
Electrophysiol. 2000;23:522–524.

Case 13C.2  u  497


CASE
13C.3 Carlos Alberto Pastore, MD, PhD
Nelson Samesima, MD, PhD

Patient History
The patient was an asymptomatic 24-year-old male. Patient had been receiving treatment for a
diffuse dilated cardiomyopathy in a local public health service ambulatory for the last 8 years.

Medications
Angiotensin-converting-enzyme (ACE) inhibitor, amiodarone, nebivolol, acetylsalicylic acid.

Clinical History
Tachycardic palpitations for the last 2 years, currently asymptomatic. Physical examination was
normal.

Echocardiography
Left atrium: 35 mm; LVDD: 49 mm; septum: 10 mm; posterior wall: 9 mm; diffuse hypokinesia.

Holter-ECG
Nonsustained ventricular tachycardia—three episodes.

Figure 13C.3.1

Right ventricular enlargement, septal hypertrophy, ventricular preexcitation, dextrocardia,


anteromedial divisional block.

Conclusion
Duchenne Muscular Dystrophy

498  u  Section 13C: Dilated Cardiomyopathy (DCM)


Discussion
The prominent R waves in V1, according to the literature, any of the following:
1. Right ventricular enlargement
2. Right bundle branch block
3. Ventricular preexcitation
4. Infarction in dorsal area
5. Septal hypertrophy
6. Anteromedial fascicular block
7. Normal variant
Duchenne muscular dystrophy has a recessive autosomal dominant inheritance. It is linked to the X
chromosome and affects 1:3500 individuals at birth. Most of the deaths occur before 20 years of
age, 10% due to cardiac causes.
Electrocardiographic alterations, which are found in 93% of all cases, are:
- According to Santos MA et al.:1
• 55.7% right bundle-branch conduction disorder.
•• 54.9% ventricular repolarization alterations.
•• 37.4% abnormal QS in inferior/high lateral walls.
•• 35.8% prolonged QTc.
•• 29.7% tall R wave in V1 (lateral myocyte degeneration with fibrosis and fat deposition).
•• 21.3% abnormal ECG.
- And according to James S et al.:2
• 68% left ventricle involvement.
•• 22% right ventricle involvement.
•• 17% biventricular involvement.
•• 16% ↑ mean precordial voltages.
•• 3% pathological ST/T.
•• 4% sinus tachycardia.
•• 4% QTc > 440 ms.
•• 4% short PR interval.

References
1. Santos MA, Costa F de A, Travessa AF, et al. Duchenne muscular dystrophy: Electrocardiographic analysis of 131
patients. Arq. Bras. Cardiol. 2010;94(5):620–624.
2. James J, Kinnett K, Wang Y, et al. Electrocardiographic abnormalities in very young Duchenne muscular dystrophy
patients precede the onset of cardiac dysfunction. Neuromuscul. Disord. 2011;21(7):462–467.

Case 13C.3  u  499


SECTION 13D
Chagas Cardiomyopathy

CASE
Carlos Alberto Pastore, MD, PhD
Horácio Gomes Pereira Filho, MD 13D.1
Patient History
The patient was a 63-year-old married, Brazilian female from small town in the state of São Paulo
who worked as a cleaner.
Patient complained of dyspnea on moderate effort, which had started 6 months prior. Two
months ago, she started to present dyspnea on mild effort associated to nocturnal episodes of
paroxysmal dyspnea and orthopnea. About 15 days ago, she noticed edema of her lower limbs,
which progressed to the area of her thighs.

Physical Examination
She presented with regular general condition, pale +/4+, anicteric, acyanotic. On cardiovascular
examination, she had regular rhythm with holosystolic murmur ++/4+ with mitral focus radiating
to the left axillary area, jugular stasis at 45°, symmetrical arterial pulses, blood pressure 90/60
mmHg, heart rate 64 bpm. Pulmonary examination revealed crepitant rales in both pulmonary
bases, respiratory rate 18/min. Abdominal examination revealed no palpable masses, liver with
enlargement 4 cm from the right costal border, painful at palpation. Extremities: edema lower limbs
up to the thighs. Neurological examination: no relevant findings.

Medical History
Former smoker; exploratory laparotomy due to acute obstructive abdomen; Chagas disease; chagasic
megaesophagus.

Family History
Mother died due to malignant ovarian neoplasm. Father died in car accident. Two brothers are
carriers of Chagas disease.

ECG Masters’ Collection: Favorite ECGs from Master Teachers Around the World © 2017 Mohammad Shenasa, Mark E. Josephson,
N. A. Mark Estes III, Ezra A. Amsterdam, Melvin Scheinman. Cardiotext Publishing, ISBN: 978-1-942909-08-8.   501
Figure 13D.1.1  Sinus rhythm, right bundle-branch block, and anterior-superior fascicular block.

Chest X-Ray (PA)


Moderate cardiomegaly, enlarged pulmonary hila, pulmonary vascular network.

Transthoracic Echocardiography
Dilated cardiomyopathy with important involvement of left and right ventricles; mild mitral
insufficiency.

Test for Detection of Antibodies Against Trypanosoma cruzi (Chagas Disease)


Reagent enzymatic immunoassay; reagent to indirect immunofluorescence.

Syndrome Diagnosis
Congestive heart failure.

Ethiology
Dilated cardiomyopathy of chagasic origin.

Evolution and Management


Initiation of ambulatory clinical compensation of congestive heart failure with progressive increase
in captopril doses associated with furosemide and spironolactone. After clinical improvement, she
was started on carvedilol (6 months after initiating ACE inhibitor). Currently, patient is dyspneic on
moderate effort, with no signs of pulmonary or systemic congestion.

Discussion
Chagas is an infective disease caused by the Trypanosoma cruzi protozoan, which is characterized
by involvement of the cardiac and gastrointestinal systems. Chagas disease is one of the major
causes of infective myocarditis in Latin America. It strikes about 25% of all infected patients. The

502  u  Section 13D: Chagas Cardiomyopathy


most frequent clinical manifestations in chagasic cardiopathy are: abnormalities of intraventricular
conduction; ventricular arrhythmias, sinus node dysfunction; segmental lesions in the left ventricle;
left ventricular dilatation and dysfunction, either with or without heart failure.
Some studies have shown that the incidence of intraventricular conduction disorders is
significantly higher in individuals with positive serology for Chagas disease, when compared with
those with negative serology. The most frequently found abnormality in this population is right
bundle branch block (RBBB), followed by left anterior superior fascicular block (LASFB), and by the
association of both.
In this case, the analysis of the rest ECG shows the presence of right bundle branch block
(widening of QRS complex > 120 ms, with rSR′ morphology in V1 and slurring of S waves in the left
lateral leads I, aVL, V5, and V6), in addition to LASFB (QRS complex axis deviation in the frontal
plane upwardly and to the left ≥30°, with rS morphology, with S in II < S in III).

Significance
Chagas disease is still a highly prevalent infection in Brazil, striking about 4–6 million people, most
of whom will develop dilated cardiomyopathy. Its most common electrocardiographic manifestation
is the presence of RBBB associated to LASFB.
These patients must be closely followed, since Chagas disease may progress to its
arrhythmogenic form, which is associated with sudden death from complex ventricular arrhythmias
originated by scar tissue (fibrosis) located in the left ventricle. Additionally, Chagas disease and its
complications have moved progressively northward to Central America, Mexico, and the United
States in recent decades.

References
1. Maguire JH, Hoff R, Sherlock I, et al. Cardiac morbidity and mortality due to Chagas’ disease: Prospective
electrocardiographic study of a Brazilian community. Circulation. 1987;45:1040–1045.
2. Ianni BM, Arteaga E, Frimm CC, et al. Chagas’ heart disease: Evolutive evaluation of electrocardiographic and
echocardiographic parameters in patients with the indeterminate form. Arq. Bras. Cardiol. 2001;77:59–62.
3. Jurado FS, Aranda MM, Becerril NH, et al. Electrocardiographic findings in Mexican chagasic subjects living in high
and low endemic region of Trypanosoma cruzi infection. Mem. Inst. Oswaldo Cruz. 2003;98(5):605–610.

Case 13D.1  u  503


CASE
13D.2 Andrés Ricardo Pérez-Riera, MD, PhD

Patient History
A 33-year-old Caucasian male who previously lived in an endemic area for Chagas disease. He
complained of atypical precordial pain.

Physical Examination
Nothing relevant.

Complementary Examinations
1. Normal chest x-ray and echocardiogram: posterior anterior and lateral, normal.
2. 24-hour Holter monitoring: permanent right bundle block pattern without premature ventricular
contractions or other arrhythmia.
3. Positive serological tests for Chagas disease.

Figure 13D.2.1  ECG performed at the first consultation.

ECG Diagnosis
Complete right bundle branch block (CRBBB) associated to left posterior fascicular block (LPFB).
The diagnosis of CRBBB is made by the following criteria: QRS duration > 120 ms in the presence of
supraventricular rhythm; lead V1 with monophasic R wave with notch in the ascending portion of
the QRS complex; (M-shaped QRS complex equivalent to rsR′ CRBBB). Prolonged ventricular
activation time or intrinsicoid deflection in V1 > 70 ms; ventricular repolarization (ST-T) opposite to
the greater final deflection of QRS with asymmetrical T wave; broad final S wave in left leads I, aVL,

504  u  Section 13D: Chagas Cardiomyopathy


V5, and V6. The diagnosis of LPFB is based on the following criteria: SÂQRS shift to the right: +120°
(isodiphasic aVR) in absence of vertical heart, right ventricular hypertrophy (RVH) or lateral
myocardial infarction; inferior leads II, III, and aVF with characteristic qR pattern; SI-Q3 pattern; R
wave in inferior leads of increased voltage; III R wave voltage > 15 mm; RIII > RII; R wave of II and
aVF with notch in the descending ramp and complexes of the rS type in I and aVL.

Observation
Clinically, vertical heart and/or RVH, or lateral myocardial infarction were ruled out.
Figure 13D.2.2 shows the ECG/VCG performed approximately one year later. A new
echocardiogram and chest x-rays showed normal parameters with no changes from the previous year.

Figure 13D.2.2  ECG/VCG correlation.

Frontal Plane ECG Diagnosis


LPFB: SÂQRS shift to the right: +160° (positive aVR) in absence of vertical heart, RVH or lateral MI;
inferior leads III, aVF, and II with characteristic qR pattern; SI-Q3 pattern; R wave in inferior leads
of increased voltage; III R wave voltage > 15 mm; R III > RII; R wave of II and aVF with notch in the
descending ramp and complexes of the rS type in I and aVL.

VCG Diagnosis
• Vector of initial 10 to 20 ms heading above and to the left (near –45°)
• QRS loop, with clockwise rotation.
• Maximal vector near +130°
• Almost all the loop is located below the X line (0 to ±1800) in the inferior quadrants

Case 13D.2  u  505


• 40% of the loop located in the right quadrants because of an association with CRBBB.
• Afferent limb heading below and slightly to the left, and the efferent one to the right.
• Terminal portion of the QRS loop (vector from 60 to 140 ms) with RECD. It reaches the right
superior quadrant
• QRS loop duration > 120 ms because LPFB + CRBBB

Horizontal Plane ECG Diagnosis


The QRS axis increases its shift to the right even more: the aVR lead that was isodiphasic on ECG-1
(qr), in this ECG-2 is predominantly positive (qR) indicating greater shift of QRS axis to the right
(now it is in +160°). The ECG modifications in the frontal plane are very slight: The LSFB pattern
remains SI-QIII, R waves in inferior leads with very increased voltage: R of III> 15 mm, RIII > RII,
and aVF with notch in the descending ramp.
The precordial leads clearly show expressive changes unlike the frontal plane where we only see
a greater shift of QRS axis to the right.
In V1, R wave with a greater voltage and preceded by a small Q wave: qR type complexes, QRS
complexes from V2 to V4 become predominantly positive, indicating anterior dislocation of the QRS
forces on the horizontal plane: prominent anterior QRS forces (PAF), Rs pattern in V2 and V3 with
decreasing R from V5 to V6. The absence of initial Q waves in left leads V5, V6, and I is due to the
absence of middle first septal vector, vector 1 or Peñaloza & Tranchesi vector.

Comments
An important decrease in S depth is observed in V5 and V6 in Figure 13D.2.2. This fact may be
attributed to the coincidence of the LSFB forces against the right final forces of the complete RBBB.

Conclusions
In Figure 13D.2.2, we should conclude that this is a trifascicular block: RBBB+LPFB+LSFB which
occur in a tetrafascicular intraventricular system: RBB, left anterior fascicle, left posterior fascicle,
and left septal fascicle.

VCG Diagnosis
First 10-ms vector directed to the back and leftward because of the absence of middle first septal
vector, vector 1, vector 1AM (anteromedial) or Peñaloza & Tranchesi vector.
First middle septal vector: QRS loop located predominantly in the anterior quadrants (more in
the left anterior one), rounded QRS loop totally dislocated to the front (prominent anterior forces),
clockwise rotation, right end conduction delay (RECD) in the right anterior quadrant. T loop
opposite to QRS loop directed to the back.

Right Sagittal Plane VCG Diagnosis


Most of the QRS loop located in the anterior quadrants PAF, QRS loop of counterclockwise
rotation, maximal vector to the front, end conduction delay (ECD) in the top quadrants, T loop
heading to the back.

506  u  Section 13D: Chagas Cardiomyopathy


Discussion
Like left posterior fascicular block (LPFB), the presence of left septal fascicular block (LSFB) is of
clinical electrocardiographic significance. It is necessary to rule out the other causes of prominent
QRS anterior forces including those listed below:
1. Normal subjects: prominent QRS anterior forces are observed in only 1% of normal subjects.12
• Normal variant with marked counterclockwise rotation of the heart around the longitudinal
axis of the heart, resulting in a shifting of the transition area (R=S) early, i.e. to the right of
the precordial lead V2.17
• Athlete’s heart.25
2. Misplaced precordial leads.10
3. Previous strictly posterior, dorsal, high posterobasal myocardial infarction (MI).13 Current
lateral MI.1
4. Right ventricular hypertrophy (RVH).
5. Diastolic left ventricular hypertrophy (LVH), volumetric or eccentric LVH, secondary to septal
hypertrophy (magnitude of increase of 1AM vector) and counter clockwise heart rotation
around the longitudinal axis.2
6. Combined or biventricular hypertrophy.6
7. Complete right bundle branch block (CRBBB).4
8. Pre-excitation variant of Wolff-Parkinson-White (WPW) type A.5
9. Hypertrophic cardiomyopathy (HCM), both obstructive and nonobstructive forms.19
10. Progressive muscular dystrophy of childhood, Duchenne’s cardiomyopathy, Duchenne’s
muscular dystrophy, X-linked muscular dystrophy, pseudo-hypertrophic muscular dystrophy,
childhood muscular dystrophy.24
11. Endomyocardial fibrosis.22
12. Dextroposition. Example: left pneumonectomy.18
13. LSFB.
14. A combination of the above.

Possible Etiologies of LSFB


Literature data and our own experience have identified the following etiological causes for LSFB:
• Chronic Chagas cardiomyopathy. Predominant in Latin America countries.14
• Coronary artery disease (CAD): critical lesion of left anterior descending (LAD) coronary artery
and/or its septal perforating branches before the first septal branch (S1).14 This is the main cause
in the developing countries.
• Coronary artery disease with Wellens’ syndrome.21
• Non-obstructive HCM.3
• Obstructive HCM.
• Aortic valve disease.20
• Diabetes mellitus.11 Magnacca et al. described the presence of LSFB pattern on ECG of patient
with diabetic mellitus. The authors attribute the dromotropic disorder to involvement of the
microcirculation of the intraventricular septum.

Case 13D.2  u  507


• Papillary muscle dysfunction16: In a case report of papillary muscle dysfunction where the
patient died suddenly, the septal fascicle of the left bundle was found to be markedly fibrotic by
serial histological examination of the intraventricular conduction system. A case of angina
pectoris with prominent QRS anterior forces later developed anteroseptal myocardial infarction.
Ischemia of the anterior wall of the left ventricle might cause prominence or absence of the
anterior QRS forces in vectorcardiogram. This prominence may be explained by LSFB, and the
absence may be caused by the loss of the electromotive forces of the heart due to myocardial
necrosis. Intermittent prominent QRS anterior forces were also observed, which was an
important finding to support the existence of this new type of fascicular block.
• Kearns-Sayre syndrome.21

Electrocardiographic Criteria8,9,14
• Normal QRS duration or with a minor increase (up to 110 ms). When associated with other
fascicular or bundle blocks, it could be ≥ 120 ms.
• FP leads with no modifications: normal QRS.
• Increased ventricular activation time or intrinsic deflection V1 and V2: ≥ 35 ms.
• R wave voltage of V1 ≥ than 5 mm.
• R/S ratio in V1 > 2.
• R/S ratio in V2 > 2.
• S wave depth in V1 < 5 mm.
• Possible small (embryonic) Q wave in V2 and V3 or V1 and V2.
• R wave of V2 > 15 mm.
• RS or Rs pattern in V2 and V3 (frequent rS in V1) with R wave “in crescendo” from V1 to V3 and
decreasing from V5 to V6.
• Absence of Q wave in left precordial leads V5, V6, and I (by absence of vector 1AM). It is necessary
to exclude incomplete or complete left bundle branch block and Wolff-Parkinson-White
syndrome.
• Intermittent prominent QRS anterior forces during hyperacute phase of MI, or during an
exercise stress test in patients with severe myocardial ischemia23 and during early atrial
extrastimuli with some degree of ventricular aberration.7
• Development of intermittent, rate-dependent q wave in V1 and V2.
• T-wave polarity most of the time, negative in right precordial leads.

Vectorcardiographic Criteria in the Horizontal Plane15,20


• QRS loop in the horizontal plane with an area predominantly located in the left anterior
quadrant (≥ 2/3 of the loop area facing the orthogonal X lead: 0° to ±180°).
• Absence of normal convexity to the right of the initial 20 ms of the QRS loop.
• Discrete dextro or rightward-orientation with moderate delay of the vector from 20 to 30 ms.
• Anterior location of the 40–50 ms vector.
• Posterior location with a reduced magnitude of the vector from 60 to 70 ms.
• Maximal vector of the QRS loop located to the right of +30°.
• Intermittent anterior displacement of QRS loop.14

508  u  Section 13D: Chagas Cardiomyopathy


• T loop with posterior orientation tendency (useful for the differential diagnosis with posterior
MI (actual inferobasal MI)).
• The QRS loop rotation may be:
• Counterclockwise: incomplete LSFB.
• Clockwise: advanced or complete LSFB or in association with complete right faciscular block,
LAFB, or LPFB.

References
1. Bayés de Luna A, Cino JM, Pujadas S, et al. Concordance of electrocardiographic patterns and healed myocardial
infarction location detected by cardiovascular magnetic resonance. Am. J. Cardiol. 2006;97(4):443–451.
doi: 10.1016/j.amjcard.2005.08.068.
2. Cabrera E, Gaxiola A. Diagnostic contribution of the vectorcardiogram in hemodynamic overloading of the heart.
Am. Heart J. 1960;60(2):296–317. doi: 10.1016/0002-8703(60)90105-8.
3. Chen CH, Nobuyoshi M, Kawai C. ECG pattern of left ventricular hypertrophy in non obstrutive hypertrophic
cardiomiopathy: the significance of the mid-precordial changes. Am. Heart J. 1979;97(6):687–695.
doi: 10.1016/0002-8703(79)90002-4.
4. Chen CH, Kawai C, Sakurai T, et al. The RSR’ pattern in right chest leads in hypertrophic cardiomyopathy:
Vectorcardiographic analysis. Jpn. Circ. J. 1980;44(9):734–739. doi: 10.1253/jcj.44.734.
5. Chung KY, Walsh TJ, Massie E. Wolff-Parkinson-White syndrome. Am. Heart J. 1965;69(1):116–133.
doi: 10.1016/0002-8703(65)90224-3.
6. Elliott LP, Taylor WJ, Schiebler GL. Combined ventricular hypertrophy in infancy: Vectorcardiographic
observations with special reference to the Katz-Wachtel Phenomenon. Am. J. Cardiol. 1963;11(2):164–172.
doi: 10.1016/0002-9149(63)90057-2.
7. Hoffman I, Mehta J, Hilsenrath J, et al. Anterior conduction delay: A possible cause for proeminent anterior QRS
forces. J. Electrocardiol. 1976;9(1):15–21. doi:10.1016/S0022-0736(76)80004-0
8. MacAlpin RN. In search of left septal fascicular block. Am. Heart J. 2002;144(6):948–956. doi: 10.1067/
mhj.2002.125503.
9. MacAlpin RN. Left septal fascicular block: myth or reality? Indian Pacing Electrophysiol. J. 2003;3(3):157–177.
10. MacKenzie R. Tall R wave in lead V1. J. Insur. Med. 2004;36(3):255–259.
11. Magnacca M, Valesano G, Rizzo G, et al. Diagnostic value of electrocardiogram in septal fascicular conduction
disorders of the left branch in diabetics. Minerva Cardioangiol. 1988;36(7–8):361–363.
12. Mattu A, Brady WJ, Perron AD, et al. Prominent R wave in lead V1: Electrocardiographic differential diagnosis.
Am. J. Emerg. Med. 2001;19(6):504–513. doi: 10.1053/ajem.2001.25776.
13. McManus K, Condos G, Lin A. Chest pain in a patient with a tall R wave in V1. BMJ Case Rep. 2014.
pii: bcr2014205923. doi: 10.1136/bcr-2014-205923.
14. Moffa PJ, Ferreira BM, Sanches PC, et al. Intermittent antero-medial divisional block in patients with coronary
disease. Arq. Bras. Cardiol. 1997;68(4):293–296.
15. Moffa PJ, Sanches PCR. Tranchesi electrocardiograma normal e patológico. Editora Roca Ltda.2001;Capítulo
19:413–461.
16. Nakaya Y, Hiasa Y, Murayama Y, et al. Prominent anterior QRS force as a manifestation of left septal fascicular
block. J. Electrocardiol. 1978;11(1):39–46. doi: 10.1016/S0022-0736(78)80028-4.
17. Paparella N, Alboni P, Cappato R, et al. Prominent anterior QRS forces: Clinical, electrocardiographic and
prospective study. J. Electrocardiol. 1987;20(3):233–240. doi: 10.1016/S0022-0736(87)80021-3.
18. Pérez Riera AR, Ferreira C, Ferreira Filho C, et al. Electrovectorcardiographic diagnosis of left septal fascicular
block: Anatomic and clinical considerations. Ann. Noninvasive Electrocardiol. 2011;16(2):196–207.
doi: 10.1111/j.1542-474X.2011.00416.x.
19. Pérez-Riera AR, de Lucca AA, Barbosa-Barros R, et al. Value of electro-vectorcardiogram in hypertrophic
cardiomyopathy. Ann. Noninvasive Electrocardiol. 2013;18(4):311–326. doi: 10.1111/anec.12067.
20. Pérez-Riera AR, Baranchuk A. Unusual conduction disorder: Left posterior fascicular block + left septal fascicular
block. Ann. Noninvasive Electrocardiol. 2015;20(2):187–188. doi: 10.1111/anec.12185.
21. Riera AR, Kaiser E, Levine P, et al. Kearns-Sayre syndrome: Electro-vectorcardiographic evolution for left septal
fascicular block of the His bundle. J. Electrocardiol. 2008;41(6):675–678. doi: 10.1016/j.jelectrocard.2008.04.001.

Case 13D.2  u  509


22. Tobias NM, Moffa PJ, Pastore CA, et al. The electrocardiogram in endomyocardial fibrosis. Arq. Bras. Cardiol.
1992;59(4):249–253.
23. Uchida AH, Moffa PJ, Riera AR, et al. Exercise-induced left septal fascicular block: An expression of severe
myocardial ischemia. Indian Pacing Electrophysiol. J. 2006;6(2):135–138.
24. Yotsukura M, Yamamoto A, Kajiwara T, et al. QT dispersion in patients with Duchenne-type progressive muscular
dystrophy. Am. Heart J. 1999;137(4 Pt 1):672–677. doi: 10.1016/S0002-8703(99)70221-8.
25. Zema MJ. Electrocardiographic tall R waves in the precordial leads. Comparison of recently proposed ECG and
VCG criteria for distinguishing posterolateral myocardial infarction from prominent anterior forces in normal
subjects. J. Electrocardiol. 1990;23(2):147–156. doi: 10.1016/0022-0736(90)90135-O.

510  u  Section 13D: Chagas Cardiomyopathy


SECTION 13E
Takotsubo (Stress) Cardiomyopathy

CASE
Andrew D. Krahn, MD
Christian Steinberg, MD 13E.1
Patient History
A 77-year-old female patient presented to her family physician for nonspecific gastrointestinal
symptoms and a generalized feeling of being unwell. The symptoms were evolving over 3 days with
an isolated episode of self-terminating, nonspecific chest pain on the first day. The patient’s medical
history was significant for well-medicated hypertension and dyslipidemia. There was concurrent
personal stress regarding her living situation.
In the office of her family physician, the patient was hemodynamically stable and had no chest
pain. The ambulatory ECG showed significant abnormalities (Figure 13E.1.1A) when compared to a
previous unremarkable ECG on file (Figure 13E.1.1C). The patient was immediately hospitalized
with a presumptive diagnosis of acute myocardial infarction. An urgent coronary angiogram was
performed that did not show any flow-limiting coronary artery disease. The left ventriculogram
demonstrated typical signs of takotsubo (stress) cardiomyopathy that was also confirmed by
complementary imaging modalities.

ECG Interpretation
A. Sinus tachycardia at a rate of 101 bpm. Diffuse ST-segment elevation of all precordial and all
limb leads except for lead aVL. Integrating the horizontal ST depression in lead aVR, the
significant ST elevation in lead II and the absence of mirror image ST depression, the overall
ECG is suggestive of takotsubo (stress) cardiomyopathy (acute phase).
B. Control ECG recorded four months later showing sinus rhythm at a rate of 97 bpm. Note the
residual ST elevation in the majority of the precordial leads and in isolated limb leads, as well as
residual repolarization abnormalities in the precordial leads.
C. Baseline ECG of the same patient, recorded several months earlier. Sinus tachycardia at a rate of
107 bpm, with borderline criteria for left ventricular hypertrophy.

Discussion
Takotsubo (stress) cardiomyopathy is a reversible disorder characterized by transient left ventricular
dysfunction and regional wall motion abnormalities involving the left mid-ventricular ± apical
segments in the absence of flow-limiting coronary artery disease.1 Excess of circulating
catecholamines is presumed to play a major role in the pathophysiology of takotsubo (stress)

ECG Masters’ Collection: Favorite ECGs from Master Teachers Around the World © 2017 Mohammad Shenasa, Mark E. Josephson,
N. A. Mark Estes III, Ezra A. Amsterdam, Melvin Scheinman. Cardiotext Publishing, ISBN: 978-1-942909-08-8.   511
Figure 13E.1.1  Takotsubo (stress) cardiomyopathy. A. Sinus tachycardia at a rate of 101 bpm. There is diffuse ST-segment elevation
of all precordial and all limb leads except for leads aVL and aVR (arrows). ST elevation is more prominent in the precordial leads with a
maximum of 10 mm in lead V2. Marked notching of the R-descent in leads V4–V6. The ST-segment elevation in lead V1 is less prominent
compared to the other precordial leads (dashed arrow). ST-segment elevation in the inferior leads is most prominent in lead II (2.5 mm,
red arrow). Marked horizontal ST-segment depression in lead aVR (blue arrows). Absence of pathological Q waves and no loss of R waves.
Another striking aspect is the lack of mirror image ST-segment depression. Altogether, the ECG is compatible with findings during the
acute phase of takotsubo (stress) cardiomyopathy. The ECG findings would be quite atypical for acute transmural ischemia caused by an
occluded left main or left anterior descending coronary artery. B. Control ECG recorded almost four months later. Sinus rhythm at a rate of
97 bpm. There is residual mostly horizontal ST-segment elevation in the majority of the precordial leads and in isolated limb leads (arrows).
Note the presence of residual repolarization abnormalities in the precordial leads. Normal R-wave amplitudes and absence of pathological
Q waves. C. Baseline ECG of the same patient, recorded several months earlier. Sinus tachycardia at a rate of 107 bpm. Some limb lead
criteria suggesting left ventricular hypertrophy.

cardiomyopathy.2,3 Clinical presentation is characterized by acute chest pain, ST elevation and


positive cardiac biomarkers mimicking acute anterior myocardial infarction.2,3 The distinction from
myocardial infarction is often challenging and the standard 12-lead ECG is a crucial element of the
diagnostic work-up. Several studies compared ECG findings of takotsubo (stress) cardiomyopathy
with those of acute anterior myocardial infarction and reported potential discriminators.4,5
ST elevation in takotsubo (stress) cardiomyopathy is often more diffuse compared to anterior
myocardial infarction, and lead V1 typically shows little or no ST elevation.6 Also, mirror image
signs and reduction of R-wave amplitudes are typically absent in takotsubo (stress) cardiomyopathy,
whereas the presence of Q waves is variable.4,5
Another predictor with good sensitivity and specificity is the presence of ST depression in aVR
(= ST elevation in –aVR) and absence of ST elevation in V1.6 A recent study identified ST elevation
≥1 mm in lead II as a useful discriminator from acute anterior myocardial infarction with good
specificity and reasonable predictive values.7
Several of the above mentioned ECG findings in takotsubo (stress) cardiomyopathy were
documented in the present case: diffuse ST elevation, less marked in V1, absence of mirror signs,
preserved R-wave amplitudes, ST depression in aVR and ST elevation ≥ 1 mm in lead II. However,
ECG findings in takotsubo (stress) cardiomyopathy have some important limitations and should
always be interpreted in the clinical context. In the present case, the delayed presentation with

512  u  Section 13E: Takotsubo (Stress) Cardiomyopathy


normal vital signs and the ECG without Q waves or R wave loss in the presence of diffuse ST
elevation make a nonrevascularized anterior myocardial infarction unlikely.

References
1. Madhavan M, Prasad A. Proposed Mayo Clinic criteria for the diagnosis of Tako-Tsubo cardiomyopathy and long-
term prognosis. Herz. 2010;35(4):240–243.
2. Kurisu S, Kihara Y. Tako-tsubo cardiomyopathy: Clinical presentation and underlying mechanism. J. Cardiol.
2012;60(6):429–437.
3. Peters MN, George P, Irimpen AM. The broken heart syndrome: Takotsubo cardiomyopathy. Trends Cardiovasc.
Med. 2015;25(4):351–357.
4. Kosuge M, Kimura K. Electrocardiographic findings of takotsubo cardiomyopathy as compared with those of
anterior acute myocardial infarction. J. Electrocardiol. 2014;47(5):684–689.
5. Ogura R, Hiasa Y, Takahashi T, et al. Specific findings of the standard 12-lead ECG in patients with ‘Takotsubo’
cardiomyopathy: Comparison with the findings of acute anterior myocardial infarction. Circ. J. 2003;67(8):687–690.
6. Kosuge M, Ebina T, Hibi K, et al. Simple and accurate electrocardiographic criteria to differentiate takotsubo
cardiomyopathy from anterior acute myocardial infarction. J. Am. Coll. Cardiol. 2010;55(22):2514–2516.
7. Jim MH, Chan AO, Tsui PT, et al. A new ECG criterion to identify takotsubo cardiomyopathy from anterior
myocardial infarction: Role of inferior leads. Heart Vessels. 2009;24(2):124–130.

Case 13E.1  u  513


CASE
13E.2 Dan Blendea, MD, PhD
Moussa Mansour, MD

Patient History
A 75-year-old patient with known sinus node dysfunction and dual-chamber pacemaker was
recently diagnosed with rectal cancer and was admitted for dyspnea and chest pain. The ECGs
obtained over a period of 8 days (day 1, day 3, day 8) while in the hospital are shown in Figures
13E.2.1–13E.2.3. The patient had mild troponin elevation (serum TnT max 0.08 ng/mL) and no
significant coronary stenosis on CT angiogram.

Figure 13E.2.1  The QT interval at baseline was normal (QT/QTc 386/404 ms).

Figure 13E.2.2  Q-T prolonged on day 3 of hospitalization (QT/QTc 486/524 ms) due to takotsubo (stress) cardiomyopathy.

514  u  Section 13E: Takotsubo (Stress) Cardiomyopathy


Figure 13E.2.3  Q-T interval returned to normal on day 8 of hospitalization (QT/QTc 378/410 ms).

The echocardiogram on day 1 was normal, and on day 3 showed that the mid-left ventricular (LV)
cavity is hypokinetic and the LV apex is akinetic. The LV ejection fraction (EF) was 24% and on day
8 the mid and apical LV was still hypokinetic but the LVEF was improved to 54%, mainly because of
the compensatory hyperkinesis of the basal segments.

Questions
1. Why did the QT prolong?
2. Is there an increased risk of ventricular tachyarrhythmias associated with the QT prolongation?

Discussion, Interpretation, and Answers


This case was interpreted as stress cardiomyopathy. It is likely that the diagnosis of rectal cancer
was the precipitant. It is known that within 24–48 hours from presentation, most patients with
typical LV apical involvement develop repolarization abnormalities, including diffuse T-wave
inversions and QT prolongation, the latter often being quite pronounced.1,2 Madias et al. found that
the QT prolongation in patients with stress cardiomyopathy can be associated with an increased
risk of torsades de pointes (TdP).1 The majority patients developed ventricular fibrillation (VF) and
TdP in the setting of substantial QT prolongation. The mechanism of TdP and VF in patients was a
pause-dependent, long-short sequence that is the classic precipitant of polymorphic VT seen in
acquired long QT syndrome.

Answers
1. It is postulated that the mechanism of ventricular dysfunction and QT prolongation in stress
cardiomyopathy is due to catecholamine-mediated myocardial toxicity. A similar pattern of
repolarization abnormalities has been described in other hyperadrenergic states, including
pheochromocytoma and subarachnoid hemorrhage.1,3

Case 13E.2  u  515


2. The QT prolongation in stress cardiomyopathy is associated with an increased risk of ventricular
fibrillation and torsades de pointes and the risk seems to be higher in patients with more
pronnounced QT prolongation.3 One example from this study is a patient who was discharged
home 1 day after presentation and then suffered an out-of-hospital cardiac arrest the day after
discharge, whereas the QTc remained prolonged. Cases like this point to the importance of
monitoring stress cardiomyopathy patients in a cardiac unit until severe QT prolongation
resolves.1

References
1. Madias C, Fitzgibbons TP, Alsheikh-Ali AA, et al. Acquired long QT syndrome from stress cardiomyopathy is
associated with ventricular arrhythmias and torsades de pointes. Heart Rhythm. 2011;8(4):555–561.
2. Sharkey SW, Lesser JR, Menon M, et al. Spectrum and significance of electrocardiographic patterns, troponin levels,
and thrombolysis in myocardial infarction frame count in patients with stress (tako-tsubo) cardiomyopathy and
comparison to those in patients with ST-elevation anterior wall myocardial infarction. Am. J. Cardiol.
2008;101(12):1723–1728.
3. Wittstein IS, Thiemann DR, Lima JA, et al. Neurohumoral features of myocardial stunning due to sudden emotional
stress. N. Engl. J. Med. 2005;352(6):539–548.

516  u  Section 13E: Takotsubo (Stress) Cardiomyopathy


Mohammad Shenasa, MD CASE
Shahriar Heidary, MD
Hossein Shenasa, MD, MS 13E.3
Patient History
A 60-year-old female was admitted to the emergency department with chest pain following an
intense emotional event.
Heart rate: 59 bpm
PR interval: 168 ms
QRS duration: 80 ms
QT/QTc: 602/595

Figure 13E.3.1  ECG at presentation.

This ECG shows mild ST elevation with significant T-wave inversion in anterolateral
distribution and prolonged QT interval. The ST/T-wave changes do not match the distribution of a
single coronary artery occlusion.
Heart rate: 87 bpm
PR interval: 154 ms
QRS duration: 76 ms
QT/QTc: 364/438 ms

Case 13E.3  u  517


Figure 13E.3.2  ECG obtained 2 years before the ECG at presentation (Figure 13E.3.1).

The ECG in Figure 13E.3.2 displays minor nonspecific ST-T wave changes. Otherwise, this ECG
is unremarkable.

Figure 13E.3.3  ECG obtained later in the day of presentation. Note that the ST/T wave abnormalities, T-wave inversion, and prolonged
QT persist.

518  u  Section 13E: Takotsubo (Stress) Cardiomyopathy


Figure 13E.3.4  ECG obtained the day after presentation.

The subsequent two electrocardiograms show the evolution of ST/T-wave abnormalities.

Question
These ECG patterns are consistent with:
1. Acute coronary syndrome
2. Congenital long-QT syndrome
3. Electrolyte imbalance
4. Takotsubo (stress) cardiomyopathy
The ECG within the next few days showed persistent ST/T-wave abnormalities. There was some
improvement as shown in Figure 13E.3.4. Note that the QT intervals are normalized.

Answer
The correct answer is 4. This ECG finding and the patient presentation is consistent with takotsubo
(stress) cardiomyopathy.

Case 13E.3  u  519


Figure 13E.3.5  ECG obtained a year later, for comparison.

The ECG in Figure 13E.3.5 returns to minor nonspecific ST/T-wave changes similar to Figure
13E.3.2. Note there are no pathological Q waves that rules out myocardial infarction. This favors a
diagnosis of takotsubo (stress) cardiomyopathy.

Discussion
Takotsubo (stress) cardiomyopathy is often observed in older female patients following an intense
emotional event. The echocardiogram on admission of this patient revealed left ventricular ejection
fraction at 40% with apical ballooning. This is consistent with takotsubo (stress) cardiomyopathy.
Since the important differential diagnosis is acute coronary syndrome, the patient underwent
diagnostic coronary angiography and cardiac catheterization that revealed patent left and right
coronary arteries. The left ventriculography showed decreased global function and also apical
ballooning. The differential diagnosis in acute phase, the clinical presentation, and ECG finding and
biomarkers are often similar to those of acute coronary syndrome; therefore, the correct diagnosis
and management is important.

Definition
Takotsubo (stress) cardiomyopathy (octopus pot in Japan) or apical ballooning syndrome is a form
of reversible cardiomyopathy characterized by transient LV systolic and diastolic dysfunction with a
variety of wall-motion abnormalities (most commonly apical ballooning) in the absence of
obstructive coronary artery disease. Takotsubo (stress) cardiomyopathy was first described in Japan
in 1990.
Complications of takotsubo (stress) cardiomyopathy includes congestive heart failure (20% of
cases), cardiogenic shock, ventricular arrhythmias due to prolonged QT/QTc syndrome, bradycardia
that may promote the occurrence of ventricular tachyarrhythmias, mostly torsades de pointes.
Therefore, sinus bradycardia and antiarrhythmic medications that are potentially torsadogenic

520  u  Section 13E: Takotsubo (Stress) Cardiomyopathy


should be avoided. Risk stratification for tachyarrhythmias and SCD is important: therefore, in
certain cases ICD implantation may be justified.
Echocardiogram and diagnostic cardiac catheterization and coronary angiography are shown in
Videos 13E.3.1–13E.3.5.

References
1. Templin C, Ghadri JR, Diekmann J, et al. Clinical features and outcomes of takotsubo (stress) cardiomyopathy. N.
Engl. J. Med. 2015;373(10):929–938.
2. Wright PT, Tranter MH, Morley-Smith AC, Lyon AR. Pathophysiology of takotsubo syndrome. Circ. J.
2014;78(7):1550–1558.
3. Kurisu S, Kihara Y. Clinical management of takotsubo cardiomyopathy. Circ. J. 2014;78(7):1559–1566.
4. Kosuge M, Ebina T, Hibi K, et al. Differences in negative t waves between takotsubo cardiomyopathy and reperfused
anterior acute myocardial infarction. Circ. J. 2012;76(2):462–468.
5. Gopalakrishnan M, Hassan A, Villines D, Nasr S, Chandrasekaran M, Klein LW. Predictors of short- and long-term
outcomes of takotsubo cardiomyopathy. Am. J. Cardiol. 2015;116(10):1586–1590.

Video Legends
Video 13E.3.1: Apical, three-chamber view echocardiogram showing akinesis in the mid to distal
anteroseptum and apex
Video 13E.3.2: Enlarged view of video 1
Video 13E.3.3: Coronary angiogram
Video 13E.3.4: Right coronary
Video 13E.3.5: Left coronary

Case 13E.3  u  521


SECTION 13F
Non-Compaction Cardiomyopathy

CASE
Sabine Ernst, MD, PhD 13F.1
Patient History
Patient with a confirmed diagnosis of left ventricular non-compaction without any visible
involvement of the right ventricle on cardiac magnetic resonance imaging. Patient presented with
multiple syncopal episodes and palpitations.

Figure 13F.1.1  12-lead ECG documentation of frequent monomorphic ventricular ectopy and short ventricular tachycardia runs.

ECG Masters’ Collection: Favorite ECGs from Master Teachers Around the World © 2017 Mohammad Shenasa, Mark E. Josephson,
N. A. Mark Estes III, Ezra A. Amsterdam, Melvin Scheinman. Cardiotext Publishing, ISBN: 978-1-942909-08-8.   523
Figure 13F.1.2  Local activation time map during frequent ectopy with demonstration of the focal origin from the free wall of the right
ventricle close to the tricuspid annulus.

Discussion
LVNC is a rare heart disease with controversies about all of the areas of its knowledge
(nomenclature, pathogenesis, diagnostic criteria, prognosis, or management). Arrhythmias are very
frequent in these patients, but the underlying substrate which predisposes this arrhythmogenesis is
actually unknown. LVNC and ventricular arrhythmia have been associated with worse prognosis
and outcome. Definitive treatment can be performed by catheter ablation and it has been shown
that the underlying disease can extend beyond the anatomically affected area, encompassing not
only the left ventricle but also the right.

524  u  Section 13F: Non-Compaction Cardiomyopathy


SECTION 13G
Pericarditis

Jonathan Bui, MD CASE


Femi Philip, MD
Ezra A. Amsterdam, MD 13G.1
Pericarditis has several clinical and ECG characteristics that can suggest acute myocardial
infarction (MI). It is typically associated with chest discomfort and is an important cause of ST
segment elevation. However, there are important clinical features that distinguish pericarditis from
acute MI.

Patient History
A male presented to the emergency department with chest pain. His ECG (Figure 13G.1.1)
demonstrates extensive ST elevation (arrows, Figure 13G.1.1).

Figure 13G.1.1

Questions
1. The ECG of pericarditis is commonly described as having “universal” ST segment elevation.
2. Is this description valid?
3. Why or why not?

ECG Masters’ Collection: Favorite ECGs from Master Teachers Around the World © 2017 Mohammad Shenasa, Mark E. Josephson,
N. A. Mark Estes III, Ezra A. Amsterdam, Melvin Scheinman. Cardiotext Publishing, ISBN: 978-1-942909-08-8.   525
Answer
The description “universal” ST elevation is an exaggeration of the extensive, but not complete, ST
elevation typically seen in pericarditis. It is not unusual for there to be no ST elevation in leads V1 and
aVL in pericarditis; of course, ST depression in aVR has the same significance as ST elevation in other
leads. Leads with small QRS voltage may not display significant ST elevation in pericarditis, as is seen
lead I (Figure 13G.1.1). In our patient’s ECG (Figure 13G.1.1), there is typical ST elevation in all leads
except aVL and V1, the two leads that most frequently have no ST elevation in pericarditis, and arguably
in lead I.

Question
What are some other characteristic features of the ECG in pericarditis?

Answer
The ST elevation is typically concave (“smiley”); there are no reciprocal ST changes, as in ST-segment
elevation myocardial infarction (STEMI); and PR segment depression (lead II, Figure 13G.1.1, closed
arrowhead) and elevation (aVR, Figure 13G.1.1, open arrowhead) are frequently present. However,
there are several potentially overlapping features in the ECGs of pericarditis and STEMI. Concave ST
segment morphology may also be seen in acute STEMI in contrast to the convex ST morphology
typical of STEMI. In pericarditis, elevated ST segments may occasionally be almost straight rather
than concave. The key distinguishing features between the ECGs of pericarditis and STEMI are:
1. Reciprocal ST depression is absent in pericarditis and is typical in acute STEMI;
2. ECG evidence of pericarditis is extensive, involving numerous leads, while in STEMI the ECG
changes are regional, reflecting the segment of myocardium subtended by the occluded coronary
artery;
3. PR depression (best seen in the inferior leads [lead II, closed arrowhead], and as elevation in aVR
[open arrowhead]) is frequent in pericarditis. In this regard, the PR segment has been considered
the “ST segment of the atria” and depression may thereby reflect atrial injury.
It may also be challenging to distinguish the ECG of pericarditis from that of benign early
repolarization (BER). In both cases, ST elevation is extensive and the ST segment is typically
concave. However, characteristics of BER in addition to widespread ST elevation are1,2: ST is
segment higher in the precordial than limb leads; a small notch at the J point, usually best seen in
the precordial leads; prominent T waves in the precordial leads; and a shortened Q-T interval.
It is also important to appreciate the ECG evolution in pericarditis. Figure 13G.1.1 shows the
classic changes in the acute stage. During evolution, the ST segments return to baseline and the T
waves become inverted. After several days or more, the T waves resolve to their original baseline.
This evolution may not be seen because of lack of follow-up ECGs after the acute phase.

Question
Is the ECG in Figure 13G.1.2 that of a patient with a STEMI or pericarditis?

Answer
This ECG was obtained on the following day from the patient with pericarditis described above. It
shows early evolution of the repolarization changes of pericarditis. The ST segments remain

526  u  Section 13G: Pericarditis


Figure 13G.1.2

elevated, but they have straightened in leads I–III, aVF, V2, V4 –V6, and the T waves are now inverted
in almost all leads. ECG alterations such as these have been mistaken for evidence of STEMI and
prompted misplaced fibinolytic therapy or emergency coronary angiography. Overlap and
differentiation of ECG in pericarditis from those of STEMI and BER are considered above in the
answer to Question 2. A subsequent ECG after acute pericarditis had completely resolved is normal,
as shown in Figure 13G.1.3.

Figure 13G.1.3

Case 13G.1  u  527


References
1. Turnipseed SD, Bair AE, Kirk JD, Diercks DB, Tabar P, Amsterdam EA. Electrocardiogram differentiation of benign
early repolarization versus acute myocardial infarction by emergency physicians and cardiologists. Acad. Emerg.
Med. 2006;13:961–967.
2. Wang K, Asinger RW, Marriott HJL. ST-segment elevation in conditions other than acute myocardial infarction.
N. Engl. J. Med. 2003;349:2128–2135.

528  u  Section 13G: Pericarditis


SECTION 13H
Other Cardiomyopathies

CASE
Mohammad Shenasa, MD 13H.1
Patient History: Patient 1
A 39-year-old female presented with recurrent palpitations.
Heart rate: 56 bpm
PR interval: 108 ms
QRS: 94 ms
QT/QTc: 462/445 ms

Figure 13H.1.1

ECG Masters’ Collection: Favorite ECGs from Master Teachers Around the World © 2017 Mohammad Shenasa, Mark E. Josephson,
N. A. Mark Estes III, Ezra A. Amsterdam, Melvin Scheinman. Cardiotext Publishing, ISBN: 978-1-942909-08-8.   529
Patient History : Patient 2
A 55-year-old female presented with recurrent palpitations as well as near-syncope.
Heart rate: 56 bpm
PR interval: 112 ms
QRS: 80 ms
QT/QTc: 432/416 ms

Figure 13H.1.2

Question
What is the diagnosis on the above ECGs?
1. Left ventricular hypertrophic due to hypertensive heart disease
2. Hypertrophic cardiomyopathy
3. Fabry disease with left ventricular hypertrophy
4. Aortic stenosis
5. Athletic heart

Answer
The correct answer is 3. These ECGs show Fabry disease, with short PR, left ventricular
hypertrophy, and ST/T-wave wave abnormalities. However, this pattern may overlap with other
diseases as outlined in the above question.

530  u  Section 13H: Other Cardiomyopathies


Discussion
Fabry disease is an X-linked lysosomal storage disease in which the cardiovascular manifestations
are left ventricular hypertrophy due to abnormal lysosomal storage and myocyte hypertrophy.
ECG manifestations include sinus bradycardia, short PR interval, left ventricular hypertrophy,
and diffuse ST/T-wave abnormalities with T-wave inversion. In the advanced stages and older
patients, first- and second-degree AV block, right bundle branch block pattern, and ventricular
arrhythmias may be present. Since Fabry disease is a treatable disease with enzyme replacement,
electrocardiographic abnormalities often regress and return to normal. Occasionally ventricular
arrhythmias and sudden cardiac death may occur.
Echocardiographically, Fabry disease may have a similar pattern to that of hypertrophic
cardiomyopathy; however, right ventricular involvement is more common in Fabry disease than
hypertrophic cardiomyopathy.1–3

References
1. Acharya D, Doppalapudi H, Tallaj JA. Arrhythmias in Fabry cardiomyopathy. Card. Electrophysiol. Clin.
2015;7(2):283–291.
2. Yousef Z, Elliott PM, Cecchi F, et al. Left ventricular hypertrophy in Fabry disease: A practical approach to
diagnosis. Eur. Heart J. 2013;34(11):802–808.
3. Nagueh SF. Anderson-Fabry disease and other lysosomal storage disorders. Circulation. 2014;130(13):1081–1090.

Case 13H.1  u  531


SECTION 14
Congenital Heart Diseases

CASE
Daniel J. Murphy Jr., MD
Inger Olson, MD 14.1
Introduction
Although echocardiography is the definitive diagnostic tool for diagnosing congenital heart disease,
electrocardiography can be useful for identifying patients at risk for congenital heart defects.
Right QRS axis deviation can be normal in the newborn and is not strongly associated with any
particular cardiac defect. However, left-axis deviation or superior QRS-axis deviation with a
counterclockwise frontal plane loop is almost invariably associated with a structural abnormality.

Atrial Septal Defect (ASD)


Children and adults with ASD experience variable degrees of right-sided volume overload,
particularly enlargement of the right ventricle. This is generally manifested on the ECG as a
terminal right interventricular conduction delay with an rSR′ in lead V1. Other associated findings
include right-axis deviation and right atrial enlargement or right ventricular hypertrophy, especially
in cases of long-standing volume overload or pulmonary hypertension.

Figure 14.1.1  This ECG is from a 4-year-old female with a secundum atrial septal defect. There is sinus rhythm with normal P waves
and a normal PR interval. She has a borderline right-axis deviation. The most significant finding is the presence of RSR’ in V1. However, it
should be noted that RSR’ can be a normal variant in children.

ECG Masters’ Collection: Favorite ECGs from Master Teachers Around the World © 2017 Mohammad Shenasa, Mark E. Josephson,
N. A. Mark Estes III, Ezra A. Amsterdam, Melvin Scheinman. Cardiotext Publishing, ISBN: 978-1-942909-08-8.   533
Figure 14.1.2  This tracing is from a 10-year-old male with a secundum atrial septal defect and pulmonary hypertension. He has sinus
rhythm with a normal PR interval, but there is significant right-axis deviation and signs of right ventricular hypertrophy. The RSR’ in lead
V1 includes a very tall secondary R wave. (Note that the precordial leads are recorded at half standard.) The signs of right ventricular
hypertrophy in children include: upright T wave in V1 (note: in children, T waves are normally inverted in leads V1 and V2); QR pattern in
leads V1 and V3; tall R wave in leads V1 and V4; and deep S wave in lead V6.

Endocardial Cushion Defects


Endocardial cushion defects encompass a spectrum of structural abnormalities, which range from
isolated ostium primum atrial septal defect to complete atrioventricular canal defects. All patients
with endocardial cushion defect share a common ECG finding, namely the presence of a leftward
superior QRS axis representative of a counterclockwise frontal plane QRS loop.

Figure 14.1.3  This is an ECG from a 13-month-old male with an ostium primum atrial septal defect. There is sinus rhythm with normal
PR and QRS intervals. The QRS axis is leftward with a QR pattern in lead aVL, suggesting a counterclockwise superior loop. In this case,
there is also an rSR’ in lead V1. However, there are no signs of right ventricular hypertrophy.

Figure 14.1.4  This tracing is from a newborn baby with a complete atrioventricular canal defect. The tracing demonstrates sinus rhythm
with an extreme rightward QRS axis. In a newborn, the rightward QRS axis can be normal; however, in this case the QRS axis is rightward
and superior, suggesting the presence of a structural heart defect. There is also right ventricular hypertrophy, which can also be a normal
finding for an infant 1 hour after birth.

534  u  Section 14: Congenital Heart Diseases


Tricuspid Atresia
There are other, less common causes of leftward or superior QRS axis in children. Tricuspid atresia
is associated with a leftward QRS axis. Other, less common causes include other forms of single
ventricle and anomalous left coronary artery from the pulmonary artery.

Figure 14.1.5  This ECG is from a 2-year-old male born with tricuspid atresia and normally related great arteries. His left ventricle was
normal and the right ventricle was hypertrophied but diminutive. The ECG demonstrates sinus rhythm with a leftward QRS axis (–83°).
There is also evidence of right ventricular hypertrophy with a biphasic T wave and tall R waves in lead V1 and deep S waves in lead V6.

Tetralogy of Fallot
Tetralogy of Fallot (TOF) is one of the more common forms of complex congenital heart disease.
The degree of right ventricular outflow obstruction varies along a continuum. Therefore, ECG
findings are somewhat nonspecific but usually include right ventricular hypertrophy.
Surgical repair of TOF includes patch closure of the ventricular septal defect and relief of right
ventricular outflow tract obstruction. In the past, virtually all patients who underwent surgery
developed complete right bundle branch block pattern on ECG. This was generally due to the fact
that surgery was either performed through a right ventriculotomy or included excision of a
substantial amount of right ventricular muscle. However, TOF repair is now performed at an earlier
age involving less right ventricular muscle resection and with an approach from the atrium
obviating the need for a right ventriculotomy. Therefore, there is a variable appearance of the
postoperative ECG in patients following repair of TOF.

Figure 14.1.6  This ECG from an 8-year-old male demonstrates a typical pattern in a patient with TOF after complete repair. There is
sinus rhythm and a pattern of a complete right bundle branch block (QRS duration = 138 ms) with a deep S wave in lead I and a wide
secondary R wave in lead V1. This pattern will persist throughout his lifetime.

Case 14.1  u  535


Figure 14.1.7  This is an ECG from a 5-year-old male who underwent neonatal repair of TOF. The QRS axis is somewhat indeterminate.
PR and QRS intervals are normal with no conduction delay and no bundle branch block. At the time of the ECG, he had moderate
pulmonary stenosis, and the QR pattern in lead V1 suggests right ventricular hypertrophy.

Ebstein’s Malformation
Ebstein’s malformation is a complex form of congenital heart disease that includes variable
displacement and dysplasia of the tricuspid valve. Tricuspid regurgitation is common, and the right
atrium is usually significantly enlarged. The ECG frequently demonstrates right atrial enlargement
and right ventricular conduction delay. In addition, Ebstein’s malformation is frequently
accompanied by accessory bypass tracts, both overt and concealed.

Figure 14.1.8  This is an ECG from a 9-year-old male with severe Ebstein’s malformation of the tricuspid valve. The P waves are gigantic
and dwarf the QRS complexes. In addition, the PR interval is prolonged due to intra-atrial conduction delay. The QRS complexes have
low voltage and a right bundle branch block pattern. It is worthwhile noting that an individual with Ebstein’s malformation who presents
without a conduction delay most frequently has a concealed accessory bypass tract which preexcites the right ventricle, narrowing the QRS
complex.

Figure 14.1.9  This is an ECG from a 29-year-old female with moderately severe Ebstein’s malformation of the tricuspid valve. The PR
interval is prolonged due to intra-atrial conduction delay. There are low voltage QRS complexes with a rightward QRS axis and complete
right bundle branch block pattern.

536  u  Section 14: Congenital Heart Diseases


Dextrocardia
It is not uncommon to encounter an individual with dextrocardia with or without congenital heart
disease. In cases of situs inversus with mirror-image dextrocardia, all of the abdominal and thoracic
organs are reversed and the cardiac structures are likewise reversed.

Figure 14.1.10  This is an ECG from a 3-year-old female with situs inversus totalis. Note that her P-wave axis is rightward, her QRS axis is
inferiorly directed and her T-wave axis is rightward. One might suspect limb lead reversal until one examines the precordial leads. The most
striking finding is that the QRS voltage consistently decreases from lead V1 to V6, suggesting that the lead placement is moving away from
the cardiac mass. It is recommended to perform standard precordial lead placement on all patients, even those with suspected or known
dextrocardia or situs inversus.

Figure 14.1.11  This tracing is from the same patient with right chest leads rather than left chest leads. The R-wave voltage increases
progressively from V1R to V6R. This confirms the presence of dextrocardia. The patient does have a deep S wave in V1R and a tall R wave
in V6R due to the presence of a ventricular septal defect with resulting left ventricular hypertrophy.

Pulmonary Atresia, Intact Ventricular Septum

Figure 14.1.12  This ECG is from a 20-year-old female with pulmonary atresia and intact ventricular septum who had undergone
balloon pulmonary valvuloplasty in infancy and had a subsequent pulmonary valve replacement 5 years ago. She presented with chronic
worsening pain, described as ‘stabbing,’ in the mid-sternum over her sternotomy scar. Her ECG shows sinus rhythm with a nonspecific
interventricular conduction delay and T-wave inversion in the inferior and lateral leads, which was unchanged from her previous tracings.
For patients who have undergone surgical “repair” of congenital heart defects, there may be a range of ECG abnormalities, which makes
it crucially important that the patient and all physicians have access to a “baseline” ECG.

Case 14.1  u  537


QUESTION
What congenital lesions are commonly associated with an extreme leftward QRS axis? (Circle all
that apply.)
1. Endocardial cushion defects
2. Truncus arteriosus
3. Ebstein’s anomaly
4. Tricuspid atresia
5. Coarctation of the aorta

ANSWER
1 and 4: Endocardial cushion defects and tricuspid atresia are almost always associated with
extreme leftward deviation of the QRS axis. Ebstein’s anomaly has a common association with
Wolff-Parkinson-White syndrome, and the accessory pathway can occasionally cause the QRS axis
to have an extreme leftward deviation. Truncus arteriosus and coarctation are not associated with
extreme axis deviation.

QUESTION
What findings may be present on the ECG of a patient with repaired Tetralogy of Fallot? (Circle all
that apply.)
1. Right ventricular hypertrophy
2. Left ventricular hypertrophy
3. Right bundle branch block
4. Normal ECG

ANSWER
1, 3, and 4: Right ventricular hypertrophy or right bundle branch block may be seen on the ECG
after repair of TOF. Patients may also have a normal ECG. Left ventricular hypertrophy is not seen
in this lesion. Left bundle branch block occurs in children after surgical manipulation of the left
ventricle, such as left ventricular myectomy for outflow tract obstruction. Ventricular septal defects
are repaired by placing the patch and sutures on the right side of the interventricular septum, so left
bundle branch block is not seen after this repair.

538  u  Section 14: Congenital Heart Diseases


CASE
14.2
Amit Noheria, MBBS, SM
Samuel J. Asirvatham, MD

Patient History
A 27-year-old male presents with dyspnea on walking up an incline. He had corrective cardiac
surgery for congenital heart disease at age 6. His ECG is shown in Figure 14.2.1.

Figure 14.2.1

Question
What cardiac anatomical information does this ECG reveal?

Discussion, Interpretation, and Answer


This ECG shows sinus rhythm with prolongation of the PR interval, left atrial abnormality, and an
unusual right bundle branch block (RBBB). This patient was born with tetralogy of Fallot (TOF)
and had pulmonary atresia, ventricular septal defect (VSD), overriding aorta, and right ventricular
hypertrophy. He received a Blalock-Taussig shunt (subclavian artery to pulmonary artery) at age 5,
and subsequently at age 6 had a complete repair with VSD patch closure and conduit placement

Case 14.2  u  539


from right ventricular outflow tract (RVOT) to pulmonary artery. He has now developed stenosis
in the conduit to the pulmonary artery with severe enlargement and hypertrophy of the right
ventricle.
The conducted QRS in this ECG has a normal appearing initial sharp component (Figure 14.2.2,
red arrows) from activation of the left bundle branch. However, the subsequent component (R′ in
lead V1) is of large amplitude, has multiple notches (blue arrows), and the QRS is quite wide (188
ms). The large amplitude of R′ in V1 (>1 mV or >10 mm) suggests severe right ventricular
hypertrophy. The notches and fractionation in R′ suggest regions of conduction block in the
interventricular septum and right ventricle (scarring from surgical intervention and
cardiomyopathy). The delayed terminal component of the QRS signifies the delay in activating the
right ventricular free wall (conduction block and slowed conduction from ventriculotomy scar and
myocardial fibrosis).

Figure 14.2.2

The findings on this ECG are typical for patients with surgically corrected TOF, who have right
ventricular hypertrophy and a high incidence of RBBB.1 The right ventricular free wall and outflow
tract predictably has myocardial scarring subsequent to corrective surgery. Widening of the QRS
beyond 180 ms has been identified as a marker of sudden arrhythmic death.
Similar findings can be observed after surgical correction of other forms of conotruncal
anomalies, or isolated VSD. Before surgery, patients with VSD with intact bundle branches
demonstrate large equiphasic RS complexes (≥5 mV or ≥50 mm) in mid precordial leads (V2–V5) due
to biventricular hypertrophy (Katz-Wachtel phenomenon). The bundle of His and proximal bundle

540  u  Section 14: Congenital Heart Diseases


branches course along the inferior margin of a membranous VSD (with the exception of
congenitally corrected transposition of great arteries and double inlet left ventricle when the
conduction axis can be anterior to the VSD2,3). RBBB can occur in patients with VSD, either
spontaneously, with progressive right ventricle cardiomyopathy, or related to surgical intervention
with right ventriculotomy and repair of the VSD.1
RBBB also occurs frequently in patients with Ebstein’s anomaly. These patients have severe
right atrial enlargement with tall, peaked P waves in inferior leads. There is a high incidence of
right-sided accessory pathways, and absence of a RBBB pattern might reflect accessory pathway
conduction, compensating for the delay in right ventricular activation from the RBBB. The RBBB
morphology in Ebstein’s anomaly is distinctly different from TOF. Though the right ventricular
activation (R′) is delayed, the degree of ventricular scarring and notches in the R′ are limited.
The anatomic right ventricle is often diminutive, and the amplitude of the R′ in lead V1 is
generally small in contrast to the large R′ with RBBB in TOF.1 An example of RBBB in a
43-year-old female with Ebstein’s anomaly following surgical tricuspid valve repair is shown
in Figure 14.2.3. The peaked P waves are lacking as she had reduction right atrioplasty during
surgery.

Figure 14.2.3

In an ECG with right ventricular hypertrophy, it is important to recognize the initial r wave (rsR′
pattern) in lead V1 (red arrows in Figure 14.2.2 and 14.2.3). Absence of the V1 r wave (qR pattern)
signifies severe pulmonary hypertension with suprasystemic right ventricular pressures (e.g.,
Eisenmenger’s syndrome), resulting in simultaneous right ventricular hypertrophy and rotation of
the heart to create an initial vector away from lead V1.4

Case 14.2  u  541


References
1. Khairy P, Marelli AJ. Clinical use of electrocardiography in adults with congenital heart disease. Circulation.
2007;116(23):2734–2746.
2. Anderson RH, Becker AE, Arnold R, et al. The conducting tissues in congenitally corrected transposition.
Circulation. 1974;50(5):911–923.
3. Anderson RH, Arnold R, Thapar MK, et al. Cardiac specialized tissue in hearts with an apparently single ventricular
chamber (double inlet left ventricle). Am. J. Cardiol. 1974;33(1):95–106.
4. Surawicz B, Knilans TK. Chou’s Electrocardiography in Clinical Practice: Adult and Pediatric. 6th ed. Philadelphia,
PA: Saunders. 2008:62.

542  u  Section 14: Congenital Heart Diseases


CASE
David J. Callans, MD 14.3
Patient History
A 34-year-old male with a complex cardiac history [situs inversus, congenitally corrected
transposition of the great arteries (L-TGA), ASD/VSD and subpulmonic stenosis status post repair
and subpulmonary muscle resection (1985), severe tricuspid, pulmonic and mitral regurgitation
status post mechanical tricuspid valve, biprosthetic pulmonary valve, and mitral valve repair
(10/21014)] presents with recurrent atrial flutter.

Figure 14.3.1

The flutter wave morphology was negative in the inferior leads, positive in V1 with an early
precordial transition (lead V3 is isoelectric), which is consistent with counterclockwise flutter in the
anterior atrium. Even though the anterior (morphologic right) atrium was on the left side of his
chest (accounting for the negative flutter wave in lead I), the logic of the ECG still made this an easy
diagnosis despite the complexity of the situation.

Case 14.3  u  543


Figure 14.3.2

544  u  Section 14: Congenital Heart Diseases


CASE
Sabine Ernst, MD, PhD 14.4
Patient History
Transposition of the great arteries with Mustard repair after balloon septostomy.
While reentry around the tricuspid annulus is the most common atrial tachycardia in these
patients, other reentrant or focal substrates may also be present. Typical hints from 12-lead ECGs
may be misleading as these patients have a complex pattern of surgical patches and scars that make
the interpretation of the activation sequence difficult.

Figure 14.4.1  Sustained atrial tachycardia after successful deployment of a linear lesion on the floor of the right atrial (RA) compartment
of the pulmonary venous atrium (PVA) which, on the first glance, presents as an atrial tachycardia with 2:1 atrioventricular (AV) nodal
conduction (doubled amplitude to depict the P-wave morphology (fused at the end of the T wave).

Case 14.4  u  545


Figure 14.4.2  Intracardiac signals of the same patient with a decapolar catheter (“CS”) positioned in systemic venous atrium (SVA) with
the distal tip displaying ventricular signals from the left ventricle. This demonstrates the 1:1 AV nodal conduction. The map is retrogradely
advanced into the PVA using remote magnetic navigation and shows continuous, low- voltage activation.

Figure 14.4.3  3D reconstruction of a contrast CT scan using CARTO 3. Contrast was applied from the left arm, which explains why
the inferior vena cava is not contrasted at all. All ventricular chambers are transparent to allow a view on the surgically corrected atrial
switch operation (Mustard repair). The venous blood is collected from the superior and inferior caval veins into the systemic venous atrium
(SVA) which then leads via the mitral valve to the left ventricle and the pulmonary artery. Oxygenated blood returns from the lungs via
the pulmonary veins (PV) which are connected to the remainder of the right atrium (RA) and together form the pulmonary venous atrium
(PVA).

546  u  Section 14: Congenital Heart Diseases


CASE
Sabine Ernst, MD, PhD 14.5
Patient History
Transposition of great arteries with a functionally single ventricle (double inlet left ventricle) that
had undergone a total cavopulmonary connection (TCPC) repair.

Figure 14.5.1  AT #1 (360 ms): Note negative P waves in the inferior leads and the 2:1 atrioventricular (AV) nodal conduction
demonstrated to be counterclockwise reentry around the tricuspid annulus. Note the very low amplitude of P waves in the chest leads as
little horizontal conduction due to the previous TCPC surgery.

Case 14.5  u  547


Figure 14.5.2  AT #2 (260 ms): Positive P waves in leads I and II, as well as throughout the chest leads, depicting a superior and posterior
site of origin of a focal atrial tachycardia from within the TCPC.

Figure 14.5.3  Left panel: Depiction of the cardiac anatomy of the native atria and the reentrant circuit around the tricuspid annulus. The
nonoperated left atrium acts like a bystander. Right panel: Depiction of the TCPC connecting the inferior vena cava (IVC) to the pulmonary
arteries (PA).

548  u  Section 14: Congenital Heart Diseases


CASE
Sabine Ernst, MD, PhD 14.6
Patient History
A male patient with double inlet left ventricle (DILV) and discordant ventricular-atrial connection.
Both the atria and the ventricles have large, nonrestrictive septal defects. There is only a
rudimentary right ventricle (RV). Besides banding of the pulmonary artery and a subsequent
classical Glenn shunt, the patient had not had any other cardiac surgery. The right atrioventricular
(AV) valve is severely regurgitant and the SaO2 is around 78% on room air.

Figure 14.6.1  12-lead electrogram in sinus rhythm demonstrates a relatively broad PV wave that ends within the QRS complex. The
initial component fits well with a right atrium origin along the crista terminalis (sinus rhythm).

Case 14.6  u  549


Figure 14.6.2  Burst pacing from the coronary sinus induces atrial tachycardia, which spontaneously changes to a second morphology
(note the change in the chest leads) after a short time.

Figure 14.6.3  After ablation for atrial tachycardia (AT) #1 and #2, a third AT was documented. Note the width of the pulmonary vein
(within the blue markers, 400 ms). This makes the mapping and understanding of the AT (TCL 460 ms) very challenging.

550  u  Section 14: Congenital Heart Diseases


Figure 14.6.4  3D reconstructions of a noncontrast cardiac magnetic resonance scan (scan uses blood as the contrast agent instead of
contrast injection, e.g. in CT angiography). All cardiac chambers are displayed in the top panels to demonstrate the arrangements of the
ventricles on top of each other (“hamburger” heart). The lower panel demonstrates the “direction” of the AV valves: the right AV valve is
directing flow upwardly in the rudimentary RV and the left AV valve downward to the left ventricle (“criss-cross heart”).

Case 14.6  u  551


CASE
14.7 Sabine Ernst, MD, PhD

Patient History
A patient with atrial isomerism (heterotaxy syndrome) and single ventricle after extracardiac total
cavopulmonary connection (TCPC) surgical correction. There is a large atrioventricular septal
defect (AVSD) with results in a common atrioventricular (AV) valve directly in contact with a large
ventricular septal defect (VSD).

Figure 14.7.1  12-lead ECG during sudden onset tachycardia with typical on–off phenomenon. Note the narrow QRS complex
tachycardia with the negative QRS in the inferior leads and the positive retrograde P wave.

552  u  Section 14: Congenital Heart Diseases


Figure 14.7.2  A resting ECG. Note the same QRS morphology (via the inferior AV node) in the slower intrinsic rate, which reverses after
the ventricular extrasystole to another narrow QRS complex with opposite polarity (superior AV node).

Figure 14.7.3  Top panels depict the anatomy with all cardiac chambers to demonstrate the functionally single ventricle. Lower panels
are with the ventricles removed to display the common AV valve. The yellow line depicts the outline of the VSD with the connecting
“Moenckeberg” sling of conductive tissue connecting the superior and the inferior AV node. Both AV nodes and the sling can be located
by high-frequency signals preceding the onset of the local ventricular activation similar to potentials along the fascicles.

Case 14.7  u  553


CASE
14.8 John M. Miller, MD

Patient History
A 15-year-old male with a history of congenital heart disease (pulmonary atresia with intact
ventricular septum) developed palpitations and tachycardia. He had undergone two Blalock-Taussig
shunts in infancy and a Fontan repair at the age of 5 with take-down of the prior shunts. At an
emergency department visit prompted by palpitations, supraventricular tachycardia (SVT) was
diagnosed, but the arrhythmia did not respond to a dose of adenosine and he was electrically
cardioverted. Palpitations recurred and the same treatment was given. He was then referred for
electrophysiologic (EP) study and possible catheter ablation. The ECG shown was obtained during
the EP study.

Figure 14.8.1

Question
What does his ECG (Figure 14.8.1) show?

554  u  Section 14: Congenital Heart Diseases


Discussion
The rhythm strip (A) is difficult to interpret; there are clearly rapid, large complexes (pointing
downwards) at 240 ms as well as less frequent, smaller deflections pointing upwards. This suggests
ventricular tachycardia (VT) with relatively narrow (125 ms) QRS complexes (the larger, negative
deflections), with intermittent retrograde conduction to the smaller, positive deflection P waves.
QRS complexes can be relatively narrow in patients without significant ventricular scarring (the
Blalock and Fontan surgeries do not involve a ventricular incision). A diagnosis of VT might explain
why the arrhythmia did not respond to adenosine. However, when reviewing the full 12-lead ECG
(panel B), it becomes clear that the large deflections on the rhythm strip were flutter waves and the
smaller deflections were the QRS complexes (with 2:1 conduction). Of note, the recording in B is
not continuous; instead, the same 5 QRS complexes are shown in each lead. The rhythm strip in A
is, however, continuously recorded.
His diagnostic EP study showed both lateral right atrial scar-based reentry as well as
counterclockwise cavotricuspid isthmus-dependent right atrial flutter; he underwent successful
cavotricuspid isthmus ablation as well as linear lateral right atrial ablation, with elimination of his
arrhythmias.

Lessons
Flutter waves in patients with congenital heart disease can be very large, even larger than the QRS
complex on occasion. Patients with Fontan connections have extremely large right atria and would
be expected to have very large P waves, but on the other hand, scarring from pressure overload may
decrease the voltage registered from the atria on the ECG.

Case 14.8  u  555


CASE
14.9 George F. Van Hare, MD

Patient History
This is the ECG from a 6-month-old infant with Tetralogy of Fallot following surgery (Figure 14.9.1).

Figure 14.9.1

Question
Explain the sudden change in heart rate.

Explanation
On first glance, one might suspect that some form of reentrant tachycardia was suddenly
terminated by a premature atrial contraction (PAC) (red arrow) (Figure 14.9.2). However, close
inspection of the T waves following the abrupt heart rate change reveals slightly subtle PACs on the
early part of the T waves (black arrow). The actual explanation is sinus rhythm (at a normal rate for
a 6-month-old child) followed by the sudden onset of blocked atrial bigeminy. The first PAC (red
arrow) conducts with aberration, and the subsequent PACs are all blocked in a bigeminal pattern.

556  u  Section 14: Congenital Heart Diseases


Figure 14.9.2

Case 14.9  u  557


CASE
14.10 George F. Van Hare, MD

Patient History
This is the ECG from a 5-year-old male with somewhat complex congenital heart disease, admitted
for surgery (Figure 14.10.1).

Figure 14.10.1

Question
What do the initial forces tell you about the nature of the patient’s heart disease?

Explanation
There is a lack of the normal “septal Q wave” in the lateral precordium (black arrow), but a clearly
evident small q wave in lead V1 (red arrow) (Figure 14.10.2). This is so-called “congenitally corrected
transposition” also referred to as SLL ventricular inversion, L-looped ventricles, L-transposition, or
atrioventricular (AV) discordance with ventriculoarterial discordance. As the ventricles are inverted
with a morphologic left ventricle to the right and the morphologic right ventricle on the left, the
conduction system is also inverted, explaining the reversal of initial forces. Also present on this
ECG is QT interval prolongation, and first-degree AV block (RP is 200 ms, which is abnormal in a

558  u  Section 14: Congenital Heart Diseases


5-year-old). The latter bears watching. The incidence of spontaneous development of complete AV
block in this condition is 2% per year.

Figure 14.10.2

Case 14.10  u  559


CASE
14.11 George F. Van Hare, MD

Patient History
A 9-month-old child with a large ventricular septal defect is sent for surgical repair. Epicardial wires
are attached to a temporary dual-chamber pacemaker. In the cardiac intensive care unit, this ECG
is obtained (Figure 14.11.1).

Figure 14.11.1

Question
What’s going on?

Explanation
One can clearly make out prominent pacemaker spikes followed by P waves (red arrow) (Figure
14.11.2), followed by QRS complexes after a normal PR interval, so it seems that there is atrial
pacing with 1:1 atrioventricular (AV) conduction. However, further careful inspection of the QRS
complexes reveals sharp pacing artifact on the latter part of the QRS (black arrow). If one had
glanced at the device, one would have noticed pacing in both chambers. So, in the presence of intact
AV conduction, why is the ventricular channel firing?

560  u  Section 14: Congenital Heart Diseases


Figure 14.11.2

Several possible explanations may be considered. First, it is possible that there is ventricular
undersensing. However, this is pretty unlikely immediately postoperatively, particularly in the
ventricle. A second possibility is safety pacing, which can occur when a ventricular event is sensed
in the safety pacing window. However, this would be a substantially shorter interval between atrial
and ventricular pacing stimuli, specifically 110 ms in this device. Finally, one can consider late
sensing. This is the best explanation. Particularly in epicardial systems, the site chosen to sense and
pace the ventricle is often a site that activates late, either due to simply being epicardial, on the right
ventricle, in a ventricle with bundle branch block, or a combination of these characteristics. In this
situation, there is no true undersensing because the sensing/pacing site “gets the news late” and so
one observes pacemaker fusion or pseudo-fusion.

Case 14.11  u  561


CASE
14.12 George F. Van Hare, MD

Patient History
A 9-month-old infant with complex congenital heart disease consisting of heterotaxy and an
atrioventricular (AV) canal defect is admitted and undergoes banding of his pulmonary artery.
There is no intracardiac repair. The first 12-lead ECG is obtained postoperatively, the second with
atrial pacing (Figure 14.12.1). The ECG on the left shows a left atrial rhythm with 1:1 AV conduction
and an inferior axis with tall R waves in the lateral precordium. The ECG on the right shows atrial
pacing, and now a superior axis and dramatically different QRS morphology in the precordial leads
is visible.

Figure 14.12.1  Left: nonpaced. Right: paced.

Question
What accounts for the change in QRS?

Explanation
The rates of atrial pacing are not fast enough to elicit rate-dependent bundle branch aberration in a
child this age, and in any case, the QRS durations are similar between paced and nonpaced

562  u  Section 14: Congenital Heart Diseases


rhythms. In this case, the most likely explanation is the presence of duplicate AV nodes and
conduction systems. This is reported in patients with heterotaxy and AV canal defects.
Embryologically, the accessory AV node does not involute but persists along with the dominant
conduction system. This is similar to the situation seen with a Mahaim atriofascicular accessory
pathway, in which the QRS morphology changes depending on the site of atrial activation. In
duplication of the AV node, some atrial sites have preferential access to one node, and other sites
have access to the other node. Each node connects to different parts of the distal conduction
system, explaining the QRS morphology shift. In such patients, AV reciprocating tachycardia is
possible due to reentry between the two conduction systems. This patient has not yet exhibited that
problem.
With these principles in mind, evaluate this 12-lead ECG (Figure 14.12.2) obtained around the
same time postoperatively.

Figure 14.12.2

Explanation
Now that a duplicated conduction system has been confirmed, it is reasonable to hypothesize that
the first five beats represent junctional rhythm with 1:1 retrograde conduction. One can see the
retrograde P waves on the early part of the T wave (red arrow). Beat 6 is a premature QRS of
different morphology (green arrow, Figure 14.12.3). It could simply be a junctional premature beat
due to automaticity, but another possible explanation is that it actually represents a single beat of
reentry between the conduction systems. It is not due to a premature atrial contraction; the
preceding P wave is not timed differently than the previous ones. Following this, there is continued
junctional rhythm with P waves in front of the QRS complexes, but with PR intervals perhaps too

Case 14.12  u  563


short for conduction, so this is nearly isorhythmic. Finally, the sinus mechanism accelerates and
captures the ventricles and the QRS complex changes for the last seven beats of the strip (black
arrow). In retrospect, the early beats represented junctional automaticity from one conduction
system, and the later beats represent preferential use of the second conduction system during AV
conduction.

Figure 14.12.3

564  u  Section 14: Congenital Heart Diseases


CASE
George F. Van Hare, MD 14.13
Patient History
A 21-year-old male with chronic congestive heart failure in the setting of a prior lateral tunnel
Fontan procedure undergoes complex heart transplantation. His ECG obtained two weeks later is
shown (Figure 14.13.1).

Figure 14.13.1

Question
Explain the P waves.

Explanation
While not often seen these days, this is a good example of atrioatrial dissociation due to the
presence of a persisting sinus mechanism in the native atrium, which was retained in the transplant
along with the donor heart sinus node, which conducts. These rogue P waves can be appreciated
occasionally on the tracing (red arrows) and are of no importance (Figure 14.13.2), except to the
extent that they may be misinterpreted as nonconducted atrial beats that would otherwise suggest a

Case 14.13  u  565


need for ventricular pacing. Matching a structurally normal donor heart to the complex abnormal
venous anatomy following a Fontan requires flexibility from the surgeon.

Figure 14.13.2

566  u  Section 14: Congenital Heart Diseases


CASE
Edward P. Walsh, MD 14.14
Patient History
The patient is a 19-year-old male who recently moved to the area with his family and was referred
for routine cardiac follow-up because of a past history of surgical closure of a ventricular septal
defect at age 9 months. He is doing well, has no cardiac symptoms, and is not taking any
medications. A baseline ECG is obtained (Figure 14.14.1A). During the physical exam, the heart
sounds are loudest in the right chest, the apex beat is displaced to the right of the well-healed
sternotomy scar, and a soft liver edge is felt along the left costal margin. The second heart sound
splits physiologically with a soft pulmonic component, and there are no audible murmurs.

Figure 14.14.1  A. Standard 15-lead ECG at time of office visit. B. Diagrammatic representation of cardiac chamber orientation.

Case 14.14  u  567


Question
What information does the ECG provide about this patient’s unusual anatomy?

Discussion
The answer in this case is “everything.” Echocardiography and magnetic resonance imaging are
certainly powerful tools for deciphering the anatomic details of congenital heart disease (CHD), but
experienced clinicians still recognize the value of physical exam and careful review of the ECG. For
this patient, the ECG confirms dextrocardia with atrial situs inversus, consistent with the physical
exam findings. Observe that the P wave axis is +120°, suggesting that sinus node is left-sided, since
atrial activation originates high-leftward. The precordial voltages are noticeably larger in the
rightward chest leads (V4R and V3R) than in the left chest leads, confirming dextrocardia.
Furthermore, the QRS activation pattern is unusual, with an RSR over the left chest, and a septal Q
wave in lead V4R, suggesting the left ventricle is right-sided. Putting this all together, the anatomy
must be as depicted in Figure 14.14.1B. Although there are no formally established normal values
for ventricular voltages in this setting, none of the precordial R waves appear excessively tall. This,
combined with absence of a murmur and a normal second heart sound, suggest the ventricular
septal defect has been closed successfully.
Note that most ECG tracings obtained in patients with CHD involve 15 leads, with the addition
of V4R and V3R over the right precordium, and V7 far leftward.1 This expanded view of the
precordium is extremely helpful in interpreting malpositions as in this case.

Reference
1. Walsh EP, Alexander ME, Cecchin F. Electrocardiography and introduction to electrophysiologic techniques. In:
Nadas’ Pediatric Cardiology. 2nd ed. Keane JF, et al. Eds. Philadelphia, PA: Saunders/Elsevier. 2006:145–182.

568  u  Section 14: Congenital Heart Diseases


CASE
Edward P. Walsh, MD 14.15
Patient History
The patient is a 35-year-old male who underwent surgical repair of Tetralogy of Fallot (TOF) as a
child. He has an episode of sustained rapid palpitations with syncope at work that terminated
abruptly just before an ambulance arrived on the scene. Upon transfer to the emergency
department, he is in sinus rhythm as shown in the ECG in Figure 14.15.1A.

(A)

(B)

Case 14.15  u  569


(C)
Figure 14.15.1  A. ECG in sinus rhythm at time of emergency department arrival. B. Sustained monomorphic ventricular tachycardia
induced at electrophysiologic study. C. A different TOF patient with well-preserved pulmonary valve function.

Question
What does this ECG tell you about his hemodynamic status and risk for serious arrhythmias?

Discussion
Adults with repaired Tetralogy of Fallot are at risk for a wide variety of late arrhythmias,1 including
ventricular tachycardia (VT). The VT in this setting is usually monomorphic macroreentry
involving the right ventricular (RV) outflow region.2 Risk-stratification in tetralogy is a very
imperfect science, but many risk factors have been proposed, including severe RV dilation and
dysfunction,3 which should be suspected in this case based on the ECG finding. Note that there is
first a pattern of right bundle branch block (RBBB) with a QRS duration that is quite prolonged
(200 ms). Nearly all TOF patients will have some degree of postoperative RBBB, but it has been
suggested that the risk for VT increases once the QRS duration begins to exceed 180 ms.4 The
“electroanatomic” concordance between RV size in TOF and the QRS duration are now well
established. Note also that that precordial pattern in this case suggests dramatic leftward
displacement of the intraventricular septum due to this severe RV enlargement, since the RSR
pattern persists out beyond V4, and a clear septal Q wave is not seen even as far leftward as V7. It
just so happens that this patient also has an initial QRS axis suggesting concomitant left anterior
hemiblock, although bifascicular block by itself is not felt to be an important risk factor for VT
unless the QRS duration is prolonged beyond 180 ms.
Based on the ECG, it should not be surprising that subsequent imaging studies showed this
patient to have severe pulmonary regurgitation with a massively dilated right ventricle. Therefore,
there was a high level of concern that his presenting symptoms were due to VT. An
electrophysiology study shortly after admission was obtained and sustained monomorphic VT was
easily induced (see Figure 14.15.1B) supporting this suspicion.5 The QRS morphology during VT
(left bundle branch block with an inferior frontal axis) is indeed consistent with VT origin in the RV
outflow region. He ultimately underwent acutely successful catheter ablation for VT, followed by

570  u  Section 14: Congenital Heart Diseases


surgical pulmonary valve replacement, and insertion of a back-up implantable
cardioverter-defibrillator.
In sharp contrast, Figure 14.15.1C is an ECG from a similar-aged TOF patient with a well-
preserved pulmonary valve and only mild right ventricular dilation whose only rhythm concern has
been atrial flutter. Note the shorter QRS duration (120 ms) and a clear septal Q wave that emerges
by lead V4.

References
1. Walsh EP, Cecchin F. Arrhythmias in adult patients with congenital heart disease. Circulation. 2007;115:534–545.
2. Walsh EP. Sudden death in adult congenital heart disease: Risk stratification in 2014. Heart Rhythm. 2014;11:1735–
1742.
3. Knauth AL, Gauvreau K, Powell AJ, et al. Ventricular size and function assessed by cardiac MRI predict major
adverse clinical outcomes late after tetralogy of Fallot repair. Heart. 2008;94:211–216.
4. Gatzoulis MA, Till JA, Somerville J, et al. Mechanoelectrical interaction in tetralogy of Fallot. QRS prolongation
relates to right ventricular size and predicts malignant ventricular arrhythmias and sudden death. Circulation.
1995;92:231–237.
5. Khairy P, Landzberg MJ, Gatzoulis MA, et al. Prognostic significance of electrophysiologic testing post tetralogy of
Fallot repair: A multicenter study. Circulation. 2004;109:1994–2000.

Case 14.15  u  571


CASE
14.16 Edward P. Walsh, MD

Patient History
Conjoined thoracopagus twins with a history of intermittent fetal tachycardia were delivered by
C-section. There was concern that the infants shared multiple organs, including liver, intestines,
and possibly heart (Figure 14.16.1A). Modified ECGs were obtained from each twin immediately
after delivery (limb leads only since precordial leads were not possible).

(A)

572  u  Section 14: Congenital Heart Diseases


(B)

(C)
Figure 14.16.1  A. Conjoined twins: (upper panel) lateral view x-ray showing conjoined thoracic and abdominal structures. (lower
panels): limb-lead ECGs from Twin A and Twin B. B. Conjoined twins: (upper panel) Cardiac MRI image showing atrial-level cardiac
connection. The ventricles were separate, so each twin had their own atrioventricular conduction system, but Twin B also had an accessory
pathway. (lower panel) Orthodromic reciprocating tachycardia (ORT) in Twin B that was terminated with adenosine. Note that there was
passive 1:1 atrial transmission of the tachycardia to Twin A, but their ventricular rate was usually titrated to a 2:1 response by their AV
node. C. Conjoined twins: (upper panels) ECGs from Twin A and Twin B following successful catheter ablation of the accessory pathway
in Twin B. (lower panels) ECGs from Twin A and Twin B following successful surgical separation.

Case 14.16  u  573


Questions
Are the hearts conjoined? If so, at what level? What do these two ECGs tell us about the potential
mechanism for fetal tachycardia?

Discussion
Despite this bizarre presentation, it should be possible to work from first principles and use the
ECGs to answer these questions. To begin, it should be apparent from these tracings that the hearts
are connected at some level. That much can be inferred from the observation that two independent
rhythms are not present. The faster of two sinus nodes appears to be driving the rate in both twins.
The precise level of the cardiac connection is less clear, but there is a hint (best seen in leads II, III,
and aVF) that the QRS is rather complicated, possibly indicating two superimposed QRS
complexes—one native to the patient, and the other far-field from the patient’s twin. This would
support the notion that the atria are connected, but the ventricles are separate. Now look more
closely at the two QRS complexes in Twin B. The native QRS appears to have a short PR interval
and slurred QRS onset suggesting preexcitation, which raises the possibility that the fetal
tachycardia may have been due to Wolff–Parkinson–White syndrome. In fact, all of these inferences
were true. The upper panel in Figure 14.16.1B shows the cardiac magnetic resonance image proving
an atrial level connection, and the lower panel documents an episode of orthodromic tachycardia in
Twin B that was terminated with adenosine. Note that there was passive conduction of the
tachycardia across the connection into the atria of Twin A, but Twin A’s ventricular response was
usually limited to a 2:1 pattern by their AV node. After a time, Twin A would begin to conduct more
rapidly, and could reach 1:1 conduction on occasion.
Both twins became quite ill during tachycardia, and medical therapy was not effective in
preventing recurrences. Prior to separation surgery, successful radiofrequency catheter ablation of a
posteroseptal accessory pathway was thus performed in Twin B.1,2 The postablation ECGs (upper
panel in Figure 14.16.1C) still both show a duplicated QRS with native and far-field activation, but
now the delta wave is gone. Following successful surgical separation (lower panel in Figure
14.16.1C), the ECGs finally have a more conventional appearance.

References
1. Correa R, Shivapour J, Johnikin MJ, et al. WPW in conjoined thoracopagus twins. Heart Rhythm. 2013;11:336–337.
2. Correa R, Lang P, Walsh EP. Catheter ablation of WPW in conjoined thoracopagus twins. Heart Rhythm.
2014;11:1070–1072.

574  u  Section 14: Congenital Heart Diseases


CASE
Edward P. Walsh, MD 14.17
Patient History
A 31-year-old female presents to the emergency department complaining of sudden onset of
uncomfortably rapid heart rate and mild dizziness. She has a history of repaired “hole in the heart” at
11 years, and underwent placement of a pacemaker for “slow heart rate” as a teenager with several
subsequent generator changes. She confesses that she has not kept her appointments for cardiac
follow-up in over 3 years, but had been feeling extremely well until she experienced the rapid heart rate
that morning. An ECG was obtained (Figure 14.17.1A) that was initially interpreted by the emergency
department physician as mild sinus tachycardia. The on-call cardiologist is asked to evaluate her.

(A)

(B)

Case 14.17  u  575


(C)
Figure 14.17.1  A. Presenting ECG with heart rate of 97 bpm. B. Arrows indicate atrial reentrant tachycardia with a cycle length 310 ms
that conducts primarily in a 2:1 fashion. Lower rhythm strip (lead V1) verifies mechanism during a transient slowing of conduction induced
with a Valsalva maneuver. C. ECG in atrial paced rhythm after atrial tachycardia was terminated.

Questions
What is the rhythm? What is the most likely underlying anatomic heart disease?

Discussion
In many ways this is a very simple case, but it still represents one of the most common clinical
problems in adults with repaired congenital heart disease that is still frequently misdiagnosed. A
trained eye will immediately focus on lead V1 and suspect atrial tachycardia at a fixed cycle length
of 310 ms conducting predominately in a 2:1 pattern (Figure 14.17.1B), which is easily confirmed
during a Valsalva maneuver that allows a brief interlude of slower conduction (lower panel in
Figure 14.17.1B). A novice may perhaps be forgiven for their initial misinterpretation of the ECG
data since this does not look at all like classic “atrial flutter.” The ventricular response rate is not
very rapid, the atrial cycle length is much longer than would be expected for flutter, and the atrial
waveform differs from the classic sawtooth pattern one expects to see in conventional hearts.
Nonetheless, this is a fairly typical pattern for the right atrial macroreentry circuits that occur in
repaired CHD.1,2 The index of suspicion for atrial reentrant tachycardia must always be high when
dealing with this population. The tachycardia in this case was easily terminated with a brief burst of
atrial pacing through the patient’s device, which restored her normal atrial paced rhythm
(Figure 14.17.1C).
This patient’s underlying defect can be deduced with reasonable certainty from the ECG viewed
in the context of her surgical history. The fact that her corrective surgery was able to be delayed
until the relatively late age of 11 years suggests this was not a major malformation such as
transposition or large ventricular septal defect, but rather a more simple lesion like atrial septal

576  u  Section 14: Congenital Heart Diseases


defect. Note also that the ECG shows a superior QRS axis in the frontal plane of –60°, which is
indicative of defects involving the atrioventricular canal region of the heart.3 A sensible unifying
diagnosis would therefore be an atrial septal defect of the primum variety, which was in fact
confirmed by review of her childhood records.

References
1. Triedman JK, Alexander ME, Berul CI, et al. Electroanatomic mapping of entrained and exit zones in patients with
repaired congenital heart disease and intra-atrial reentrant tachycardia. Circulation. 2001;103:2060–2065.
2. Walsh EP, Cecchin F. Arrhythmias in adult patients with congenital heart disease. Circulation. 2007;115:534–545.
3. Thiene G, Wenick ACG, Frescura C, et al. Surgical anatomy and pathology of the conduction tissues in
atrioventricular defects. J. Thorac. Cardiovasc. Surg. 1981;82:928–937.

Case 14.17  u  577


CASE
14.18 Edward P. Walsh, MD

Patient History
A 3-year-old child with complex single ventricle physiology is being considered for surgery with a
Fontan operation. There is a history of recurrent supraventricular tachycardia of an uncertain
mechanism that has been poorly controlled on medications. An electrophysiology study is performed
preoperatively in an effort to establish mechanism and consider catheter ablation. The surface
electrocardiogram (ECG) (leads I, aVF, V1, and V6) at baseline reveals a peculiar pattern of
spontaneous QRS changes (Figure 14.18.1A), shifting from a superior axis (QRS #1) to a normal axis
(QRS #2) that appears dependent on subtle changes in the P-wave morphology (arrows) without
significant change in the atrial rate. This makes rate-related aberration unlikely, and suggests that the
QRS pattern varies according to changes in the earliest site of atrial activation. Both QRS patterns
have a normal PR interval, a normal HV interval, and a sharp upstroke for the QRS, making an
atrioventricular or atriofascicular accessory pathway unlikely.

(A)

578  u  Section 14: Congenital Heart Diseases


(B)

(C)

Case 14.18  u  579


(D)
Figure 14.18.1  A. Tracing from EP study showing spontaneous QRS changes that occur with subtle changes in P wave (arrows). See
text for more details. B. Site-specific atrial pacing causing shifts in QRS axis. C. Episode of SVT featuring QRS #1 with that terminates
spontaneously. D. Diagrammatic model of node-to-node reentry in twin AV node physiology.

The QRS shift can be recreated by site-specific atrial pacing (Figure 14.18.1B). Pacing in the low
right atrium favors QRS #1, while pacing in the high right atrium favors QRS #2. An atrioventricular
(AV) reciprocating tachycardia is easily induced with S2 atrial stimulation, and displays QRS #1
(Figure 14.18.1C) with earliest retrograde atrial activation time that was mapped to the high right
atrium. Following termination with retrograde block, the QRS again shifts with subtle changes in
the P-wave morphology (arrows).

Question
What possible conduction model might explain these findings?

Discussion
This is an example of a rare but fascinating condition referred to as “twin AV nodes”1,2 that can be
detected in certain specific forms of complex CHD featuring AV discordance and a malaligned AV
canal defect that results in one ventricle being very hypoplastic. Histopathologic studies have
confirmed the presence of a duplicated conduction system in such hearts involving two separate
compact nodes, two His bundles, and a “sling” of conduction tissue connecting the two systems.3,4
The potential for node-to-node reentry can be easily appreciated (Figure 14.18.1D). In this patient,
the anterior AV node was ablated after confirming normal conduction in the posterior node. All
tachycardia was eliminated, and he did well following Fontan surgery.

580  u  Section 14: Congenital Heart Diseases


References
1. Epstein MR, Saul JP, Weindling SN, et al. Atrioventricular reciprocating tachycardia involving twin atrioventricular
nodes in patients with complex congenital heart disease. J. Cardiovasc. Electrophysiol. 2001;12:671–679.
2. Wu MH, Wang JK, Lin JL, et al. Long-term outcome of twin atrioventricular node and supraventricular tachycardia
in patients with right isomerism of the atrial appendage. Heart Rhythm. 2008;5:224–229.
3. Mönckeberg JG. Zur entwicklungsgeschichte des atrioventrikularsystems. Verhandl. Deutsch Path. Gesellsch.
1913;16:228–249.
4. Symons JC, Shinebourne EA, Joseph MC, et al. Criss-cross heart with congenitally corrected transposition: Report
of a case with d-transposed aorta and ventricular preexcitation. Eur. J. Cardiol. 1977;5:493–505.

Case 14.18  u  581


SECTION 15
Special Considerations: Age, Race, Gender, and Athletes

CASE
Alan Cheng, MD
Jane E. Crosson, MD 15.1
Patient History
A 19-year-old college athlete underwent a standard precollegiate ECG screening (Figure 15.1.1).

Figure 15.1.1

Question
Should additional testing be performed, or is she cleared to participate?

Discussion
The ECG demonstrates T-wave inversions in the septal precordial leads in a 19-year-old, a finding
that should prompt further evaluation of arrhythmogenic right ventricular cardiomyopathy
(ARVC). T-wave inversions in the precordial leads are normal in pediatric patients, but the axis is

ECG Masters’ Collection: Favorite ECGs from Master Teachers Around the World © 2017 Mohammad Shenasa, Mark E. Josephson,
N. A. Mark Estes III, Ezra A. Amsterdam, Melvin Scheinman. Cardiotext Publishing, ISBN: 978-1-942909-08-8.   583
typically upright by the age of 14. Given these findings, a signal-averaged ECG was performed
(Figure 15.1.2), which demonstrated abnormal depolarization. Normal criteria for signal-averaged
ECGs include a total filtered QRS duration ≤ 114 ms, duration of high-frequency, low-amplitude
signals below 40 μV ≤ 38 ms, and the root mean squared voltage in the terminal 40 ms of the QRS
complex ≥ 20 μV. While the patient’s signal-averaged ECG is abnormal, establishing a diagnosis of
ARVC requires additional criteria including radiographic abnormalities seen with advanced cardiac
imaging.1 The patient is currently undergoing further testing to determine whether the diagnostic
criteria for ARVC are fulfilled.

Figure 15.1.2

Reference
1. Marcus FI, McKenna WJ, Sherrill D, et al. Diagnosis of arrhythmogenic right ventricular cardiomyopathy/dysplasia:
Proposed modification of the task force criteria. Circulation. 2010;121:1533–1541.

584  u  Section 15: Special Considerations: Age, Race, Gender, and Athletes
CASE
N. A. Mark Estes, MD 15.2
Patient History
ECG of a 32-year-old male with palpitations and recurrent syncope while pitching in baseball. He
collapsed during competition and was cardioverted from ventricular flutter (rate 260 bpm).

Figure 15.2.1

Questions
1. What is the ECG abnormality?
2. What cardiovascular condition is likely to cause the ECG abnormality and cardiac arrest?

Discussion
The ECG demonstrates normal sinus rhythm with left ventricular hypertrophy with strain and deep
symmetrical T wave inversion in leads V2–V5. Apical hypertrophic cardiomyopathy was confirmed
by an echocardiogram and magnetic resonance imaging with gadolinium enhancement of the apical
septum.

Case 15.2  u  585


CASE
15.3
Pieter Koopman, MD
Hein Heidbuchel, MD, PhD

Patient History
A 34-year-old female mountain biker, training 30 hours per week and internationally competitive,
presents for a yearly check-up. During thorough history taking, she admits that her results have
declined somewhat over the last year, despite an intensified training schedule.

Figure 15.3.1

Question
Can she continue to compete or not?

Discussion
At first look, there seems to be complete heart block. However, the second and third RR intervals
are slightly shorter than the other intervals. The longer RR intervals are due to junctional rhythm,
but the two shorter intervals are due to conducted sinus impulses. Sinus rhythm is slightly faster
than the junctional rhythm, explaining intermittent sinus capture. AV dissociation with
competition between sinus rhythm and junctional rhythm is very often seen in highly trained
athletes at rest. With exercise, there was normal sinus tachycardia to 175 bpm at a maximum of
300 W, with 1:1 AV conduction throughout the test. Therefore, the finding at rest is physiologic
(adaptive) and not pathologic. Based on the ECG and the normal chronotropic response during
exercise, there is no explanation for the slightly decreased performance, nor is there a cardiac
reason to exclude her from further training and competition. This does not negate to look for other

586  u  Section 15: Special Considerations: Age, Race, Gender, and Athletes
explanations, including an overtraining syndrome. The final diagnosis was physiologic sinus
bradycardia in competition with junctional rhythm.

Figure 15.3.2   Explanatory ECG.

Reference
1. Heidbüchel H, Panhuyzen-Goedkoop N, Corrado D, et al. Recommendations for participation in leisure-time
physical activity and competitive sports in patients with arrhythmias and potentially arrhythmogenic conditions
part I: Supraventricular arrhythmias and pacemakers. Eur. J. Cardiovasc. Prev. Rehabil. 2006;13(4):475-484.

Case 15.3  u  587


CASE
15.4 Robert J. Myerburg, MD

Patient History
The patient is a 19-year-old male college basketball player who was referred because of a
presyncopal episode 1 minute after a 1.5-hour weight-training program. He became dizzy and
diaphoretic, and had problems with balance. The symptoms abated within 10–15 minutes.
He denied ever having had an episode like this previously and his family history was negative for
premature heart disease or sudden cardiac death over three generations. He denied symptoms of
cardiac arrhythmias, and had no history of dyspnea or chest pain on exertion or at rest. He did have
a history of a mild upper respiratory infection about 2 weeks prior to the event.
His resting blood pressure was 128/81 mmHg and an echocardiogram revealed septal and
posterior wall thickness of 1.2 and 1.1 centimeters, respectively. A stress test and cardiac MRI were
both normal. His 12-lead electrocardiogram is shown below (Figure 15.4.1).

Figure 15.4.1

588  u  Section 15: Special Considerations: Age, Race, Gender, and Athletes
Question
Which of the following statements is correct?
1. The patient can be cleared to participate in college-level basketball without further work-up.
2. Repeat ECG and echocardiograms are not necessary in the future unless there is a change in
symptoms.
3. This ECG is abnormal, even for a young athlete, and he should not be cleared for competitive
athletics.
4. Clearance to participate should be deferred, pending genetic testing to exclude hypertrophic
cardiomyopathy (HCM), arrhythmogenic ventricular cardiomyopathy/dysplasia (ARVC/D), and
Brugada syndrome.
5. The most likely cause for the ECG changes in this athlete is hypertensive left ventricular
hypertrophy.

Discussion, Interpretation, and Answer


The correct answer is A, the patient can be cleared to continue participation as a college basketball
player without further work-up. Certainly, an athlete with unexplained presyncope or syncope
under the circumstances of this case does require evaluation, particularly in the presence of an ECG
with dramatic but nonspecific changes. In this case, even though the circumstances of the
presyncopal event suggested a vagal response or dehydration, the level of work-up was appropriate
in the context of the ECG. That said, he had a sufficient work up to rule out any identifiable cause of
concern.
Nonspecific ECG changes in athletes are very common and have to be taken in the context of
objective findings. In the absence of demonstration of hypertension, the consideration of
hypertension-type LVH is less likely. Since the patient had a history of a recent viral infection,
consideration of myocarditis should be included in the differential, and the echo and MRI
contributed to ruling that out. Had there been evidence for myocarditis, he would have been
restricted from high intensity athletics for 3 months after resolution and then allowed to resume his
normal activities.
Since abnormal ECGs have been noted to precede the anatomic expression of hypertrophic
cardiomyopathy in a subset such of patients, serial ECGs on an annual basis would be appropriate in
this case. It should also be noted that in a smaller subset of HCM patients, the anatomic changes
have been reported to precede the ECG changes.

Case 15.4  u  589


CASE
15.5 Melvin Scheinman, MD

Patient History
A 20-year-old male presents with an irregular rhythm detected on a routine pre-employment
examination. He is without symptoms.

Figure 15.5.1

Question
What are the diagnoses and why is the ejection fraction abnormal?

Discussion
Figure 15.5.1 shows wide and narrow QRS complexes. The second to last beat shows a narrow QRS
complex preceded by a sinus P wave and likely conducted over the normal atrioventricular (AV)
node–His axis. The wide complex beats show a typical left bundle branch (LBB) pattern with a
narrow r deflection followed by a rapid descent. This pattern would not be compatible with origin
from the myocardium. Review of the rhythm strip on the bottom clearly shows varying degrees of
fusion, which indicates a ventricular origin. The typical LBBB pattern discussed above can therefore
only arise from the AV junction or from the right bundle branch (RBB) fascicle. A junctional

590  u  Section 15: Special Considerations: Age, Race, Gender, and Athletes
rhythm with aberrant conduction would be unlikely since the ventricular rate remains about the
same whether arising from a sinus-conducted beat or from ventricular focus. This would not
explain the fusion complexes. Hence, the most likely explanation is an accelerated fascicular rhythm
arising from the RBB itself. Optimal treatment would involve attempted ablation of the focus rather
than attempted lifelong drug therapy or chronic atrial pacing in this young male.

Figure 15.5.2  Intracardiac mapping shows a discrete fascicular potential preceding the ventricular electrogram. The focus was localized
over the region of the moderator band. Ablation just superior to the moderator band was successful in ablation of one focus but another
focus was localized beneath the moderator band. Ablation in this area abolished the ectopic activity and the ejection fraction normalized
after a 3-month follow-up period.

Case 15.5  u  591


CASE Mohammad Shenasa, MD

15.6
Shahriar Heidary, MD
Hossein Shenasa, MD, MS

Patient History
A 28-year-old African-American male referred for screening to play soccer. He is asymptomatic
with a negative family history.
Heart rate: 64 bpm, PR interval: 112 ms, QRS: 82 ms, QT/QTc: 455/462 ms

Figure 15.6.1

Figure 15.6.1 shows left ventricular hypertrophy (LVH), repolarization abnormalities, and
T-wave inversion in leads I, II, III, aVF, and V3–V6.
The echocardiogram showed LVEF of 74% with mild concentric LVH (Video 15.6.1).
MRI (Video 15.6.2): four-chamber view. Morphologically normal LV with normal function. LVEF
of 59%. Concentric LV hypertrophy, septal wall thickness 12 mm. Normal RV size. No evidence of
arrhythmogenic right ventricular cardiomyopathy or delayed enhancement of the RV and LV.

Question
What is the most likely ECG diagnosis?
1. Hypertrophic cardiomyopathy
2. Athletic heart
3. African-American ECG pattern (may be normal for race)

Answer
Considering the ECG pattern, cardiogram, and MRI findings, it is consistent with athletic heart in
an African-American.

Video Legends
Video 15.6.1 Echocardiogram showing LVEF of 74% with mild concentric LVH
Video 15.6.2 MRI, 4-chamber view

592  u  Section 15: Special Considerations: Age, Race, Gender, and Athletes
SECTION 16
Syncope and ECG Troubleshooting

CASE
16.1
Balaji Krishnan, MD
David G. Benditt, MD

Patient History
These tracings were obtained in a 58-year-old female with a prior history of right middle cerebral
artery stroke with subsequent seizure disorder treated with valproate. One day before this
admission, the patient felt lethargic and developed sudden loss of consciousness. She was found face
down on the floor and unresponsive but recovered spontaneously. There were no convulsions
observed at any time. After admission to the neurology service, she underwent video EEG
monitoring to evaluate for seizures.
Five-day video EEG monitoring showed the presence of continuous slowing in the right fronto-
temporal-parietal region and intermittent generalized slowing, but no epileptiform discharges.
ECG monitoring showed the following evolution in heart rhythms. Baseline ECG obtained one
month prior to admission when the patient was seen in clinic shows normal sinus rhythm, heart
rate (HR) 65 bpm (Figure 16.1.1). On admission in a postictal state, the ECG showed sinus
bradycardia with competing junctional rhythm, HR 58 bpm (Figure 16.1.2). Two hours after
admission the patient developed a junctional rhythm with HR 47 bpm (Figure 16.1.3).

ECG Masters’ Collection: Favorite ECGs from Master Teachers Around the World © 2017 Mohammad Shenasa, Mark E. Josephson,
N. A. Mark Estes III, Ezra A. Amsterdam, Melvin Scheinman. Cardiotext Publishing, ISBN: 978-1-942909-08-8.   593
Figure 16.1.1  A. Baseline 12-lead ECG showing normal sinus rhythm, a heart rate (HR) of 65 bpm and normal QRS and QT intervals.
B. EEG obtained at same time as Figure 16.1.1A ECG is normal.

594  u  Section 16: Syncope and ECG Troubleshooting


Figure 16.1.2  A. Admission ECG: Patient has a sinus bradycardia with competing junctional rhythm, HR 58 bpm. The QRS and QT
intervals remain normal. B. The EEG associated with the admission ECG reveals continuous slowing in the right fronto-temporal-parietal
region (see arrows) and intermittent generalized slowing at the same time.

Case 16.1  u  595


Figure 16.1.3  A. Approximately 2 hours after admission, the patient now shows a somewhat more dramatic bradycardia (junctional
rhythm, HR 47 bpm). B. The EEG associated with the bradycardia in Figure 16.1.3A shows continuous slowing in the right fronto-
temporal-parietal region and intermittent generalized slowing (arrows).

596  u  Section 16: Syncope and ECG Troubleshooting


Discussion
Over the duration of in-hospital monitoring, the degree of EEG regional slowing decreased. This
observation tends to support the possibility that the EEG slowing being due to recovery from a
postictal state. In concert, her bradyarrhythmia tendency diminished, and ECG returned to normal.
The ECG evolution during an ictal-bradyarrhythmia episode is essentially indistinguishable
from that of a vasovagal syncope or near-syncope, with sinus tachycardia being followed by
progressive heart-rate slowing, which may at times progress to an asystolic pause.1–3 Most
documented pauses are of non–life-threatening duration (range, 3–20 seconds), but longer pauses
have been observed.3,4 The majority of documented ictal-induced bradyarrhythmias accompany
complex partial seizures of temporal lobe origin.1–3

References
1. Freeman R. Cardiovascular manifestations of autonomic epilepsy. Clin. Autonom. Res. 2006;16:12–17.
2. Schuele SU, Bermeo AC, Alexopoulos AC, et al. Video-electrographic and clinical features in patients with ictal
asystole. Neurology. 2007;69:4341–4341.
3. Benditt DG, van Dijk G, Thijs RD. Ictal asystole: Life-threatening vagal storm or a benign seizure self-termination
mechanism? Circ. Arrhythm. Electrophysiol. 2015;8(1):11–14.
4. van Dijk JG, Thijs RD, van Zwet E, et al. The semiology of tilt-induced reflex syncope in relation to EEG changes.
Brain. 2014;137(pt2):576–585.
This work was supported in part by a grant from the Dr. Earl E. Bakken Family in support of heart-brain research.

Case 16.1  u  597


CASE
16.2 Haran Burri, MD

Patient History
A patient was admitted to the stroke unit and placed upon rhythm monitoring. A 12-lead ECG was
recorded with a monitor, showing ST-segment elevation. The patient did not report any chest pain.

Figure 16.2.1  ECG recorded by a monitoring system showing ST-segment elevation in the anterior precordial leads.

Questions
1. How do you interpret this tracing?
2. What needs to be done?

Discussion
The ST elevation was due to the high-pass filter setting (set to 0.5 Hz as shown in the bottom of
Figure 16.2.1). This filter is designed to avoid baseline wander to facilitate rhythm monitoring.
Non-linear high-pass filters have been shown to result in artifactual ST-segment elevation, which
may mimick anteroseptal myocardial infarction.1 The ECG was recorded again using standard filter
settings (0.05–150 Hz), and was found to be normal (Figure 16.2.2).

598  u  Section 16: Syncope and ECG Troubleshooting


Figure 16.2.2  New ECG recording in the patient using standard filters settings (0.05–150 Hz, shown within the red circle).

Reference
1. Burri H, Sunthorn H, Shah D. Simulation of anteroseptal myocardial infarction by electrocardiographic filters.
J. Electrocardiol. 2006;39:253–258.

Case 16.2  u  599


CASE
16.3
Alan Cheng, MD
Jane E. Crosson, MD

Patient History
A 72-year-old female with systolic heart failure being treated with beta-blockers, angiotensin-
converting enzyme inhibitors, and digoxin presented with one day of confusion and an ECG (Figure
16.3.1) revealing frequent ectopy.

Figure 16.3.1

While on telemetry, she exhibited periods of additional ectopy and occasional sinus beats that
failed to conduct to the ventricle (Figure 16.3.2). A prior baseline ECG was obtained for comparison
(Figure 16.3.3).

600  u  Section 16: Syncope and ECG Troubleshooting


Figure 16.3.2

Figure 16.3.3

Question
What is the most likely explanation for these acute abnormalities?

Discussion
This case illustrates a classic example of digoxin toxicity with manifestations of neurologic and
electrophysiologic abnormalities.1 Cardiac glycosides impart their electrophysiologic effects
through reversible inhibition of the sodium-potassium ATPase exchanger, a cellular channel found

Case 16.3  u  601


in gastrointestinal, ocular, neurologic, and cardiac cells. This effect results in higher levels of
intracellular sodium and activation of the sodium/calcium exchanger, thereby increasing the resting
membrane potential and resulting in activation of voltage-gated calcium channels. This
phenomenon induces activation of calcium channels along the sarcoplasmic reticulum and a release
into the intracellular space. The higher intracellular calcium is thought to provide the increased
cardiac inotropy observed with these agents. However, there is also a negative chronotropic effect of
these agents, largely due to their ability to increase vagal tone.
Digitalis toxicity results in preferential discharge of the fascicles. Note the patient shows a RBBB
and inferior axis compatible with origin from the left anterior fascicle.
At toxic levels, cardiac glycosides can result in delayed afterdepolarizations that can result in
ventricular extrasystoles and episodes of heart block (Figure 16.3.2). Given the acute presentation of
this patient, Digibind2 was administered, and after several hours, resolution of her symptoms and
her electrocardiographic abnormalities was observed (Figure 16.3.3).

References
1. Kanji S, MacLean RD. Cardiac glycoside toxicity: More than 200 years and counting. Crit. Care Clin. 2012;28:
527–535.
2. Chan BS, Buckley NA. Digoxin-specific antibody fragments in the treatment of digoxin toxicity. Clin. Toxicol.
2014;52:824–836.

602  u  Section 16: Syncope and ECG Troubleshooting


CASE
N. A. Mark Estes III, MD 16.4
Patient History
ECG of a 56-year-old female presenting with hypertension, atrial fibrillation, and frequent
premature ventricular contractions.

Figure 16.4.1

Question
What is the ECG abnormality?

Discussion
The ECG shows left ventricular hypertrophy with strain with marked T-wave inversion across the
precordial leads. Her echocardiogram and cardiac magnetic resonance imaging were normal. She
was found to have a pheochromocytoma. Once the tumor was surgically removed, her ECG
normalized.

Case 16.4  u  603


CASE
16.5
Charles Jazra, MD
Simon Abou Jaoude, MD

Patient History
A 67-year-old male underwent mitral valve repair (degenerative mitral regurgitation).

Figure 16.5.1  Permanent atrial fibrillation. Non specific ST abnormalities.

Medications included warfarin and bisoprolol.


Six years after initial treatment, he presented with presyncope, confusion, lethargy, and severe
hypothermia, suggesting severe sepsis.
Medical intensive care HR 32/min – SBP 85 mmHg possible indication for cardiac pacing?

604  u  Section 16: Syncope and ECG Troubleshooting


Figure 16.5.2  Atrial fibrillation with bradycardia and Osborn wave (see arrow).

Discussion
The Osborn wave (J wave) is a deflection with a dome or hump configuration occurring at the R-ST
junction (J point) on the ECG. In the historical view, different names have been used for this wave
in the medical literature, such as “camel-hump sign,” “late delta wave,” “hathook junction,”
“hypothermic wave,” “J point wave,” “K wave,” “H wave,” and “current of injury.” Although there is
no definite consensus about terminology of this wave, “Osborn wave” and “J wave” are the most
commonly used names for this wave in the current clinical and experimental cardiology. The
Osborn wave can be generally observed in hypothermic patients; however, other conditions have
been reported to cause Osborn waves, such as hypercalcemia, brain injury, subarachnoid
hemorrhage, cardiopulmonary arrest from oversedation, vasospastic angina, or idiopathic
ventricular fibrillation. Our knowledge about the link between the Osborn waves and cardiac
arrhythmias remains sparse and the arrhythmogenic potential of the Osborn waves is not fully
understood.

Reference
1. Maruyama M, Kobayashi Y, Kodani E, et al. Osborn Waves: History and Significance. Indian Pacing Electrophysiol.
J. 2004;4(1):33–39.

Case 16.5  u  605


CASE
16.6
Charles Jazra, MD
Simon Abou Jaoude, MD

Patient History
A 27-year-old female with a family history of coronary artery disease and sudden cardiac death,
admitted to the emergency department for severe chest pain and hyperventilation. The ECG
(Figure 16.6.1) showed a long QT with ST-T modification.

Figure 16.6.1  ECG during hyperventilation.

Figure 16.6.2  ECG after hyperventilation stopped.

The ST-T modification returned to normal after she has been calmed down.

606  u  Section 16: Syncope and ECG Troubleshooting


Discussion and Comment
ECG changes are common in patients with hyperventilation syndrome (HVS). Abnormalities
may include prolonged QT interval, ST depression or elevation, and T-wave inversion.
In patients with subcritical coronary artery stenosis, the vasospasm induced by hypocarbia may
be sufficient to provoke myocardial injury.
The incidence of HVS is high among patients with mitral valve prolapse (MVP), and the chest
pain associated with MVP may be due to hyperventilation.

Reference
1. Chenivesse C, Similowski T, Bautin N, et al. Severely impaired health-related quality of life in chronic
hyperventilation patients: Exploratory data. Respir. Med. 2014; 108(3):517–523.

Case 16.6  u  607


CASE
16.7 Philip Podrid, MD

Patient History
A 75-year-old male with a history of a dilated cardiomyopathy presents to the emergency
department with worsening symptoms of heart failure. Physical examination is consistent with
pulmonary edema, and this is confirmed by a chest x-ray. He is treated with intravenous diuretics. A
12-lead ECG is obtained (Figure 16.7.1). He is admitted to the hospital. Several hours later a change
in his rhythm is noted on telemetry and another 12-lead ECG is obtained (Figure 16.7.2).

Figure 16.7.1  Sinus bradycardia, low voltage, right bundle branch block/right ventricular intraventricular conduction delay, premature
ventricular complexes with AV dissociation.

608  u  Section 16: Syncope and ECG Troubleshooting


Figure 16.7.2  Accelerated idioventricular rhythm, retrograde (VA) Wenckebach, echo beat with right bundle branch block.

Questions
1. Although all of the QRS complexes in Figure 16.7.1 have a right bundle branch block, what
accounts for the different in QRS width and the subtle change in morphology?
2. What is the rhythm in Figure 16.7.1?
3. Why does every fourth QRS complex in Figure 16.7.2 have a different morphology?

Discussion
Figure 16.7.1 shows a regular rhythm at a rate of 48 bpm. There is a P wave (+) before each QRS
complex with a constant PR interval (0.20 seconds). The P wave is positive in leads I, II, aVF, and
V4 –V6. This is a normal sinus rhythm. QRS complexes 1, 3, and 5 are wide (0.16 seconds) and have a
morphology of a right bundle branch block with a tall broad R wave in V1 (→) and a broad terminal
R wave in leads I, V5, and V6 (←). Complexes 2 and 4 also have a morphology resembling a right
bundle branch block but they are slightly narrower (0.11 seconds) and there is no broad S wave in
leads I, V5, and V6. These QRS complexes represent an intraventricular conduction delay to the right
ventricle and not a right bundle branch block. The QRS complexes have low voltage (i.e., less than
<5 mm in each limb lead and <10 mm in each precordial lead). The axis is normal between 0° and
90° (positive QRS complex in leads I and aVF). The QT/QTc intervals are slightly prolonged
(520/465 ms) but are normal when corrected for the prolonged QRS complex width (460/410 ms). At
the end of the tracing there are two wide QRS complexes (0.18 seconds) (^) that have a different
morphology, resembling a left bundle branch block, and are occurring at a slower rate (42 bpm).
There are no P waves before these QRS complexes, but there are P waves after them, within the ST
segments (*); the RP intervals are different and the P waves are dissociated from the QRS

Case 16.7  u  609


complexes. These P waves have the same PP interval as the sinus rhythm (└┘). Therefore, there is
AV dissociation present and these two QRS complexes are ventricular, and they are premature
ventricular complexes.
Figure 16.7.2 shows an irregular rhythm and there is group beating with four QRS complexes
and a pause. Therefore, the rhythm is regularly irregular. The first three QRS complexes of the
group are regular, at a rate of 68 bpm. These complexes are wide (0.18 seconds). They have a same
morphology as the two ventricular complexes seen in Figure 16.7.2. Therefore, there is an
accelerated idioventricular rhythm. The fourth QRS complex (v) is occurring earlier and it has a
right bundle branch block morphology with a broad R wave in lead V1 (←) and a broad terminal S
wave in leads I, V5, and V6 (↑). It has the same morphology as the sinus complexes 1, 3, and 5 seen in
Figure 16.7.1; thus this complex is supraventricular. There are no obvious P waves seen before the
ventricular complexes, but there are alterations of the ST-T waves. As seen in lead aVL, these
waveforms on the ST-T waves are P waves (^, +, *). After the first wide complex, there is a P wave on
the ST segment (^), after the second wide QRS complex there is a P wave at the nadir of the T wave
(+), causing it to have a sharp negative morphology , while after the third wide QRS complex, the P
wave is at the end or upstroke of the QRS complex (*). The P waves are negative in leads II and aVF
and are positive in lead aVR. Hence these are retrograde P waves and are due to VA conduction
from the ventricular complex. There is a progressive lengthening of the RP interval (└┘) which is
VA or retrograde Wenckebach. The fourth QRS complex is supraventricular and is preceded by a
negative P wave; this is therefore termed an echo beat. This occurs as a result of the retrograde
atrial impulse that then conducts antegradely through the AV node to restimulate the ventricle.
Since antegrade conduction to the ventricles is through the normal AV node–His-Purkinje system
the echo beat is supraventricular. An echo beat will occur whenever there is a preceding QRS
complex that can result in VA conduction, i.e., a junctional complex, ventricular complex, or a
ventricular paced complex. These complexes do not have a P wave before them and hence there is
intact VA conduction and retrograde activation of the atria. The VA conduction may occur via an
overt or concealed bypass tract, one of two dual AV nodal pathways, or through the same AV nodal
pathway. In this case, retrograde conduction is through the AV node as there is retrograde
Wenckebach, which is due to decrimental conduction. Generally, only the AV node manifests
decrimental conduction.

610  u  Section 16: Syncope and ECG Troubleshooting


CASE
16.8
Scott Sakaguchi, MD
Baris Akdemir, MD

Patient History
A 44-year-old male had the ECG in Figure 16.8.1 obtained as part of a drug study in which he was
participating. He had no symptoms but the study coordinator was concerned that the ECG had
changed significantly from enrollment (Figure 16.8.2).

Figure 16.8.1  Presenting ECG. The ECG is notably different from a prior ECG. Lead II is strikingly isoelectric.

Case 16.8  u  611


Figure 16.8.2  Comparison ECG. Lead III is similar to the presenting ECG.

Questions
1. Why has the ECG changed so dramatically?
2. Why is lead II isoelectric?

Discussion
The presenting ECG (Figure 16.8.1) shows an electrode reversal of the right arm and right leg.
The isoelectric recording in lead II is striking. ECGs record the electrical difference between
two points. No electrical difference will be recorded if the two recording electrodes are in the same
location. An ECG technician is unlikely to put two electrodes on the same limb, but they can do
something physiologically equivalent by placing two active electrodes on the right and left leg.
Relative to the heart, the two legs meet at a “common” point at the torso, distant from the heart.
Normally the left leg electrode is actively used in ECG recordings while the right leg electrode is
tied to ground. Einthoven noted that there was virtually no difference in the ECG whether he used
the left or right leg as an active recording site with the other as ground.1
ECG lead II normally records between the right arm and left leg. The simplest electrode reversal
that produces an isoelectric recording in lead II then is to place the right arm electrode on the right
leg (and the right leg electrode, or ground, on the right arm) so that lead II will record the electrical
difference between the two legs. Examination of lead II in the ECG shows how close this recording
truly is to isoelectric: deflections of < 0.5 mm are seen with each QRS complex (Figure 16.8.1, red
arrows).
Figure 16.8.3A, left, shows the normal vectors for ECG leads I (solid line), II and III (dotted
lines). ECG lead III records from the left arm to the left leg. These two electrodes (left arm and left
leg) are correctly placed in Figure 16.8.1 so lead III is correct. This is confirmed in the comparison
ECG (Figure 16.8.2, red circle), while the other five limb leads are markedly different between the

612  u  Section 16: Syncope and ECG Troubleshooting


two recordings. In Figure 16.8.1, the incorrectly recorded ECG, lead I, is an inversion of lead III.
Lead I records from the misplaced right arm electrode on the right leg to the (correctly placed) left
arm electrode (Figure 16.8.3A, right). Since the two leg electrodes are electrically equivalent relative
to the heart this recording is the inverse of lead III.

Figure 16.8.3  Normal and incorrect recording vectors. The left side diagrams of panels A, B, and C show the normal recording vectors
of the three leads I, aVL, and aVR, respectively. The right side diagrams show the corresponding recording vectors when the right arm
electrode is misplaced on the right leg. Lead I incorrectly records an inverted lead III (panel A). The augmented leads aVL (panel B), aVR
(panel C), and aVF (not shown) tie two limb leads together to create a virtual reference. Misplacing the right arm electrode changes the
virtual references such that the augmented leads provide no more electrocardiographic information than that contained in the correctly
recorded lead III.

The augmented leads are particularly interesting. The augmented leads are recorded by tying
two of the limb electrodes together and recording from that combined electrode to the remaining

Case 16.8  u  613


limb electrode.2 Thus, to record lead aVL, the right arm and left leg electrodes are tied together to
create a reference that is effectively midway between the right arm and left leg. This virtual
reference is then used to record to the left arm electrode (Figure 16.8.3B, left). In the ECG of Figure
16.8.1 with incorrectly placed electrodes, however, the right arm electrode is on the right leg so that
lead aVL compares the (correctly placed) left arm electrode against the reference formed by
combining the two electrodes on the electrically “equivalent” right and left legs (Figure 16.8.3B,
right). Thus lead aVL, like lead I, also shows an inversion of lead III (Figure 16.8.1, red circles).
Leads aVR and aVF are essentially half-amplitude recordings of lead III (Figure 16.8.1, blue
circles). Lead aVR records between the right arm electrode and a reference formed by the
combination of the left arm electrode and the left leg electrode. This reference is a point midway
between the left arm and left leg (Figure 16.8.3C, left). When the right arm electrode is incorrectly
placed on the right leg, the recording will compare the leg potential to the same reference. The
vector will have the same orientation as lead III, but half the amplitude (Figure 16.8.3C, right).
Likewise, lead aVF uses a reference that combines the correctly placed left arm electrode and the
right arm electrode that has been incorrectly placed on the right leg. This reference is, again,
midway between the left arm and the “common” legs. The correctly placed left leg electrode is then
recorded against this reference. The resulting vector will have the same orientation as lead III, but
of half the amplitude (not shown).
One of the interesting outcomes of the electrode reversal seen in this ECG is that nothing can
be discerned from limb leads other than lead III. Of the four limb electrodes, one is used for ground
and is incorrectly placed on the right arm. One active electrode is on the left arm and the remaining
two electrodes are on the “electrically equivalent” legs. Thus, no combination of the three active
electrodes can provide any information beyond the electrical difference between the left arm and
the (common) legs; in other words, lead III or variations in its orientation or amplitude.

References
1. Einthoven W, Fahr G, De Waart A. Über die Richtung und die manifeste Grösse der Potentialschwankungen im
menschlichen Herzen und über den Einfluss der Herzlage auf die Form des Elektrokardiogramms. Pflüger’s Archiv
für Physiologie. 1913;150:275–315.
2. Goldberger E. A simple, indifferent, electrocardiographic electrode of zero potential and a technique of obtaining
augemented, unipolar, extremity leads. Am. Heart J. 1942;23:483–492.

614  u  Section 16: Syncope and ECG Troubleshooting


CASE
16.9
Scott Sakaguchi, MD
Baris Akdemir, MD

Patient History
A 35-year-old female whose ECG is among the daily ECGs for reading. History is otherwise
unknown.

Figure 16.9.1  Presenting ECG. Leads I and aVL show a prominent rS pattern. The precordial leads show normal R/S progression.

Question
What is the differential diagnosis of deep Q waves or rS waves (red arrows) in leads I and aVL?

Discussion
The differential diagnosis of Q waves in leads I and aVL includes recording error, situs inversus, and
high lateral infarction. In this case, the arm electrodes have been reversed. Computerized ECG
scoring algorithms often include steps to identify this recording error. Lead I records the electrical
difference between the left and right arm; reversing these electrodes will invert the recording (turn
it “upside down”). Leads II and III are “swapped” because they normally record the electrical
difference from the right and left arms, respectively, to the left leg. Likewise, leads aVL and aVR are
swapped (green arrows). Lead aVF is unaffected. A comparison ECG confirmed that the electrodes
had been misplaced.
Lateral infarction is unlikely because lead I shows an inverted P wave (blue arrows) that would
require an additional explanation (e.g., ectopic atrial rhythm). Reversal of the arm electrodes inverts

Case 16.9  u  615


the entire recording in lead I including P wave, QRS complex, and T wave. Closer inspection of
leads I and aVL shows a tiny R wave (red arrows). Had the arm electrodes been placed correctly,
these would have been tiny Q waves that represent normal septal activation.
Ruling out situs inversus requires examination of the precordial leads. Although the arm
electrodes have been reversed, the precordial leads show a normal progression of R waves and R/S
ratio from V1 to V6 (Figure 16.9.1, inset). For comparison, the ECG in Figure 16.9.2 shows a 43-year-
old male with situs inversus (the otherwise de-identified header was included in the scan to show
that the computerized scoring system flagged the possibility of arm lead reversal and diagnosed an
infarction in the absence of such a recording error).

Figure 16.9.2  Situs Inversus. Leads I and aVL show an rS and Q pattern, respectively. The R/S progression is abnormal and the amplitudes
of the QRS complexes are progressively smaller toward V6.

Although all of the electrodes are correctly placed, the limb leads show the same abnormal
pattern as the ECG with the arm electrodes reversed because, in this case, the heart and its position
are “reflected” to the right chest. In this case, however, the precordial leads show progressively
smaller QRS complexes and little or no R waves moving from lead V1 toward lead V6 because these
leads are progressively farther away from the ventricle (Figure 16.9.2, inset). The right-sided heart
also reverses the functional relationship between leads V1 and V2. Lead V2 is a right-sided V1
(sometimes labeled RV1) and lead V1 is a right-sided V2 (sometimes labeled RV2). Lead V2, i.e., RV1,
shows a P-wave morphology (negative or biphasic) expected in lead V1 with a left-sided heart (blue
arrow). The P wave in lead V1 (RV2) is upright as would be expected in lead V2 with a left-sided heart
(red arrow).

616  u  Section 16: Syncope and ECG Troubleshooting


CASE
16.10
Paolo China, MD
Sakis Themistoclakis, MD

Patient History
A 22-year-old female who had complained of pain and swelling of her legs in the previous weeks
was admitted at the emergency department (ED) for cardiac arrest. She was taking oral estrogen-
progestin contraceptive therapy and her mother and grandmother were on blood-thinning
medicines for unknown reasons. In the ED, a first ECG was done (Figure 16.10.1).

Figure 16.10.1  12-lead resting ECG at hospital admission (paper speed 25 mm/s).

Case 16.10  u  617


Question
What is the diagnosis?

ECG
The first ECG (Figure 16.10.1) at rest revealed sinus tachycardia with heart rate of 130 bpm. In the
peripheral leads a peaked P wave in lead II, right axis deviation and the “S1Q3T3” pattern
characterized by the combination of deep S wave in leads I and aVL, Q wave with inverted T wave
in lead III were seen. An incomplete right bundle branch block (RBBB) with large terminal R wave
in leads V1–V3 associated with inverted T waves in the same leads were also observed.

Discussion, Interpretation, and Answer


The echocardiogram showed a moderate right atrial dilatation, severe right ventricular dilatation
with impaired systolic function and severe pulmonary hypertension while left ventricular chambers
had normal dimension and function. No interatrial shunts were detected. A peripheral Doppler
ultrasound showed left occlusive thrombosis extended from the distal femoral superficial vein to
the popliteal and peroneal vein. An urgent chest angio-CT scan found a massive bilateral acute
pulmonary embolism.
The patient was supported with ventilator assistance, treated with unfractionated heparin, and
underwent percutaneous suction thrombectomy with aspiration catheters. After a few days, the
patient completely recovered and was hemodynamically stable. The transthoracic echocardiogram
repeated 14 days after admission showed normal right atrial and ventricular function as well as
minimal mitral and tricuspid regurgitations with normal pulmonary pressure and no pericardial
effusion.
Serial 12-lead ECGs (Figure 16.10.2) repeated daily after the beginning of therapy showed the
evolution and recovery from the right atrial and ventricular overload with P-wave changes in lead II,
disappearance of incomplete RBBB, and appearance of more prominent negative and diphasic T
waves in precordial leads with a QTc prolongation. Interestingly, Q wave in leads III disappeared,
suggesting the normalization of the initial septal superior-leftward depolarization vector as well as
terminal forces (S in leads I and aVL) together with reduction of right axis deviation on the frontal
plane.

618  u  Section 16: Syncope and ECG Troubleshooting


Figure 16.10.2  12-lead ECGs obtained at day 1, 3, 4, and 12 after hospital admission (paper speed 25 mm/s).

Case 16.10  u  619


ECG abnormalities in pulmonary embolism (PE) become more evident with the relevance of the
hemodynamic overload in the right heart sections, which reflects the extent of obstruction of the
pulmonary vascular tree. However, even with major pulmonary artery obstruction, ECG
abnormalities cannot be observed and minimal or nonspecific waveform changes or even normal
ECG can be registered. The more common abnormalities observed in patients with PE are sinus
tachycardia or atrial fibrillation. The pressure and volume overload in the right chambers is
reflected by right ventricular conduction delay with appearance of large R wave in leads V1–V3, right
axis deviation, and T-wave inversion in leads V1–V4. Although less frequent, a tall and peaked P
wave in inferior leads with right axis deviation called “P pulmonale” can appear.1,2 The “S1Q3T3
pattern” described in 1935 by McGinn and White3 has a reasonable explanation due to the effect of
right ventricular overload on septal and free wall depolarization forces: the rotation of septal
depolarization vector in superior and leftward direction (Q wave in lead III), and the right and
inferior deviation of the last depolarization vector (S wave in leads I and aVL). However, this classic
pattern occurs only in about 10% of cases with acute PE. Voltage reduction in the peripheral leads
and clockwise rotation in precordial leads, not present in this case, are another typical feature of
severe right ventricular impairment. ECG abnormalities usually disappear in the days after the
acute episode with treatment of PE and a stabilization of the hemodynamics of patients, but they
can also give important prognostic information. Indeed, a previously unknown complete RBBB,
atrial fibrillation, peripheral low voltage, and Q wave in leads III and aVF (but not in lead II and ST
elevation or depression in leads I, II, and V4 –V6) have been associated with increased 30-day
mortality4,5 while resolution of anterior T-wave changes has been linked to an improved prognosis.
Differential diagnosis includes changes of right ventricular overload that can also be seen in
other pathological conditions such as acute exacerbation of chronic pulmonary disease,
myocarditis, and arrhythmogenic right ventricular cardiomyopathy/dysplasia.

References
1. Sreeram N, Cheriex EC, Smeets JL, et al. Value of the 12-lead electrocardiogram at hospital admission in the
diagnosis of pulmonary embolism. Am. J. Cardiol. 1994;73:298–303.
2. Yoshinaga T, Ikeda S, Shikuwa M, et al. Relationship between ECG findings and pulmonary artery pressure in
patients with acute massive pulmonary thromboembolism. Circ. J. 2003;67:229–232.
3. McGinn S, White PD. Acute cor pulmonale resulting from pulmonary embolism its clinical recognition. JAMA.
1935;104(17):1473–1480.
4. Toosi MS, Merlino JD, Leeper KV. Electrocardiographic score and short-term outcomes of acute pulmonary
embolism. Am. J. Cardiol. 2007;100:1172–1176.
5. Geibel A, Zehender M, Kasper W, et al. Prognostic value of the ECG on admission in patients with acute major
pulmonaryembolism. Eur. Respir. J. 2005;25:843–848.

620  u  Section 16: Syncope and ECG Troubleshooting


Appendix
Contributor Case Number
Abdel Aziz, Ahmed 4.4
Abou Jaoude, Simon 13A.5, 16.5, 16.6
Akdemir, Baris 16.8, 16.9
Amsterdam, Ezra A. 10.1, 10.2, 11.1, 13G.1
Andrade, Jason 8C.1, 8C.2, 9D.1
Asirvatham, Samuel J. 2.1, 4.1, 9A.1, 12.1, 14.2
Badhwar, Nitish 8C.12
Baranchuk, Adrian 8F.1
Barbosa-Barros, Raimundo 9A.12
Bayés de Luna, Antoni 2.2, 7.1, 8F.1, 10.3
Belhassen, Bernard 2.3, 4.2, 8D.1, 9A.2
Benditt, David G. 8A.1, 16.1
Blendea, Dan 4.11, 9A.9, 12.12, 13E.2
Brugada, Pedro 7.2, 7.3, 7.4
Bui, Jonathan 10.1, 10.2, 11.1, 13G.1
Burri, Haran 11.2, 12.2, 16.2
Buzea, Catalin A. 2.4, 2.5, 10.5
Callans, David J. 3.1, 4.3, 10.4, 14.3
Carreras Costa, Francesc 10.3
Chen, Lily 11.1
Cheng, Alan 7.5, 15.1, 16.3
China, Paolo 9A.19, 16.10
Copie, Xavier 8C.7
Crosson, Jane E. 7.5, 15.1, 16.3
Dalili, Mohammad 3.2
Dan, Andrei G. 2.4, 2.5, 10.5
Daniels, Judith 6C.5, 6C.6, 8D.7, 12.14
de Abreu, Luiz Carlos 10.9
Denis, Arnaud 3.3, 3.4, 4.7, 7.8, 10.6
Derval, Nicolas 7.8
Dubuc, Marc 8C.1, 8C.2, 9D.1
Ellenbogen, Kenneth A. 12.3, 12.4, 12.5
Epstein, Andrew E. 2.6, 12.6, 12.7, 12.8, 12.9, 12.10
Ernst, Sabine 13F.1, 14.4, 14.5, 14.6, 14.7
Escobar Robledo, Luis Alberto 2.2
Estes III, N. A. Mark 2.7, 2.8, 5.1, 6C.1, 7.6, 13A.1, 13B.1, 13C.1, 15.2, 16.4
Fahmy, Tamer S. 4.4, 8D.2, 9A.3
Appendix  u 621
Fontaine, Guy 6C.2
Frank, Robert 4.5, 9A.4
García-Niebla, Javier 7.1
Goldwasser, Diego 10.3
Groeneweg, Judith A. 13A.2
Haghjoo, Majid 5.2, 7.7
Haïssaguerre, Michel 4.7
Han, Frederick T. 4.6
Hauer, Richard N. 13A.2, 13A.3, 13A.4
Heidary, Shahriar 13B.5, 13B.6, 13E.3, 15.6
Heidbuchel, Hein 8D.3, 8D.4, 11.3, 15.3
Hocini, Mélèze 10.6
Hsia, Henry H. 4.8, 8D.5
Huikuri, Heikki V. 5.3, 5.4, 5.5
Ip, James E. 3.5, 3.6, 3.7, 3.8, 9A.8
Jaïs, Pierre 3.3
Jazayeri, Mohammad-Ali 2.9, 7.9, 9B.1, 11.4
Jazayeri, Mohammad-Reza 2.9, 7.9, 9B.1, 11.4
Jazra, Charles 13A.5, 16.5, 16.6
Josephson, Mark E. 9A.18
Junttila, Juhani 5.3, 5.4, 5.5
Kalahasty, Gautham 12.3, 12.4, 12.5
Kalman, Jonathan 8C.3, 8C.4, 8C.5, 8C.6, 9A.5, 9C.1
Katritsis, Demosthenes G. 8B.1, 8D.6, 8E.1, 8E.2, 9B.2
Katritsis, George D. 8B.1, 8D.6, 8E.1, 8E.2, 9B.2
Kenttä, Tuomas 5.3, 5.4, 5.5
Knight, Bradley P. 2.10, 12.11, 13C.2
Koopman, Pieter 8D.3, 8D.4, 15.3
Kowey, Peter R. 6A.1, 6B.1, 6C.3
Krahn, Andrew D. 5.6, 6C.4, 7.10, 9A.6, 13E.1
Krishnan, Balaji 8A.1, 16.1
Lascault, Gilles 2.11, 6A.2, 8C.7, 8C.8
Lemery, Robert 4.9, 8F.2, 9A.7, 13B.2
Lepillier, Antoine 8C.8
Lerman, Bruce B. 3.5, 3.6, 3.7, 3.8, 9A.8
Magdy, Mohamed 4.10, 8C.9
Mansour, Moussa 4.11, 9A.9, 12.12, 13E.2
Marcus, Frank I. 13A.6, 13A.7
Massó van Roessel, Albert 2.2
Miller, John M. 8C.10, 8C.11, 9A.10, 9A.11, 14.8
Morady, Fred 2.12, 2.13, 2.14, 3.9, 8G.1
Murphy Jr., Daniel J. 14.1
Myerburg, Robert J. 4.12, 6A.3, 13B.3, 15.4
Nakazato, Yuji 2.15, 7.11, 12.13

622  u  Appendix
Noheria, Amit 2.1, 4.1, 9A.1, 12.1, 14.2
Obel, I. W. P. 6C.5, 6C.6, 8D.7, 12.14
Okutucu, Sercan 3.10, 6D.1, 10.7
Olson, Inger 14.1
Oto, Ali 3.10, 6D.1, 10.7
Padala, Santosh K 12.3, 12.4, 12.5
Pastore, Carlos Alberto 10.8, 13A.8, 13B.4, 13C.3, 13D.1
Paziaud, Olivier 6A.2
Pereira Filho, Horácio Gomes 13A.8, 13B.4, 13D.1
Pérez-Riera, Andrés Ricardo 9A.12, 10.9, 10.10, 13D.2
Philip, Femi 10.1, 10.2, 11.1, 13G.1
Piot, Olivier 2.11, 6A.2
Podrid, Philip 2.16, 3.11, 3.12, 3.13, 4.13, 9A.13, 10.11, 16.7
Pons-Lladó, Guillem 10.3
Richter, Sergio 7.2, 7.3, 7.4
Riley, Michael P. 2.6, 12.6, 12.7, 12.8, 12.9, 12.10
Saba, Magdi M. 3.14, 4.14, 6D.2, 9A.14
Sacher, Frédéric 3.4
Sadr-Ameli, Mohammad Ali 8D.9, 9A.15, 12.15
Saenen, Johan 11.3
Sakaguchi, Scott 16.8, 16.9
Samesima, Nelson 10.8, 13C.3
Santini, Luca 10.12, 11.5
Santini, Massimo 10.12, 11.5
Scheinman, Melvin 4.6, 4.15, 5.7, 8C.12, 9B.3, 13A.9, 15.5
Schwartz, Peter J. 6A.4, 6A.5, 6A.6, 6A.7, 6A.8, 6A.9
Shenasa, Hossein 4.17, 8D.8, 9A.17, 13B.5, 13B.6, 13E.3, 15.6
Shenasa, Mohammad 1.1, 4.16, 4.17, 6C.7, 8D.8, 8D.9, 8D.10, 8D.11, 9A.16,
9A.17, 9A.18, 13B.5, 13B.6, 13E.3, 13H.1, 15.6
Smith, Mariah 4.16
Steinberg, Christian 5.6, 6C.4, 7.10, 9A.6, 13E.1
Stripe, Benjamin 11.1
Themistoclakis, Sakis 9A.19, 16.10
Traykov, Vassil 2.17
Tseng, Zian H. 4.15
Van Hare, George F. 14.9, 14.10, 14.11, 14.12, 14.13
Verma, Nishant 2.10, 12.11, 13C.2
Walsh, Edward P. 14.14, 14.15, 14.16, 14.17, 14.18
Ward, David E. 3.14, 4.14, 6D.2, 9A.14
Woods, Christopher E. 8C.12
Yetiş Sayın, Begüm 3.10, 6D.1, 10.7
Zhang, Li 6A.1, 6B.1, 6C.3

Appendix  u 623

You might also like